Download as pdf or txt
Download as pdf or txt
You are on page 1of 537

Part One

Applied
~ Science
Review Guide
Volume Two
X

KMK Part I Applied Science Review


Guide
7th Edition, Volume 2

Kyle M. Cheatham , O.D., F.A.A.O.


Melissa A. Cheatham , MPAS, PA-C
Kevin B. Wood, Ph.D .

Copyright 2014 by KMK Educational Services, LLC


11

N OTICE

Optometry and vision science are dynamic fields, each subject to the rapid
modification brought about by the continuing progress in medical and biolog­
ical science. Because of the breadth and ever-changing nature of these fields,
the authors, publishers, and all parties involved with the production and prepa­
ration of this text cannot be held responsible for possible omissions or errors
throughout the book. While we have gone to painstaking lengths to ensure
the information is correct and complete to the best of our knowledge, we en­
courage the reader to consult other sources in both designing a study plan for
board exams and, of course, when gathering information for an accurate clinical
decision.
The National Board of Examiners in Optometry (NBEO) is a registered trade­
mark, NBEO does not endorse this guide.

The following is copyrighted material. It is unlawful to distribute without


express written consent from the copyright holder.
Copyright 2014 by KMK Educational Services, LLC.

Copyright 2014 by KMK Educational Services, LLC


Ill

C ontributors

Kyle M. Cheatham, O.D., F.A.A.O. graduated from Centre College with


a B.S. in Biology and completed optometry school at Indiana University School
of Optometry before accepting a residency at the Kansas City VA Hospital. Dr.
Cheatham practices at Heartland Eye Consultants, a secondary eyecare center
in Omaha, Nebraska, where he specializes in ocular disease and low vision.
He serves as an adjunct faculty member of six colleges of optometry teaching
optometric interns at Heartland Eye Consultants. Dr. Cheatham also provides
consultative care within the ophthalmology department at Creighton University
Medical Center and is a diplomate of the American Board of Optometry (ABO).

Melissa A. Cheatham, MPAS, PA-C graduated from the University of


Nebraska, Lincoln, with a Bachelor of Science degree in Communication Studies
before receiving her Master’s in Physician’s Assistant Studies from University
of Nebraska Medical Center. She has clinical experience in family practice
and obstetrics and gynecology and now practices as a board certified physician
assistant in cardiology with the Bryan Heart Institute in Lincoln, NE.

Kevin B. Wood, Ph.D. graduated Magna Cum Laude from Centre College
(KY) with a B.S. in Chemical Physics before completing an M.S. in Biology, a
double Ph.D. in Theoretical Physics and Physical Chemistry at the University
of California, San Diego (UCSD), and a postdoctoral research fellowship in the
Department of Molecular and Cellular Biology at Harvard University. Kevin
is currently an Assistant Professor of Biophysics and Physics at the University
of Michigan, where he leverages theoretical tools from statistical physics to
study population dynamics and the evolution of drug resistance in microbial
infections and human cancers.

Sarah Dougherty Wood, O.D., M.S., F.A.A.O. graduated Magna Cum


Laude from University of Evansville with a dual major in Biology/Chemistry
and graduated with honors from IU School of Optometry. She completed a
residency at the Kansas City VA Hospital, where later she served as director
of the low vision program, VICTORS, for 3 years. Sarah completed a two
year research fellowship at the Boston VAMC in 2009 where her research was
published in Journal of Glaucoma. Dr. Wood received a Master?s degree in
Vision Science from The New England College of Optometry in 2010. She is a
member of the Optometric Glaucoma Society and serves as chair of the AAO
Glaucoma SIG. She currently practices at the Kellogg Eye Center in Ann Arbor
and is a clinical instructor of ophthalmology and vision science at University
of Michigan.
Copyright 2014 by KMK Educational Services, LLC
IV

Additional contributors:
The authors would like to thank Chad Reade, M.D., Chris Wolfe, O.D.,
and Ryan Fenska, O.D., all of whom provided excellent suggestions and
insights which have improved several sections of the text.

Copyright 2014 by KMK Educational Services, LLC


V

Chief Medical Editor


Kendra R. Dalton, O.D. graduated from Emory University in Atlanta, GA
with a B.S. in Biology and graduated summa cum laude from Southern College
of Optometry. She completed a residency in ocular disease and low vision re­
habilitation at the Kansas City VA Medical Center. She is currently practicing
at the Philadelphia VA Medical Center, where she serves as the Externship
Program Director and provides direct patient care. She is an adjunct faculty
member of New England College of Optometry and Pennsylvania College of
Optometry at Salus University.

Copyright 2014 by KMK Educational Services, LLC


f
r
f
(
r
c
(
c
c
(
c

c
(
(
(
(
Copyright 2014 by KMK Educational Services, LLC (
c
(
(
vii

Special Thanks to:


Larry Alexander, O.D.
Paula Campbell
Rxjwan Candy, O.D.
Ryan Fenska, O.D.
Timothy Harkins, O.D.
Chad Reade, M.D.
Thoroughbred Printing, Lexington, KY
Christopher Wolfe, O.D.
The UPS Store (Georgetown, KY)
Matthew Wood
Laura Ruwe

Copyright 2014 by KMK Educational Services, LLC


c
c
(
(
(
c
(
(
(
(
(
(
(
(
(
c
(
(
(
(
(
(
(
Copyright 2014 by KMK Educational Services, LLC (
(
(
(
IX

Foreword

Passage of the National Boards has always been a source of angst for a physician
in training. One’s future in their chosen profession hinges on successful passage
of the series of examinations. The National Boards in Optometry now even take
on more importance, as all states as well as the District of Columbia and Puerto
Rico require passage of the National Boards Parts 1 and 2 for consideration of
licensure. All states likewise require graduation from an accredited school or
college of optometry to sit for the state board. Forty-seven (47) states as well
as District of Columbia and Puerto Rico require passage of Parts 1, 2 and 3
for licensure. The National Boards in Optometry also represent a considerable
investment for the future doctors. The fees for the National Board Examination
exceed $3000 assuming that the tests are passed the first time taken. While
stressful to the optometry student, there is a reason for requiring passage of
the National Boards; the profession of optometry wants to assure the general
population a basic level of competency for optometric physicians.
The pressure to commit to a full-time-job curriculum (often over 32 hours
of classroom time per week) in the schools and colleges of optometry while
attempting to review all necessary materials for the National Boards in Op­
tometry has become a daunting task. Schools and colleges of optometry have
excellent curricula and superb instructors and the subject matter is very rel­
evant to the requirements of the practice of optometry. Unfortunately, the
amount of information covered, classroom time, and required study time leave
little time to organize all information into a format that would be applicable
to an effective review strategy for National Boards. While schools and colleges
of optometry often organize National Boards review courses and student orga­
nizations like the AOSA make refresher courses available at various meetings,
the majority of students are not readily afforded the benefits of these review
courses. The National Boards review course for Part 1 presented herein has
been constructed under the direction of KMK. This course will not only impact
positively on the knowledge base of optometry students and future practition­
ers but will likewise facilitate their passage of the National Boards. The course
simply organizes volumes of information in an integrative fashion. The pieces
of the puzzle are put together to bring a sense of organization to the learning
process. The need for a methodology to attack the integration problem has
never before been satisfied within the constructs of the National Boards in Op­
tometry. The format chosen by KMK makes learning pleasant and deepens the
knowledge base through integration of previously presented course material.
Very difficult principles are simplified, restructured and reassembled to bring a
sense of organization to the learning process.
While the text herein serves as an excellent study guide, a full face-to-face
course is also an option available to interested students. These courses will
be offered at different schools and colleges of optometry on a rotational basis.
Copyright 2014 by KMK Educational Services, LLC
X

Without doubt, the live course will offer a more complete learning situation,
but the study guide will provide more than has ever been accessible to the
optometry student in the past. The KMK organization, comprised of a prac­
ticing optometrist, a physician’s assistant and a physicist brings a fascinating
dimension to the production of a study guide for basic science in optometry.
The result is exquisite in both its depth and its simplicity. This text, as well
as the supplementary face-to face course, is also being constantly updated by
the authors to provide cutting edge information.
The pride that I have for the three individuals who developed this concept is
indescribable. I have always been an optometry student advocate. I believe
that the optometry students are my future colleagues and they are the future
of optometry. Why would we all not want to make their learning process both
more pleasant and effective? This National Boards Part 1 study course is big.
It represents movement in a positive direction and will help to modify and im­
prove the delivery of information to doctors both pre and post graduation. My
congratulations go to the authors for a significant contribution to the profession
of Optometry and in the art of education.
Larry J. Alexander, O.D.
Private Practice
Jeffersonville Indiana/Louisville Kentucky

Copyright 2014 by KMK Educational Services, LLC


XI

Preface to the Seventh Edition


We are happy to introduce our seventh edition study guide, which features
a substantial number of corrections, revisions, and additions. Many of the
improvements in this edition are attributable to our new Chief Medical Editor,
Dr. Kendra Dalton, who brings to the job a great deal of academic expertise
and has shepherded many chapters of the book to their current forms. We are
indebted to Dr. Dalton for her outstanding work, and we’re excited to welcome
her to the KMK team.
Overall, we have attempted to incorporate new clinical ideas and further refine
the presentation of all the material. We hope this book will serve as a useful
reference and a beneficial starting point for board exam preparation. As always,
we encourage students to approach the exam with optimism and focus. We’ve
found that when students view the board exams as an opportunity to improve
their clinical skills and increase their knowledge base, the experience can be
rewarding on both a professional and personal level.
We wish you all the best as you begin your studies.
KMK Educational Services, LLC
July 2014

Copyright 2014 by KMK Educational Services, LLC


(
(
c
(
c
(
(
(
(
(
f

(
(
(
(
(
(
(
c
(
Copyright 2014 by KMK Educational Services, LLC (
(
(
(
Contents

Contents xiii
1 General Physiology 1
1.1 Cellular Functions .................................................................... 3
1.2 Respiration................................................................................ 7
1.3 Circulatory System.................................................................... 18
1.4 Urinary System.......................................................................... 32
1.5 Endocrine System....................................................................... 42
1.6 M uscles...................................................................................... 56
1.7 Gastrointestinal Physiology......................................... 63
2 Ocular Physiology 73
2.1 Eyelids....................... 75
2.2 Tears..................................................... 78
2.3 Extraocular M uscles................................................................. 84
2.4 Cornea......................................................................................... 86
2.5 L e n s............................................................................................ 95
2.6 U vea............................................................................................ 99
2.7 Vitreous...................................................................................... 101
2.8 Circulation ................................................................................ 104
2.9 R etina......................................................................................... 108
2.10 Neurophysiology....................................................................... 113
2.11 Visual Pathway.......................................................................... 122
2.12 Pupillary Pathways.................................................................... 129
2.13 Intraocular P ressure................................................................. 130
2.14 Aqueous...................................................................................... 134
3 Geometrical Optics 141
3.1 Notation................................................................. 143
3.2 Vergence, Objects, and Images ............................................... 143
3.3 Single Spherical Refracting Interfaces (SSRIs)........................ 145
3.4 Snell’s Law ................................................................................ 149
3.5 Thin Lenses................................................................................ 149
3.6 Thin Lens Systems: Thick Lenses............................................ 152
3.7 Stops, Pupils, and P o rts........................................................... 158
xiii
xiv CONTENTS
3.8 Spherocylindrical Lenses........................................................... 161
3.9 Mirrors......................................................................................... 166
4 Physical Optics 169
4.1 Electromagnetic Waves.............................................................. 171
4.2 L asers......................................................................................... 172
4.3 Diffraction................................................................................... 176
4.4 Thin Films ................................................. 177
4.5 Scattering................................... 178
4.6 Polarization................................................................................ 179
5 Physiological Optics I 183
5.1 Ophthalmic Instruments............................................................ 185
5.2 Ophthalmic Characteristics of Lenses....................................... 192
5.3 Frame and Lens Specifications................................................... 197
5.4 Aberrations and Lens D esign................................................... 204
5.5 Prisms and Ophthalmic L enses................................................ 212
5.6 Relative Magnification and Image Sizes.................................... 219
5.7 Comparative Retinal Image siz e s............................................. 221
5.8 Absorptive Lenses ................................ 225
5.9 Contact Lenses.......................................................................... 227
5.10 Magnification and Low Vision................................................... 242
5.11 Clinical Low Vision.................................................................... 250
6 Physiological Optics II 257
6.1 Models of the Eye....................................................................... 259
6.2 Refractive Anomalies: Ametropia............................................. 260
6.3 Ametropia: Clinical Considerations.......................................... 265
6.4 Astigmatism................................................................................ 281
6.5 Optical Effects in Vision............................................................ 283
7 Ocular Motility 287
7.1 Ocular Rotations....................................................................... 289
7.2 Laws of Eye Movement Dynamics.................................................291
7.3 Eye Movement Dynamics ......................................................... 292
7.4 Diagnostics for EOM dysfunction............................................. 303
8 Binocular Vision 311
8.1 Introduction................................................................................ 313
8.2 Non-strabismic BV disorders ................................................... 314
8.3 Assessment of Accommodative Disorders................................. 327
8.4 Common Non-strabismic BV / Accommodative Disorders . . 333
8.5 Sensory Anomalies of BV/Strabismus....................................... 340
8.6 Strabismic disorders................................................................. 356
8.7 Acquired Brain Injury (ABI) ................................................... 359
Copyright 2014 by KMK Educational Services, LLC
CONTENTS xv

9 Visual Perception 367


9.1 Psychophysics............................................................................. 369
9.2 Radiometry vs. Photometry..................................................... 375
9.3 Light Perception....................................................................... 381
9.4 Spatial and Temporal Summation............................................ 387
9.5 Color Perception....................................................................... 392
9.6 Color Vision Anomalies ........................................................... 398
9.7 Perception of Form and Space.................................................. 405
9.8 Depth Perception....................... 409
9.9 Motion Perception , ................................................................. 410
9.10 Temporal Perception................................................................. 412
9.11 Magno vs. Parvo Pathw ays..................................................... 415
10 General Pharmacology 417
10.1 Basic Principles of Pharmacology............................................ 419
10.2 Antimicrobials .......................................................................... 423
10.3 Anti-Inflammatory Medications............................................... 435
10.4 Cold and Allergy Medications.................................................. 439
10.5 Gastrointestinal Medications..................................................... 441
10.6 Respiratory Medications........................................................... 442
10.7 Chemotherapeutic A gents........................................................ 444
10.8 Medications Acting on C N S ..................................................... 445
10.9 Endocrine System Medications ............................................... 450
10.10 Genitourinary Medications........................................................ 452
10.11 Cardiovascular M edications..................................................... 453
10.12 Dermatologic Medications........................................................ 460
10.13 Hallucinogens and Drugs of A b u se......................................... 461
10.14 Disinfectants and A ntiseptics.................................................. 462
10.15 Autonomic / NMJ Drugs ........................................................ 462
10.16 Toxicology................................................................................... 464
10.17 Drug Use in Pregnancy and Breast Feeding.......................... 464
11 Ocular Pharmacology 469
11.1 General Principles ................................... 470
11.2 Autonomic Drugs....................................................................... 471
11.3 Glaucoma D ru g s....................................................................... 484
11.4 Properties of Topical Ocular Anesthetics................................ 486
11.5 Antihistamines.......................................................................... 487
11.6 Anti-Inflammatory Agents........................................................ 488
11.7 D yes............................................................................................ 491
11.8 Agents for Exudative A R M D .................................................. 492
11.9 Hyperosmotic Agents................................................................. 492
11.10 Tear Substitutes ....................................................................... 493
11.11 Preparations Used with Contact Lenses ................................ 494
11.12 Toxicology................................................................................... 494
Copyright 2014 by KMK Educational Services, LLC
f
(
C
CONTENTS
XVI
(
Index 503 £
c
(
(
(
(
(
(
(
(
(
(
(
(
(
(
(
(
(
Copyright 2014 by KMK Educational Services, LLC
(
(
(
(
Chapter

General Physiology

Melissa A. Cheatham, MPAS, PA-C

1
('
f
(
(
(
(
(
(
(
(
(
(
(
(
(
(
(
(
(
(
(
(
(
Copyright 2014 by KMK Educational Services, LLC (
(
(
(
CHAPTER 1. GENERAL PHYSIOLOGY 3

i— SECTION 1.1

Cellular Functions

ffipiillSIl
Intracellular Com partment - Environment inside the cell.
• Consists mostly of water.
• Separated from the extracellular compartment by a cell membrane.
• Comprised of all intracellular organelles.
• M ain electrolyte is K +.
Extracellular Com partment - Environment outside the cell.
• Divided into two parts - blood plasma and interstitial fluid.
• Communicates with the intracellular compartment via movement of
molecules and ions through the cell membrane.
• M ain electrolyte is N a+.

Membrane potential is a function of both electrical (charge) and chemical


(concentration) gradients.
• The cytoplasm of the cell consists of negatively charged organic ions (an­
ions) that are “fixed” inside the cell (i.e. they cannot leave). These fixed
anions attract K+ ions, which are inorganic ions that are able to easily
diffuse through the cell membrane.
• K+ will flow in and out of the cell until the intra and extracellular K+
concentration is in equilibrium. However, since there is still a concentra­
tion of negative “fixed” ions INSIDE the cell, the the potential difference
of the cell is -90mV.
• The concentration of K-fi within a cell is greater than the concen­
tration of K+ in the extracellular fluid.

The equilibrium potential of the cell is the potential difference


between the intra and extracellular environments when there is no
NET movement of ions. This potential is -90mV in cells (2).

Copyright 2014 by KMK Educational Services, LLC


4 1.1. CELLULAR FUNCTIONS
N af concentration will always be greater in the extracellular fluid compared to
the intracellular environment. However, some Na-f is capable of slowly diffusing
through the cell membrane, creating a resting membrane potential that is
between -65 and -85 mV. N af will always be more highly concentrated outside
the cell than inside.
• N af/K f ATPase pumps help to alter membrane potential by pumping
out 3 Na-f for every two K f that enter the cell. This counterbalances
the leakage of K f into the cell, allowing for constant ion concentrations
and a constant resting potential.

Depolarization occurs when the inside of the cell becomes more


positive than the resting state (due to influx of Naf). Hyperpo­
larization occurs when the inside of the cell becomes less positive
than the resting state (due to efflux of Naf). An action poten­
tial occurs when the membrane is depolarized beyond a certain
threshold (13).

Hirarisjioft M
Cellular transport of molecules may occur in two directions:
1 Up-hill: Transports molecules from an area of lower to higher concen­
tration (against the concentration gradient). Up-hill transport requires
energy. There are two main types of up-hill transport:
• Prim ary active transport: Directly uses ATP.
• Secondary active transport: Indirectly uses ATP by using a
concentration gradient set up by primary active transport (usually
Na-f / K f pump) to transport two or more ions. There are two types
of secondary active transport:
- Co-Transport: Both ions are moved in the same direction. Ex:
Naf/Amino Acid in PCT of kidneys, N af /K f /2C1- in the As­
cending Loop of Henle.
- Counter-Transport: Moves ions in opposite directions. Ex: N af/C a2f
transport in muscle cells.
2 Downhill: Transports molecules from an area of higher to lower con­
centration (down the concentration gradient). Downhill transport does
NOT require energy. There are two types of downhill transport systems:
• Simple: Non-electrolytes (no charge) diffuse across a membrane.
Copyright 2014 by KMK Educational Services, LLC
CHAPTER 1. GENERAL PHYSIOLOGY 5

* Carrier-M ediated Transport: Utilizes integral membrane pro­


teins to move charged molecules across the membrane (e.g. facili­
tated diffusion).

The passage of glucose, amino acids and other polar molecules


through the cell membrane is mediated by carrier proteins in the
cell membrane. Carrier mediated transport exhibits the properties
of specificity, competition and saturation (13), The transport rate
of molecules such as glucose reaches a maximum when the car­
riers are saturated. This maximum rate is called the transport
maximum.

Adrenoreceptors
• Alpha 1: Increases [IP 3], which t intracellular [Ca2+], Receptors are
located in the radial muscle of the iris, vascular SM, and the GI tract
(sphincters).
• Alpha 2: Inhibits adenylyl cyclase, which 4- cAMP. Receptors are located
in the presynaptic adrenergic neurons and the walls of the GI tract.
• Beta 1: Stimulates adenylyl cyclase, which f cAMP. Receptors are located
in the heart, salivary glands, adipose tissue, and kidneys.
• Beta 2: Stimulates adenylyl cyclase, which f cAMP. Receptors are located
in the bronchioles, GI tract, bladder wall, and vascular SM.

Topical bet a-blockers act on beta 1 and 2 receptors and may cause
a decreased heart rate and bronchoconstriction. Ask about pul­
monary symptoms and check pulse on all patients using topical
beta-blockers!

Cholinoreceptors
• Nicotinic: Opens Na+ and K+ channels, which causes depolarization.
Receptors are located in the motor end plates of skeletal muscles, post­
ganglionic neurons of the SNS and PNS, and the adrenal medulla.

Copyright 201,4 by KMK Educational Services, LLC


6 1.1. CELLULAR FUNCTIONS

Nicotinic receptors are unique! They are the only receptors


that open Na+ and K-j- channels. The other receptors alter either
calcium concentration or cAMP.

• Muscarinic: Increases [IP3], which increases intracellular [Ca2+]. Re­


ceptors are located in the vascular SM, sweat glands, and PNS effector
organs.

Remember that Alpha 1 and Muscarinic receptors both increase


intracellular Ca2+. Beta 1 and 2 receptors increase cAMP. Alpha
2 decreases cAMP.

Body Fluids
Intracellular Compartment - 25 L; K+ is the major component.
Extracellular Compartment - 15 L total; composed of interstitial fluid and
plasma.
1 Interstitial Fluid - 12 L; Na+ is the major component.
2 Plasma - 3 L; Na+ is the major component, but also has protein (9).

60% of our body is composed of water!

The ECF and ICF have the same osmolality. The ICF is slightly more acidic.
Intracellular Volume = Total Volume —Extracellular Volume (1.1)
Control Systems of the Body
Diffusion
Diffusion is the net movement of molecules or ions from a region of higher to
lower concentration. It is a form of passive transport and utilizes thermal
energy from molecules, rather than ATP produced by cellular respiration. Dif­
fusion stops when the concentration of molecules is equal on both sides of the
membrane. The rate of diffusion increases under the following conditions:
Copyright 2014 by KMK Educational Services, LLC
CHAPTER 1. GENERAL PHYSIOLOGY 7
• Decreased membrane thickness*
• Increased membrane permeability to the diffusing substance.
• Increased temperature.
Simple Diffusion
Simple diffusion is a type of passive transport that involves the movement of
small molecules and inorganic ions through the cell membrane. Inorganic
ions (e.g. Na+ and K+) pass through specific channels within the membrane,
while steroid hormones and other lipids pass directly through the phospholipid
bilayer.
Osmosis
Osmosis is the simple diffusion of water across a semipermeable membrane
(more permeable to water than solute) from an area of low to high solute
concentration.
• Osmolality is the concentration of total solute in osmoles/kg of wa­
ter (4).
• The solution with the higher osmolality has the higher osmotic pres­
sure.

Water moves by osmosis to the solution of higher osmolality and


osmotic pressure.

- SECTION 1.2 ----------- ;------------------------------------

Respiration
We now introduce concepts related to respiration (4, ch. 14).

There are two lungs in the body, each composed of multiple surfaces and zones:
• The right lung has 3 lobes and is larger than the left lung.
• The left lung has 2 lobes and is smaller because it shares the left thoracic
cavity with the heart.
Copyright 2014 by KMK Educational Services, LLC
8 1.2. RESPIRATION
Each lung has 3 surfaces:
1 Costal surface faces the sternum, costal cartilage, and the ribs.
2 Mediastinal surface faces the hilum of the lung and is medial to the
mediastinum.
3 Diaphragm surface rests on the dome of the diaphragm.
Each lung has two zones:
1 Conducting zone includes the upper airways, trachea, bronchi, and
bronchioles. This zone warms and humidifies the air before it reaches
the respiratory zone. It is innervated by the sympathetic nervous sys­
tem through /?2 receptors, which cause relaxation of smooth muscle and
broncho dilation, allowing the inward flow of air.
2 Respiratory zone includes the respiratory bronchioles, alveolar ducts,
alveolar sacs, and alveoli.

In order for air to reach the lungs, it must pass through the pharynx
and trachea before reaching the main bronchi that lead to each
lung. Because the right main bronchus is shorter and wider than
the left, particles that pass down the trachea are more likely to end
up in the right main bronchus.

Mechanics.of:Bre . bTi'T > V- ' b


The following muscles are involved in respiration: external intercostals, internal
intercostals, parasternal intercostals, scalenes, pectoralis minor, and sternoclei-
domastoids (6).
Inspiration (Active Process)
• Diaphragm contracts, lowers and flattens, increasing thoracic volume.
• Parasternal and external intercostal muscles contract to expand the
thoracic volume laterally.•
• The scalenes lift the rib cage in an anterior-posterior position.

Copyright 2014 by KMK Educational Services, LLC


CHAPTER 1. GENERAL PHYSIOLOGY 9

The combined effort of these muscles increases thoracic volume,


causing intrapulmonary pressure to decrease and allowing air to
flow into the lungs.

Expiration (Passive Process)


• After active inspiration, the stretched thoracic cavity and lungs pas­
sively recoil due to their elastic tension, and the diaphragm relaxes.
• The subsequent decrease in lung volume causes the pressure in the alveoli
to increase above the atmospheric pressure, causing air to be pushed out
of the lungs.
• In forced expiration, the internal intercostal muscles contract and
squeeze the rib cage and the abdominal muscles force the organs up
against the diaphragm, further decreasing the volume of the thorax (2).
Lung Volumes and Capacities
We can directly measure 5 lung volumes:
1 Tidal Volume (TV) - Air volume inspired or expired after each
normal breath.
2 Inspiratory Reserve Volume (IR) - The maximum volume in­
spiration after a tidal volume (normal) inspiration.
3 Expiratory Reserve Volume (ER) - The maximum volume of
expiration that can be pushed out after tidal volume (normal)
expiration.
4 Inspiratory Capacity (IC) - The volume of air inhaled during a
maximal inspiration. Measurement of the IP begins at the end
of a normal tidal volume and is approximately 3L.
5 Vital Capacity (VC) - The total lung capacity that can be ex­
pired voluntarily after a maximum inspiration. It is the com­
bined total of the tidal volume, inspiratory reserve volume and
expiratory reserve volume and is approximately 4.5L.

Respiratory Rate (RR) = Number of breaths per minute.


Normal RR is 12 breaths/minute at rest. Minute ventila­
tion = Tidal Volume x Breaths/minute (TV x RR).

Copyright 2014 by KMK Educational Services, LLC


10 1.2. RESPIRATION
j

We can also measure lung volumes indirectly:

1 Residual Volume (RV) - The volume that remains after a max­


imum exhalation of the lung (RV = FRC - ER).
2 Functional Residual Capacity (FRC) - The volume left in the
lungs after a normal passive expiration (FRC = ER -f- RV).
3 Total Lung Capacity (TLC) - The entire volume of the lung
that can exchange gas (TLC = VC + RV).
4 Forced Vital Capacity (FVC) - The volume of air that is expired
during a maximal expiration after a maximal inspiration.
5 Forced Expiratory Volume (FEV) - Often measured over one
second (FEV1) and compared to the FVC to determine the
respiratory function of the patient and to classify lung disease
as restrictive or obstructive (12).

Spirogram of Lung Volumes

Figure 1.1: Spirogram

Copyright 2014 by KMK Educational Services, LLC


CHAPTER 1. GENERAL PHYSIOLOGY 11

Spirograms are used clinically to monitor lung function


and disease. Obstructive airway disease (e.g. COPD,
chronic bronchitis, emphysema, asthma) is characterized
by a reduction in the FEVl/FVC ratio (how much air
the patient can force out in one second/total forced lung
capacity). A ratio of 80% or less indicates obstructive
lung disease, with a lower percentage indicating worse dis­
ease (12).

Restrictive lung disease is characterized by poor expan­


sion of the lungs with a decrease in lung volume and a
normal to elevated FEVl/FVC ratio. Examples include
toxoplasmosis, histoplasmosis, and sarcoidosis (12).

There is also air in the lungs that does NOT participate in gas ex­
change.
Anatomic Dead Space - Volume of air in the conducting zone of the
lungs (bronchi and bronchioles).
Functional Dead Space - Air volume in the alveoli.
Physiologic Dead Space - Anatomic plus the functional dead space.

Once air reaches the alveoli in the lungs, O2 and <702 will diffuse
across the alveolar and capillary walls to and from the bloodstream.
Blood flow through the lungs is not uniform: the base of the lungs
has the highest blood flow and the greatest exchange of 02 (4).
Oxygen
• 98% of 02 is bound to hemoglobin, a protein with 4 peptide sub­
units. Each subunit contains a heme group with a reduced iron
core that shares electrons with an 02 molecule. Hb is consid­
ered oxygenated when it is saturated with 4 02 molecules. Hb
carrying less than 4 02 molecules is considered deoxygenated.•
• The remaining 2% of 02 is dissolved in the plasma (13).
Copyright 2014 by KMK Educational Services) LLC
12 1.2. RESPIRATION
Carbon Dioxide
• The concentration of C0 2 is 20X greater than O2 in the blood­
stream. CO2 is transported from tissues to the lungs in 3 forms:
1 90% of CO2 is transported to the lungs in the form of bi­
carbonate. Carbonic anhydrase within RBCs catalyzes the
first step of the following reaction:
C 0 2 + H20 H 2CO3 IICO 3- + H + (1.2)
2 5% of C 0 2 is bound to hemoglobin as carbaminohemoglobin.
3 5% is dissolved as C0 2 within plasma (13).
Transport of C0 2
Transport of GO2 from the tissues to the lungs occurs through the
following steps:
1 CO2 moves across the cell membrane of RBCs via simple diffu­
sion.
2 CO2 is hydrated to H 2 CO3 within RBCs.
3 H 2CO3 is broken down into IICO 3 — and H-f- by carbonic anhy­
drase.
4 Deoxyhemoglobin acts as a buffer within the bloodstream to
counteract released H-|~ ions from the breakdown of H 2 CO3 .
5 Cl- and HCO^ —are exchanged across the RBC membrane. HCO 3 —
within the plasma is then taken to the lungs.
6 After reaching the lungs, Cl- and HCO 3— are again exchanged
across the RBC membrane in the veins of the lungs. HCO 3 —
then combines with H + to form H 2CO3 .
7 H 2CO3 dissociates into C0 2 and H 20 , which are expired by the
lungs (13).
The air we breath has an atmospheric pressure of 760 mmHg. 21%
of the air is composed of 0 2>resulting in a partial pressure of oxygen
(P 0 2) of 160 mmHg. There is very little CO2 in the air, so the partial
pressure of C0 2 (PCO 2 ) is considered about zero,
• As air enters the trachea, the P 0 2 decreases due to the vapor­
ization of water. P 0 2 is decreased even further as air enters the
lungs and 0 2 dissociates to travel into the bloodstream.•
• The partial pressure of C0 2 increases within the lungs as C 0 2
diffuses from the bloodstream into the alveoli (4) (13).
Copyright 2014 by KMK Educational Services, LLC
CHAPTER 1. GENERAL PHYSIOLOGY 13

Diffusion Rates of CO2 and O2


Diffusion rates across the respiratory membrane of the lungs is in­
fluenced by the following factors:
• Thickness of the membrane.
- The diffusion rate is inversely proportion to the thickness
of the diffusion membrane (i.e, diffusion distance).
• Surface area of the membrane.
- Surface area and rate of diffusion are directly proportional.

Emphysema decreases the surface area of the lungs, re­


sulting in a decrease in the exchange of CO2 and 0 2 *

• Diffusion coefficient of the gas.


—Measures how easily a gas will pass across a given mem­
brane based on the solubility and size of the gas.
— Remember that CO2 is approximately 20X more soluble
than 0 2 *
• Partial pressure differences.
— Gas will always flow from an area of high to lower pressure.

Oxygen Dissociation Curve


The oxygen dissociation curve describes the oxygen carrying capacity of
the blood. It relates the partial pressure of oxygen to the ability of hemoglobin
to bind to oxygen (i.e. hemoglobin saturation).
• When the P02 of surrounding tissues is high, hemoglobin can more read­
ily bind to oxygen molecules, resulting in hemoglobin saturation.•
• When the PO% is low, hemoglobin is less capable of binding to oxygen.

Copyright 2014 by KMK Educational Services, LLC


14 1.2. RESPIRATION

Hemoglobin
saturation (%)

P02 (mm Hg)

Figure 1.2: Oxygen Dissociation Curve

Copyright 2014 by KMK Educational Services, LLC


CHAPTER 1. GENERAL PHYSIOLOGY 15

The binding of 0 2 to hemoglobin is cooperative; once one O2


molecule binds to hemoglobin, it can more easily bind to addi­
tional O2 molecules. The phenomenon of cooperation underlies
the sigmoidal shape of the oxygen dissociation curve.

The behavior of hemoglobin and 0 2 has important consequences for the body (4):
• When the partial pressure of 0 2 is high (e.g. in the vasculature of the
lungs), hemoglobin easily binds to 0 2 and becomes nearly saturated.
• As the O2 rich blood circulates throughout the body, the partial pressure
of 0 2 decreases, causing hemoglobin to release 0 2 molecules into the
bloodstream for diffusion into the surrounding tissues.
• There are a number of factors that can shift the location of the curve
along the X axis:
- A shift in the curve towards the left indicates that hemoglobin more
readily binds to 0 2 at a given partial pressure.
- A shift in the curve to the right indicates that hemoglobin is LESS
capable of binding to 0 2 at a given partial pressure. The following
factors result in a decreased hemoglobin affinity for 0 2 and a shift
in the curve to the right:
* A decrease in pH (higher acidity).
* An increase in temperature.
* An increase in CO2 levels.
- The opposite conditions (higher pH, lower temperature, reduced
CO 2 levels) will shift the curve to the left, representing an increase
in hemoglobin’s affinity for 0 2.

Carbon monoxide affects hemoglobin’s carrying capacity for oxygen


and it’s ability to release oxygen in the following ways:
• Carbon monoxide binds to hemoglobin more readily com­
pared to O2 ■
• Once bound, it decreases the ability of hemoglobin to release
0 2 into the bloodstream.

Copyright 2014 by KMK Educational Services, LLC


16 1.2. RESPIRATION

Respiration is controlled through neural and chemical regulators (4, ch. 16).
Neural Control
The rhythmicity center in the medulla oblongata directly and repeatedly acti­
vates neurons that control respiratory muscles to produce an automatic breath­
ing cycle.
• The activity in the medulla is influenced by the apneustic and pneumo-
taxic centers in the pons, as well as sensory feedback information.
• Activation of pulmonary stretch receptors in the bronchioles leads to
negative inhibition, helping to prevent lung over-inflation.
• Conscious breathing includes direct control by the cerebral cortex via the
corticospinal tracts.
Chemical Control
1 Peripheral chemoreceptors include the carotid and aortic bodies, which
are located within the walls of the carotid arteries and aorta, respectively.
They monitor O2 and CO2 levels within the arterial blood and provide
feedback to the medullary centers.
• They detect changes in CO2 concentration through increased acidity
of the plasma and increased levels of carbonic anhydrase.
• O2 levels affect the sensitivity of the carotid and aortic bodies to
CO 2 levels in the blood. A decline in O2 increases their sensitivity
to CO2 .
2 Central Receptors include the medullary chemoreceptive neurons that are
concentrated in the carotid and aortic bodies.•
• The central receptors are very sensitive to changes in the concen­
tration of H+ ions. Remember that H+ ions cannot penetrate the
blood brain barrier, but CO2 CAN. Higher levels of CO2 lead to
higher levels of H+ ions, resulting in an increase in acidity (decrease
in pH) and an increase in ventilation.
• The buffering ability of cerebrospinal fluid is very limited, so it is
very sensitivity to changes in CO2 and H+. As an example, a change
in PCO 2 from 40 to 44 within the bloodstream causes the respiration
rate to double!

Copyright 2014 by I<MK Educational Services, LLC


CHAPTER 1. GENERAL PHYSIOLOGY IT

Increased acidity within the blood leads to increased ventilation.

The following is a review of conditions caused by an imbalance in acid-base


concentrations within the bloodstream (4, ch. 16).
Respiratory Acidosis
• Caused by hypoventilation, resulting in an increase in dissolved CO2 ,
with a subsequent increase in plasma H+ concentration and a decrease
in pH.
• The kidneys compensate by decreasing excretion of HCO$~, which re­
sults in an increase in HCO 3— in the plasma and a return to the normal
pH.
Respiratory Alkalosis
• Caused by hyperventilation, resulting in a decrease in dissolved CO2
and H+ ions and an increase in pH.
• The kidneys compensate by increasing excretion of HCO 3 —, resulting in
an increase in dissolved CO2 and a return of pH to normal.
Metabolic Acidosis
• Caused by a loss of HCO 3 —, often due to diarrhea. This results in an
increase in plasma H+ concentration and a decrease in pH.
• Respiratory compensation involves an increase in ventilation, resulting
in a greater amount of dissolved CO2 within the plasma and a resulting
increase in pH.
• The kidneys compensate by increasing the excretion of H+ in the urine.
Metabolic Alkalosis
• Caused by an increase in HCO 3 —, primarily due to vomiting (results
in a loss of H+).•
• Respiratory compensation involves a decrease in ventilation, which in­
creases the amount of dissolved CO2 and H+ concentration in the plasma,
re,suiting in a decrease in pH.
Copyright 2014 by KMK Educational Services, LLC
18 1.3. CIRCULATORY SYSTEM
• The kidneys compensate by excreting HCO 3— through the urine.
- SECTION 1.3

Circulatory System

The heart has 4 chambers:


Right Atrium - Forms the right border of the heart and sits on top of the
right ventricle. This chamber receives venous blood from the body.
Right Ventricle - Largest part of the anterior and inferior surface of the
heart. The blood from this chamber goes through the pulmonary valve
-> pulmonary artery —>lungs.
Left Atrium - Forms the base of the heart. Recall that the base of the heart
is the “top” of the heart and the apex is the “bottom” of the heart. The
interior of this chamber is smooth and serves as the entry site for the four
pulmonary veins.
Left Ventricle - Forms the apex and the diaphragmatic surface of the heart.
Blood from this ventricle is pumped into the aorta.
The arteries that supply the heart muscle come off of the aorta. The two main
arteries are the left and right coronary arteries, which then divide into
several branches to reach the heart.

Blood Plow through the Heart:


• Deoxygenated blood comes from the body through the vena
cava to the right atrium. The blood flows through the tricus­
pid valve to the right ventricle. The blood is pumped through
the pulmonary valve to the pulmonary artery.
• It is oxygenated by the lungs and comes back to the heart
through the pulmonary veins into the left atrium. The blood
goes through the mitral valve into the left ventricle.
• Blood is then pumped through the aortic valve to the aorta
to be taken to the rest of the body (4).

Copyright 2014 by KMK Educational Services, LLC


CHAPTER 1. GENERAL PHYSIOLOGY 19

Figure 1.3: Blood Flow Through the Heart

Systole = Contraction of the ventricles. This is the first heart sound


(lub) during auscultation of the heart.
• During systole, the m itral and tricuspid valves close. The contraction
of the ventricles forces blood to open the pulmonary and aortic valves in
order to flow to the lungs and body, respectively. The blood then leaves
the heart and returning blood fills the atriums at the same time.

Systole is the Q-T on the EKG.

Diastole = Relaxation of the ventricles. This is the second heart sound


(dub) that occurs when the ventricles relax and the atriums contract.*
* After blood leaves the heart, the ventricles relax and the pressure in
the ventricles decreases, causing the aortic and pulmonary valves to
close.
Copyright 2014 by KMK Educational Services, LLC
20 1.3. CIRCULATORY SYSTEM
• Contraction of the atriums forces the tricuspid and m itral valves to
open. Here the ventricles relax, while the atriums contract.

Diastole is the T-R on the EKG.

The Mechanical Events:


1 Isovolumic Contraction - Contraction of the ventricles that occurs after
mitral valve closure but before opening of the aortic valve during systole.
The ventricles contract but do not force blood out of the heart, thus the
volume remains the same.
2 Ventricular (Systolic) Ejection - Pressure in the ventricles exceeds the
pressure in the aorta, forcing the aortic valve open so blood can flow
from the ventricles into the aorta. The aortic valve closes at the end of
this cycle.
3 Isovolumic Relaxation - The ventricles relax after the aortic valve closes
but before the mitral valve opens. This occurs during diastole. NO filling
occurs in this cycle.
4 Ventricular Filling - Immediately after the mitral valve opens, the ven­
tricles fill with blood. This occurs during diastole. There is NO change
in pressure within the ventricles (4).
The above processes cause our Heart Sounds:
• SI - Mitral and tricuspid valves close (Lub).
• S2 - Aortic and pulmonary valves close (Dub).
• S3 - Occurs with the rapid filling of ventricles.
• S4 - High atrial pressure/ stiff ventricles.

When you are checking blood pressure on a patient, you are hear­
ing systole over diastole. When you listen to the heart with your
stethoscope, the first sound (lub) is SI, and the second sound (dub)
is S2. S3 and S4 heart sounds are not normal for a healthy heart.
An S3 beat could indicate congestive heart failure, while an S4
sound points to a hypertrophic ventricle (10).

Copyright 2014 by KMK Educational Services, LLC


CHAPTER 1. GENERAL PHYSIOLOGY 21

Mechanical Events

Pressure

Isovolur lie
Relaxatii >n

Volume

Figure 1.4: Mechanical Events of the Heart

The conduction system of the heart is designed to provide coordinated contrac­


tion of the atria, a pause to allow filling of the ventricles, and then a coordinated
contraction of the ventricles. The electrical cycle occurs in the following steps:
1 Spontaneous depolarization of the SA node.
2 Rapid spread of depolarization through the atria by specialized conduct­
ing fibers and by contraction of atria muscle.
3 The impulse enters and moves slowly through the AV node.
4 The impulse enters the His Purkinje libers and spreads rapidly to the
ventricles.
5 Ventricular muscle cells depolarize, leading to coordinated contraction (2) (8).

The SA Node is considered to be the pacemaker of the heart as


it self-generates action potentials that then spread to other cells.

Copyright 2014 by KMI< Educational Services, LLC


22 1.3. CIRCULATORY SYSTEM

Arrhythmias
Occur when the timing of electrical depolarization in the heart is altered.from
abnormal stimuli of cardiac action potentials (drug induced or myocardial cell

Figure 1.5: EKG

Copyright 2014 by KMI< Educational Services, LLC


CHAPTER 1. GENERAL PHYSIOLOGY 23

death), or when there or when there are abnormal conduction pathways. Ex­
amples of arrhythmias include A-fib or heart block.

Think of the heart as an old house. You have walls (the structure
of the heart), plumbing (the vessels and valves) and the electrical
current (the conduction system). When the heart has a problem,
you need to determine which of the three aspects of “the house” is
not working correctly.

Fast Myocardial Action Potentials


The contraction of the heart muscle is due to a ventricular action potential.
1 Phase 0 (Upstroke) - Action potentials in the heart cause voltage-dependent
Na+ channels to open. Na+ enters the cells and causes rapid depolariza­
tion.
2 Phase 1 - K+ channels and slow Ca2+ channels begin to open, allowing
K+ to escape the cell and Ca2+ to flow inside the cell. At the same time,
there is a decrease in Na+ influx. These changes result in transient
repolarization.
3 Phase 2 - Plateau due to a balance between entering Ca2-f- currents
through L-gated voltage-dependent Ca2-b channels, and leaving currents
of I<+ through voltage gated K+ channels.
4 Phase 3 - Both Ca2Y and K+ channels close at the same time, but
Ca2+ influx diminishes slowly while RY quickly leaves the cell, caus­
ing membrane repolarization (may hyperpolarize) and action potential
termination.
5 Phase 4 - Return to resting membrane potential established by K+ cur­
rents.

Remember: Depolarization ~ inward currents of positive ions.


Hyper polarization or Repolarization = outward currents of
positive ions.

Copyright 2014 by KMK Educational Services, LLC


1.3. CIRCULATORY SYSTEM

Figure 1.6:' Fast Myocardial Action Potentials

Figure 1.7: Slow Myocardial Action Potentials

Copyright 2014 by KMK Educational Services, LLC


CHAPTER 1. GENERAL PHYSIOLOGY 25

Slow Myocardial Action Potentials


The SA and AV nodes are much slower and do not have a phase 1 or 2. This
is called the pacemaker action potential (4).
1 Phase 0 (Upstroke) - T type voltage gated Ca2+ channels open. The
cells of the SA and AV nodes DO NOT have Na+ channels. Ca+2 influx
results in depolarization and a slow conduction velocity from the AV
node to prolong transmission of the action potential from the atria to the
ventricles to allow filling.
2 Phase 1 and 2 - Absent.
3 Phase 3 - Repolarization due to calcium leaving the cell.
4 Phase 4 - Slow diastolic depolarization. The membrane potential spon­
taneously depolarizes as sodium conductance increases. This causes the
SA and AV nodes to fire spontaneously.

The speed of the SA node determines heart rate (4).

WiSISBSMB:
>v '-i - f

Flow and Resistance


Blood flow through a vessel is determined by two factors:
1 The resistance to blood flow through the vessel.
2 The pressure differences at the beginning and end of the vessel.
Pressure G rad ien t

Resistance

Figure 1.8: The difference in pressure (P) between the two ends of the vessel
determines the rate of flow (4).

Copyright 2014 by KMK Educational Services, LLC


26 1.8. CIRCULATORY SYSTEM
The resistance to blood flow through the arterioles is high due to the small
diameter; the larger the diameter, the less the resistance. Poiseuille’s Law tells
us that the rate of blood flow is directly proportional to the fourth power of the
radius of the vessel and inversely proportional to vessel length and viscosity (4).

Flow (Q) = r4/n L (1.3)

The change in pressure divided by resistance equals the blood flow.

Flow (Q) = AP /R (1.4)

The sum of all the vascular resistances within the systemic circulation is the
total peripheral resistance. Vasodilation can decrease the total peripheral
resistance (4).

Blood Flow Across Capillary Walls

Starling Forces
The fluid exchange across capillary walls is dependent on a balance of four
different pressures (13):
1 Oncotic pressure within the capillary.
2 Oncotic pressure outside of the capillary wall.
3 Hydrostatic pressure within the capillary.
4 Hydrostatic pressure outside the capillary wall.

Copyright 2014 by KMK Educational Services, LLC


CHAPTER 1. GENERAL PHYSIOLOGY 27

Starling Equation:
J v = K f((P c -P i)-(n c -m )) (1.5)
• Jv = Fluid movement (ml/min)
• Kf —Hydraulic Conductance (ml/min mm Hg)
• Pc —Capillary Hydrostatic Pressure (mm Hg)
• Pi = Interstitial Hydrostatic Pressure (mm Hg)
• n c = Capillary Oncotic Pressure (mm Hg)
• II i = Interstitial Oncotic Pressure (mm Hg)

Summary of the Starling Equation:


• Fluid movement across a capillary wall can be determined by finding the
oncotic pressure difference across the blood vessel wall and subtracting
that from the hydrostatic pressure across the blood vessel wall.
• The above calculation is multiplied by Kf, a constant that differs among
tissues, to determine fluid movement.
• A negative Jv indicates the fluid is returning to the vessel from the tissue.
A positive Jv indicates the fluid is moving from the capillaries to the
tissue (4).

Oncotic pressure is dependent on protein content. Hydrostatic


pressure is fluid pressure that is generated from the heart.

sSliSH
Neural Regulation of Blood Flow
Baroreceptors in the aortic arch and carotid sinuses affect heart rate and total
peripheral resistance via the sym pathetic nervous system. The baroreceptors
sense blood pressure and send a signal via the vagus (aortic baroreceptors) and
glossopharyngeal nerve (carotid baroreceptors) to the medulla oblongata to
increase or decrease the total peripheral resistance and cardiac output (4).
Copyright 2014 by KMK Educational Services, LLC
28 1.3. CIRCULATORY SYSTEM
Blood Volume and Pressure
Blood pressure can be elevated via the following mechanisms (4) (13):
1 ADH release from the posterior pituitary by osmoreceptors in the hy­
pothalamus.
2 Angiotensin II (most potent vasoconstrictor in the body).
3 Aldosterone (released by the adrenal cortex:).
The amount of blood pumped out of a ventricle with each heartbeat is called
the ejection fraction (EF). This is a percentage or fraction because it looks
at the proportion of blood left in the ventricle after a contraction.
• The blood within a ventricle before it contracts is known as the end-
diastolic volume.
• The blood that is left in a ventricle after it contracts is called the end-
systolic volume.
• The difference between the end-diastolic volume and the end-systolic vol­
ume is the stroke volume.
Let’s summarize this concept with an equation:
Ejection Fraction — End Diastolic Volume (EDV) —End Systolic Volume (ESV)
End Diastolic Volume (EDV)
( 1.6 )
This is equivalent to:
EF = Stroke Volume (SV)
ED V
(1.7)

A normal EF is 50% or greater. An EF less than this means that


the patient has heart damage from a heart attack or congestive
heart failure.

Example: A healthy man with a SV of 72 ml and EDV of 120 ml would have


an Ejection fraction of .60 or 60%.
Cardiac O utput (CO) is the amount of blood ejected from the heart per
unit of time. During exercise, cardiac output increases initially as a result of
an increase in stroke volume. Over time, cardiac output increases as a result
of the heart rate. If the heart rate is too high, diastolic filling is incomplete
Copyright 2014 by KMK Educational Services, LLC
CHAPTER 1. GENERAL PHYSIOLOGY 29

Figure 1.9: The force of contractions are proportional to the length of cardiac
muscle fibers (preload).

because there is not enough time to fill the ventricles. This results in a decrease
in CO.
Cardiac Output ~ Stroke Volume X Heart Rate (1.8)
Cardiac O utput Variables
Stroke volume is affected by the following:
1 Contractility.
2 Afterload = diastolic arterial pressure + peripheral resistance.
3 Preload —ventricular end-diastolic volume.

Stroke volume increases when preload increases, afterload de­


creases, and contractility increases.

Factors that increase Stroke Volume and Contractility


• Increased intracellular Ca2+.
* Decreased extracellular Na+.
Copyright 2014 by KMK Educational Services, LLC
30 1.3. CIRCULATORY SYSTEM
• Digitalis (increases intracellular Na+.
• Sympathetic stimulation.
Factors that decrease Stroke Volume and Contractility
• Heart Failure.
• Loss of heart cells Hue to infarction.
• Acidosis and hypoxia.

Sympathetic system
Fight or flight conditions cause constriction of the afferent arterioles leading
to -the kidney. This decreases the glomerular filtration rate (GFR) and the
production of urine, resulting in an increase in blood volume. The sympa­
thetic system also increases cardiac output and total peripheral resistance so
that blood flow can be directed toward the heart and muscles.
Angiotensin II
Low Cl- concentration signals the body is not absorbing enough water and
leads to the release of angiotensin II, the most potent vasoconstrictor in the
body. It can t blood pressure directly by constricting the afferent arterioles,
or indirectly by inducing the following mechanisms (13):
• t ADH levels (stored in the posterior pituitary).
• t Aldosterone levels (produced in the adrenal cortex).
• t Na+ reabsorption in the DOT of the kidneys.
• f Water reabsorption in the collecting duct of the kidneys.
Regulation by Antidiuretic Hormone (ADH)
ADH, also known as vasopressin, regulates the reabsorption of water. An
increase in blood osmolality due to dehydration or increased salt intake will
cause osmoreceptors in the hypothalamus to signal the posterior pituitary to
release ADH. ADH then works in the collecting duct to increase water retention,
resulting in an increase in blood volume (8).
Copyright 2014 by KMK Educational Services, LLC
CHAPTER 1. GENERAL PHYSIOLOGY 31

Figure 1.10: Angiotensin II

Regulation by Aldosterone
Aldosterone release occurs in the following manner:
• A decrease in blood volume, a decrease in blood flow to the kidneys, and
an increase in K+ concentration cause the juxtaglomerular cells to release
renin.
* Renin converts angiotensinogen to angiotensin I. The ACE enzyme then
converts angiotensin I to angiotensin II, which stimulates the adrenal
cortex to release aldosterone.
Aldosterone acts on the kidneys to increase salt and water retention, resulting
in an increase in blood volume (4) (8).
Copyright 2014 by KMK Educational Services, LLC
32 14- URINARY SYSTEM

Blood Volume Increases, Blood Osmolality Decreases

Figure 1.11: ADH

- SECTION 1 .4 ----------------------- -------------------------------------------------------------------------

U rinary System *•
We know introduce the functions and structure of the kidney (4, ch. 17).
Functions of the Kidney
• Eliminates drugs (along with the liver).
• Excretes waste products and toxic material from protein metabolism (e.g.
urea, uric acid).
• Maintains extracellular volume and ionic concentration.
Copyright 2014 by KMK Educational Services, LLC
CHAPTER 1. GENERAL PHYSIOLOGY 33

• Maintains blood plasma volume.


• Regulates blood pressure.
• Produces and secretes erythropoietin (for RBC formation).

The kidneys are bean-shaped and have a convex outer surface with a capsule,
and an internal concave hilum surface. The hilum is the location where the
renal artery enters and the renal vein and ureter exit the kidneys (6).
• Renal Pyramids: The medulla contains 8-15 renal pyramids that are
separated by renal columns. Each renal pyramid projects into a small
depression called a minor calyx.
• Major calyx: Several minor calyces empty into a major calyx.
• Renal Pelvis: Several major calyces join to form the renal pelvis.
• Ureter: The renal pelvis transports urine to the ureter and then to the
urinary bladder.
• Urinary Bladder: Storage sac for urine that is drained by a urethra into
the penis or the labia minora.
• Internal and External Sphincters: Contain smooth and voluntary muscle,
respectively, and surround the urethra. The sphincters regulate mictura-
tion (i.e. urination).

Blood Flow Through the Kidney


• Renal Artery -* Interlobar —>■Arcuate —>Interlobar —i Afferent Arterioles
Interlobar arteries pass in the renal columns between the pyramids until
they reach the cortex/medulla border, where they branch into the following:
1 Arcuate arteries: arch all the way around the kidney on the cor­
tex/medulla border.
2 Interlobar arteries: branch off the arcuate arteries toward the cortex.
3 Afferent arterioles: travel in the cortex and deliver blood into the
glomeruli.
Copyright 2014 by KMK Educational Services, LLC
34 Uh URINARY SYSTEM

The nephron is the functional unit of the kidney (1 million/kidney). The


nephron is composed of a glomerulus, a high pressure capillary bed that
filters fluid into Bowman’s capsule, to the PCT, then through the Loop of
Henle, DCT and Collecting duct (which collects fluid from many nephrons),
before finally emptying into the pelvis of the kidney. Nephrons are classified
into two types according to their location in the kidney and the length of their
loops:
1 Cortical nephrons (80%): Originate in the outer 2/3 of the renal cortex.
2 Juxtam edullary nephrons (20%); Originate in the inner 1/3 of the
cortex next to the medulla. They have long loops (composed of simple
squamous epithelium) that extend deep into the medulla and allow for
the production of concentrated urine.

The Pathway of the Nephron


Bowman’s capsule: Within Bowman’s capsule, the afferent arteriole enters
into the glomerulus and leaves as the efferent arteriole. The glomerulus is
a capillary network that allows blood to filter into the nephron. The blood
and components that are not filtered into the nephron leave in the efferent
arteriole. Bowman’s capsule contains a vascular pole (glomerulus side) and a
urinary pole which leads to the rest of the nephron. It also has a parietal layer
and an inner visceral layer. It is important to understand terminology related
to filtration in the glomerulus:
* Glomerular Filtrate: Fluid that filters through the glomerular mem­
brane.•
• Glomerular Filtration Rate (GFR): The volume of filtrate produced
by both kidneys in each minute.
— Filtration is favored by the hydrostatic pressure of blood.
- Filtration is opposed by the hydrostatic pressure of fluid in the
glomerular capsule and the osmotic pressure of proteins in plasma.
Protein concentration is higher in the plasma than in the glomerular
filtrate, promoting the return of filtered water into the blood vessels.
- Small molecules are filtered, but most salts, water, and metabolites
are reabsorbed.
— Large molecules, including serum, globulins, albumin, and fibrino­
gen, are generally NOT filtered.
Copyright 2014 by KMK Educational Services, LLC
CHAPTER 1. GENERAL PHYSIOLOGY 35

KIDNEY

Figure 1.12: Kidneys are are bean shaped organs located in the retroperitoneal
area (posterior to the abdominal fascia). The right kidney is usually lower than
the left kidney because it lies underneath the liver. The kidneys are fed by the
renal artery, a branch off of the abdominal aorta.

Factors that favor Increased GFR:


—Increase in renal blood flow.
- Increase in mean arterial pressure.
~ Efferent arteriolar constriction.

The GFR is commonly used in medicine to determine kidney


health. The lower the GFR., the worse the renal function.

Factors that favor Decreased GFR:


— Sympathetic stimulation and Afferent arteriolar constriction.

Copyright 2014 by KMK Educational Services, LLC


36 14- URINARY SYSTEM

The average amount of filtrate is about 180 L/day (45 gallons).


However, only 1 to 2L is excreted in the urine! Approximately 99%
of the filtrate must be reabsorbed back into the bloodstream (4).

PROXIMAL CONVOLUTED TUBULE (PCT)


Workhorse of the nephron! Here all of the glucose and amino acids
and most of the bicarbonate, sodium and water are absorbed.
• 67% (of the overall 99%) of Na is reabsorbed here.
• 67% (of the overall 87%) of K is reabsorbed here (4) (13).
Epithelial cells in the PCT have a lumen side and an interstitial side, The
epithelium is simple cuboidal and contains a brush border appearance due to
the prominent microvilli utilized for absorption. Cells in the PCT stain darker
than the DOT because this region is more convoluted. In the PCT there are
both active and countertransport mechanisms:
Active transport: Na-h/K-f- ATPase pumps create a gradient that allows
Na+ to leave the nephron. Cotransport mechanisms utilize the gra­
dient to move two molecules across the capillary wall. Examples include
Na+/glucose, Na+/Amino Acids, Na+/Cl-.
Countertransport: Na+/H+ pump is important for reabsorption of bicar­
bonate. Na+ is reabsorbed from the nephron, and water and CO2 follow.
H+ is then excreted into the loop in exchange for other ions, allowing H 2O
+ CO 2 to be transformed to H+ and bicarbonate.

Acetazolamide is an important drug that works on the PCT!


It inhibits carbonic anhydrase production, therefore inhibiting the
reabsorption of Na (with water following) and bicarbonate back
into the body, which can lead to metabolic acidosis.

LOOP OF HENLE (LOH) - Descending and Ascending loop


Histologically, both ascending and descending loops have simple squamous ep­
ithelium. The purpose of the LOH is to set up a concentration gradient in
which the extracellular environment (outside the loop) is hypertonic compared
to the tonicity of the loop. This allows more fluid to leave the loop, leading to
more concentrated urine.
Copyright 2014 by KMK Educational Services, LLC
CHAPTER 1. GENERAL PHYSIOLOGY 37

Descending Loop
The descending loop is highly impermeable to NaCl. Thus, these ions are NOT
reabsorbed into the interstitium (the area outside of the LOH that carries
fluid back into the body via the venous system). Only water leaves the
descending loop!

Water passively leaves the descending LOH via osmosis because


of the surrounding hypertonic interstitial fluid, and is eventually
reabsorbed back into the bloodstream. There are no diuretics that
act on the descending LOH.

Ascending Loop
Highly permeable to NaCl! It actively pumps Na-f- and Cl- from the loop into
the interstitium. Because water is unable to follow, the bottom of the loop has
a very high ionic concentration (4). Remember, water DOES NOT leave the
ascending loop!

The thick ascending loop absorbs 25% of Na+ and 20% of K+ (13).
Chloride also leaves the ascending loop for reabsorption.

* The Na/2C1-/K+ cotransport mechanism is on the luminal side and


moves Na+, C1-, and K+ passively into the cell for movement out of
the loop.
* The Na+/K+ pump on the interstitial side actively pumps Na+ into the
interstitium. Cl- and K+ then follow, although K+ may leak back into
the lumen of the ascending loop.
On luminal side, Na+ binds and moves passively with Cl- and K+ into cell for
movement out of the loop. The K+ can then leak out back into the lumen or
go into the interstitium.

Furosemide acts upon the thick Ascending Loop of Henle!


It inhibits the Na+/2C1-/K+ cotransporter and can cause a state
of hypokalemia.

Copyright 2014 by KMK Educational Services, LLC


38 1.1 URINARY SYSTEM

Figure 1,13: Look at the two transporters shown. The Na-f/K-b pump pushes
Na+ into the interstitium for reabsorption and pushes K+ into the cells of the
ascending LOH. This creates a gradient that favors movement of Na-f- (and
K+ and C1-) into the cell via the Na+/K-f/2Cl- transporter. Because both
transporters push K+ inside the cell, K+ travels through channels either into
the inter stitium for reabsorption (20% of K+ reabsorbed) or into the filtrate
within the lumen.

Distal Convoluted Tubule (DCT)


This part of the nephron is located in the cortex. It contains fewer microvilli,
has shorter cuboidal cells, and is paler on staining compared to the PCT.
The DCT is impermeable to water (i.e. water reabsorption back into the
bloodstream does NOT occur) unless acted on by aldosterone.
• Na+, Cl- and Ca2+ can all leave the filtrate here. Just like the Ascending
Loop of Henle, a Na-b/K+ pump is again used on the interstitial side to
create a gradient in which Na+ is less inside the cell.
• In the DCT, a Na+/Ca2+ pump also exists on the interstitial side and
pushes Na+ into the cell and Ca2+ into the interstitium. These pumps
create a gradient that favors movement of Nad- and Ca2-f from the loop
to be reabsorbed into the cell.•
• On the lumen side, a Na+/Cb pump is present and pushes both Na+ and
Cl- into the cell. Also present on the lumen side is a channel that allows
consistent passive absorption of Ca2+. Parathyroid Hormone (PTH)
enhances this channel and thus increases the absorption of Ca2+ into the
interstitium.
Copyright 2014 by KMK Educational Services, LLC
CHAPTER L GENERAL PHYSIOLOGY 39

Thiazide diuretics work on the Early Distal Convoluted


Tubule. They inhibit the Na+/Cl- pump, resulting in a decrease
in the reabsorption of Na+ and Cl- into the bloodstream. Thi­
azides also increase Ca2-f- reabsorption similar to PTH, likely due
to increased passive absorption of Ca2+ (although the exact MOA
is unknown).

Aldosterone and the K+ sparing diuretics work on the Late DCT.

Aldosterone
This is a mineralocorticoid that acts on the late DCT and collecting duct. It
is released in response to decreased blood volume.
• Aldosterone acts on Na+/K+ ATPase pumps, leading to pump phospho­
rylation and a subsequent change in the shape of the pumps (4) (13).
This shape change causes Na-t- to be pumped outside of the cell.
• Aldosterone ultimately increases the number of Na+/K+ ATPase pumps
and Na-t- and K+ channels, allowing for greater Na+ and H 2 O reabsorp­
tion and K+ and H-f excretion in the urine.
• These effects ultimately increase blood pressure but also result in hy­
pokalemia. K-t- sparing diuretics antagonize the actions of aldosterone,
resulting in opposite effects (most notably the “sparing of K+”).
Collecting Duct
Eight to ten distal convoluted tubules empty into each collecting duct. The
collecting duct has only small amounts of microvilli (similar to the DCT),
allowing for limited NaH- reabsorption. This is the last location for absorption
of Na+ and C1-.
• Similar to those mentioned previously, a Na+/K+ pump exists on the
interstitial side that creates a gradient for the absorption of Na-t- from
the lumen.•
• Unlike the other areas of the nephron, the lumen side of the collecting
duct has separate channels for Na+, K-t-, and water. The Na+ and water
channels are used for reabsorption from the lumen to the interstitial space,
Copyright 2014 by KMK Educational Services, LLC
40 Uh UMNAR Y SYSTEM
while the K+ channel favors secretion of K+ into the lumen. Aldosterone
will enhance this channel and can lead to hypokalemia.
• The lumen side also contains a pump that causes H+ to be secreted into
the lumen. Similar to the DCT, Na+ and H2 O are reabsorbed into the
bloodstream, and K+ and H+ are excreted into the urine.

7% of Na+ reabsorption and less than 1% of I<+ reabsorption oc­


curs in the DCT and the Collecting Duct (13).

Tubuloglomerular Feedback
Decreased blood flow leads to a decrease in the pressure in the glomerulus.
This is sensed by a group of cells called the macula densa that communicate
with the Juxtaglom erular cells (JG cells). JG cells are specialized muscle
cells that produce and release renin. Renin acts as an enzyme and convert
Angiotensinogen to Angiotensin I. Angiotensin I is released into the blood

Afferent Arteriole

Glomerulus
Collecting Duct

Efferent Arterioie
A scending Loop
o fH en le

Figure 1.14: The Nephron

Copyright 2014 by KMK Educational Services, LLC


CHAPTER1. GENERAL PHYSIOLOGY 41

and goes to the lung, where ACE (Angiotensin converting enzyme) converts
Angiotensin I to Angiotensin II. This will cause a constriction of the afferent
arterioles to change the glomerular filtration rate (GFR), allowing a steady
filtration of blood through the kidney despite blood pressure changes (4).

Peritubular capillary network


This is an extension of the efferent arterioles that allows for fluid to be
reabsorbed from the interstitial space for re-entry into the venous system. A
portion of this network is called the vasa recta, a section of capillaries that
descends down into the medulla of the kidney around the lower portion of the
Loop of Henle. The vasa recta extends back upwards to the cortex before
draining back into the veins.

Counter-current Exchange
To concentrate urine, water must leave the nephron. The vasa recta is a highly
hypertonic collection of arterioles that causes water to leave the descending
LOH and re-enter the bloodstream. NaCl readily goes into the vasa recta,
causing the osmolar concentration to increase to a maximum concentration of
1200 milliosmoles/liter at the tip of the vasa recta. Water follows the absorbed
ions in the vasa recta via osmosis. In the ascending limb, more ions readily
diffuse into the interstitial space. In the end, the osmolar concentration within
the vasa recta (400) is only slightly higher than when it began (320). However,
the blood pressure is increased due to increased water reabsorption (4).

Water and ions that are reabsorbed from the loop must enter the
interstitium (outside the nephron) and flow into the peritubular
capillary network (venous) system.

Acid-Base Balance: Kidneys regulate blood pH


When the body is in a state of acidosis (4- pH), there are high concentrations of
H+ in the bloodstream. To compensate for this, hydrogen ions are excreted in
urine and bicarbonate is reabsorbed back into the venous system. The opposite
occurs when pH is too high (4). The normal pH of urine varies from 4.0-7.0,
depending on the needs of the body. Kidney regulation of pH is a long term
mechanism that acts over a period of hours to days.

Copyright 2014 by KMK Educational Services, LLC


42 1.5. ENDOCRINE SYSTEM

Remember the two other major mechanisms the body employs to


regulate pH: alveolar ventilation (acts in a matter of minutes)
and the buffer system in the blood (acts immediately).

Erythropoiesis
Low PO 2 levels detected in the kidneys cause erythropoietin to be released.
This hormone is the primary regulator of erythropoiesis (production of RBCs).
One of the main causes of decreased O2 content is a low red blood cell count (4).

1- SECTIO N 1.5

Endocrine System

The endocrine system is a group of cells that secrete substances (e.g. hormones)
into the systemic circulation that have a specific effect on tissues in other parts
of the body (1) (8). In general, these hormones integrate and coordinate the
following processes: digestion, utilization and storage of nutrients, growth and
development, ion and water metabolism, and reproduction.

H ypothalam ic Hprniones: 4 4 ^ i t c
There are 2 types of hypothalamic hormones:

1 Releasing hormones that stimulate the synthesis and release of the


anterior pituitary hormones.

2 Inhibitory hormones that inhibit the synthesis and release of the an­
terior pituitary hormones.

Copyright 2014 by KMK Educational Services, LLC


CHAPTER 1. GENERAL PHYSIOLOGY 43

HORM ONE A C T IV IT Y
Thyroid Releasing Hormone (TRH) Stim ulates synthesis and release
of thyroid stim ulating hormone (TSH)
and prolactin.
Corticotropic Releasing Hormone (CRH) Stim ulates th e synthesis
and release of Adrenocorticotropic
hormone (ACTH).
Gonadotropin Releasing Hormone (GnRH) Stimulates th e synthesis and
release of gonadotropins.
(FS11 and 1,11)
■Growth Hormone Releasing Hormone (GHRH) Stim ulates th e synthesis
and release of GH.
Som atostatin Inhibits th e release of GH.
Prolactin Inhibiting Factor Inhibits th e release
of prolactin.

The pituitary gland is located on the inferior aspect of the brain in the
region of the diencephalon. It is attached to the hypothalamus by a stalk-like
structure called the infundibulum. It has an anterior (adenohypophysis)
and a posterior (neurohypophysis) lobe. The anterior pituitary lies outside
the blood brain barrier.
Anterior Pituitary Hormones
1 ACTH (Adrenocorticotropic hormone)
• Target Tissue: Adrenal Cortex.
• Principal Action: Stimulates secretion of glucocorticoids from the
adrenal gland.
• Regulation of secretion: Stimulated by CRH and inhibited by glu­
cocorticoids.
2 TSH (Thyroid-stimulating hormone)
• Target Tissue: Thyroid Gland.
• Principal Action: Stimulates the secretion of thyroid hormones.
• Regulation of secretion: Stimulated by release of TRH, inhibited by
T3 and T4.
TSH
• The hypothalamus releases TRH (Thyroid Releasing Hormone), which
stimulates the anterior pituitary to release TSH (Thyroid Stimulat­
ing Hormone). TSH stimulates the release of T3 and T4 from the
Copyright 2014 by KMK Educational Services, LLC
44 1.5. ENDOGRINE SYSTEM

Synthesis of Protein and Peptide Hormones

Figure 1.15: Synthesis of Protein and Peptide Hormones

Cholesterol 17-0H- Pregnenolone Dehydroepiandrosterone

X
Pregnenolone
*
17-OH - Progesterone c Andrestenedlone
\
DHEA - sulfate

X x X
Progesterone 11-Deoxycortisol Testosterone

X
11 - Deoxycorticosterone
* I
Cortisol
Estradiol
X
Corticosterone

X
18-OH-Corticosterone
Synthesis of a Steroid Hormone
X
Aldosterone

Figure 1.16: Synthesis of Steroid Hormones

Copyright 2014 by KMK Educational Services, LLC


CHAPTER 1. GENERAL PHYSIOLOGY 45

GABA, Serotonin, NE

ACTH
Negative
s
Stress, Endorphins, Ac FI

Feedback

Aldosterone

Figure 1.17: Anterior Pituitary Hormones

Figure 1.18: Thyroid Hormones

Copyright 2014 by KMK Educational Services, LLC


46 1.5. ENDOCRINE SYSTEM
thyroid gland. When circulating levels of T3 and T4 are high, they
provide negative feedback to decrease the synthesis and release of
additional TRH and TSH (8).
• A healthy thyroid gland secretes sufficient amounts of the thyroid
hormones T3 and T4 to maintain normal growth and development,
temperature regulation, and daily energy requirements. Thyroid
hormones are largely made up of iodine.
3 Growth Hormone (GH)
• Target Tissue: Cells throughout the body
• Principal Action: Stimulates movement of amino acids into cells for
protein production. Ultimately promotes tissue and organ growth.
• Regulation of Secretion: Stimulated by GHRH (from the hypotha­
lamus) and inhibited by somatostatin; GH is sent to the liver to
promote the release of IGH-I (insulin like growth hormone). This
stimulates the hypothalamus to release more somatostatin, which
decreases the release of GH and stimulates more GHRH from the
hypothalamus.
GH is a single chain peptide that has a diabetogenic effect: it causes
a decrease in the amount of glucose that can enter cells.
GH deficiency can result from the following:
a Hypothalamic dysfunction.
b Pituitary tumor.
c Abnormal synthesis.
d Receptor disorders.

HORMONE TARGET RESU LT


G ROW TH HORM ONE Adipose Lowers glucose uptake.
Increases lipolysis (will lower adipose).
GROW TH HORMONE Metabolism Increases plasm a glucose
and FFA. Lowers plasm a urea
and amino acids.
GROW TH HORM ONE Muscle Lowers glucose uptake.
Increases amino acids
and protein synthesis.
IGH-I Kidney, pancreas, skin, Increases organ size
CT, bone, heart, lungs and function.
IG H-I Chondrocytes Increases cell size,
number, and collagen.
IGH-I Liver Increases gluconeogenesis.

Copyright 2014 by KMK Educational Services, LLC


CHAPTER 1. GENERAL PHYSIOLOGY 47

GH excess can cause the following:


a Gigantism (during pre-pubertal development),
b Acromegaly (after puberty).
GH deficiency can cause the following:
a Short stature,
b Delayed ossification of bones,
c Mild obesity.
4 FSH (Follicle Stimulating Hormone)
• Target Tissue: Ovaries and Testes.
• Principal Action: Stimulates development of follicles in the ovaries
and causes spermatogenesis in the testes.
• Regulation of secretion: Stimulated by Gonadotropin releasing hor­
mone (GnRH) and inhibited by sex steroids.
5 LH (luteinizing hormone)
• Target Tissue: Gonads.

Figure 1.19: FSH and LH Pathway in Females

Copyright 2014 by KMK Educational Services, LLC


48 1.5. ENDOCRINE SYSTEM

Figure 1.20: FSH and LH Pathway in Males

• Principal Action: Stimulates secretion of sex hormones, as well as


ovulation and corpus luteum formation in females.
• Regulation of Secretion: Stimulated by GnRH.
ens and
Estrogens are female sex hormones. Progestins are hormones that promote
gestation.

Estrogens stimulate growth of the endometrium. Progesterone


inhibits endometrial growth and maintains the vascular supply.
Withdrawal of progesterone leads to menstruation.

Physiology of the Hormones


• Estrogen levels are very low until puberty.
• At puberty, estrogen levels rise and sexual maturation and development
begin.
Copyright 2014 by KMK Educational Services, LLC
CHAPTER 1. GENERAL PHYSIOLOGY 49

• Cyclic changes of estradiol and progesterone occur during the menstrual


cycle.
• Secretion of gonadal hormones is under the control of FSH and LH from
the anterior pituitary.
• Secretion of the gonadotropins is under the control of gonadotropin re­
leasing hormone from the hypothalamus (1 1 ).
On average, the female menstrual cycle is 28 days. The first day of menstrual
bleeding is day one of the cycle. The menstrual cycle is divided into two phases:
1 The first cycle is the follicular phase, which starts on day one of the
cycle.
2 The second phase is the luteal phase that lasts from the first day of
ovulation to the next bleeding cycle.
• During the follicular phase, secretion of FSH is relatively high. FSH
stimulates follicular development, maturation of an ovum and secre­
tion of estrogen.
• About mid-cycle, rising levels of estrogen and FSH stimulate a surge
of LH. This leads to a rupture of follicles and release of ova (ovula­
tion).
• LH then causes the differentiation of follicles into the corpus luteum,
which secretes estrogen and progesterone. Estrogen and proges­
terone provide feedback inhibition to the pituitary gland to decrease
the release of gonadotropin.
• Estrogen and progesterone levels fall as corpus luteum function fails.
A decrease in these hormones then leads to menstruation (unless
fertilization of the ovum occurs) (1 1 ).
Progesterone is released during the luteal phase and inhibits further uterine
endometrial growth but promotes maintenance of the endometrium. If implan­
tation of a fertilized ovum does not occur, progesterone levels decrease, result­
ing in a loss of support of the endometrium and the start of menstruation. If
implantation occurs, chorionic gonadotropin (secreted by the embryo) main­
tains the corpus luteum, progesterone secretion continues, and menstruation
does not occur (1 1 ).
Prolactin: Targets the mammary glands and other sex accessory organs.
• Principal Action: Increases breast size during puberty and preg­
nancy, and stimulates milk production during pregnancy.
• Regulation of secretion: Inhibited by PIF and dopamine. Prolactin,
together with estrogen, cortisol and GH, stimulates the proliferation
and branching of the breast ducts. These hormones also induce
alveolar development and milk synthesis.
Copyright 2014 by I<MK Educational Services, LLC
50 1.5. ENDOCRINE SYSTEM

Figure 1,21: Prolactin Pathway

Posterior Ilorrriones A t T A y T T ?iy-1


Arginine Vasopressin/Antidiuretic hormone (
ADH is produced by neurons in the hypothalamus and then stored in Herring
bodies in the posterior pituitary. It is secreted due to nerve impulses from
(
the hypothalamic osmoreceptors, carotid and aortic baroreceptors, and stretch
receptors in the walls of the left atrium and pulmonary veins (4). The main (
function of ADH is blood volume regulation.
(

Diabetes Insipidus is due to an ADH deficiency. Symptoms include


(
t urination, t thirst, headaches, and possible coma. (
(
Oxytocin (
Oxytocin acts on the myoepithelial cells to stimulate milk ejection. The suck­
ling from a baby and myometrial stretching of the uterus causes the release of
oxytocin. Oxytocin secretion is inhibited by stress, fear and opioids (4). Oxy­
(
tocin is also released in childbirth and stimulates uterine contractions during
labor. c
Copyright 2014 by KMK Educational Services, LLC (
(
(
(
CHAPTER 1. GENERAL PHYSIOLOGY 51

Endocrinology of Calcium and Phosphate Homeostasis


Calcium Turnover
Most of our intracellular calcium is stored in the ER, mitochondria, and the
plasma membrane. Extracellular calcium is stored in the skeleton. Plasma
Ca2 + is always in balance with bone calcium: a fall in plasma calcium will
induce a fall in bone density.
Phosphate Turnover
Phosphate is in the form of high energy compounds such as ATP. Renal excre­
tion and reabsorption constitute the major controlling mechanisms that main­
tain plasma phosphate levels. Phosphate is stored in soft tissues and in the
skeleton. Plasma phosphate is directly related to bone turnover.
Bone Formation and Reabsorption
Bone formation is..,
• Stimulated by GH, Insulin, Estrogen, Androgen, Vitamin D, and Calci­
tonin.
• Inhibited by PTH and Cortisol.
Bone reabsorption is...
• Stimulated by PTH, Vitamin D, and Cortisol.
• Inhibited by Estrogen, Androgen, and Calcitonin.

Bone Formation and Reabsorption


• Active osteoblasts synthesize collagen that forms the linear matrix
(osteoid). Calcium and phosphate are deposited in the osteoid and com­
bine with hydrozide and bicarbonate to form hydroxyapatite crystals for
bone mineralization.
• Gradually, the mineralized bone surrounds the osteoblasts and other os-
teocytes by flowing through the tiny fluid filled channels in the bone called
canaliculi. Bone calcium and phosphate are transported from the inside
of the bone to the outside and vice versa via this osteocyte activity (4).•
• Bone reabsorption destroys the entire organic mass and removes the cal­
cium. The reabsorption is done by giant osteoclasts that attach to the
bone surface to be remodeled. The destroyed bone serves as the site for
osteoblastic activity and bone regeneration (4).
Copyright 2014 by KMK Educational Services, LLC
52 1.5. ENDOCRINE SYSTEM

Bone remodeling is a continuous process in the adult. Inadequate


plasma calcium and phosphate levels will result in a loss of bone
mass. Even though there are changing rates of bone formation and
reabsorption, blood levels of calcium and phosphate stay consistent
due to urinary excretion and intestinal absorption of the ions (4).

Functions of Vitamin D
Vitamin D stimulates intestinal Ca2+ and phosphate absorption and bone reab­
sorption. It promotes the synthesis of calcium binding protein in the intestine,
bone, kidney, and parathyroid gland. Vitamin D stimulates osteoid forma­
tion, increases bone density, and promotes calcium uptake by the sarcoplasmic
reticulum.
The Parathyroid Glands
Composed of 2 cell types 1 ) Chief and 2 ) Oxyphil
Parathyroid Hormone (PTH)
Synthesized in Chief cells. Calcium controls PTH synthesis and release. In­
creased levels of serum calcium result in a decrease in the release of PTH. Low
levels of serum calcium stimulate the parathyroid glands to synthesize and re­
lease PTH. PTH stimulates osteoclast activity via cell surface receptors
and cyclic AMP (5).
Calcitonin
Synthesized by parafollicular cells of the thyroid gland. Calcitonin decreases
plasma calcium (the opposite effect of PTH). It also inhibits osteoclast ac­
tivity and decreases plasma calcium levels by enhancing calcium deposition in
the bone (5).
Pancreatic Hormones
There are 3 cells in the Islets of Langerhans within the pancreas:
1 Alpha Cells

2 Beta Cells

3 Delta cells

Copyright 2014 by KMK Educational Services, LLG


CHAPTER 1. GENERAL PHYSIOLOGY 53

Alpha cells secrete glucagon in response to a fall in blood glucose. Glucagon


stimulates the liver to convert glycogen to glucose (glyc.ogenolysis) >resulting
in an increase in blood glucose levels. Glucagon also stimulates the hydrolysis
of stored fat (lipolysis) and the subsequent release of free fatty acids into the
blood (4).
Beta cells secrete insulin in response to a rise in blood glucose. Insulin
promotes the entry of glucose into tissue cells for conversion into energy storage
molecules, including glycogen and fat. Insulin also aids in the entry of amino
acids into cells for the production of cellular protein.

The actions of insulin and glucagon are antagonistic. After a meal,


insulin secretion is increased and glucagon secretion is decreased.
Fasting causes a rise in glucagon and a fall in insulin secretion (4).

Delta Cells secrete somatostatin , an inhibitory hormone that is also


produced by the cells of the hypothalamus, stomach, and intestine. It inhibits
the release of growth hormone (GH), thyroid-stimulating hormone (TSH), and
gastrointestinal hormones.
Insulin Actions - (Moves glucose into cells)
• Increased glucose taken up by fat and muscle.
• Increased glucose .storage as glycogen (glycogenesis).
• Decreased breakdown of glycogen (glycogenolysis) and decreased forma­
tion of glucose (gluconeogenesis).

Copyright 2014 by KMK Educational Services, LLC


54 1.5. ENDOCRINE SYSTEM
• Increased lipid synthesis (lipogenesis).
• Increased amino acid uptake and protein synthesis.
• Decreased protein degradation (5).
• Decreased ketone body formation (ketogenesis). This explains why pa­
tients with diabetes who do not take their insulin develop diabetic ke­
toacidosis.
Disorders of Insulin Action
When the pancreas cannot produce insulin, and/or insulin receptors do not
respond to insulin, glucose concentration increases, resulting in diabetes. Dia­
betes is classified into two categories: Type 1 and 2 .
Type I (Insulin Dependent Diabetes)
Results from the destruction of beta cells of the Islets of Langerhans. It may
be chemically induced or genetic. Findings include:
• Hyperglycemia and glucosuria due to the inability of glucose utilization
and increased glycogenolysis and gluconeogenesis.
• Hyperlipidemia due to decreased storage of free fatty acids (FFAs) and
triglycerides, and increased lipolysis and FFAs to the mitochondria.•
• Ketoacidosis can also occur due to increased FFA oxidation and produc­
tion of acetoacetic acid. This causes a fall in blood pH and subsequent
acidemia. You will be able to smell acetone on their breath.
• Treatment includes insulin injections.
Type II (Non-Insulin Dependent Diabetes)
Results from defects in glucose transporters, malfunctions of insulin receptors,
and peripheral insulin resistance due to fat (4) (7). Treatment includes diet,
exercise, oral agents, and insulin.
Somatostatin
This hormone is also released from the pancreas and is sent to the GI tract.
Its functions include the following:
• Inhibition of intestinal glucose and triglyceride absorption.
• Decreasing the rate of digestion and absorption of nutrients from the GI
tract.
Copyright 2014 by KMK Educational Services, LLC
CHAPTER 1. GENERAL PHYSIOLOGY 55

• Inhibition of insulin and glucagon secretions.


• Release from the pancreas is stimulated by amino acids, glucose, FFAs,
and GI hormones.

The adrenal glands are located above each kidney. Each gland is divided into
2 zones: the outer cortex and the inner medulla. The adrenal cortex produces
2 types of cortical steroids:

1 Glucocorticoids: Cortisol and Corticosterone.

2 Mineralocorticoids: Aldosterone and Deoxycorticosterone.


Please see the beginning of this section for more on the synthesis of these
hormones.

mm
Glucocorticoids
Cortisol
• Mobilizes movement of amino acids from muscles for gluconeogenesis.
• Increases blood glucose.
• Promotes glycogenesis so that other hyperglycemic hormones can utilize
stored glycogen.
• Promotes peripheral insulin resistance.
• Decreases the synthesis and increases catabolism of proteins in all tissues
except the liver.
• Increases protein synthesis in the liver.
• Enhances lipolysis in adipose tissue and decreases conversion of glucose
to fatty acids.
• Increases oxidation of fatty acids to meet energy requirements.
• Increases ketoacid formation.
• Maintains contractility and work output by muscles.
• Slows bone formation by blocking synthesis and function of Vitamin D.
• Chronic use leads to immunosuppression.
• Continuous effects lead to loss of muscle, bone, skin and connective tissue
mass.
Copyright 2014 by KMK Educational Services, LLC
56 1.6. MUSCLES
2. Mineralocorticoids
Aldosterone: Two main functions:
1 Renal Na+ reabsorption into the bloodstream.
2 Renal K+ excretion into the urine.
Its main sites of action include the DCT and collecting duct. It binds to a type
of cortisol receptor and stimulates Na+ absorption in the distal tubules and
collecting ducts, resulting in water reabsorption and an increase in extracellular
fluid volume.
The Adrenal Medulla
Catecholamine release results from “fight or flight” conditions, stress, or hypo­
glycemia. This specialized sympathetic ganglion releases 2 major catecholamines:
1 Norepinephrine (NE)
2 Epinephrine (Epi)
Actions of catecholamines on the body:
• Increased liver and muscle glycogenolysis.
• Increased lactate production in the muscle for hepatic gluconeogenesis.
• Inhibition of insulin release and glucose uptake by the muscle.
• Stimulation of glucagon release and gluconeogenesis.
• Enhanced lipolysis and fatty acid oxidation.
• Increased cardiac contractility and heart rate.
• Increased blood pressure.
• Adrenergic effects such as dilated pupils, sweating, etc.
- SECTIO N 1.6 --------------------------------------------- ---------------------- --------------------------------

Muscles
Muscles are the largest tissue type in the human body and serve as effector
organs for the nervous system. Skeletal muscles carry out the orders of the
somatic nervous system, and smooth and cardiac muscle are controlled by the
autonomic nervous system. All muscles are excitable tissues that generate
action potentials which induce contractions and generate force.
Copyright 2014 by KMK Educational Services, LLC
CHAPTER 1. GENERAL PHYSIOLOGY 57

Striated, voluntary muscles comprised of individual myofibers. Each muscle


fiber is innervated by a nerve terminal, which is called a neuromuscular
junction.
Excitation-Contraction Coupling
Acetylcholine (Ach) produces an action potential that is spread across the
sarcolemma until it reaches the transverse tubules, which are continuous
with the sarcolemma and allow action potentials to spread deep into the muscle
fibers. Calcium is normally present in the sarcolemma within the sarcoplasmic
reticulum. When Ach is present, the calcium is released into the sarcoplasm and
binds to the troponin complex (changing the troponin structure), causing the
attached tropomyosin to shift position on the actin filament; the myosin binding
site is now exposed on the actin strand, allowing for cross bridge formation.
Sliding Filament Theory of Contraction
When a muscle contracts, it decreases in length as a result of the shortening
of its individual fibers (myofibrils). This is produced by decreasing the space
between sarcomeres - the distance from Z line to Z line.
• The sarcomeres shorten in length, but the A bands remain constant. The
I bands (the distance between A bands of successive sarcomeres) decrease
in length.
• The thin filaments that compose the I band do not shorten. The H band
shortens during contraction.
• The thick and thin filaments remain the same length, but they slide
across each other in an effort to shorten the sarcomeres (4). The sliding
of filaments is caused by the action of numerous cross bridges.

Remember, Z, I, and H bands shorten during contraction.

Reflex Arc (Muscle Spindles, Golgi Tendon Organs)


A reflex arc is the neuronal circuit that is responsible for a particular reflex.
A reflex arc includes sensory receptors that cause the reflex by exciting a set
of interneurons and motor neurons. Another set of interneurons and motor
neurons may also be inhibited so that a pattern of muscle contractions occur
around one or more joints.
Copyright 2014 by KMK Educational Services, LLC
58 1.6. MUSCLES

Figure 1.22: Excitation-Contraction Coupling in Skeletal Muscle

Muscle Spindles: Determines the muscle length and rate of change on the
muscle length. Muscle spindles are spindle shaped organs of modified muscle
fibers. They are found in most skeletal muscles but are concentrated in mus­
cles involving fine motor control (eye and finger movement). The spindles lie
parallel to the regular muscle fibers with the distal ends attached to the con­
nective tissue of the muscle. The fibers of the muscle spindles are too weak to
contribute to contraction, but have both sensory and motor axons (9).

Golgi Tendon organs: Signal the force on a muscle. They have stretch
receptors in the organ that are sensed by nerve fibers. The terminals of the
Golgi tendons are wrapped around bundles of collagen fibers in the tendon of
the muscle. The stimulus then is the force that develops in the tendon. The
golgi tendon organ can be activated by muscle stretch or contraction (9).
Copyright 2014 by KMI< Educational Services, LLC
CHAPTER 1. GENERAL PHYSIOLOGY 59

1 . 1

1
Relaxation

L_____ - 1

<■ +• A = Thick Filament

J= Thin Filament

Contraction

Types of Contractions
Isotonic: These contractions result in muscle shortening. Their force of con­
traction remains fairly constant throughout the shortening process.
Isometric: These are contractions that have opposing forces that are too
great, so the number of muscle fibers activated is too few to shorten the muscle.
These can be voluntarily produced (4) (9).
Isometric and Isotonic: When a muscle receives a stimulus to fire, the
mechanical response of the muscle is called a twitch. The muscle contractions
that result from this twitch are all generated the same way, but differ in whether
the muscle shortens as it contracts.
Muscle Fiber Types
Muscles contract at different rates depending on the type of fibers present:
• Slow-twitch fibers (Type I) contract rather slowly and with low intensity
and have a greater resistance to fatigue.
—Type I fibers contain slow myosin, which slowly hydrolyzes ATP
(serves as the rate limiting step in myosin cross-bridge formation)
obtained from oxidative phosphorylation.
- Type I fibers also have a high oxidative capacity due to the following:
Copyright 2014 by KMI< Educational Services, LLC
60 1.6. MUSCLES
1 Multiple capillaries.
2 High myoglobin content.
3 Numerous mitochondria.
—Type I fibers are red and small and are well suited for activities such
as maintaining body posture or running a marathon.
• Fast-twitch fibers (Type II) contract relatively quickly with a high inten­
sity, but they also fatigue quickly.
— Type II fibers contain fast myosin that quickly hydrolyzes ATP de­
rived from anaerobic glycolysis.
—Type II fibers have a low oxidative capacity due to the following:
1 Few capillaries.
2 Low myoglobin content.
3 Few mitochondria.
—Type II fibers are large and white and provide rapid, powerful action
such as jumping or sprinting.
Muscle Fibers
Muscles are composed of intrafusal and extrafusal fibers:
• Extrafusal fibers make up the majority of muscle tissue and are re­
sponsible for contraction. They are attached to tendons on both ends
and are innervated by alpha motor neurons.
• Intrafusal fibers (also discussed above in reflex arcs) are muscle spin­
dles designed to measure muscle length and provide feedback. They are
encapsulated within sheaths and rim parallel within the belly of the mus­
cle between the extrafusal fibers. Intrafusal fibers attach to tendons on
both ends and are innervated by gamma motor neurons,
—Nuclear bag fibers transmit information about muscle length and
tension. They are innervated by fast group I afferents.
— Nuclear chain fibers transmit information about muscle length. They
are innervated by slow group II afferents.

Muscles involved in fine movements require greater feedback and


control and therefore contain a greater number of muscle spindles
within the muscle.

Copyright 2014 by KMK Educational Services, LLC


CHAPTER 1. GENERAL PHYSIOLOGY 61

Muscle control is also determined by the number of muscle fibers recruited to


perform an action. When a muscle fiber is signaled to fire, it fires completely.
This is termed the all or nothing principle. Muscle force is therefore controlled
by fractioning. The more muscle fibers that are signaled to fire, the stronger
the force of the resulting contraction. While a single fiber is only innervated by
one neuron, a single neuron may innervate multiple fibers. In muscles requiring
greater fine motor control, each nerve innervates fewer fibers.

Muscle Metabolism
Muscle utilizes ATP as fuel and must have a readily available supply.
• At rest, a muscle cell’s demand for ATP is low. Upon contraction, a
muscle cell’s ATP demand soars. Each muscle cell contains a small supply
of ATP that is quickly used up during high demand.
• Just as in other cells, muscles can generate more ATP through substrate
level phosphorylation and oxidative phosphorylation. These reactions can
take a few seconds to come up to speed.
• To ensure a steady supply of ATP, muscles utilize stores of high-energy
creatine phosphate to allow regular metabolism time to gear up. Creatine
phosphate combines with ADP when catalyzed by creatine kinase to form
creatine and ATP.
The rate of ATP usage and the mechanism of replenishment depends on the
intensity of exercise and the blood’s ability to supply oxygen.
• During light to moderate intensity activities when 0 2 supply can
keep up with demand, ATP is supplied by oxidative phosphorylation.
- Muscle cells rely on their own glycogen stores until they are de­
pleted; at this point, muscle cells increase their expression of insulin-
dependent glucose transporters (GLUT-4) in order to obtain glucose
supplied by the blood stream.
- After approximately 30 minutes, muscle cells begin to use fatty acids
in the blood stream as the dominant energy source rather than glu­
cose.
• During heavy exercise, muscle cells use anaerobic glycolysis to produce
ATP and pyruvate.
- When pyruvate is produced faster than it is oxidized in the Krebs
cycle, it is converted to lactic acid.
Copyright 2014 by KMK Educational Services, LLC
62 1.6. MUSCLES
S m ooth M ils cl cs
Smooth muscles are non-striated, involuntarily controlled muscles that
are located on hollow organs. The autonomic nervous system (sympathetic
and parasympathetic) is the primary stimulus for smooth muscle contraction
or relaxation, although hormones also play a role.
• Smooth muscles contain actin and myosin interactions that cause short­
ening of the cells, which allows the muscle fibers to curve. This provides
the motility to alter the dimensions of an organ.
• Unlike skeletal muscles, the cytoskeletal structure of smooth muscles is
not organized in sarcomeres and there is no troponin. This unor­
ganized arrangement gives a homogeneous appearance under the micro­
scope.
• Smooth muscles must be able to sustain contractions to maintain organ
structure despite external forces,
Smooth Muscle Excitation-Coupling
Smooth muscle contraction does not involve the same process as skeletal muscle
because smooth muscle has a relatively sparse nerve supply. Instead, neuro­
transmitters are released at a greater distance from the muscle and must diffuse
through tissue to reach the muscle cells.
• Numerous gap junctions between these cells allow fast integration and
contraction of all the cells at once. Gap junctions electrically couple the
cells and permit ions and chemicals to flow from one cell to another (13).•
• The action potentials in smooth muscle are generated by the ligand­
gated Ca2 + channels. They allow calcium to enter the muscle fibers
and bind to an enzyme called calmodulin. This activates myosin light
chain kinase (MLCK) which then interacts with the G-actin active site
allowing cross-bridges to form. The cross bridges can then cause the
muscle filaments to move across one another.

Ca2+/calmodulin complex activates myosin light chain kinase.


This is changed by phosphatase and allows myosin and actin to
interact, allowing for contraction.

If the Myosin light chain kinase cannot by phosphorylized, the MLCK can still
bind to the actin via latch-bridge formation. No energy is required in this
union and the muscle can still hold tension (13).
Copyright 2014 by KMK Educational Services, LLC
CHAPTER 1. GENERAL PHYSIOLOGY 63

Cardiac muscle is similar to skeletal muscle because it is striated, has sar­


comeres, and contracts by the troponin-tropomyosin system. It is similar to
smooth muscle because it is extensively connected by gap junctions so that a
single action potential activates the whole cell network.
• Cardiac muscle has a relatively long action potential, allowing the heart
to relax completely and fill with blood before contracting again.
• Cardiac muscle is unique in that it does not depend on neural input to
produce a contraction. The sinoatrial (SA) and atrioventricular (AV)
nodes originate action potentials and trigger the heartbeat. For this
reason, the heart’s contractions are said to be myogenic rather than
neurogenic, like other muscles.
• The heart is still regulated by the ANS, as it regulates the frequency and
the force of heart contraction.
“ SECTION 1.7 ------------------------------------------------------------------ --

G astrointestinal Physiology •

The GI system is designed to break down and absorb all the nutrients we
consume, regardless of whether we need them or not.
• The digestive process starts in the mouth where mastication and salivary
components begin digestion; it continues down the esophagus and into the
stomach, small intestine, and ultimately the colon before the unabsorbed
waste is secreted out the rectum and anus.
• The absorption of small nutrient molecules into the blood stream
is the ultimate goal of this system. Along the way, fluids and en­
zymes are secreted into the GI system to facilitate the breakdown of the
large nutrient molecules found in food.
• Smooth muscle contractions throughout the system continually mix and
propel the food onward.

The GI system is regulated by neural and endocrine pathways. While the au­
tonomic nervous system regulates the GI system indirectly, the enteric nervous
system has its own receptors in the GI tract that act on effector cells in GI
organs. There are three types of receptor neurons in the GI tract:
Copyright 2014 by KMK Educational Services, LLC
64 1.7. GASTROINTESTINAL PHYSIOLOGY
1 Mechanoreceptors - detect the degree of distension of the GI wall.
2 Chemoreceptors - monitor concentrations of substances such as hydrogen
ions and fats.
3 Osmoreceptors - monitor osmolarity.
Signals from these receptor neurons, as well as indirect signals from the GNS,
go to GI organs and act on smooth muscle (controlling GI motility) or secretory
cells (releasing hormones). There are four main GI hormones; the first three
listed are known as the enterogastrones:
1 Cholecystokinin (CCK)

• Secreted in the duodenum and jejunum.


• Stimulated for secretion by fats or amino acids in the duodenum.
• Inhibits gastric secretion and motility.
• Stimulates pancreatic enzyme secretion.
• Stimulates bile secretion by acting on both the liver and the gall­
bladder.
2 Secretin

• Secreted in the duodenum and jejunum.


• Stimulated for secretion by acid in the duodenum.
• Inhibits gastric secretion and motility.
• Stimulates the pancreas to secrete bicarbonate (buffers the acid).
• Stimulates pancreatic enzyme secretion (works with CCK).
• Stimulates bile secretion by the liver.
3 GIP
• Secreted in the duodenum and jejunum.
• Stimulated for secretion by fats, glucose, or acid in the duodenum,
and by distension of the duodenum.
• Inhibits gastric secretion and motility.
• Stimulates insulin secretion by the pancreas.
4 Gastrin
• Secreted by the stomach.
• Stimulated for secretion by protein in the stomach, distension of the
stomach, and parasympathetic input.
• Stimulates gastric motility and HC1 secretion.
Copyright 2014 by KMK Educational Services, LLC
CHAPTER 1. GENERAL PHYSIOLOGY 65

As food passes through the GI system, the digestion process is aided by nu­
merous secretions at each stage.

Mouth
Saliva is secreted into the mouth where digestion begins and has 2 primary
functions:
1 Lubricates and moistens food for swallowing.
• Mucins - coat food for swallowing.
• Water - moistens food and washes away debris.
2 Initiates digestion.
• Amylase initiates starch digestion.
• Lipase aids in lipid digestion.

Esophagus
Food then passes through the pharynx into the esophagus.
• An upper and lower esophageal sphincter control the entry of food through
the pharynx and its exit into the stomach. Both sphincters are normally
closed and only open when food is swallowed.

Gastric reflux and the resultant heart burn occurs when the lower
esophageal sphincter opens, allowing stomach acid to back flow into
the esophagus.

Mucous
Mucous is secreted by goblet cells that line the entire GI tract. Mucous is a
sticky, viscous fluid that contains glycoproteins. It coats food to protect the
GI system from abrasion and chemical attack.
Copyright 2014 by KMK Educational Services, LLG
66 1.7. OASTROINTESTINAL PHYSIOLOGY
Stomach
When food enters the stomach, it is mixed with gastric juice to produce
chyme. Gastric juice is produced from cells in the gastric pits lining the
stomach. Gastric pits contain 4 types of cells:
1 Neck cells - secrete mucous.
2 Parietal cells - secrete hydrochloric acid and intrinsic factor (aids in vi­
tamin B12 absorption).
3 Chief cells - secrete pepsinogen, a precursor for pepsin that serves as a
proteolytic enzyme.
4 G cells - secrete the hormone gastrin, which directly enters the blood­
stream.

Gastrin is an intrinsic secretion because it is delivered directly


to the blood stream. The other three stomach secretions from the
neck, parietal, and chief cells are extrinsic secretions because
they stay in the lumen of the GI system.

Note that the stomach is the only section of the GI system that is acidic. The
gastric mucosal barrier (mucous and bicarbonate) covers and protects the
stomach from acid and pepsin.

Stomach ulcers result when acid penetrates the gastric mucosal


barrier. Aspirin (salicylic acid) and other NSAIDs are a common
cause of stomach ulcers.

Small Intestine
When partially digested food (chyme) passes through the pyloric sphincter
and leaves the stomach, it enters the small intestine. The small intestine is
where most of digestion happens and where the resulting nutrients are absorbed
by the body. The first section of the small intestine is called the duodenum
(about 30 cm long) and is the site where chyme is mixed with 2 fluids:
1 Pancreatic juice (produced by the pancreas) contains a wide variety -
of digestive enzymes and is rich in bicarbonate to help neutralize the
stomach acid.
Copyright 2014 by KMK Educational Services, LLC
CHAPTER 1. GENERAL PHYSIOLOGY 67

2 Bile is secreted by the liver and stored in the gall bladder. It contains
bicarbonate to buffer the stomach acid, as well as bile salts to aid in the
digestion of fats.
As chyme passes through the small intestine from the duodenum and into the
jejunum (about 1 meter long), enzymes continue to break down nutrients into
molecules that are readily absorbed into the bloodstream through the lining of
the small intestine for distribution throughout the body. Villi and microvilli
line the walls of the small intestine, increasing its absorptive area and allowing
for a rich network of arteries and veins.
• Water soluble materials absorbed from the intestines into the blood are
taken to the liver for further processing.
• Other nutrients remain in the blood in the general circulation.
The small intestine is very efficient at processing and absorbing nutrients, usu­
ally completing the process within the first 2 0 % of its length and well before
it reaches the third segment of the small intestine called the ileum (1.5 meters
long).
Colon
Nutrient depleted chyme passes from the small intestine into the colon, which
is larger (6 cm diameter), but shorter (1.5 meters long). The ileocecal sphincter
controls passage of the chyme from the small intestine into the cecum and on
into the colon, which is divided into the ascending, transverse, descending and
sigmoid colon. Very few nutrients remain in the chyme by the time it reaches
the colon. The colon’s primary function is volume reduction through water
absorption, resulting in feces stored in the sigmoid colon.
Rectum
When feces is ready to be eliminated from the body, the colon contracts, push­
ing fecal material into the rectum. Two anal sphincters, the internal (controlled
by smooth muscle) and the external (controlled by skeletal muscle) must relax
before feces is eliminated (defecation).
• There are more than 700 types of bacteria that live in the colon and are
commonly called the intestinal flora. They aid in many processes such
as the digestion of fiber and the production of vitamin K and vitamin B7
(biotin).•
• The intestinal flora also play a role in second pass metabolism and
aid in the uptake of certain drugs.
Copyright 2014 by KMK Educational Services, LLC
68 1.7. GASTROINTESTINAL PHYSIOLOGY
• Their by-products are often gases such as nitrogen, carbon dioxide, and
small amounts of hydrogen, methane, and sulfur dioxide; these gases are
collectively known as flatus.
• Antibiotics may attack the intestinal flora and cause intestinal discomfort.
Foods such as yogurt aid in replenishing the intestinal flora.

Several organs play a significant role in the digestion process. As described


above, the enterogastrones exert their effects on the pancreas, liver and gall­
bladder.
Pancreas
The pancreas is located behind and beneath the stomach. The pancreas has
both exocrine and endocrine functions:
• Exocrine functions:
— Pancreatic juice - produced by acini within the pancreas when stim­
ulated by CCK and secretin. It is rich in bicarbonate and digestive
enzymes and is delivered to the duodenum via the pancreatic duct.
— Pancreatic amylase - breaks down starch and glycogen (similar to
salivary amylase),
— Pancreatic lipase - breaks down fats.
— Several proteases - break down proteins.
— Several nucleases - break down nucleic acids.
• Endocrine functions
—Islet cells - produce and release insulin into the bloodstream when
stimulated by GIP.
Liver
The liver is the largest organ in the abdominal cavity and has several functions:
1 Produces and secretes bile.
• Bile contains bicarbonate, phospholipids, inorganic ions, and bile
salts.
• Bile is continually secreted by the liver, but is only released into the
GI tract when food is present;. Between meals, bile is stored in the
gall bladder.
Copyright 2014 by KMK Educational Services, LLC
CHAPTER 1. GENERAL PHYSIOLOGY 69

2 Metabolizes carbohydrates, proteins, and lipids.


• Regulates blood glucose levels through glucose metabolism, storage
of glucose as glycogen, and gluconeogenesis.
—Glucose is primarily released during exercise and between meals.
— Remember, the brain has no significant storage of glycogen and
is therefore completely dependent on the availability of glucose
in the blood.
• Converts amino acids to fatty acids and plasma proteins (fibrinogen,
prothrombin, clotting proteins). Can also synthesize amino acids or
send them through the urea cycle.
— Non-essential amino acids can be synthesized in the liver.
• Regulates lipid metabolism and synthesizes lipoproteins.
3 Removes old red blood cells from the blood and reuses (iron) or eliminates
(bilirubin) the breakdown products.
4 Eliminates waste from the body.
• Bilirubin and other heme waste is secreted into the bile to be elimi­
nated in feces. Bilirubin gives bile a greenish hue (known as the bile
pigments).
• Normal bilirubin concentration is 0.5 mg/100 mL. Excess bilirubin
(excessive production or lack of elimination) results in jaundice,
where concentrations can rise to 40 mg/100 mL.
• Cholesterol, insulin, drugs, poison, and alcohol are all eliminated
by the liver through bile. Many chemicals and/or molecules are
processed into more hydrophilic compounds so they can be more
readily eliminated.
5 Synthesizes most of the plasma proteins, including albumin, fibrinogen,
prothrombin, angiotensinogen, etc.
6 Secretes and modifies hormones.
• Works with the kidneys to activate Vitamin D.
• Secretes insulin like growth factors.
• Also eliminates many hormones by metabolizing them.
7 Stores essential molecules, including vitamins A, D, and B12, and metals
such as iron and copper.
Copyright 2014 by KMK Educational Services, LLC
70 1.7. GASTROINTESTINAL PHYSIOLOGY
Gall Bladder
The gall bladder stores and concentrates the bile produced by the liver. Bile has
a pH of approximately 7,8 and helps with lipid digestion and absorption. The
gall bladder is stimulated to contract and release bile by CCK. The sphincter
of Oddi regulates the delivery of bile into the duodenum. It is normally closed,
but opens when food is present.
• Note: When someone has their gallbladder removed, all bile secreted by
the liver gets delivered directly to the GI tract.

The contents of the GI system are propelled forward and mixed through con-
trolled muscle movements.
Peristalsis: Refers to sequential smooth muscle contractions while the mus­
cle ahead relaxes, propelling the contents of the GI tract forward.
Gastric contractions: Help to mix chyme within the stomach. Remember,
the pyloric sphincter stays closed, allowing only small particles and liquids to
pass. Gastric contractions cause most of the chyme to flow backward and mix
within the stomach.
* Liquids may exit the stomach and enter the duodenum through the py­
loric sphincter within 3 min of ingestion.
# Small particles (<1 mm) exit in 20-30 minutes, while large particles may
remain in the stomach for further breakdown for up to 9 hours.
• R,emember, gastric contraction force increases in response to gastrin, and
decreases in response to CCK, secretin, and GIP.
Segmentation: Alternating contractions between intestinal segments that
serves to mix the chyme. Segmentation primarily occurs in the small intestine
and is stimulated by distension. In the duodenum, segmentation may occur at
a rate of 1 1 - 1 2 cycles per minute; the rate decreases as you progress through
the small intestine.
Haustration: Similar to segmentation in the small intestine, haustration
refers to the mixing of chyme within the large intestine. Permanent folds in the
large intestinal wall delineate it into segments called haustra between which the
mixing occurs. Haustration occurs at a much slower rate than segementation
(usually about two per hour).
Copyright 2014 by KMK Educational Services, LLC
CHAPTER 1. GENERAL PHYSIOLOGY 71

Mass movement: Similar to a peristaltic wave. It occurs in the large in­


testine 3-4 times per day with prolonged contractions that rapidly propel the
contents forward.
Defecation reflex: Triggered by distention of the rectum, usually during
mass movements of the colon. This is an involuntary response, but may be
controlled by the voluntary contraction of the external anal sphincter.

References
[1 ] Anderson, Kenneth N. Ed. Mosby’s Medical, Nursing, and Allied Health
Dictionary, 4th ed. St, Louis: Mosby, 1994.
[2 ] Anthony, Cathrine and Norma Kolthoff. Textbook of Anatomy and Phys­
iology, 9th ed. St. Louis: Mosby, 1975.
[3] Dubin, Dale. Rapid Interpretation of EKG’s, 6 th ed. Tampa: Cover Pub­
lishing Company, 2004.
[4] Fox, Stuart Ira. Human Physiology, 6 th ed. Boston: McGraw-Hill, 1999.
[5] Katzung, Bertram G. Basic and Clinical Pharmacology, 8 th ed. New York:
Lange Medical Books/McGraw Hill, 2001.
[6 ] Moore, Keith and Arthur Dailey. Clinically Oriented Anatomy, 4th ed.
Baltimore: Lippincott Williams and Wilkins, 1999.
[7] Noble, John, Ed. Textbook of Primary Care Medicine, 3rd ed. St. Louis:
Mosby 2001.
[8 ] R,oy Shymal. K. 306/606/806 “Endocrine and Reproductive Physiology”
Class Notes. University of Nebraska Medical Center. 2001.
[9] Sansom, Steven C. 306/606/806 “Cell and Neurophysiology” Class Notes.
University of Nebraska Medical Center. 2 0 0 1 .
[10] Seidel, Henry M. et al. Mosby’s Guide to Physical Examination, 4th ed.
St. Louis: Mosby 1999.
[1 1 ] Stenchever, Morton et al. Comprehensive Gynecology, 4th ed. St. Louis:
Mosby 2001.
[12] Tierney, Lawrence M. et al. 2006 Lange Current Medical Diagnosis and
Treatment. 45th ed. 2006 New York: McGraw-Hill, 2006.
[13] Viswanathan, Suresh. V542 “Systemic Physio-Pharmacology,” Class Notes.
Indiana University. 2003.

Copyright 2014 by KMK Educational Services, LLC


c(--, '

(( ' \

((- :
;

(c-:
(( -- .
cc
rc·- !

c( - '
'

((
c(
((
((
c(
((
(c
((
(c
(c
(c
((
(_(

((
(( /

t(
(( /

(_1

(
(
I

i
Chapter 2 ________

Ocular Physiology
t
Kyle Cheatham O.D., F.A.A.O.

73
(
(
(

(
(
f
(
(
(
(
c
(
(
(
(
(
(
(
(
(
(
(
(
Copyright 2014 by KMK Educational Services, LLC (
(
(
(
CHAPTER 2. OCULAR PHYSIOLOGY 75

r ~ SECTION 2.1

Eyelids
Eyelid closure is a result of contraction of the orbicularis oculi muscles,
NOT relaxation of the levator muscle. There are 3 main types of eyelid closure:
blinking, winking, and spasm. Because winking is a form of voluntary blinking,
we will divide the following discussion into blinking and spasm (18).

There are three main types of blinking: spontaneous, reflex, and voluntary
blinking (18, pp. 2 2 ).
1. Spontaneous Blinking:
The most common type of blinking; it results from contraction of the palpebral
portion of the orbicularis oculi in the absence of an external stimulus and
occurs at an average rate of 12-15 blinks per minute (18).
* Spontaneous blinking helps to maintain the optics and comfort of the eye
by stabilizing the tear film. During a spontaneous blink, new tears are
secreted and spread across the ocular surface while old tears are pushed
medially towards the nasolacrimal drainage system (see below for further
information).

A decreased rate in spontaneous blinking results in decreased tear


secretion and an increase in tear evaporation, resulting in dry eye
syndrome and secondary epiphora. A decreased blink rate com­
monly occurs when reading, watching TV, or after LASIK surgery
due to a decrease in corneal sensitivity (18).

2. Reflex Blinking:
A blink caused by sensory stimuli, including:
• Auditory: Loud noises sensed by CN V III can elicit a blink reflex.
• Touch or Irritation: Mediated by CN V.

Copyright 2014 by KMK Educational Services, LLC


76 2.1. EYELIDS

C otton swab testing evaluates the health of V I by determining


its ability to initiate reflex blinking in response to an irritating
stimulus.

• Dazzle: Bright light detected by CN II stimulates a reflex blink.


• Menace: CN II detects an unexpected object threatening the eye and
initiates a blink reflex.

The efferent loop of reflex blinking in response to auditory,


touch/irritation, and menacing stimuli begins in the frontal lobe
(i.e. involves cortical input). The dazzle reflex is the only reflex
blink that does NOT involve the cortex. Remember that the effer­
ent loop involves stimulation of the orbicularis oculi via CN VII.

Unlike reflex blinking, spontaneous blinking occurs in the ab­


sence of an external stimulus. The palpebral portion of the or­
bicularis is responsible for both spontaneous and reflex blinking.

3. Voluntary blinking:
The amplitude and duration of voluntary blinking is varied and more prolonged
compared to spontaneous and reflex blinking (18).
• Winking: A form of voluntary blinking that requires simultaneous con­
traction of the orbital and palpebral portions of the orbicularis oculi (18).

Includes the condition benign essential blepharospasm:


• Characterized by bilateral, involuntary, sustained twitching and/or clos­
ing of the eyelids.
* Results from spasms of the orbicularis oculi, procerus, and corrugator
musculature.
Copyright 2014 by KMK Educational Services, LLC
CHAPTER 2. OCULAR PHYSIOLOGY 77

Tight or forced eyelid closure requires contraction of the or­


bital portion of the orbicularis oculi. Bell’s phenomenon, a
normal defense reflex present in about 75% of the population, oc­
curs after forced lid closure and is characterized by an upwards and
outwards rotation of the globe.

PRODUCTION
• Meibomian glands: Sebaceous glands located within the upper (30-
40) and lower (20-30) tarsal plates of the eyelids that are responsible for
secreting the anterior lipid layer of the tears (26). Blinking stimulates
lipid release via holocrine secretion (18, pp. 32).
• Glands of Zeis and Moll: Modified sebaceous and modified sweat
glands, respectively, located next to hair follicles that make a minor con­
tribution to the anterior lipid layer of the tears.
• Accessory lacrimal glands: Tubuloacinar exocrine glands that con­
tribute to the aqueous layer of the tears.
— Glands of Krause are more numerous and are located in the for-
nices.
— Glands of Wolfring are less numerous and are found in the tarsal
conjunctiva.
DISTRIBUTION
The upper eyelid closes laterally to medially during a blink, spreading the
mucin layer of the tears evenly across the corneal epithelium and bulbar
conjunctiva to aid in proper tear film formation.
DRAINAGE
The lacrimal pump theory, popularized by Jones and Wobig, summarizes how
eyelid closure affects tear drainage (18, pp. 40).
• When the eye is OPEN, tears passively drain into the puncta via capil­
lary attraction, •
• When the eyelids close during a blink, the muscle of Horner con­
tracts, causing the canaliculi to shorten as they move medially towards
Copyright 2014 by KMK Educational Services, LLC
78 2.2. TEARS
the lacrimal sac. This action helps to “pump” the tears into the lacrimal
sac.
- Remember that the muscle of Homer is part of the palpebral portion
of the orbicularis oculi and surrounds the canaliculi.
• As the eyelids close, the orbicularis oculi also contracts, stretching the
temporal wall of the lacrimal sac away from the nose. This action creates
a negative pressure that helps to draw tears into the lacrimal sac.
Newer theories suggest that contraction of the orbicularis oculi causes lacrimal
sac compression (NOT dilation), forcing tears into the nasolacrimal duct (26).

Blinking occurs from the lateral to the medial canthus and helps
to move tears towards the puncta. Blinking also lowers the pres­
sure in the canaliculi, creating a pressure difference between the
atmosphere and the lacrimal sac that promotes tear drainage.

SSIlSISISlillM
Cilia (eyelashes): Responsible for screening and sensing the environment and
inducing blink reflexes when necessary. There are approximately 150
eyelashes in the upper eyelid and 75 in the lower eyelid (26).
Glands of the eyelids: Secretions from the glands of the eyelids not only
help to produce the tear film, but also help to move debris away from the
cornea in concert with spontaneous blinking.

We now introduce the various functions of the tear film (18, pp. 32) (26).
1 Optical: The primary role of the tear film is to create a smooth optical
surface for clear vision. Remember, the largest change in refractive index
occurs between the air/tear film interface.
2 Nutritional: The primary source of O2 for the corneal epithelium is
from diffusion of atmospheric O2 through the tear film.
Copyright 2014 by KMK Educational Services, LLC
CHAPTER 2. OCULAR PHYSIOLOGY 79

3 Mechanical: The tear film collects debris and moves it away from the
cornea during a blink. It also helps to remove metabolic waste products
from corneal epithelial cells.
4 Antibacterial: The aqueous layer of the tears (secreted by the main
and accessory lacrimal glands) contains lysozyme, lactoferrin, IgA, and
other proteins of the immune system.
5 Corneal Transparency: The tear film has a specific osmolarity and
pH that is maintained by the secretory glands and the corneal epithelial
cells, thus helping to prevent corneal edema.

psitioiiyofV Tears
Although widespread research has been conducted, there has been a lack of
consensus regarding the total thickness of the tear film, with studies reporting
a range of results from 7 um to 45 um. Despite continued variability in results,
newer research performed with non-invasive techniques suggests the total tear
film thickness is approximately 3 um (14).
Anterior Lipid Layer
The anterior lipid layer is composed of free fatty acids, cholesterol and waxy
esters (2 1 ). It is secreted primarily by the meibomian glands and, to a lesser
extent, the glands of Zeis and Moll. The main purpose of the anterior lipid
layer is to slow the evaporation of the aqueous layer of the tears. It also
helps to maintain optical clarity.*
* Although blinking is the primary method for releasing lipids from the
glands, parasympathetic innervation from nerves surrounding the glands
may also increase lipid secretion (18, pp. 33).
Aqueous Layer
The aqueous layer of the tears serves the following functions (18) (26):
1 Provides protection through antibacterial proteins.
2 Provides nutrition by supplying glucose to the corneal epithelium.
3 Adds thickness to the tear film.
The aqueous layer contains the following components (26) (18):
• Water (the main component of the tears).
• Electrolytes: Na+, K+, and C1-.
Copyright 2014 by KMK Educational Services, LLC
(

(
2.2. TEARS
80
C
• Antimicrobial components: IgA, lactoferrin, lysozyme, beta-lysin,
and interferon. c
—Lysozyme cleaves peptidoglycan bonds in bacterial cell walls.
—Lactoferrin chelates Fe2-b, an essential nutrient for bacterial cell ('
growth and metabolism.
—Beta-lysin destroys bacterial cytoplasmic membranes and acts
in concert with lysozyme.
c
• Lipocalins: Decrease the surface tension of the tears to enhance
spreadability and act as a carrier for all-trans retinol. Also block
(
Fe2+ binding receptors on the surface of bacteria. (
• Vitamin A: Present within tears in the form of all-trans retinol;
necessary for the development of goblet cells of the conjunctiva. (
• Enzyme cofactors (Fe2+, Mg2+, Cu2+, Ca2+): Help maintain the
membrane permeability of corneal epithelial cells. ( :
• HCO 3 —: Acts as a buffer for tears.
• Solutes: Glucose, urea, lactate, citrate, ascorbate, and amino acids. (
• Additional proteins including albumin, growth factors, interleukins,
and VEGF. ( -

The composition of the aqueous layer of the tears changes with increasing age,
contact lens wear, and under closed eye conditions.
(
• Increasing age is associated with a decrease in the levels of lysozyme and (
lactoferrin proteins within the tears, as well as a.n overall decrease in
aqueous secretion (26). (
• Contact lens wear causes an increase in electrolyte and protein concen­
tration due to increased evaporation of the tears (26). (
• The aqueous layer of the tears under closed eye conditions has a higher
concentration of IgA and serum albumin compared to open eye condi­
(
tions. Lysozyme and lactoferrin levels are essentially the same. (
The main lacrimal gland and the accessory lacrimal glands of Krause
and Wolffing secrete the aqueous layer of the tears. (
• The main lacrimal gland is innervated by parasympathetic fibers from
CN VII, sympathetic fibers, and sensory nerves of VI.•
C
• The accessory lacrimal glands are thought to be innervated by parasym­ c
pathetic nerves; however, neural control of the accessory lacrimal gland
secretions is not well understood and conclusive research is unavailable (18,
PP- 35). (.
Copyright 2014 by KMK Educational Services, LLC (
(,
c,
(_
CHAPTER 2. OCULAR PHYSIOLOGY 81

Although conventional theory states that the main lacrimal gland is responsible
for reflex and emotional tearing and the accessory lacrimal glands are respon­
sible for maintenance (i.e. basal) tearing, a more recent theory holds that both
the main AND the accessory lacrimal glands are responsible for basal tearing.

The sensory nerves (VI) of the cornea are involved in a reflex arc
that causes lacrimation (through parasympathetic stimulation of
the lacrimal gland via CN VII), miosis, and a protective blink.
The dazzle blink reflex can also stimulate lacrimal gland secre­
tion (26).

Mucous Layer
Consists of an outer mucin layer that interacts with the glycocalyx of the
corneal epithelium and helps to spread the tears across the corneal surface, as
well as trap debris, bacteria, and sloughed corneal epithelial cells.

Remember that mucin molecules are unique in that they are ca­
pable of mixing with lipid AND water. This property allows the
mucous layer to mix with the aqueous layer of the tears and spread
it evenly over the hydrophobic corneal epithelial surface.

The mucous layer is produced by the goblet cells of the conjunctiva and the
squamous cells of the cornea and conjunctiva.
• Goblet cells are predominately found in the inferonasal fornix and the
bulbar conjunctiva (most concentrated temporally) (18).•
• Goblet cells require Vitamin A for development, which is found in the
aqueous layer of the tears as all-trans retinol. Vitamin A deficiency results
in keratinization of the conjunctiva and cornea.

Vitamin A deficiency can result in B itot’s spots (foamy build-up


of keratin) on the conjunctiva.

Copyright 2014 by KMK Educational Services, LLC


82 2.2. TEARS
Sensory nerves in the corneal and conjunctival epithelium stimulate sympa­
thetic and parasym pathetic nerve endings surrounding goblet cells; parasym­
pathetic stimulation causes an increase in mucous secretions (18, pp. 39).

Mucous Fishing Syndrome occurs as a result of patients “fish­


ing” for and removing excess mucous in the conjunctiva. This re­
sults in damage to the conjunctival epithelium and a subsequent
increase in mucous production, creating an unfortunate cycle that
exacerbates symptoms. Dry eye syndrome is the most common
cause of mucous fishing syndrome.

Although traditionally the tear film is viewed as three separate


layers, new research suggests that the aqueous and mucin layers
are intermixed, with a greater mucin concentration towards the
corneal surface of the tears.

Recall that the eyelids play an important role in spreading the mucous layer
evenly over the corneal epithelium. The mucin layer interacts with the gly-
cocalyx of the corneal epithelium, allowing the tear film to be evenly spread
across the corneal and conjunctival epithelium.

The stability of the tears can be examined clinically by analyzing


the tear break-up time (TBUT). Fluorescein is instilled in the eye
and spreads evenly throughout the tears. Over time, the aqueous
layer evaporates as a result of an insufficient lipid layer, resulting
in a break-up of the tears. Blinking promotes secretion of the
anterior lipid layer and restores the tear film. A TBUT less
than 10 seconds is considered abnormal.

Copyright 2014 by KMK Educational Services, LLC


CHAPTER 2. OCULAR PHYSIOLOGY 83

Approximately 25% of secreted tear volume is continuously lost via evaporation.


The remaining 75% of the tear volume drains through the nasolacrimal system
or into the systemic circulation via absorption into the conjunctival and/or
nasolacrimal vasculature (4, pp. 17).

The total tear volume on the ocular surface is approximately 7-9


uL. The maximum amount of fluid the eye can hold within the
tear film and the fornices is 20-30 uL. Normal tear production is
approximately 1 uL/minute and the average eye drop contains 50
uL. Drop instillation or tear production greater than 1 uL/min
results in tear overflow onto the cheeks (i.e. epiphora) (18, pp. 95).

• The normal tear film on a healthy ocular surface has an osmolarity of


approximately 308 raOsm/L and is isotonic to the healthy corneal
surface.
— Tear film osmolarity can vary depending on the blink rate, humidity,
contact lens wear, and ocular pathology
• Na-t- and Cl- ions within the aqueous portion of the tears are the main
contributors to tear osmolarity. Ca2-f- and K-(- are also important com­
ponents of the aqueous portion of the tears:
— Calcium is essential for hemidesmosome formation in the basement
membrane of the corneal epithelium. Excess calcium can deposit
on contact lenses, forming “jelly bumps” that may decrease visual
acuity.
— Potassium helps to maintain the health of the corneal epithelium
and has a 4X greater concentration in the tears compared to
blood plasma.

Dry eye syndrome causes an increase in tear osmolarity. Hypo­


tonic eye drops (osmolarity of 150 mOsm/L) are often utilized
in treatment.

Copyright 2014 by 1<MK Educational Services, LLC


84 2.3. EXTRAOCULAR MUSCLES
pH, Buffering, Temperature
The bicarbonate ions (HCO 3 —) within the aqueous layer of the tears are an
excellent buffer and can tolerate ophthalmic solutions with a pH ranging from
3.5 —10.5. The average pH of the tears is 7.45 (4, pp. 28).
• The pH of the tears during sleep decreases (i.e. becomes more
acidic) due to the byproducts of anaerobic respiration.
• The pH of the tears in dry eye syndrome increases (i.e. becomes
more basic) due to an increase in tear osmolarity.

Most ophthalmic topical solutions are weak bases. Because


the pH of the tears is 7.45, most ophthalmic drugs are present in the
non-ionised form within the tear film, promoting drug absorption
across the hydrophobic corneal epithelium and endothelium (4).

r - SECTIO N 2.3

E xtraocular Muscles

jVestibular Cbntrol dVIechanisms ; - :; ( ]


MIDDLE EAR: Separated from the external ear by the tympanic mem­
brane. Sound waves are amplified 10-20X by the tympanic membrane (ear
drum) before being converted into mechanical vibrations and sent into the
inner ear. The middle ear contains the following;
• Tympanic Cavity: The space internal to the tympanic membrane (aka
“middle ear”).
• Auditory Ossicles; The small ear bones, including the malleus, incus,
and stapes. These bones are located in a series between the tympanic
membrane and the oval window. The malleus is first in the series and
is attached to the tympanic membrane, followed by the incus, and then
the stapes, which is attached to the oval window. The auditory ossicles
amplify and transmit vibrations received by the tympanic membrane.
• Stapedius and Tensor tympani muscles: These muscles dampen the amount
of vibrations placed on the auditory ossicles.
—The stapedius muscle is innervated by a branch of CN VII just before
it exits the skull via the stylomastoid foramen.
Copyright 2014 by KMK Educational Services, LLC
CHAPTER 2. OCULAR PHYSIOLOGY 85

— The tensor tympani muscle is innervated by a branch from the


mandibular division (V3) of CN V.

The chorda tympani nerve of GN VII (carrying taste sensations


from the anterior 2/3 of the tongue) and the tympanic nerve plexus
(branching from CN IX) travel within, but do not innervate, the
middle ear cavity.

INNER EAR: Converts mechanical vibrations into neural signals.


* Vestibulocochlear organs help to maintain balance, receive sound, and
contribute to ocular reflex actions.
• The bony labyrinth consists of three parts that are innervated by CN
VIII:
1 Cochlea: Shell shaped portion of the inner ear. Contains the organ
of Corti that contains hair cells that control hearing.
2 Vestibule: Contains the utricle and saccule that help maintain bah
ance. These organs detect static linear acceleration (movement of
the head or body from side to side) and cause reflex eye movements
(Linear VOR) that are equal and opposite to the motion of the
head.
- The utricle detects horizontal linear movement.
- The saccule detects vertical linear movement.
- The vestibule area is continuous with the cochlear duct for hear­
ing.
3 Semi-circular canals: Communicate with the vestibule and con­
tain ampullae that detect angular acceleration (rotational move­
ments of the body or head) and cause reflex eye movements know
as the angular VOR.

In summary, the tympanic membrane separates the external


and middle ear. The oval window separates the middle and inner
ear. The tympanic membrane is much larger, allowing amplifica­
tion of sound. The malleus, incus and stapes bones lie between the
tympanic membrane and the oval window.

Copyright 2014 by KMK Educational Services, LLC


86 2.1 CORNEA
.................ir Goilir<>1.of -Eye
Most voluntary eye movements are a combination of saccades and pursuits (3).
Saccades
• Rapid eye movements that maintain fixation (aka foveation) on the object
of regard.
• Horizontal saccades are controlled by the contralateral frontal eye
field in the frontal lobe and the superior colliculus. For example, the
right frontal lobe controls saccades towards the left.
Pursuits
• Smooth tracking movements that maintain foveation on slow-moving ob­
jects.
• Controlled by the ipsilateral parietal lobe. For example, the right
pursuit is driven by the right parietal lobe, and the left pursuit is driven
by the left parietal lobe.
Vergence
• Control of vergences is presumably located at the level of the brainstem.
• Divergence and convergence (i.e. motor fusion) eye movements are likely
driven by retinal disparity and help to maintain sensory fusion and
stereopsis (18, pp. 835).•

Please see the ocular motility chapter for further details regarding
the extraocular muscles and eye movements.

• Corneal epithelium: Contains zonula occludens junctions (tight junc­


tions) that force molecules to travel THROUGH the cells rather than
passing between cells. The epithelium is highly lipophilic (aka hydropho­
bic), limiting the absorption of hydrophilic, ionized molecules.
Copyright 2014 by KMK Educational Services, LLC
CHAPTER 2. OCULAR PHYSIOLOGY 87

• Corneal stroma: Highly hydrophilic. Hydrophilic, ionized substances


can easily pass through the corneal stroma.
• Endothelium: Contains macula occludens junctions. The endothelium
is highly lipophilic (similar to the epithelium) and allows only lipophilic,
non-ionized substances to pass through.

The following facts regarding light transmission through the cornea are note­
worthy:
• Recall that the UV spectrum can be divided into UV-C (200-290 nm),
UV-B (290-320 nm), and UV-A (320-400 nm) light waves. The energy
per photon increases as the wavelength decreases (14).
• The corneal epithelium and Bowman’s layer protect the inner layers
of the eye by absorbing shorter wavelengths of UV light (UV-C and UV-B
below 300 nm).
• The cornea transm its light with a wavelength of 300 nm (UV) to 2,500
nm (infrared).
• The visible wavelengths of light (400 to 700 nm) are transmitted through
the cornea with a high degree of precision. More than 99% of light above
a wavelength of 400 nm is transmitted through the cornea (18, pp. 56).

The corneal epithelium is most sensitive to radiation in the UV~


C range (particularly 260-280 nm); snow-blindness, welder’s ker­
atitis, and tanning sun lamps can cause ultraviolet keratitis (14).

The following factors contribute to minimal light scattering, allowing for opti­
mal corneal transparency:
* Corneal crystallins are located in the cytoplasm of epithelial and en­
dothelial cells and help to maintain corneal transparency by limiting light
.scattering, similar to crystallins in the lens.*
* Ascorbate (Vitamin C) and glutathione are located within the epithe­
lial cells and help to protect the cornea from UV rays and free radical
scavengers.
Copyright 2014 by KMK Educational Services, LLC
88 24- CORNEA
• The corneal stroma contains approximately 200-250 layers of 30 nm lamel­
lae composed of collagen fibrils (n —1.55) that lie within a network of
GAGs (n = 1.345). Collagen fibrils have a uniform size and are pre­
cisely spaced less than one half the wavelength of visible light from one
another (18, pp. 80) (26).
— Proteoglycans (PGs) are present within the ground substance that
fills the space between the corneal cells and collagen fibrils and lamel­
lae. The glycosaminoglycan side chains of PGs help to maintain
appropriate collagen spacing by forming negatively charged bonds
with water molecules.
—Precise spacing of the collagen fibrils increases destructive inter­
ference, thereby minimizing light scattering and increasing corneal
transparency.
• The avascular nature of the cornea minimizes light scattering and con­
tributes to its transparency.
• The high water content of the cornea helps maintain the regular spacing
between collagen fibrils.
— Remember that the sclera has a lower concentration of GAGs (75%
less) compared to the cornea, and is thus dehydrated (65% H 2O) and
less transparent compared to the corneal stroma (78% H 2O) (26).

Proteoglycans are composed of a core protein with one or more


covalently linked glycosaminoglycan (GAG) side chains. Sulfona-
tion of the GAG side chains in the corneal stroma allows proteo­
glycans to bind to water, creating a hydrophilic environment that
helps to maintain the precise spacing of collagin fibrils. The major
proteoglycan in the corneal stroma is keratin sulfate (26).

The most important factors that influence corneal thickness (hydration) and
maintain corneal deturgescence include epithelial pump mechanisms, endothe­
lial pump mechanisms, and aquaporins (26) (18, pp. 77).

Corneal deturgescence is the state of relative dehydration main­


tained by the normal cornea that is necessary for transparency;
75 —80% stromal water content is optimal (23). Deturgescence re­
lies on the endothelial (main contributor) and epithelial transport
mechanisms.

Copyright 2014 by KMK Educational Services, LLC


CHAPTER 2. OCULAR PHYSIOLOGY 89

Epithelial pump mechanisms


The basal membrane on the posterior surface of corneal epithelial cells (sur­
face closest to the stroma) contains two transport mechanisms: Na+/K+ AT-
Pase pump and the Na-f/K+/Cl- cotransporter.
• N a+ passively enters the epithelial cell from the tear film covering
the anterior surface of the cell. The Na+/K+ ATPase pump actively
moves Na+ from the epithelial cell into the corneal stroma, creating a
higher Na-f- concentration in the stroma compared to the epithelium.
• The Na+/K+/Cl- cotransporter utilizes the Na-f- concentration gradient
to passively move Na+, K+, and 2C1- from the stroma into the epithelial
cells. Cl- and K-f- each have their own channels that allow for passive
diffusion back into the tears and towards the aqueous humor, respectively.
• Movement of K+ into the aqueous humor will stimulate Cl- to move
into the tears. Water will follow C1-, contributing to the dehydration
of the cornea.
—The K+ channel has been shown to respond to pH changes within
the cornea. For example, a hypoxic cornea (e.g. after prolonged
contact lens wear) will have higher acidity (lower pH) and increased
thickness due to corneal swelling,
—The K+ channel responds by moving more K+ into the aqueous,
causing more Cl- and H2 O to move into the tear film to restore
normal corneal thickness (18, pp. 79).
Endothelial pump mechanisms
There are multiple mechanisms (most are poorly understood) that are believed
to play a role in the transport of ions and water across the endothelium. The
most thoroughly understood mechanism is the Na+/K+ ATPase pump (26).
• Na-f- enters the endothelial cell from the corneal stroma via ion exchang­
ers. The Na-b/K4- ATPase pump, located on the basolateral mem­
brane of the endothelial cell, pumps Na+ out of the endothelial cell into
the aqueous humor, establishing a higher Na+ concentration in the aqeu-
ous humor compared to the corneal endothelium.
• The Na+/H+ pump utilizes the Na+ concentration gradient to move
H+ ions out of the endothelial cells into the aqueous in exchange for the
transfer of Na-f- ions back into the endothelial cells. Movement of H+
ions into the aqueous humor results in a decrease in extracellular pH,
causing GO2 to diffuse into the endothelial cell (18).•
• CO2 is combined with H^O to form H 2GO3 , which then dissociates into
H+ and HCOs~ (bicarbonate) ions.
Copyright 2014 by KMK Educational Services, LLC
90 24- CORNEA
• Bicarbonate and Cl- move out of the endothelial cell and into the aqueous
humor. H 2O will then follow, contributing to the dehydration of the
cornea (26).

Cl- excretion and N a+ absorption are the major factors


for water transport across the corneal epithelium and endothe­
lium (22) (26).

Aquaporins
Proteins embedded within the apical and basal membranes of corneal epithelial
and endothelial cells that regulate bi-directional water transport.

The entire cornea receives oxygen primarily from the atmosphere. The aque­
ous humor, limbal vasculature, and palpebral conjunctival capillaries provide a
minor contribution of O2 to the cornea under open-eyed conditions.
• The total amount of pressure in the atmosphere is 760 mmHg. Since 1/5
of the atmosphere is oxygen, the partial pressure of oxygen in the air is
155 mmHg.
• Because the cornea receives O2 from the atmosphere under open eye
conditions, the partial pressure (PP) of O2 within the tears is 155
mmHg. This PPO 2 Is high enough to provide O2 to the entire cornea,
although as previously mentioned, the aqueous humor and limbal capil­
laries provide additional support.
• During closed eye conditions, the PPO 2 is approximately 55 mmHg.
O2 supply varies depending on the layer of the cornea:
1 The superior palpebral conjunctiva (primary contributor) and the
limbal vasculature supply the epithelium and the anterior stroma.
2 The aqueous humor supplies the posterior stroma and endothelium.

Mild corneal edema occurs after awakening in all healthy indi­


viduals; in fact, the cornea is always thickest in the morning! Mild
corneal edema is due to a build-up of lactate from anaerobic respi­
ration and the limited supply of O2 when the eye is closed (26).

Copyright 2014 by KMK Educational Services, LLC


CHAPTER 2. OCULAR PHYSIOLOGY 91

The critical PPO 2 for the cornea is 10-20 mmHg. A contact lens
that is worn while sleeping must maintain a partial pressure of O2
above the critical value. Remember that minus lenses are thinner
in the center and thus are likely more capable of transporting O2
compared to plus lenses.

The following formula is useful in looking at how O2 flow to the cornea is


affected during contact lens wear:
J/A = Dk/t (P 1 -P 2 ) (2 .1)

• J/A refers to how much oxygen flows over a certain area.


• Dk refers to the oxygen permeability of a material. The higher the Dk,
the more readily oxygen will diffuse through a specific area of a material.
• Transmissibility (D k/t) is a measure of how much oxygen will diffuse
through a contact lens of a given thickness. It is determined by dividing
the Dk by the thickness of the lens (t).

Transmissibility is given as a number X 10“9 (cm/sec) (ml


Og/mL mmHg). The initial number is quoted with the units and
power of 10 assumed. For example, the Dk of Acuvue Oasys is 103,
and the Dk/t is 147 (6).

* Proper control of pH within the cornea (pH = 7-7.3) is essential for main­
taining corneal transparency (26). Decreased levels of O2 (hypoxia) can
lead to an accumulation of H+ ions produced in glycolysis, resulting in
increased acidity of the corneal cells.•
• Decreased corneal pH causes a change in K+ channels, resulting in a mas­
sive efflux of K+ from the keratocytes with subsequent collagen damage
and scar formation.

Copyright 2014 by KMK Educational Services, LLC


92 2.1 CORNEA

Glucose is produced for the cornea via anaerobic glycolysis (85%), aerobic
glycolysis, and the hexose monophosphate shunt (26).
• Glucose concentration is low in the tears but is high in the aqueous
humor. As a result, the aqueous humor is the primary contributor of
glucose to all corneal layers.
• The aqueous humor also serves as the primary source for amino acids and
vitamins for all layers of the cornea.

Corneal epithelial cells are unique because they can store large
amounts of glycogen for basal cell mitosis and epithelial wound
healing. The endothelium also requires large stores of energy in
order to maintain the function of the Na+/I<+ ATPase pumps
that contribute to corneal transparency (14) (26).

Maintenance Epithelial Regeneration


The entire corneal epithelium replaces itself every 7-14 days (26) (21) (1).
Recall the following noteworthy facts about epithelial regeneration:
• Basal cells are the only mitotic cells in the epithelium. They are derived
from differentiating limbal stem cells from the Palisades of Vogt.•
• Basal cells differentiate into wing cells and then squamous cells before
reaching the corneal surface. Old superficial corneal cells are shed as this
process occurs.
Traumatic Epithelial Regeneration
• First step: Basal cell mitosis is inhibited (26) (1).
• Second step: After epithelial or stromal injury occurs, fibronectin is
released and serves as a scaffolding for epithelial cells to migrate over
the wound in response to the release of cytokines and growth factors.
Hemidesmosomes are then created to allow for proper adhesion between
the migrated epithelial cells and the basement membrane (18, pp. 70) (26) (14).
Copyright 2014 by KMK Educational Services, LLC
CHAPTER 2. OCULAR PHYSIOLOGY 93

• Third step: Basal cell mitosis resumes at a rapid rate. This will oc­
cur once the wound is closed with a single layer of cells and cell-to-cell
junctions are created (26).
If the basement membrane remains intact, corneal regeneration occurs quickly.
If the basement membrane is damaged (most commonly a result of sharp cut­
ting objects such as fingernails or paper cuts), corneal regeneration occurs more
slowly.
• Complete healing of the BM (with creation of intact hemidesmosomes)
takes approximately 8 weeks (32).

Matrix metalloproteinases can degrade hemidesmosome formation.


Because corticosteroids and tetracyclines have been shown to de­
crease the activity of metalloproteinases, they are often included in
the treatment regimen for RCEs.

Corneal abrasions result from trauma. RCEs occur in eyes with


poor adhesion between the corneal epithelium and basement mem­
brane from previous abrasions or corneal dystrophies.

Summary of All Cornea Layers


• Epithelium and Descemet’s membrane CAN regenerate.
• Bowman’s layer and the endothelium CANNOT regenerate.
• Stroma will replace itself if damaged, but with a very different textured
tissue. The new collagen is larger and less organized, resulting in a
scar (26) (21).

Bowman’s “bows out” (will not regenerate) if damaged. De­


scemet’s “D-3” will regenerate via the endothelium. Remember
that Descemet’s membrane is always growing and triples in thick­
ness (5 um to 15 um) from youth to adulthood (26).

Copyright 2014 by KMK Educational Services, LLC


94 2.1 CORNEA

Recall that there are no nerves in Descemet’s membrane or the endothelium.


Corneal nerves enter at the level of mid-stroma and travel through Bowman’s
layer to the corneal epithelium.
• Corneal nerves respond to several different types of sensory stimuli
including mechanical, thermal, and chemical factors (e.g. low pH).
• The corneal nerves can detect foreign agents and elicit the blink reflex,
protecting the epithelium from potential invasion (18, pp. 68).
• The majority of sensory nerves are considered nociceptors, which have a
low threshold and mediate pain.
• The corneal nerves serve a trophic function: sensory innervation is
essential for epithelial cell maintenance and regeneration.
• Reduced corneal sensitivity is typical after LASIK and with aging.

Neurotrophic keratitis is characterized by CN V damage and


decreased corneal sensitivity and can be diagnosed with the cotton
swab test. Herpes simplex and zoster, stroke, and DM are
common causes of neurotrophic keratitis.

• The vertical meridian flattens, resulting in an increase in ATR astigma­


tism.
• Light scattering increases.
• Corneal sensitivity decreases.
• The basement membrane thickens.
• The degree of corneal arcus in the peripheral stroma increases.
• Descemet’s membrane thickens, causing an increase in the number of
Hassail-Henle bodies in the corneal periphery.•
• The endothelial cell density decreases as the endothelium becomes thinner
with age.
Copyright 2014 by KMK Educational Services, LLC
CHAPTER 2. OCULAR PHYSIOLOGY 95

Provides one-third of the total dioptric power of the eye and allows for accom­
modation to near objects. The following changes occur during accommoda­
tion;
1 Parasympathetic stimulation causes contraction of the ciliary muscle, re­
sulting in a decrease in the tension in the lens zonules.
2 The anterior pole of the lens moves forward and the anterior curvature
increases.
3 The posterior pole of the lens moves back slightly and the posterior cur­
vature increases.
4 The lens thickness (anterior-posterior dimension) increases and the an­
terior chamber depth decreases.
5 The lens diameter decreases.
6 The lens power increases.

Note that accommodation can cause a temporary decrease in


IOP because ciliary muscle contraction pulls the scleral spur pos­
teriorly and opens up the pores of the TM. Accommodation also
leads to a decrease in the depth of the anterior chamber because
the anterior pole of the lens moves forward, the anterior lens cur­
vature increases, and the lens thickness increases. These changes
may induce “pupillary block” and result in elevated IOP, important
adverse effects of miotic drugs such as pilocarpine.

M m M a m m srn m M sm M i
Although the lens is avascular, it has the largest concentration of proteins of any
structure in the body and thus requires glucose and oxygen from the aqueous
in order to maintain the following functions:
1 Production of new lens fibers and protein synthesis (18, ch. 3).
Copyright 2014 by KMK Educational Services, LLC
96 2.5, LENS
2Maintenance of the N a+ /K + ATPase pump that helps to establish
a balance between H 2 O and ions within the lens to maintain lens trans­
parency.
• The Na+/K+ ATPase pump on the epithelial cells constantly move
Na+ into the aqueous humor (and K-f into the lens). H 2 O ulti­
mately follows Na+ into the aqueous, contributing to lens dehydra­
tion and transparency.
The following are noteworthy facts regarding lens metabolism:
• Over 70% of the glucose required by the lens is produced through anaer­
obic glycolysis. Aerobic metabolism (via the Kreb’s cycle and the elec­
tron transport chain) is limited to the lens epithelium (9).
• The first step in both aerobic and anaerobic respiration involves the con­
version of glucose to glucose 6-phosphate via the enzmye hexokinase.
If hexokinase is not available, glucose is converted to sorbitol via the
enzyme aldose reductase.
• Excess sorbitol can accumulate in the lens, creating an osmotic gradient
that favors the movement of H 2 O into the lens, ultimately causing lens
swelling, lens fiber damage, and cataract formation.

Excessive levels of glucose in diabetes leads to the accumulation of


sorbitol within the lens, ultimately leading to early cataract devel­
opment and an acute shift in the refractive error secondary to lens
swelling.

it®»
We now introduce the effects of glutathione and ascorbic acid on lens clarity (18,
pp. 130).
Glutathione: The primary protector against oxidative damage in the lens.
• Glutathione acts as a reducing agent and detoxifies hydrogen per­
oxide.
• Glutathione is transported into the lens from the aqueous, but can
also be synthesized from lens epithelial cells and superficial fiber
cells.
• Deep fiber cells and nuclear cells produce minimal glutathione and
thus rely on diffusion of glutathione from the superficial fibers and
lens epithelial cells.
Copyright 2014 by KMK Educational Services, LLC
CHAPTER 2. OCULAR PHYSIOLOGY 97

Glutathione diffusion diminishes with age and is a factor in the


formation of cataracts.

Ascorbic Acid (Vitamin C ): Helps to protect the lens from oxidative dam­
age. Ascorbic acid is found in a much higher concentration in the lens
compared to the aqueous humor.

The following factors help to maintain lens transparency (26) (18):


• Active Na+/I<+ ATPase pumps located on the lens epithelial cell mem­
branes maintain electrolyte balance by pumping Na+ into the aqueous
humor and K+ into the lens.
• The lens is avascular (dependent on nutrients that diffuse from the aque­
ous humor), which minimizes light scattering.
• Lens fiber cells lack membrane-bound organelles to minimize light scat­
tering.
• Lens fiber cells are packed very close together and are uniformly spaced.
• The cytoplasm of the lens fibers are crowded with crystallins that mini­
mize light scattering by destructive interference.
• Multiple transport processes are present to carefully monitor and limit
the concentration of Ca2+ inside the lens in order to prevent cataract
formation.

* The embryonic nucleus is formed from primary lens fibers of the posterior
lens epithelium during embryological development. All remaining growth
of the lens is due to the production of secondary lens fibers by the anterior
lens epithelium.•
• Mitosis of fiber cells occurs in the germinative zone of the ante­
rior lens epithelium. After mitosis, lens fiber cells gradually migrate
through the transition zone and into the equator, where fiber elonga­
tion occurs. During this process, lens fibers lose their membrane-bound
organelles and acquire crystallins.
Copyright 2014 by KMK Educational Services, LLC
98 2.5. LENS
—The anterior lens epithelium has the greatest metabolic demand of
all lens components and thus contains a significant amount of mi­
tochondria. to produce energy for mitosis. Remember that aerobic
respiration is limited to the anterior lens epithelium!
* Aqueous humor travels over the anterior surface of the lens and provides
nutrients (including glucose and 0 %) to the anterior lens epithelium for
mitosis.

Remember that the anterior lens epithelium is responsible


for transporting nutrients from the aqueous humor into the lens.
N a+ /K + ATPase pumps located on the epithelial cell mem­
branes are responsible for maintaining the appropriate ion balance
to control lens dehydration.

Remington describes the following age-related changes in the lens (26):


• Decrease in crystalling (soluble lens proteins) and an increase in insoluble
lens proteins due to increased cross-linking between lens fiber cells. Cross-
linking results in the formation of lens protein aggregates that alter the
amount of H 2 O within the lens.
— Alpha crystallins decrease dramatically with age; by age 45, there
are NO alpha crystallins in the lens nucleus (26).
- Remember that alpha crystallins function as molecular chaper­
ones by preventing the degradation of other crystallins. A reduc­
tion in alpha crystallins results in an increase in degradation of lens
fiber cells and ultimately contributes to cataract formation.
• Lens thickness increases 0.02 mm per year. The lens diameter is relatively
stable after the teenage years (26).
• The anterior lens capsule thickness (produced by the anterior lens epithe­
lium) increases with age. The posterior lens capsule thickness is relatively
stable with age (26).

Remember that the lens capsule is thickest at the anterior mid­


peripheral portion of the lens (pre-equatorial region) and is thinnest
at the posterior pole. It is the thickest BM in the entire body and
is comprised of type IV collagen (29).

Copyright 2014 by KMK Educational Services, LLC


CHAPTER 2. OCULAR PHYSIOLOGY 99

* The radius of curvature of the anterior and posterior lens decreases with
age.
• The center of the lens moves anteriorly, causing a decrease in the anterior
chamber depth with age.
• Amino acid concentration decreases with age. Remember that the lens
usually contains a higher concentration of amino acids for protein syn­
thesis compared to the aqueous humor.
* Glutathione activity decreases. Na-f, Ca2+, and H^O concentrations in
the lens increase.

An increase in intracellular Ca2+, a decrease in glutathione, and


a decrease in crystallins are factors that significantly contribute to
cataract formation with aging.

• Nuclear fibers begin to lose their nucleus and organelles. They also ac­
cumulate a yellow-brown pigment that contributes to the formation of a
nuclear sclerotic cataract.
- Nuclear sclerosis begins in the embryonic nucleus and expands to
include the fetal and adult nuclei. It is the most common cataract
due to aging (26).

Remember that the embryonic nucleus (center of the lens) has


the highest refractive index (1.41) because it has the highest con­
centration of crystallins in the lens. Also recall that the Y sutures
seen during slit lamp biomicroscopy demarcate the boundaries of
the fetal nucleus.

We now overview the function and aging changes of the components of the
uvea (26).
Copyright 2014 by KMK Educational Services, LLC
100 2.6. UVEA
rasp o f th c:Tils
• Regulates pupil size to control the amount of light entering the eye
and to reduce spherical and chromatic aberrations.
- Pupil size helps to optimize retinal image quality by regulating the
amount of light that reaches the retina. For example, the pupil
size is larger under dim illumination to increase the amount of light
reaching the retina.
- Smaller pupils reduce spherical and chromatic aberrations and in­
crease the depth of field.

• Increase in pigment deposition on the anterior structures of the eye from


the posterior pigmented iris epithelium.
• The pupil becomes more resistant to dilation due to aging changes in the
iris sphincter and dilator muscles.

• Contains the ciliary muscle for accommodation.


• Produces and secretes aqueous humor (see section 8.14).
• Contains the ciliary stroma that contributes to aqueous humor drainage
via the uveoscleral meshwork.

^h^ng^s o f the Giiiary


• Aqueous humor formation decreases with age.
• Ciliary muscle contraction DOES NOT decrease with age (26) (27). Loss
of accommodation with age (presbyopia) is secondary to changes in the
lens.

o f the C ^orpid;
• Provides nutrients to the outer layers of the retina and contains pigment
that absorbs excess light that passes through the RPE.•
• Contains the suprachoroidal space that serves as a passageway for nerves
and arteries from their posterior insertion sites to the anterior segment
of the eye.
Copyright 2014 by KMK Educational Services, LLC
CHAPTER 2. OCULAR PHYSIOLOGY 101

• Contains a high protein content relative to the retina, establishing a pro­


tein gradient that promotes the absorption of excess H 2O from the retina
into the choroid.

Remington describes the following age-related changes in the choroid (26):


• Bruch’s membrane increases in thickness with age; drusen accumulates
on Bruch’s membrane with age.
• The choriocapillaris decreases in thickness with age.
• The overall choroidal thickness decreases with age.
SECTIO N 2.7

Vitreous

We start by summarizing the functions of the vitreous (18, pp. 306), (26).
• Provides a transparent, unhindered medium for the passage of light. Also
acts as a UV filter by decreasing the transmission of light at 300-350 nm
in order to protect the retina.
• Provides structure to the eye and likely cushions the globe (especially the
retina and lens) by absorbing vibrations and other external forces during
eye movements and trauma.
• Serves as a storage area for ions and nutrients for the retina and lens,
including O2 , H 2O , Na-f, K+, C1-, phosphate, glucose, and proteins.

The gel-like consistency of the vitreous decreases the bioavailability


of topical drugs entering the posterior segment.

Remember that the vitreous is approximately 4 mL in volume, making it about


80% of the total volume of the eye (5 mL).
Copyright 2014 by KMK Educational Services, LLC
102 2.7. VITREOUS
• The vitreous weighs approximately 4 grams and is composed of approxi­
mately 99% H 2 O combined with Type II collagen fibrils and hyaluronic
acid (GAG) molecules that create a gel-like consistency (18).
• Hyaluronic acid is a non-sulfated GAG in the vitreous that provides
support to collagen libers, helps maintain proper collagen fibril spacing,
and maintains the viscosity of the vitreous.
• Vitamin C concentration is higher in the vitreous compared to the blood
plasma. Amino acids are found in a lower concentration in the vitreous
compared to the blood plasma.•

Vitamin C (aka ascorbate) concentration in the vitreous is as


high as 40X greater than in the blood plasma. Vitamin C buffers
O2 as it travels from the retinal vessels towards the lens, which
helps to reduce oxidative stress and minimize tissue damage. It
also protects the retina from other metabolic and light-induced
free radicals (26) (18).

Summary of Physical Characteristics of the Vitreous


Visible light Vitreous transmits > 90%
UV light Transmission drops at 300-350
nm
IR light Transmission drops off at 800 nm
Volume 3.9 mL (80% of the globe)
Water content 99%
n 1.3345-1.3348 (similar to aque­
ous)
Remember the following values in regards to UV radiation:
• Wavelengths below 300 nm are absorbed by the cornea.
• Wavelengths between 300 and 400 nm are absorbed by the lens.
• Wavelengths above 400 nm are transmitted to the retina.

The lens absorbs the majority of UV-A and UV-B light, which
protects the retina from UV damage (14).

Copyright 2014 by K'MK Educational Services, LLC


CHAPTER 2. OCULAR PHYSIOLOGY 103

We now summarize metabolic functions of the vitreous (18, pp.311):


• Minimal metabolic activity occurs in the vitreous. Instead, the vitreous
acts as a metabolic buffer and storage area for the retina and lens.
• As an example, glucose and glycogen stored in the vitreous can be used
to supplement metabolic activities in the retina during anoxic condi­
tions (18, pp.311).

The gel structure of the vitreous becomes more liquefied with age. The follow­
ing is a summary of the aging changes that result in vitreous liquefaction:
• With increased age, the liquid portion of the vitreous increases as H 2O
starts to collect into pockets in a process known as liquefaction.
• These water pockets cause the vitreous gel to break down, resulting in
the aggregation of collagen fibrils (i.e. condensation) into floaters.

The processes of vitreous liquefaction and condensation are col­


lectively referred to as vitreous syneresis. Oxidative damage is
thought to induce structural changes in hyaluronic acid, causing
changes in the hyaluronic acid-collagen network that promote vit­
reous syneresis (13).

Vitreous syneresis is the most common cause of a posterior vitre­


ous detachment (PVD); risk factors for PVDs include age, my­
opia, diabetes, intraocular surgery, intraocular inflammation, and
trauma.

. • Remember that collagen concentration is highest at the vitreous base


and is lowest in the center of the vitreous. Loss of structure is thus
most likely to occur within the center of the vitreous.*
* The concentration of hyaluronic acid remains stable from approximately
age 20-50. After age 50, hyaluronic acid concentration increases (26) (13).
Copyright 2014 by KMK Educational Services, LLC
104 2.8. CIRCULATION
SECTION 2.8

Circulation

Blood flow and pressure differences in the eye play a pivotal role in maintaining
normal ocular function. Current research is focused on this area because dis­
ruptions in normal blood flow have been found to be related to several of the
core ocular disease processes, including diabetes, macular degeneration, and
glaucoma (18, pp. 757). The following equation demonstrates the components
that influence blood flow through the vessels:
_ P arteries (entering a tissue) - P veins (leaving the tissue) ( 2. 2)
R (resistance)
where F = Flow, P = Pressure, and R = Resistance.
Mean arterial pressure of the arteries entering the eye is around 65 mmHg.
The pressure in the episcleral veins leaving the eye is around 15 mmHg.
Perfusion pressure indicates how easily blood can pass through a given
tissue and is the difference between the pressure of blood flow entering and
leaving the eye. Perfusion pressure in the eye is approximately 50 mmHg.
Ocular perfusion pressure (OPP) = diastolic blood pressure - IOP. Glau­
coma patients with low OPPs are 1.5X more likely to develop progressive optic
neuropathy secondary to ischemia (31).
• If IOP decreases, OPP increases. Conversely if IOP increases, OPP
decreases.
• If diastolic BP decreases, OPP decreases.

Autoregulation is the process by which blood vessels alter their diameter


(in the absence of neural control) in order to increase or decrease resistance
to blood flow. Pericytes are most likely responsible for autoregulation within
the blood supply of the retina and the optic nerve (2),•
• Remember that a smaller blood vessel diameter offers more resistance,
resulting in a decrease in blood flow. Retinal arteries are smaller than
choroidal arteries and thus offer more resistance.

Copyright 2014 by KMK Educational Services, LLC


CHAPTER 2. OCULAR PHYSIOLOGY 105

Autoregulation allows blood flow to be maintained at a constant


rate despite moderate variations in the mean arterial pressure and
the intraocular pressure.

Transmural pressure describes the pressure across the blood vessel wall and
is determined by subtracting the pressure outside the vessel from the pressure
inside the vessel.

Critical closing pressure is the pressure at which the blood vessel collapses
and blood flow stops.

Elevated IOP in acute angle-closure causes a reduction in blood


flow through the central retinal artery, leading to a decrease in the
perfusion pressure of the retinal tissue. The retinal vessels detect
this change in transmural pressure and increase their vessel diame­
ter through autoregulation to improve perfusion. However, if IOP
remains acutely elevated long enough, the central retinal artery will
reach its critical closing pressure, resulting in a CRAO. Remem­
ber that the immediate threat to vision in acute angle closure is a
CRAO!

Sympathetic Innervation
• Sympathetic fibers are prevalent throughout the uveal tract but they DO
NOT innervate the central retinal artery past the lamina cribrosa and
therefore do not influence retinal blood flow (18).
* Sympathetic innervation causes vasoconstriction of uveal blood vessels.
It is important in maintaining adequate blood flow through the uvea
during sudden increases in blood pressure.
- A sudden spike in blood pressure increases the force of blood flow
through the small vessels of the uvea. The sympathetic system re­
sponds by causing constriction of the blood vessels, resulting in a
compensatory reduction in blood flow.
Copyright 2014 by KMK Educational Services, LLC
106 2.8. CIRCULATION
Parasympathetic Innervation
• Parasympathetic fibers from the oculomotor and facial nerves are also
prevalent throughout the uveal tract. Parasympathetic innervation is
most prominent in the anterior uvea and has a minimal influence on
choroidal and retinal blood flow.
• Parasympathetic innervation causes vasodilation of the uveal blood ves­
sels in response to sudden decreases in blood pressure.

Unlike the choroid, retinal vasculature is NOT under autonomic


control.

The following unique characteristics of the eye allow for appropriate structure
and function:
• IOP must be greater than the episcleral venous pressure so aqueous
humor can drain from the anterior chamber, through the corneoscleral
meshwork, and into the venous system.
• IOP must be greater than the intracranial pressure in order to maintain
an axoplasmic (i.e. pressure) gradient that flows from the optic nerve
towards the brain.

Papilledema results from a reversal in the axoplasmic gradient


between the eye and the brain due to an increase in intracranial
pressure. This reversal causes cerebrospinal fluid to spill from the
subarachnoid space onto the optic disc margins and the surrounding
RNFL.

• IOP must be lower than the pressure in the retinal and uveal arteries,
allowing nutrients to be delivered from the chorio capillaris to the RPE
cells.•
• Protein content must be highest in the choroidal vasculature so excess
water is pulled from the retina, across the RPE, and into the choroid,
which promotes the adherence between the RPE and the neurosensory
retina.
Copyright 2014 by KMK Educational Services, LLC
CHAPTER 2. OCULAR PHYSIOLOGY 107

Choroid: The majority of blood flow in the ocular vessels occurs in the chori-
ocapillaris (approximately 60%).
• Huge fenestrations within the choroidal vessels allow nutrients to
easily diffuse out of the vessels and into the RPE and the outer 5
layers of the retina.

Remember that the primary responsibility of the choroid is to pro­


vide the outer retina with nutrients such as oxygen, glucose, and
vitamin A (non-exhaustive list).

Ciliary Body: Contains the Major Arterial Circle of the Iris that is formed
by anastomoses between the anterior ciliary arteries (ACAs) and the long
posterior ciliary arteries (LPCAs).
• Major arterial circle of the iris —>- ACAs + LPCAs.
• Remember, the ciliary body and choroidal capillaries are fen­
estrated.
Iris: Contains the Minor Arterial Circle of the Iris that is formed by anas­
tomoses of the iris radial vessels. Blood flows from the major circle —>
minor circle -4 pupillary margin, and then back again.
• Remember, the iris and retinal capillaries are non-fenestrated
and contribute to the blood-aqueous and blood-retinal barriers, re­
spectively.

The retina has a dual blood supply. The inner two-thirds of the retina is
supplied by the central retinal artery, while the outer one-third is nourished by
the choroid. Remember that the outer plexiform layer is a watershed area
that is supplied by both the CRA and the choroid.
• The central retinal artery forms two networks of capillaries within the
inner retinal layers. The superficial capillary network is located in the
RNFL, and the deep capillary network is located in the INL.•
• The retinal capillary networks become very dense around the fovea, but
remember that the center of the fovea is avascular! The central fovea
obtains its blood supply from the underlying choriocapillaris.
Copyright 2014 by KMK Educational Services, LLC
108 8.9. RETINA
• The extreme anterior edges of the peripheral retina (approximately 0,5
mm from the ora serrata) are also avascular.

Blood Retinal Barrier


Because blood is toxic to the retina, it is essential that blood perfusing through
the retinal vessels and choriocapillaris is kept isolated from the retinal tissue.
The blood retinal barrier is formed by tight junctions in two locations:
1 Between endothelial cells lining the retinal vessels.
2 Between RPE cells.

Diabetic retinopathy results from damage to the blood reti­


nal barrier. Increased blood glucose levels damage pericytes and
basement membranes of the retinal capillaries, allowing blood and
plasma to leak into the surrounding retinal tissue. The most im­
portant risk factor for the development of retinopathy in a patient
with insulin-dependent diabetes is the duration of diabetes.

SECTION 2.9

R etina

The outer segment contains stacks of discs that enclose photopigments (rhodopsin
and iodopsins). In rods, rhodopsin is embedded within the disc membranes.
In cones, iodopsins are stored in invaginations of the plasma membrane (NOT
within discs). Please see ocular anatomy for further details.

Photopigments are produced in the photoreceptor inner segment and then


travel through the cilium to the outer segment. The discs and plasma mem­
branes that enclose the photopigments are produced in the outer segment.
Copyright 2014 by KMK Educational Services, LLC
CHAPTER 2. OCULAR PHYSIOLOGY 109

teiiliSiMiss
The three cone photopigments (cyanolabe, erythrolabe, and chlorolabe) are
used for photopic vision and color vision. Rhodopsin is the photopigment
found in rods that is used for scotopic vision. Cone and rod visual pigments
contain the same basic structure:
• Opsin: Membrane apoprotein (allows for varied absorption spectrum).
• Chromophore: ll-cls retinal (Vitamin A derivative).

Vitamin A gains access to the RPE via diffusion through the large
fenestrations in the choriocapillaris. Vitamin A is an alcohol retinol
that is oxidized in the RPE to form 11-cis retinal (18, pp. 373).

I
Light absorption in the outer segment damages the photoreceptors and renders
them in need of constant replacement.
* Rod outer segments are shed in the morning via phagocytosis by the
RPE.
* Cone outer segments are shed and renewed in the evening.

The transformation of retinoids due to light exposure and their subsequent


movement between the retina and RPE is called the visual cycle (18, pp.350).
* Light absorption by a photoreceptor results in the transformation of 11-
cis retinal to all-trans retinal.
• All-trans retinal moves from the disc lumen into the cytoplasm where it
is reduced to all-trans retinol.
* All-trans retinol is transported to the RPE cells where it is converted to
11-cis retinol. 11-cis retinol is then oxidized to 11-cis retinal.•
• 11-cis retinal is shuttled back to the photoreceptors for incorporation into
the photopigments in the disc outer segments (18, pp. 350).
Copyright 2014 by KMK Educational Services, LLC
110 2.9. RETINA
) v;■ Elcct.ropliysio
' ■■ :■ . : ■.■
1og:^ t \ . ■:.■*,■

We now introduce the process of phototransduction (18, ch. 13).


• Photoreceptors maintain a slight negative electrical charge of about -50
mV in the dark. Na+/K+ ATPase pumps on the inner segment plasma
membrane use ATP to pump Na+ out of the inner segment while moving
KT inside the inner segment.
• Na-f- then re-enters the inner segment through Na+ channels located in
the outer segment.
• This flow of cations into and out of the cell under dark conditions is
referred to as the dark current.

The membrane potential of a photoreceptor in the dark (-50 mV) is


more positive than normal cells (around -70 mV) due to the dark
current. Thus, photoreceptors are depolarized and constantly
release glutamate to bipolar cells (they are constantly turned “on”).
cGMP keeps NaT channels open to promote depolarization.

• The absorption of light by rhodopsin triggers phototransduction, a


biochemical cascade that converts absorbed light into an electrical signal.
During this process, the dissociation of rhodopsin triggers the activation
of the G protein transducin, leading to a cascade that ultimately results
in a decrease in the concentration of cGMP.*
* Decreased cGMP levels results in the closure of the sodium channels,
resulting in an increase in the negative charge of the cell membrane to -65
mV. This hyperpolarization of the cell membrane results in a decrease
in the release of glutamate to bipolar cells.

The closure of sodium channels is a key event during phototrans­


duction and effectively “shuts off” the photoreceptors by decreas­
ing their release of glutamate.

Note that photoreceptors do not produce action potentials but


rather produce graded potentials.

Copyright 2014 by KMK Educational Services, LLC


CHAPTER 2. OCULAR PHYSIOLOGY 111

S il
■ . ■ ;

• Glutamate: An excitatory neurotransmitter released by all rods, cones,


bipolar cells, and most ganglion cells in the vertebrate retina.
• GABA and Glycine: Inhibitory neurotransmitters that are released
by horizontal cells and most amacrine cells (18, pp. 397).

Bipolar cells have center-surround receptive fields (i.e. spatial antagonism).


• Cone cells can hyperpolarize or depolarize the center of bipolar cells.
Thus there are two types of cone bipolar cells that are classified
by their response to light: ON-center depolarizing or OFF-center
hyperpolarizing bipolar cells.
— ON-center bipolar cells are inhibited by glutamate and are
thus hyperpolarized in the dark. When light is present, less
glutamate is released, resulting in depolarization of the ON-
center bipolar cell (26) (18).
— OFF-center bipolar cells are excited by glutamate and are thus
depolarized in the dark. When light is present, less glutamate
is released, resulting in hyperpolarization of the OFF-center
bipolar cell (26) (18).
• Rod bipolar cells always depolarize in response to light.

Regardless of their response, bipolar cells respond to glutamate


with graded potentials (18, pp. 342).

Horizontal cells receive input from a large number of photoreceptors. They


DO NOT have center/surround receptive fields.•
• Horizontal cells respond with graded potentials and hyperpolarize
in response to light.
• Horizontal cells impact the surround responses of bipolar cells
by providing inhibitory feedback to photoreceptor cells (which then
impacts the bipolar cell), or by directly synapsing with the bipolar
cell (feed-forward synapse) (18, pp.428).
• Horizontal cells provide lateral inhibition, which helps to fine-tune
the neural signal sent from neighboring photoreceptors.

Copyright 2014 by KMK Educational Services, LLC


112 2.9. RETINA

Remember, photoreceptors and horizontal cells are unique because


they hyperpolarize in response to light.

Amacrine cells have center/surround receptive fields. They respond with


action potentials and always depolarize in response to light.
• Amacrine cells fine-tune the signal between bipolar and ganglion
cells (similar to the horizontal cells at the level of the photorecep­
tors).
Ganglion cells have center/surround receptive fields and respond to bipolar
cells with action potentials. They are classified into two types based
on their response to light: ON-center/OFF-surround or OFF-center/ON-
surround ganglion cells.
• ON-center/OFF-surround ganglion cells synapse with ON-center bipo­
lar cells and depolarize in response to light.
• OFF-center/OFF-surround ganglion cells synapse with OFF-center
bipolar cells and hyperpolarize in response to light.
Remember that midget ganglion cells are small ganglion cells that
have a single dendrite that synapses with one midget bipolar cell, which
synapses with a single cone in the fovea, allowing for the resolution of
very fine details (18, pp. 339).

To summarize:
1 Action potentials - All or nothing response.
2 Graded potentials - Response is influenced by the number
of photons absorbed (not an all or nothing response) (18).
3 Amacrine and ganglion cells respond with action po­
tentials; all other retinal cells respond with graded poten­
tials.

Remington overviews aging changes of the retina (26):


Copyright 2014 by KMK Educational Services, LLC
CHAPTER B. OCULAR PHYSIOLOGY 113

• Retinal nerve fibers within the optic nerve decrease, resulting in an in­
crease in the diameter of the vertical cup,
• ILM thickens with age, causing the foveal reflex to become dimmer.
• Rod density decreases with age, although note that scotopic function
DOES NOT decline,
• The total number of RPE cells decreases significantly with age. Lipofus-
cin within RPE cells and drusen increases with age.
• Atrophy increases throughout the retina, including around the optic disc
(peripapillary atrophy}, throughout the posterior pole (as seen by a de­
crease in pigmentation in the RPE/choroid), and in the periphery (e.g.
pavingstone degeneration),
- SECTIO N 2.10 ---------------------------------------------------------------------------------------------------

N europhysiology

We introduce several major neural pathways, paying particular attention to


the points of midline crossover.

The pyramidal motor pathway (PMP) (24, pp. 377) (17, pp. 346) begins in
the motor cortex (located in the precentral gyrus) and plays a large role in
complicated voluntary movements (Figure 2.1).
• Pyramidal motor cell axons come together, forming the internal capsule
in the forebrain. These fibers then travel through the cerebral peduncles,
pons, and medulla and form the medulla pyramids.
—Note that fibers that innervate cranial nerves break away from this
path at certain regions of the middle pons and middle medulla; this
“break away” tract is called the corticobulbar tract.
• The major pathway continues until it reaches the pyramidal decussa­
tion in the caudal medulla, where most (85-90%) of the fibers cross to the
opposite side of the spinal column and become the lateral corticospinal
tract, which controls the proximal musculature (2 0 ).•
• The remaining fibers make up the anterior corticospinal tract and
eventually decussate at the level of the spinal cord. These fibers control
the distal musculature (2 0 ).
Copyright 2014 by KMK Educational Services, LLC
114 2.10, NEUROPHYSIOLOGY
Cerebrum

Internal
Location of cerebrum cross section
Capsule

Locations of Sections through Brainstem

Midbrain
Mid-Pons

Caudal Medulla

Caudal Medulla

Pyramidal Decussation (85 % cross)


Lateral Cortico­ Anterior Cortico­
spinal Tract spinal Tract

Cervical Spinal Cord

Alpha Motor Neuron

Figure 2.1: Pyramidal Motor Pathway. Drawing modified from SA Kinkel


original.

A lesion above the medulla will lead to problems with motor


control on the contralateral side..

The reticulospinal tracts are also involved in the control of complex volun­
tary movements, as well as the integration of sensory information to direct
motor control. These pathways offer an alternative to the pyramidal motor
pathway for muscle control (2 0 ).*
* Fibers originate from the reticular formation (diffuse collection of neu­
rons) within the pons and the medulla. They descend ipsilaterally and
eventually synapse with neurons at all levels of the spinal cord.
Copyright 2014 by KMI< Educational Services, LLC
CHAPTER 2. OCULAR PHYSIOLOGY 115

Although the exact functions of the tectospinal tract are unknown, it is thought
to play a role in reflexive head movements in response to visual stimuli (2 0 ).
• Fibers originate in the superior colliculus. They immediately cross the
midline and then descend through the pons and the medulla, traveling
anterior to the medial longitudinal fasciculus (MLF).
• Fibers eventually synapse at the cervical level of the spinal cord.

The cochlear and vestibular nerves combine to form the vestibulocochlear


nerve (CN V III), which carries information to the primary auditory cortex,
the cerebellum, and the spinal cord for hearing and balance (1 0 ) (1 1 ).
Cochlear nerve: Composed of fibers that originate from the spiral ganglion
of the cochlea. These fibers travel through the organ of Corti before
exiting via the internal meatus and ending at their cell bodies located in
the cochlear nuclei of the medulla (1 0 ).
• The second order neuron axons ascend on both sides (i.e. crossed
and uncrossed fibers) of the trapezoid body to the superior olivary
complex within the brainstem. This is the first location of bilateral
auditory input.
• Fibers from the superior olivary complex (third order neurons) form
the lemniscus pathway and eventually synapse in the inferior
colliculus of the midbrain and the medial geniculate body in the
thalamus (fourth order neureons) before traveling to the primary
auditory cortex.
Vestibular nerve: Composed of axons originating from the vestibular ganglia
at the distal end of the internal auditory meatus. These fibers join the
cochlear nerve of CN VIII and carry sensory information from the semi­
circular canals and otolith organs of the ear. Most of the fibers synapse
with 4 vestibular nuclei in the medulla and pons. The remaining fibers
directly project to the cerebellum via the inferior cerebellar peduncle to
control movements necessary for balance (1 1 ).•
• Primary ascending fibers from the superior and lateral vestibular
nuclei carry sensory information to the thalamus, which then sends
fibers to the primary vestibular cortex (exact location in the cere­
brum is unknown).
• Ascending fibers from the superior and medial vestibular nuclei
travel through the medial longitudinal fasciculus to the nuclei
of CN 3, 4, and 6 and help to coordinate head and eye movements.
Copyright 2014 by KMK Educational Services, LLC
116 2.10. NEUROPHYSIOLOGY

Cerebrum

vps

Location of cerebrum cross section

Spinothalamic
Tract Locations of Sections through Brainstem

Midbrafn
Mid-Pons

.Caudal Medulla

Synapse in
Substansia
Gelatinosa

Pain/Temp Info from


Upper Body (not face)

Pain/Temp Info from


Lower Body

Figure 2.2: Spinothalamic Pathway. Drawing modified from SA Kinkel original.

* Ascending fibers from the inferior and medial vestibular nuclei travel
to the cerebellum to help coordinate balance.
* Descending fibers from the lateral vestibular nuclei form the lateral
vestibulospinal pathway that travels along the ipsilateral spinal
cord and helps control movements that allow us to walk upright.*
* Descending fibers from the medial vestibular nuclei form the medial
vestibulospinal pathway that travels along either side to the tho­
racic segments of the spinal cord. This pathway helps to integrate
head movements with eye movements.

Copyright 2014 by KMK Educational Services, LLC


CHAPTER 2. OCULAR PHYSIOLOGY 117

Cerebrum

Location of cerebrum cross section

Midbrain

Locations of Sections through Brainstem


Mid-Pons

Midbrain
Paln/Temp Info
Mid-Pons
from face
.Caudal Medulla
Spinal tract
ofTrigeminal
Nerve

Nucleus of spinal
tract ofTrigeminal
Nerve

Figure 2.3: Trigeminothalamic Pathway. Drawing modified from SA Kinkel


original.

The spinothalamic pathway (17, pp. 482) (24, pp. 213) carries pain and tem ­
perature information from the body. Note that this overall pathway is
sometimes called the anterolateral system.
• Nerve endings in the periphery synapse at the substantia gelatinosa
within the dorsal horn of the spinal cord. Fibers that leave the substan­
tia gelatinosa cross the midline and become the lateral spinothalamic
pathway.•
• The fibers remain contralateral until they terminate in the ventral pos­
terior thalamus (VPL) (see Figure 2.2).

The trigeminothalamic pathway (TGP) (24, ch. 10) (17, ch. 23,24) carries pain
and tem perature information from the face. The pathway originates in
the trigeminal ganglion cells, as well as facial pain and temperature receptors
that extend into the brainstem at the level of the pons.
Copyright 2014 by KMK Educational Services, LLC
118 2.10, NEUROPHYSIOLOGY
• These axons descend into the medulla (forming a tract known as the
spinal tract of cranial nerve V), where they synapse onto second
order neurons in one of two sub-regions of the trigeminal complex of the
spinal cord.
• Axons from the neurons within the trigeminal complex then cross the
spinal column in the medulla and ascend contralaterally until they ter­
minate in the thalamus (see Figure 2.3).

A lesion to the trigeminothalamic pathway above the crossover


point will result in a loss of pain or temperature information from
the contralateral side of the face.

M edial Lemniscus Pathw ay


The medial lemniscus pathway (17, pp. 34) (24, pp. 200) carries information
about touch, pressure, and vibration.
• Peripheral information from mechanoreceptors in the upper body travels
along the cuneate tract (located more laterally), while information from
the lower body travels along the gracilis tract (located more medially).
• These tracts enter at the cervical and lumbar regions of the spinal cord,
respectively, and ascend to the cuneatus and gracilis nuclei in the caudal
medulla, respectively.•
• Axons from the secondary neurons in this region cross the midline at the
level of the medulla and become the internal arcuate fibers. These
libers continue to travel contralaterally until terminating in the VPL (see
Figure 2.4).

A lesion in the medial lemniscus pathway below the crossover


point affects the ipsilateral side, while a lesion above the
crossover point affects the contralateral side.

Copyright 2014 by KMK Educational Services, LLC


CHAPTER 2. OCULAR PHYSIOLOGY 119

Cerebrum

Location o f cerebrum cross section

Locations of Sections th ro u g h Brainstem

Cervical
Spinal Cord
M echanoreceptors
from upper body (not face)'

Lumbar
Spinal Cord
M echanoreceptors
from lower body

Figure 2,4: Medial Lemniscus Pathway. Drawing modified from SA Kinkel


original.

Remember that the autonomic nervous system (ANS) is composed of neurons


within the central and peripheral nervous systems that control input to the
visceral organs, secretory glands, and smooth muscle of the cardiovas­
cular, digestive, excretory, and thermoregulatory systems of the body. Input
from the ANS is NOT voluntary and helps to maintain homeostasis (5).•

• The ANS is composed of a sequence of two neurons between the CNS and
the target tissue. The first (pre-ganglionic) neuron is located within
the brainstem or spinal cord. The second (post-ganglionic) neuron is
located in the autonomic ganglia in the periphery (outside the CNS).
Copyright 2014 by KMK Educational Services, LLC
120 2.10. NEUROPHYSIOLOGY
The autonomic nervous system is separated into two divisions: the sympathetic
nervous system and the parasympathetic nervous system (5).
Sympathetic nervous system: Responsible for the “fight or flight” response.
It increases heart rate and blood pressure, dilates the bronchioles, causes
vasodilation within skeletal muscles, increases blood glucose levels, and
decreases GI motility and blood flow.
• Pre-ganglionic neurons are located in the thoracic and lumbar sec­
tions of the spinal cord in the lateral horn of the grey matter. Their
axons ascend the spinal cord to enter the sympathetic chain of gan­
glia located along the vertebral column.
• Fibers that carry information to the head and thorax regions synapse
within the ganglia of the sympathetic chain. Post-ganglionic fibers
then continue to travel up the spinal cord to their target tissue.
• Fibers carrying information to the pelvic and abdominal viscera pass
through the sympathetic chain WITHOUT synapsing. They travel
to the autonomic plexi that surround the large branches of the ab­
dominal aorta, where they eventually synapse. Post-ganglionic fibers
then travel a short distance from the autonomic ganglia to the target
tissue.
- Autonomic ganglia include the celiac, superior mesenteric, and
inferior mesenteric ganglia.
• Pre-ganglionic sympathetic fibers release acetylcholine. Post-ganglionic
sympathetic fibers release norepinephrine.

The adrenal gland is the ONLY gland that is innervated directly


by pre-ganglionic sympathetic fibers (5).

Parasympathetic nervous system: Responsible for the “rest and digest”


response. It decreases heart rate, constricts the bronchioles, increases
salivary and lacrimal gland secretions, increases GI motility, and causes
pupil constriction and accommodation (5).
• Pre-ganglionic neurons are located within the cranial nerve nuclei of
the brainstem, or in the 2nd-4th sacral segments of the spinal cord.
The brainstem parasympathetic fibers innervate structures of the
head, thorax, and abdomen. The sacral spinal cord parasympathetic
fibers innervate pelvic viscera.
• Post-ganglionic neurons are located within ganglia that are very
close or adjacent to their target tissue.
• Pre- AND post-ganglionic parasympathetic fibers release acetyl­
choline.
Copyright 2014 by KMK Educational Services, LLC
CHAPTER 2. OCULAR PHYSIOLOGY 121

5 . ; - . v: ' - v r: ■. o v . v .-Wm
Computed Tomography (CT): Scan of choice for analyzing bone and
calcification and for emergent situations (CT scan is faster than an MRI).
• Uses ionizing radiation to create approximately 3 mm thick cross-sectional
images of tissue in order to compare the calcium density of neighboring
tissues.
• As tissues undergo apoptosis, calcium enters cells and increases the den­
sity of the tissue. This denser tissue appears whiter on CT scans.

CT scans are often utilized in the diagnosis and management of


orbital fractures.

Positron Emission Tomography (PET): Scan that is used to analyze the


metabolic activity of tissues by comparing glucose uptake of neighboring
tissues.
• PET scans are often used in conjunction with CT scans to monitor metas­
tasis in cancer.

Magnetic Resonance Imaging (MRI): Scan of choice for analyzing de­


tailed pathology in soft tissue.
• The MRI machine uses a strong, static magnetic field of radiofrequency
energy to excite free protons (located mainly in water) to a higher energy
state.
• As the protons relax back to their baseline level, they give off energy that
is detected by antennae inside the MRI unit.
• The MRI unit then performs a series of complex mathematical computa­
tions to produce a detailed spacial map of the tissues in question.•
• As a general rule, diseased tissue has higher water content than healthy
tissue and will thus have more free protons.

MRI contraindications include severe claustrophobia and magnet­


ically activated implanted devices (e.g, pacemakers, defibrillators,
cochlear implants).

Copyright 2014 by KMK Educational Services, LLC


122 2.11. VISUAL PATHWAY
l - SECTION 2.11

Visual Pathw ay

TYWi
The LGN is located on the dorsolateral aspect of the thalamus. The following
is noteworthy information regarding the LGN:
• The main purpose of the LGN is to process visual information from the
retina before relaying only the most relevant information to the visual
cortex (18, ch. 29). Thus, the LGN helps to regulate the strength of the
visual signal sent to the primary visual cortex.
• The axons of the retinal ganglion cells terminate in the LGN and are
thought to be the “drivers” for LGN output (18).
• The LGN also receives input from the superior colliculus and feedback
from the visual cortex regarding the visual signal. These inputs are be­
lieved to be “modulators” of LGN output (18).
• Axons that leave the LGN are called the optic radiations.

The LGN is not just a simple relay station, but rather a center for
processing input from multiple sources to allow filtration of only
the most relevant information to VI.

• There is an LGN located on the left and right sides of the thalamus.
Remember that each LGN contains six layers:
- Magnocellular layers: Layers 1 and 2 on the ventral side (i.e.
bottom) of the LGN.
- Parvocellular layers: Layers 3, 4, 5, and 6 on the dorsal side (i.e.
top) of the LGN.
- Koniocellular layers: Located between each of the 6 layers through­
out the LGN.•
• Each layer receives input from only one eye; the type of input is depen­
dent on the location of the object in the visual field. This organization
allows fibers from each eye that carry information from the same parts of
the visual field to lie adjacent to one another in the LGN (i.e, retinotopic
mapping) (26).
Copyright 2014 by KMK Educational Services, LLC
CHAPTER 2. OCULAR PHYSIOLOGY 123

—As an example, when an object is located in the right hemifield of


each eye, the layers of the LGN on the left side of the brain will
respond in the following manner:
* Layers 1,4, and 6 receive fibers from the contralateral (right
eye) nasal retina.
* Layers 2, 3, and 5 receive fibers from the xpsilateral (left eye)
temporal retina,
• Traveling medial lateral in the LGN corresponds to moving from the
fovea -P peripheral visual field.
• Traveling from anterior ~¥ posterior LGN corresponds to moving from
the inferior —>superior visual field.
• Traveling from dorsal —» ventral LGN corresponds to the SAME spot in
the visual field, but the eye providing input differs with each layer (26).

Binocular visual processing does NOT occur at the LGN level


- the LGN neurons are still monocular. The visual cortex (VI) is
the first location along the visual pathway to combine monocular
input for binocular processing.

Receptive Fields of the LGN


Magno and parvo cells of the LGN have center-surround receptive fields,
similar to the bipolar and ganglion cells of the retina. Magno and parvo cells
differ in their response to illumination, color, contrast, and motion (28):
• Parvo cells are most sensitive to red-green, fine details (high spatial fre­
quencies), and slow motion (low temporal frequencies) and have a slower
speed of transmission of visual signals.
• Magno cells are monochromatic and are most sensitive to fast move­
ments (high temporal frequencies) and large details (low spatial frequen­
cies). They have a higher speed of transmission of visual signals due to
larger axons compared to the parvo cells.•
• Konio cells respond to blue-yellow contrast.
Copyright 20.14 by KMK Educational Services, LLC
124 2.11. VISUAL PATHWAY

..¥--.v
£M .

Also called the striate cortex, Broadmarm Area 17 or VI. The primary visual
cortex begins on the outer surface-of the occipital lobe and extends anteriorly
along the medial surface of the lobe to the parieto-occipital sulcus. It contains
six layers (similar to the LGN), with each layer containing two maps (one for
each eye) of the opposite visual hemifield (unlike the LGN). Remember the
following points regarding VI:
• The activity of VI cells depends on the input from the LGN via the
optic radiations (the “drivers” of VI activity); VI also receives input
from several cortical areas (18, pp. 684).
• VI is the first location in the visual pathway that combines monocular
input for binocular processing and evaluation of binocular disparity.
• VI is the first location in the visual pathway that begins evaluating visual
input based on the size, orientation, and direction of movement of the
stimulus. It also discriminates the shape and texture of objects.
• Layer 4 receives the primary visual input from the LGN. Remember
that cells are organized into ocular dominance columns that respond
to visual input from one eye only. Ocular dominance columns are fur­
ther organized into hypercolumns, which combine an ocular dominance
column from one eye with an orientation column (contains cells that re­
spond to a specific stimulus orientation).
• Layer 3 sends axons to other cortical layers.
• Layers 5 and 6 send axons to subcortical areas (e.g. superior collicu­
lus, thalamus, midbrain, pons). Remember that layer 6 provides direct
feedback to the LGN, allowing VI to regulate its own input (18, pp. 674).

VI examines basic stimulus features before relaying information


to more complex processing centers known as the extrastriate
cortex (V2-V5) for further processing.

V2-V5: Located on the lateral aspect of the occipital cortex. These areas are
responsible for complex processing of visual information. Visual input
ultimately travels to two locations within the extrastriate cortex:•
• Inferotemporal (IT) cortex: Allows for identification of the ob­
ject (“what”).
Copyright 2014 by KMK Educational Services, LLC
CHAPTER 2. OCULAR PHYSIOLOGY 125

• Middle tem poral (MT) cortex: Allows for identification of the


spatial relationship of the object to its surroundings (“where”) (18,
pp. 669).
Superior Colliculus (SC): Receives information from VI and from fibers
that exit the posterior optic tract prior to reaching the LGN. The SC
controls saccades, visual orientation, and foveation. It does NOT analyze
visual input for perception (26).
Frontal Eye Fields: Receives information only from VI. The frontal eye
fields are located in the frontal lobe and have two primary functions:
1 Pupillary response to near objects.
2 Activates during initiation of voluntary and reflex eye movements.

Voluntary saccades are initiated by input from the frontal eye


fields and the superior colliculus.

We now summarize receptive field properties of cells of the visual cortex (18,
pp. 674-78). Remember that the “average” visual scene that elicits a response
in a cell is its receptive field. Cortical neurons have more complex receptive
fields than lower level cells (retinal ganglion cells and LGN cells), and are thus
tuned to respond to edges and specific orientations of stimuli. The following
cells are found in the visual cortex:
• Simple cells have elongated center-surround receptive fields that re­
spond to the orientation of stimuli and can detect complex structures
including bars and edges. Their receptive fields are likely a product of
the combined input of multiple circular center-surround RFs of LGN cells.
• Complex cells process higher levels of perceptual detail and respond
to the motion and orientation of visual stimuli. Their receptive fields do
NOT have a center-surround orientation and are a product of combined
input from many simple cortical cells.•
• Hypercomplex cells process combined input from multiple complex
cells. They can respond to line stimuli of a specific length in addition to
orientation.

Copyright 2014 by KMK Educational Services, LLC


126 2,11. VISUAL PATHWAY

In summary, the visual system processes images in a hierarchi­


cal fashion, with neurons responding to more basic stimuli feeding
information to higher order neurons that respond to increasingly
complex images.

Visual input from the fovea makes up a large percentage of the


visual cortex. This phenomenon, known as the cortical magni­
fication of the fovea, allows us to identify small central objects
and fine details more easily.

Electrooculogram (EOG)
The EOG measures the difference in electrical charge between the front and
back of the eye.
• The EOG analyzes the health of the RPE by examining differences in
electrical potentials that are generated as patients perform eye movements
under dark adapted and light adapted conditions (18, ch. 14).
• Electrodes are attached near the inner and outer canthus of the eye. The
patient is instructed to make a series of right and left movements and the
electrical potential is recorded over a period of about 30 minutes.
• The electrical potential is lowest after about 8 minutes of dark adaptation
(dark trough), and is highest after about 1 0 minutes of light adaptation
(light rise).
The ratio of light peak/dark trough is the Arden ratio and provides an indi­
cation of the health of the RPE:
Arden ratio = light rise/dark trough (2.3)

An Arden ratio greater than 1 . 8 is considered normal. 1.65-1.80 is


considered subnormal, and less than 1.65 is considered very abnor­
mal (28).

Copyright 2014 by KMK Educational Services, LLC


CHAPTER 2. OCULAR PHYSIOLOGY 127

The EOG is not clinically useful in differentiating between diseases that affect
the RPE but it may be helpful in diagnosing Best’s disease, Stargardt’s
disease, advanced drusen, and patterned RPE anomalies.
Electroretinogram (ERG)
The ERG records graded potentials produced within the retina in response to
light.
• The ERG represents the activity of the outer retinal layers (pho­
toreceptor cells and bipolar cells); it does not include the ganglion cell
layer (18, pp.410).
* Prior to performing an ERG, the patient is maximally dilated and dark
adapted for about 45 minutes. The retina is then flooded with various
rates, wavelengths and intensities of light stimuli (18, pp. 411).
• The patient is tested under dark-adapted and light-adapted conditions,
allowing for the isolation of cone and rod function for analysis.
- Rod function is isolated by using a blue flash with a slow flicker in
a dim background.
- Cone function is isolated by using a red flash with a fast flicker in a
bright background.
* The ERG response is composed of three waves:
- A-wave: A negative wave that represents photoreceptor activity,
- B-wave: A positive wave that represents activity of bipolar and
Muller cells.
- C-wave: A positive wave that represents RPE cells. The C-wave is
rarely evaluated clinically; an EOG is the test of choice to analyze
RPE function.*

An electronegative ERG is characterized by loss of the b-wave.

* Remember that the overall proportion of rods to cones within the retina
is 13:1. Thus, rods contribute approximately 75% and cones contribute
approximately 25% to the amplitude of the b-wave under dark adapted
conditions (18, pp.4'13).

Copyright 2014 by KMK Educational Services, LLC


128 2.11. VISUAL PATHWAY

• Pattern ERGs target the ganglion cells by using a complex


stimulus rather than a simple flash of light.
• Multifocal ERGs record responses at multiple locations
within the retina, allowing for localization of retinal disease.
• Serial ERGs can be used to track intraocular foreign bodies
that cannot be removed (7).

Retinitis pigmentosa (RP) is characterized by vessel attenua­


tion, bone-spicule pigmentation, and waxy optic disc pallor. In
early cases of RP, only the scotopic (rod) ERG is abnormal. In
late stages of RP, the ERG is completely extinguished due to poor
function of the rods and cones.

Visual Evoked Potentials (VEP)


The VEP analyzes the electrical response (latency) of brain activity to a visual
stimulus.
• Wires are placed on the area of the scalp overlying the primary visual
cortex within the occipital lobe. The patient sits in front of a screen that
displays an alternating checkboard pattern.
• The abrupt pattern differences of the alternating checkerboard produce
responses in the visual cortex within 100 milliseconds in adults (4).•
• A normal VEP graph contains a large positive wave that peaks at 90-110
msec after the initial stimulus presentation. Waves that peak after 110
msec are abnormal.
• The VEP can detect an anomaly between the fovea and VI, but it cannot
localize the defect!

Visual evoked potentials can be helpful in the diagnosis and


evaluation of conditions including optic neuritis, optic nerve tu­
mors, retinal disorders, and demyelinating diseases (e.g. multiple
sclerosis).

Copyright 2014 by KMK Educational Services, LLC


CHAPTER 2. OCULAR PHYSIOLOGY 129

SECTIO N 2.12

Pupillary Pathways
We now coyer the fundamentals of pupillary pathways (18, ch. 32) (26).

k-' •''.?V^hY (,1'r rjj


• Afferent pupillary fibers travel with the ganglion cell fibers until the pos­
terior 1/3 of the optic tract, when the pupillary fibers exit and travel
within the brachium of the superior colliculus to synapse at the pretec­
tal nucleus in the midbrain.
• The fibers then project from the pretectal nucleus to the ipsilateral and
contralateral Edinger Westphal (EW) nuclei, forming the tectotegmen-
tal tract.

Damage to the tectotegmental tract can lead to an Argyll-


Roberston pupil, which is characterized by light-near response
dissociation. Argyll-Robertson pupil is associated with neu­
rosyphilis.

* P re-ganglionic parasympathetic fibers leave each EW nucleus and travel


to the ciliary ganglion within the orbit.
• Post-ganglionic parasympathetic fibers project from the ciliary ganglion
to the iris sphincter and the ciliary muscles.•

Efferent parasympathetic fibers responsible for miosis and accom­


modation begin in the EW nucleus. Remember that anisocoria is
ALWAYS a result of an efferent pathology.

• The near reflex triad of convergence, accommodation, and pupillary


constriction occurs when fixation is shifted from a far to a near object.
Copyright 2014 by KMK Educational Services, LLC
130 2.13. INTRAOCULAR PRESSURE
• In this case, pupillary constriction is mediated by supranuclear input from
the frontal eye fields instead of the pretectal nucleus.
• The FEF activates the EW nucleus, which projects fibers to the ciliary
ganglion and then on to the sphincter muscle and ciliary muscle, similar
to the light response pathway.

The light and near pupillary responses both utilize the EW nu­
clei and the ciliary ganglion as the efferent pathway for pupillary
constriction.

The sympathetic nervous system actively inhibits the EW nuclei through supranu­
clear control.
• When uninhibited, EW neurons continuously fire action potentials to the
sphincter muscle for miosis.
• Sympathetic stimulation (e.g. during waking hours) results in supranu­
clear inhibition, causing a decrease in EW activity and normal pupil size.
• During sleep or anesthesia, supranuclear input is reduced, causing an
increase in EW activity with resulting miotic pupils (18, pp. 721).
SECTION 2.13

Intraocular Pressure •

Goldmann Applanation Tonometry


* The probe is designed with a precise size (diameter of 3.06 mm) and
weight in order to minimize potential error from tear film surface tension
and corneal elasticity. The probe is used to gently flatten (i.e. applanate)
the cornea to obtain a measurement of intraocular pressure (IOP) in
mmHg.
• Goldmann tonometry is based on the Imbert-Fick Law, which states
that the pressure inside an infinitely thin, dry sphere covered by a thin
membrane is equal to the force necessary to just flatten that sphere. It
assumes that the force from the surface tension of the tear film cancels
the opposing elasticity of the cornea.
Copyright 2014 by KMK Educational Services, LLC
CHAPTER 2. OCULAR PHYSIOLOGY 131

• Goldmann’s method assumes that all corneas have the same average
thickness of approximately 520 um. This assumption causes us to over­
estimate IOP in thicker corneas and to underestimate IOP in thinner
corneas.

Current studies have shown an average corneal thickness of approx­


imately 555 um.

Noncontact Tonometry
• A form of indentation tonometry that utilizes an airstream of known
force to flatten a circular area of the cornea. The NCT machine contains
a photocell that reaches its optimal output when air returns from the
corneal surface.
• The amount of time between the initiation of the airstream and the peak
response of the photocell is converted to mmHg (30, pp. 111).
• IOP measurements are variable and less predictable compared to Gold-
mann applanation tonometry.
PASCAL Tonometry
• Also known as dynamic contour tonometry. The tonometry tip is con­
toured and resembles the shape of the cornea when pressure on both sides
of the probe is equal.
• The contoured tip helps to minimize the effect of the unique character­
istics of the patient’s cornea (e.g. corneal thickness) on the intraocular
pressure measurement (31).

Average IOP anc Variation


Average IOP 15.5 mmHg
Two Standard Deviations 21 mmHg / (97.5%)
Three Standard Deviations 22 mmHg / (99.9%)
The statistical curve on these measurements is skewed to the right, meaning
that more people have an IOP > 2 2 mmHg than the above numbers indi­
cate (30, pp.97).
Copyright 2014 by KMK Educational Services, LLC
132 2.13. INTRAOCULAR PRESSURE
Diurnal variation
• IOP varies throughout the course of the day. Studies have shown that
10P is highest during nocturnal hours (1200-600 AM) with the peak IOP
commonly occurring between 330-530 AM.
• 24 hour variations of 2-5 mmHg in TOP are common in the normal pop­
ulation (29).
• Glaucoma patients can have pressure differences of 10 mmHg or more
throughout the course of the day.

Agents that decrease aqueous production include:


• B-blockers
• Alpha.-2 agonists
• Carbonic anhydrase inhibitors
• Cardiac glycosides
• Hyperosmotic agents
• Significant decline in blood pressure (although minimal effects)
• Uveitis (the inflammed, sick ciliary body produces less aqueous)

In order to maintain the pressure gradient between the posterior and anterior
chambers that is necessary for aqueous flow, the amount of aqueous that enters
the posterior chamber must be equal to the amount of aqueous that leaves
the anterior chamber. This is further complicated by the resistance to outflow
inherent in the conventional corneoscleral outflow pathway. Recall that aqueous
leaves the anterior chamber through two different routes:
Corneoscleral outflow
Drains 2.25 uL/min (80% of aqueous outflow).
• Aqueous flows from the anterior chamber across the trabecular meshwork
into Schlemm’s canal. The episcleral veins drain aqueous from Schlemm’s
canal.
Copyright 2014 by KMK Educational Services, LLC
CHAPTER 2. OCULAR PHYSIOLOGY 133

• The rate of drainage in this pathway is pressure dependent. In gen­


eral, as IOP increases, aqueous drainage increases. However, if IOP is
acutely elevated, Schlemm’s canal may collapse on itself, preventing en­
try of aqueous humor into the venous system.
Uveoscleral outflow
Drains only 0.25 uL/min (2 0 % of aqueous outflow).
• Aqueous drains through the ciliary stroma into the surrounding vessels
of the venous system.
• The rate of aqueous outflow is independent of IOP.
Summary Equation for Aqueous Outflow
• The following equation, described in Adler’s Text (IS, pp. 249) and with
the variables manipulated and renamed by this author, can be used to
summarize the key points regarding aqueous outflow.
F out = Corneoscleral (IOP - EVP) T Uveoscleral (2.4)
- F out = Aqueous outflow
- EVP = Episcleral venous pressure
• Recall that the aqueous produced in the healthy eye must be equal to the
amount of aqueous that is drained from the eye.
- The total amount of aqueous drainage is a combination of outflow
through the corneoscleral meshwork (pressure dependent) and the
uveoscleral meshwork (pressure independent).•
• The total amount of aqueous outflow is about 2.5 uL/min. The total
volume of aqueous humor is 250 uL; thus, the total volume of aqueous
humor fluid is replaced every 1 0 0 minutes (18).

As can be seen from the above equation, an increase in episcle­


ral venous pressure will increase IOP. Wearing a necktie can
compress the external jugular vein, which leads to an increase in
EVP and a reflex increase in IOP. An acute rise in EVP will result
in a 1:1 ratio of increased IOP (e.g. increase in EVP by 5 mmllg
will cause an increase in IOP by 5 mmHg) (29). Sturge-W eber
syndrome and arteriovenous fistulas are two conditions that
can increase EVP (29).

Copyright 2014 by KMK Educational Services, LLC


134 2.11 AQUEOUS
y■ :,:-yy‘ * Li
• Body position - IOP is highest in the supine (lying on back) position.
• Corneal thickness - thicker corneas cause artificially high readings, while
thinner corneas cause artificially low readings.
• Blood pressure - no consistent effect on IOP.
• Prolonged exercise - can decrease IOP.
• Blinking/squeezing the eyes/straining - can increase IOP.
• Caffeine - can occasionally cause a transient rise in IOP (29).
i - SECTION 2.14

Aqueous

We now summarize the roles of the aqueous humor as described in the following
sources (18, pp.237) (26):
• Maintains the pressure and shape of the eye and provides a transparent,
colorless refractive index to enhance the overall optics of the globe.
• Provides nutrition for the avascular cornea, lens, anterior vitreous, and
TM.
• Serves as a collection bin for metabolic waste products of surrounding
tissues and clears out inflammatory products and blood from the globe.
A fa lu rcie ^O sta q ^

# Volume: 250 uL that is completely replaced around every two hours (9).
Recall that 2.5 uL of aqueous is produced and 2.5 uL of aqueous is drained
every minute in healthy eyes.
• Osmolarity: Slightly hyperosmotic to plasma.
• Viscosity: 1.025-1.040 relative to water.
PS
Recall that aqueous humor is produced and secreted by the non-pigmented
ciliary epithelium of the ciliary processes. Aqueous production involves the
processes of diffusion, ultrafiltration, and active secretion (18, ch. 8 ).
Copyright 2014 by KMK Educational Services, LLC
CHAPTER 2. OCULAR PHYSIOLOGY 135

1. Diffusion
Involves the passive movement of ions across a membrane based on ion size and
solubility. Small lipid soluble substances are able to easily diffuse out of the
fenestarted capillaries of the ciliary body vasculature into the ciliary stroma.
Diffusion plays a minimal role in aqueous humor production.
2, Ultraflit rat ion
Involves the passive flow of blood plasma from the capillaries into the ciliary
stroma and is caused by an increase in hydrostatic pressure (pressure from the
heart) compared to pressure within the surrounding tissue.

Although substances can leave the blood through diffusion and ul-
trafiltration, most substances must be actively secreted across the
non-pigmented ciliary epithelium in order to produce aqueous hu­
mor. Active secretion of ions across the ciliary processes into the
posterior chamber creates a gradient where the aqueous is hyper­
tonic to the blood by approximately 5 mOsm (18).

3. Active Secretion
Involves the active transport of large, water-soluble, charged substances across
the non-pigmented epithelial cell membranes against an electrochemical gradi­
ent; requires ATP.
• Active secretion accounts for 80-90% of total aqueous humor formation,
providing further evidence that alterations in blood pressure have little
effect on aqueous humor formation (26).
• The pigmented ciliary epithelium and non-pigmented ciliary epithelium
have several ion transport mechanisms that are essential for aqueous for­
mation. A detailed overview can be found in Remington’s text (26). The
following is an over-simplified summary of key parts of active secretion:
- Na-b/K-f- ATPase pump: Located within the NPCE cell. It uti­
lizes ATP to pump Na+ out of the NPCE cell into the posterior
chamber (with water following); this pump helps to maintain a gra­
dient that constantly moves Na-t- from the ciliary stroma into the
PCE.
— Carbonic anhydrase: Catalyzes the following reaction in the PE
cells that yields bicarbonate: CO% + H^O —> H^CO^ —» H+ +
R C O z~.

Copyright 2014 by KMK Educational Services, LLC


136 2.11 AQUEOUS
—Bicarbonate ions are believed to increase aqueous production by
increasing Cl- and Na+ flux into the posterior chamber (26) (18).

Active transport facilitates the movement of sodium, chloride,


and bicarbonate ions to create a gradient for water movement and
aqueous humor production. Agents that disrupt this process in­
clude oral cardiac glycosides (alter the Na+/K+ ATPase pump)
and carbonic anhydrase inhibitors.

Each of the factors discussed below impede aqueous outflow and may lead to
glaucoma.
COVERING OF THE TRABECULAR MESHWORK
• Diabetes: Proliferative diabetic retinopathy may lead to neovascular­
ization and accompanying fibrous tissue in the angle, causing obstruction
of the TM and acute angle closure secondary to PAS formation.
— CRVO, OIS, and retinal detachments may also lead to neovascular­
ization of the angle.
• Uveitis: Inflammatory cells may impede outflow by clogging the TM.
Posterior and peripheral anterior synechiae can cause angle closure.
• Hyphema: Blunt trauma can lead to bleeding of the iris and/or ciliary
body, causing blood to accumulate in the anterior chamber and impede
aqueous outflow through the angle.

INJURY TO THE TRABECULAR MESHWORK


• Fuchs’ heterochromic iritis: Results in chronic inflammation that can
permanently damage the TM.
• Glaucomatocyclitic crisis: Acute inflammation of the TM (i.e. trabe­
culitis) that leads to an acute and dramatic rise in IOP.•
• Angle recession glaucoma: Trauma to the iris can cause separation of
the iris from the iris root, resulting in angle recession and damage to the
TM. Angle recession is seen as a very wide ciliary band on gonioscopy.

Copyright 2014 by KMK Educational Services, LLC


CHAPTER 2. OCULAR PHYSIOLOGY 137

OCCLUSIONS OF THE TRABECULAR MESHWORK


• Pseudoexfoliative Glaucoma: Aging epithelial cells of the iris base­
ment membrane and lens capsule release pigment and pseudoexfoliative
material, respectively, that accumulates within the angle and damages
the TM.
• Pigment Dispersion Glaucoma: Pigment is released from the pos­
terior layer of the iris (usually as a result of posterior bowing of the
iris against the lens zonules) and accumulates within the angle, causing
damage to the TM.

The following is a summary of aqueous humor composition (18) (26):


• The aqueous humor has less protein but more amino acids than plasma.
The concentration of protein in the aqueous is < 1 % that of plasma,
which limits light scattering.
• The aqueous humor has high amounts of ascorbate (vitamin C); the con­
centration of vitamin C in the aqueous is 20X higher than in plasma.
• The aqueous humor has more lactate than plasma, primarily a result of
anaerobic glycolysis in the lens and cornea.
• The aqueous humor has less bicarbonate ions than plasma and is slightly
more acidic (pH — 7.2).
;v.v-v-V'-...
We now summarize the blood aqueous barrier (18, ch. 8 ). Aqueous formation
begins with approximately 2 0 % of the substances trickling through the capillary
walls of the major arterial circle of the iris, then through the ciliary stroma, and
finally across both ciliary epithelial layers before begin transmitted through the
tight junctions of the NPCE into the posterior chamber via active secretion.
* Remember, the ciliary stromal capillaries are fenestrated and allow sub­
stances to diffuse out of the vessels. The tight junctions of the NPCE that
line the posterior chamber help to regulate the substances that ultimately
form aqueous humor.•
• The blood aqueous barrier consists of tight junctions located in three
places: the iris vessels, the endothelium of Schlemm’s canal, and
the non-pigmented ciliary epithelium.

Copyright 2014 by KMK Educational Services, LLC


138 2.11 AQUEOUS

Uveitis is secondary to a breakdown in the blood aqueous barrier.

References
[1] Ang LPK, Azar DT. Ocular surface epithelial stem cells and corneal wound healing response
to injury and infection. In: Albert DM, Miller JW. Albert and Jacltobiec’s principles and
practice of ophthalmology. 3rd ed. Philadelphia 2008: Saunders-Elsevier Inc. 475-484.
[2] Anderson DR. Glaucoma, capillaries and pericytes. 1. Blood flow regulation. Ophthalmology
ica. 1996;210(6):257-62.
[3J Banks MS, Bennett PJ: Optical and photoreceptor immaturities limit the spatial and chro­
matic vision of human neonates, Opt Soc Am A 5:2059, 1988.
[4] Bartlett, Jimmy D., Jaanus, Siret D. Clinical Ocular Pharmacology. Boston: Butterworth,
1984.
[5] ’ Crossman AR, Neary D. Neuroanatomy: An illustrated colour text. 4th ed. China: Churchill
Livingstone, 2010.
[6] Flattau, Pamela Ebert Considerations in Contact Lens Use Under Adverse Conditions: Pro­
ceedings of a Symposium. 1991.
[7] Friedbert, M. Rapuano, C. The Wills Eye Manual, 3rd edition. Philadelphia: Lippincott
Williams and Wilkins, 1999.
[8] Gipson IK, Spurr-Michaud SJ, Tisdale AS, Anchoring fibrils forma complex network in hu­
man and rabbit cornea. Invest Ophthalmol Vis Sci 1987; 23(2):212
[9] Gipson IK, Spurr-Michaud SJ, Tisdale A, et al. Reassembly of the anchoring structures of
the corneal epithelium during wound repair in the rabbit. Invest Ophthalmol Vis Sci 1989;
30:425
[10] Gray L. Auditory system: Pathways and reflexes. In: Neuroscience online. University of Texas
Medical School at Houston. 1997. http://neuroscience.nth.tmc.edu/s2/chapterl3.html.
[11] Gray L. Vestibular system: Pathways and reflexes. In: Neuroscience online. University of
Texas Medical School at Houston. 1997. http://nenroscience.uth.tmc.edu/s2/chapterll.html.
[12] Hanna, C, O’Brien JE. Cell production and migration in the epithelial layer of the cornea.
Arch Ophthalmol 1960;64:536.
[13] Hogan RN. The Eye in Aging, In: Albert DM, Miller JW, Albert and Jackobiec’s principles
and practice of ophthalmology. 3rd ed. Philadelphia 2008: Saunders-Elsevier Inc. 4605-4663.
[14] Holland, E., Cornea, 2nd edition, volume 1, Fundamentals, Diagnosis and Management
(2005), pp 1335-1340.
[15] Jones, LT. Anatomy of the Tear System. Int Ophthalmology Clin 1973; 13(1):3
[16] Jordan A, Baum JL. Basic Tear Flow, does it exist? Ophthalmology 1980;95:1.
[17] Kandell, E. Schwartz, J. Jessel, T. (2000). Principles of Neural Science, 4th Ed. McGraw-Hill.
[18] Kaufman, P, Aim, A. Adler’s Physiology of the Eye, 10th ed. St. Louis: Mosby, 2003.
[19] Kessler TL et al: Stimulation of goblet cell mucous secretion by activation of nerves in rat
conjunctiva, Curr Eye Res 14: 985, 1995.

Copyright 2014 by KMK Educational Services, LLC


CHAPTER 2. OCULAR PHYSIOLOGY 139

[20] Knierim J. Spinal reflexes and descending motor pathways. In: Neu­
roscience online. University of Texas Medical School at Houston. 1997.
http: / /neuroscience.uth.tmc.edu/s3/chapter02.html.
[21] Krachmer JH, Mannis MJ, Holland EJ. Cornea. 2nd ed, Philadelphia: Mosby, 2011.
[22] Levin, M. Verkman, A. Aquaporin-3-Dependent Cell Migration and Proliferation during
Corneal Re-epithelialization. Invest. Ophthalmol. Vis. Sci, October 2006 vol. 47 no. 10 4365-
4372.
[23] Pepose, J.S, and Ubeis, J.L., The cornea. In W.M. Hart Jr. (Ed.), Adler’s Physiology of the
Eye, Mosby, St. Louis, 1992, pp,
[24] Purves D, Augustine G, Fitzpatrick D, Katz L, LaMantia A, McNamara J, Williams S.
Neuroscience, 2nd Ed, Sinauer Associates, Inc, 2001.
[25] Rap nano, Christopher J. Heng, Wee-Jin. Color Atlas and Synopsis of Clinical Ophthalmology.
Wills Eye Hospital. Singapore: McGraw-Hill, 2003.
[26] Remington, Lee Ann. Clinical Anatomy and Physiology of the Visual System, 3rd Ed. Boston:
Butterworth-Heinemann, 1988.
[27] Strenk SA, Strenk LM, Guo S. Magnetic resonance imaging of the anteroposterior posi­
tion and thickness of the aging, accommodating, phakic, and pseudophakic ciliary muscle. J
Cataract Refract Surg. 2010 Feb;36(2):235-41.
[28] Schwartz S. Visual perception: A clinical orientation. 4th ed. New York: McGraw Hill. 2010.
[29] Tamesis, Richard R. Ophthalmology Board Review., 2nd Edition. McGraw-Hill, 2006.
[30] Terry, Jack, Ocular Disease Detection Diagnosis and Treatment. Thomas, Charles
C. (Publisher). 1984
[31] The Glaucoma Handbook, Optometric Glaucoma Society. Review of Optometry, August
2008.
[32] Thomas, R, Melton, R, http://www.eyenpdate.com/

Copyright 2014 by KMK Educational Services, LLC


c( -- :
J

( -, '
(

(( : -
J

(
(
c--.
((-, '

(c·:
(
( -, .

(( : '

(( .
((

((
(( .

((
((
((
((
((
((
((
((
((
((
(. (
((
(( ~­
((
I
I

Chapter

Geometrical Optics

Kevin B. Wood, Ph.D.

141
c
(
(
(
(
(
(
c
(
(
(
(
(
(
(
(
(
(
(
c
(
(
(
Copyright 2014 by KMK Educational Services, LLC (
<L
C
(
CHAPTER 3. GEOMETRICAL OPTICS 143

A uthor’s note: We would like to thank Dr. Chris Wolfe and Dr.
Ryan Fenska, who provided a number of suggestions to improve
the following 3 chapters

l - SECTION 3.1

Notation

There are several different notational schemes commonly used for studying
optometry-based optics. For this text, we chose the common optometrist nota­
tion (left column), but we offer here the conversions to the sometimes preferred
notation used in, for example, (5). In any event, the intended meaning should
always be clear from the context.

Description Notation 1 Notation 2


Power F P
Vergence L and V U and V
Object location l u
Image location V V
Primary Focal Point P F1
Secondary Focal Point P f2
Focal lengths f and / ' h and f 2
Primary Principal Plane P Hi
Secondary Principal Plane P' h2

- SECTION 3.2 -------------------------- ------------------- -------------------

Vergence, Objects, and Images


We start with a brief discussion of objects and images (5, pp. 29-35) (4, pp. 149-
53). Much of geometric optics can be summarized by a simple idea: an optical
system (a lens) will alter the properties of the light striking it in a predictable
way. The property that is altered is called vergence. Vergence characterizes the
curvature of the wavefront of light, and it is intimately related to the concepts
of objects and images.
Vergence: Vergence measures the degree of convergence or divergence of light
at a given point in space. By definition, diverging light has a negative
vergence and converging light has a positive vergence.
Copyright 2014 by KMK Educational Services, LLC
144 3.2. VERGENCE, OBJECTS, AND IMAGES

Real object: Incident light with a negative vergence corresponds to a real


object.
Virtual object: Incident light with a positive vergence corresponds to a vir­
tual object.
Real image: Exiting light with a positive vergence corresponds to a real im­
age.
Virtual image: Exiting light with a negative vergence corresponds to a vir­
tual image.

Sign convention:
• L < 0 indicates a real object.
• L! < 0 indicates a virtual image.
• L > 0 indicates a virtual object.
• V > 0 indicates a real image

Dioptric Power (F): When light strikes an interface, the vergence of its
wavefront is altered. We use F to quantify the change in vergence across
an interface.
General Vergence Equation: The vergence equation establishes a central
idea of geometric optics. An optical system alters the wavefront of the
light that strikes it, and the outgoing light reflects this change. Specif­
ically, an optical system alters the curvature of the wavefront, thereby
altering the degree of convergence (or divergence) of the light. This idea
is captured by the general vergence equation.
V = F + L, (3.1)
where V is the vergence of the exiting wavefront (in “image space”), L is
the vergence of the incident wavefront (in “object space”), and F is the
dioptric power of the optical system.
Object and Image Locations: In fact, vergence can be simply related to
object and image locations.

(3.2)

where l is the distance to the object and V is the distance to the image,
as measured from the optical system. Note that m is the index of the
Copyright 2014 by KMK Educational Services, LLC
CHAPTER 3. GEOMETRICAL OPTICS 145

material in which the light travels before striking the optical system or
interface (typically on the left in drawings), and ?i2 is the index of the
material in which light travels after passing through the optical system
or interface (typically on the right in drawings).
- SECTIO N 3.3 ---------------------------------------------------------------------------------- ------------------

Single Spherical Refracting Interfaces (SSRIs)


We now deal with the simple case of a curved interface between two media (5,
ch. 7) (1 , ch. 15) (4, ch. 5). This is very fundamental to optometry. For exam­
ple, a spherical lens is simply an optical system with two spherical refracting
interfaces - the front and back surfaces.
Concave/Convex Convention: We will label an interface as concave or con­
vex by considering the point of view of the medium with lower n (5,
pp. 107-108). It does not matter which way light is traveling.
Concave: Interface wraps around the lower n medium.
Convex: Interface wraps around the higher n medium.

Using the Concave/Convex convention:


Concave interface is ALWAYS a diverging interface.
Convex interface is ALWAYS a converging interface.

Power of an SSRI: The power of an SSRI is given by

r
(3.3)
where n 2 is the index of refraction of the final media (typically on the
right), n \ is the index of refraction of the original media (typically on
the left), r is the radius of curvature of the SSRI, and F is the power
of the SSRI. Note that we can use the convex/concave rule above to
double check if F < 0 (diverging) or F > 0 (converging). You can use
Equation 3.3 along with the general vergence equation to solve many
types of problems.

Exam ple 3 .1 : W h a t is th e p o w e r o f a p la n o c o n v e x th in lens whose cu rv e d surface


has a ra d iu s o f c u rv a tu re o f 3 cm ?

Copyright 2014 by KMK Educational Services, LLC


146 3.3. SINGLE SPHERICAL REFRACTING INTERFACES (SSRIS)
Solution 3 .1 :If the lens is made of glass (n—1.5), we can find the power of the
curved surface using equation 3.3.
F = (1.0 - 1.5)/(—0.03) = +16.6 D. (3.4)
Since the other surface is flat, it does not contribute to the power of the lens.

Focal Points: Focal points play a key role in optics. We begin to study them
in simple SSRIs.
Secondary Focal Point {F!)i F ’ is the location of the image point when
plane-polarized light (light from infinity) is incident on the interface. For
a converging interfacej this point is to the right of the interface (real
image). See Figure 3.1. For a diverging interface, this point is to the left
of the interface (virtual image).
Secondary focal length (/'): f is simply the distance from the interface to
F '. This is the image location when the object is at l ~ oo.

Finding /':
We can find f using Equation 3.1 with L = 0 (because the object
is located at l = oo, by definition). We have F = L' = yj-, which
gives
(3 .5)

The secondary focal point is associated with the light leaving the
interface.

Prim ary Focal Point (F): F is the location of the object point from which
light can leave such that, after interacting with the interface, it exits the
interface as plane waves.
Prim ary focal length (/): / is the distance from the interface to F . This
is the object location when the image is at V — oo.

Finding /:
We can find / using Equation 3.1 with L' = 0 (because the image
is at V = oo, by definition). We have F = —L = -y-> which gives
(3.6)

The primary focal point is associated with light arriving at the


interface.

Copyright 2014 by KMK Educational Services, LLC


CHAPTER 3. GEOMETRICAL OPTICS 147

Figure 3.1: An example of a lens with a secondary focal point to the right of
the lens is shown. Incoming plane waves converge to the secondary focal point.
The primary focal point is similarly defined, but incoming rays originating
there exit as plane waves.

Nodal point: The Nodal point is the point on the axis through which light
passes undeviated. For a single spherical refracting interface, the nodal
point is the center of curvature.

Example 3,2: Consider a very simple eye model that consists of only a single
spherical refracting interface (the "cornea'1) and a screen (the "retina"). Assume
that the radius of curvature of the SSRI is 6 mm, and the axial length of the eye
is 23 mm. If an object, 35 cm tall, is placed 1 m in front of the eye, how large is
the retinal image?

Solution 3.2: The nodal point can be used to find the retinal image size in simple
eye models (6). In this case, the nodal point is located at the center of curvature
of the SSRI, so it is 6 mm from the cornea and (23-6), or 17 mm, from the retina.
Because rays pass undeviated through the nodal point, we can use geometry to
find the size of the retinal image.
In this case, the distance between the object and the nodal point is 1000 mm + 6
mm. Using similar triangles, we therefore have
350 _ I (3.7)
1006 “ 17
Copyright 2014 by KMK Educational Services, LLC
148 3.3. SINGLE SPHERICAL REFRA CTING INTER,FA CES (SSRIS)
w here I is th e size o f th e re tin a l im ag e. T h e im a g e is th e re fo re 5 .9 m m .

Lateral Magnification; Lateral magnification, m, is the ratio of image size


to object size. It is sometimes called linear magnification.
hi L
(3.8)

where hi is the the height of the image, hQis the height of the object, L
is the incoming vergence and V is the outgoing vergence.

If the object height is given by #, the image height is given by m x}


where m is the lateral magnification of the system.

In the case of a flat surface, ideas from SSRIs lead to an equation for apparent
image locations (e.g. apparent depth) (5, ch. 8 ). For a flat surface, r = oo, so
F — 0. Now use Equation 3.1 and Equation 3,2 to arrive at
m _ n2
T ” T’
where l is the object’s actual distance from the interface and V is the object’s
apparent distance from the interface (i.e. the image location).
Exam ple 3 .3 : A person looks a t an o b je c t e m b e d d e d in glass 30 cm b elo w th e surface.
W h a t is th e a p p a re n t d ep th o f th e o b je c t?

Solution 3 .3 : T his is e x a c tly like a single s p h erical re fra c tin g in te rfa c e pro blem , b u t th e
p o w e r o f th e surface is 0. C onsider tig h t tra v e lin g fro m th e o b je c t e m b e d d e d in th e glass
to th e air. U sing th e vergence e q u a tio n , we h ave

F =0=V - L,

L' = L, (3.10)
n%fl! = ni/l.
F o r this problem , m = 1.5 a n d n 2 = 1.0. So w e have

1.0 ( t ! = - 1 . 5 / 0 .3 0 ,
(3 .11)
l ’ = — 20 cm .

S o the o b je c t app ears h ig h e r th a n it a c tu a lly is. I t a p p ears to s it o n ly 20 cm from th e


surface. N o te also th a t th e im ag e is virtu a l.

Copyright 2014 by KMK Educational Services, LLC


CHAPTER S. GEOMETRICAL OPTICS 149

SECTIO N 3.4

Snell’s Law
Snell’s law details the relationship between the angle of refraction and the angle
of incidence (5, ch. 17) (1, ch. 14) (4, pp. 100-101). Snell’s Law is given by
ni sin cf)j = ri2 sin <f>R} (3.12)
where n is the index of refraction and <f>is the angle of refraction or incidence.

Calculations on the exam will not involve trigonometric functions,


but a qualitative familiarity with Snell’s Law and Total Internal
Reflection may be helpful.

Total Internal Reflection: For n 2 < m, there exists some angle </>c, called
the critical angle, such that light at any angle of incidence <pi > <j>c will
be totally internally reflected. That is, light incident at an angle greater
than the critical angle </>c will be reflected internally. Mathematically,
this is simply saying that at —<f)c, </># = 90°. So we have
sin^c = ~ni. (3.13)
Note that total internal reflection is only possible if < ni!

Total internal reflection makes unaided viewing of the anterior


chamber angle impossible. This limitation is overcome with the
use of gonioscopy lenses.

I - SECTIO N 3.5

Thin Lenses
Thin lenses are a simple extension of SSRIs (5, ch. 5) (1 , ch. 15) (4, ch. 5).
We treat a lens as two SSRIs, calculate the power of each, and then add them
to get the total lens power. This leads to
F ~ Fi + F 3 , (3.14)
where F\ is the power of the front lens surface and F2 is the power of the back
lens surface.
Copyright 2014 by KMK Educational Services, LLC
150 3.5. THIN LENSES

Exam ple 3.4: W h a t is th e p o w e r o f a th in lens (n ? = 1.5,) in a ir whose fro n t radius o f


c u rv a tu re is 3 cm ( w ith a convex s u rfa c e ) a n d whose b ack rad ius o f cu rv a tu re is 2 cm
(w ith a concave s u rface)?

Solution 3.4: W e can t r e a t each side s e p a ra te ly as a n S S R I a n d add. E q uivalently, one


c ou ld ju s t p lu g d ire c tly in to th e e q u a tio n above.

L e ft side:
Fl = (1.5 - 1)/0.03 = +16.67 D; (3.15)
R ig h t side:
F2 = (1 - 1.5)/0.02 = -25.00 D\ (3.16)
So
F = 16.67 - 25.00 = -8.33 D. (3.17)

Exam ple 3 .5 : A re a l o b je c t is 35 c m fro m a +3.00 th in lens p la c e d in c o n ta c t w ith a


+1.00 th in lens. F in d th e c o n ju g a te im a g e a n d th e fo c a l lengths.

Solution 3.5 : W h en th in lenses are p la c e d in c o n ta c t, th e ir pow ers ju s t add, so this is


e q u iv a le n t to a th in lens o f + 4 . 0 0 D . W e can firs t fin d th e fo c a l len g th s f a n d f o f this
c o m p o u n d lens.
E = n /f,
f = +25 cm, (3.18)
/ = - / ' = -25 cm .
To solve th e p ro b lem , we h ave

l = ~ 3 5 cm ,
L = Ijl = -2.86 D ,
L = F + L gives

H =4.00+ (-2.86), (3.19)


L' 1.14 D ,
=
l ! = 1/1/ = 1/1.14 = 0.88 m = 88 cm (to rig h t).

m = / L * = -2.86/1.14 = -2.5.
l

So th e im ag e is inverted , m ag nified, a n d real.

Copyright 2014 by KMK Educational Services, LLC


CHAPTER 3, GEOMETRICAL OPTICS 151

When you place a lens some distance (x) in front of a screen, the vergence
of light striking the screen is different than the vergence of light leaving the
lens (5, clr. 6 ) (3, cli, 13). But the vergence of light at the screen under some
conditions is more relevant than the vergence of light leaving the lens. Our
equations above give us only the latter. It is therefore helpful to define an
Effective Vergence (or Downstream Vergence) to account for location. The
relevant equation for the Effective Vergence at a location r = x is
=r+ r LeIf (3,20)
where L is the vergence of light at position r = 0 , x is the direct distance from
position r = 0 to the point in question, and L ef / is the effective vergence (the
vergence at position r = x of a wave with vergence L at position r ~ 0).

Exam ple 3.6: W h e n a s ta n d m a g n ifie r is used to view re a d in g m a te ria l, th e lig h t le a vin g


th e m a g n ifie r has a vergence o f - 3 .0 0 D . T h a t is, an im ag e o f th e m a te ria ! is lo c a te d 3 3
cm on th e o p p o s ite side o f th e m a g n ifie r from th e p a tie n t. W h a t a d d is re q u ire d fo r this
p a t ie n t to use th e m ag n ifier, assum ing th e p a tie n t is 4 0 cm fro m th e m a g n ifie r?

S olu tion 3 .6: W e can use th e effe c tiv e vergence eq u atio n to fin d th e vergence o f lig h t
a rriv in g a t th e eye. T h a t is, w e w a n t th e effective vergence 4 0 cm d o w n s tre a m fro m a
- 3 .0 0 D w a v efro n t. W e have:

-3.00
Leff = 1 - 0.4(—3.00) = -1.36 D (3.21)
So th e a d d re q u ire d to p e rfe c tly n e u tra lize th e divergence o f th e lig h t w o u ld b e +1.36 D .

Spectacle lenses sit away from the cornea, while contact lenses sit at the cornea.
This difference must be accounted for in prescribing corrective lenses. Consider
plane waves incident on a spectacle lens of a given power. Since light will gain
vergence between leaving a lens and arriving at the cornea, the vergence of
light reaching the cornea is more positive than that of light leaving the lens (5,
ch. 6 ) (3, ch. 13). As a result, we need different correcting lenses for different
lens locations. In equation form, we have
1 - |x\Fg Fc = (3.22)
where Fg is the power of the correcting lens needed at a vertex distance of x
and Fc is the desired power at the cornea. One can easily confirm that more
Copyright 2014 by KMK Educational Services, LLC
152 3.6. THIN LENS SYSTEMS: THICK LENSES
plus power (less negative power) is needed in prescriptions that sit closer to
the cornea.

Equation 3.22 is a direct consequence of the idea that the secondary


focal point of a correcting lens must be placed at the far point of
an ametropic eye.

Exam ple 3 .7 : A p a t ie n t ’s a m e tro p ia is p e rfe c tly c o rre c te d by c o n ta c t lenses t h a t are


+ 10 .0 0 D . W h a t p o w e r w o u ld be re q u ire d a t a v e rte x distance o f 1 2 m m ?

Solution 3 .7 : W e can d ire c tly use e q u a tio n 3 ,2 2 to solve this p ro b le m . In th a t case,


x = 0 .0 12 a n d Fc := + 1 0 .00 , a n d w e m u s t solve th e e q u a tio n fo r Fg. A lte rn a tiv e ly ;
we can th in k a b o u t this in term s o f fa r p o in ts . T h e p a tie n t is h yp ero pic, a n d th e fa r
p o in t, w hich is virtu a l, is lo c a te d 1 / 1 0 = 0 .1 0 m b e h in d th e cornea. I f a re fra c tio n were
p erfo rm e d a t a distance o f 1 2 m m , this w o u ld b e a to t a l o f 0 .1 0 + 0 .0 1 2 = 0 .1 1 2 m from
th e fa r p o in t. So th e p o w e r need ed in a lens a t this distance is s im p ly 1 /0 .1 1 2 = + 8 .9 3
D.

- SECTION 3.6 ---------------------------------------------------------------------------

Thin Lens Systems: Thick Lenses


When several lenses are placed in a series, we can use the image from the 1 st
lens as the object for the 2 nd lens, and so on. Furthermore, a thick lens can
be thought of as two thin lenses, representing the front and back lens surfaces,
that are separated by some material. We only briefly mention some techniques
for solving multiple lens problems (5, ch. 9,11) (4, ch, 5,6).
There are two common approaches to deal with thick lenses:
1 Treat the thick lens as two thin lenses separated by some material of
index n equal to the index of the lens. This is sometimes referred to
as successive imaging. It is no more difficult than doing two thin lens
problems in succession.
2 Use the Gauss System and Cardinal Points.
Copyright 2014 by KMK Educational Services, LLC
CHAPTER 3. GEOMETRICAL OPTICS 153

Equivalent Power: It is sometimes convenient to approximate a thick lens


by using a thin lens that behaves similarly. In those cases, the concept of
equivalent power is useful. Equivalent power is the power of a thin lens that
would be equivalent (optically) to a particular thick lens. It is given by
Ee — F i + E % -----E iF 2 y (3 .2 3 )
U2
where Fe is equivalent power, Fi is the power of the front surface, F2 is the
power of the back surface, t the is thickness of the lens, and 112 is the index of
refraction of the lens material.

Equivalent power is often useful for characterizing a two lens com­


bination such as a thick lens. When t —>0 , the power is just the
sum of the surface powers (i.e. it is like a thin lens).

Back vertex power Back vertex power is a critical idea for optometry. It
provides the vergence of light exiting the optical system given that plane waves
entered the system. Most prescriptions are specified using the back vertex
power. The back vertex power for a thin lens is given by
Fv — F 2' T ___ ^ > (3 .2 4 )
nsr 1
where Fv is the back vertex power, Fi is the power of the front surface of the
lens, F2 is the power of the back surface of the lens, t is thickness, and 112 is the
index of refraction of the lens material. A similar equation exists for what is
called the front neutralizing power, but it is less commonly used in optometry.
Note that front and back vertex powers are reciprocally related to front and
back focal lengths.

Exam ple 3.8: A crow n glass sp e c ta cle lens ( n = 1.5 ) has a fro n t s u rface p o w e r o f + 1 5 . 0 0
D a n d a b a c k s u rface p o w e r o f - 5 .0 0 D . T h e c e n tra l thickness o f th e lens is 2 m m . W h a t
is th e b a c k v e rte x p o w e r o f this lens?

Solution 3.8 : T h e in d e x o f re fra c tio n is 1.5, so t f n = 0.0013. T h e back v e rte x p o w e r is


given b y

- - 5-0 0 + = +wmD- (3'26)


N o te t h a t this is s lig h tly m o re p o s itiv e th a n th e sum o f th e surface powers.

Copyright 2014 by KMK Educational Services, LLC


154 3.6. THIN LENS SYSTEMS: THIGK LENSES
Successive Imaging
For this method, we treat a thick lens as two thin lenses, as mentioned above.
The front surface is considered to be a thin lens, and the back surface is also
a thin lens. The vergence of light L 2 hitting the back surface of the thick lens
is not the same as the vergence of light L[ leaving the front surface. Thus, we
must treat the front and back separately (see Figure 3.2). This gives rise to the
notion of front and back focal lengths, but these topics are beyond our limited
scope.

When finding object and image distances, L\ is paired with m , L[


and L 2 with ri2 , and V2 with 723.

Gauss System and Cardinal Points


When we use the Gauss System (5, ch. 11) (4, ch. 6 ), we essentially treat the
optical system like a black box. That is, we do not care what goes on inside the
system; instead, we are only concerned with the relationship between incoming

Left Right
surface surface

Figure 3.2: The setup and notation for Method 1, Thick lenses, is shown above.
Li is the incident vergence (with corresponding Ji), L[ is the vergence exiting
the front surface (with corresponding Zj), L 2 is the vergence incident on the
back surface (with corresponding fa)} and U2 is the vergence exiting the back
surface (with corresponding V2)

Copyright 2014 by KMK Educational Services, LLC


CHAPTER 3, GEOMETRICAL OPTICS 155

and outgoing rays (specifically, the incoming and outgoing vergences). To get a
handle on this relationship, we characterize the system using Cardinal Points.
Cardinal Points: We can define six Cardinal points:
1 Two Focal Points The focal points are similar to the previous defini­
tions of focal points. Sometimes the lengths associated with these focal
points are referred to as “equivalent” focal lengths, which means that the
locations of the focal points are measured relative to the principal planes.
Alternatively, these are referred to as “back” or “front” focal lengths when
their positions are specified relative to the front or back of the optical sys­
tem.
2 Two Nodal Points The nodal points are similar to previous definitions
of nodal points, but with added complexity. In ray tracing, a ray that
strikes the first nodal point N at an angle 0 with respect to the optical
axis will leave the second nodal point N’ at an angle <p with respect to
the optical axis, as in Figure 3.4.
3 Two Principal Points Principal planes represent something new, and
we need new definitions to capture these ideas. We introduce these defi­
nitions below (and see Figure 3.3).

Secondary Principal Plane (P f)i Consider incident rays parallel to the


axis. Outgoing rays will point to the focal point E \ by the definition of EL
If we now extend the incident rays and the outgoing rays and see where they
intersect, we will find that all intersections lie in one plane. We call this plane
the secondary principal plane PL P ’ can be in front of, inside, or behind the
optical system. The point where P{ intersects the optical axis is the Secondary
Principal Point.
Prim ary Principal Plane (P): Consider a point source at the primary focal
point E. By the definition of E , rays from E will leave the system as plane
waves. The primary principal plane P is the plane containing the intersection
of these parallel rays (extended backwards, if needed) and the rays leaving E
(extended if needed).

An object at P always has an image at P f with m = +1.

Copyright 2014 by KMK Educational Services, LLC


156 S. 6. THIN LENS SYSTEMS: THICK LENSES

Lens / Interface

Figure 3.3: Principal planes are shown for an optical system. In this example,
both planes fall inside the optical system itself. Note that we treat the system
as a black box: we do not care exactly what is happening inside the box, so
long as we can make statements about incoming and exiting light. This is the
idea behind the Gauss system. Principal planes can also be found in front of
or behind the system, depending on the optical setup of the system itself.

Why is this useful? If we know the locations of P and P*, we have


a system whose behavior resembles that of an SSRI. One major
difference exists: in an SSRI, one interface (the SSRI) interacts
with both incoming and outgoing rays; in the Gauss System, P
interacts with the incident rays and P' interacts with the exiting
rays, at least insofar as we are speaking heuristically.

Problem Solving with Cardinal Points: The idea is simple: we want to


use the cardinal points to understand the behavior of some very complicated
systems. We will assume that we know the location of the principal planes (if
not, see below). Note that the procedure is similar to that used for an SSRI.
Problem Solving Steps
1 Pretend incident light travels all the way to P and compute the vergence
(Lp) at that location.

2 Pretend exiting light leaves P f and compute the vergence (Lp,) at that
location.
Copyright 2014 by KMK Educational Services, LLC
CHAPTER 3. GEOMETRICAL OPTICS 157

3 Now use the following generalized Vergence Equation:


Fe = LfP, - L P, (3.26)
where Fe is the equivalent power, Equation 3.23.
4 Similarly, the magnification formula keeps the same form
"> = #■ • (3.27)

Finding Principal Plane Locations: There are several important equa­


tions for locating principle planes.
y = -n3—
K2 —
t Fi
»
Fe
(3.28)
x - m — —,
t F2
n 2 Fe
where y is the directed distance from the back edge to P ', F\ is the power of
the front surface, x is the directed distance from the front edge to P, and F^
is the power of the back surface.
Finding Nodal Point Locations: The locations of the nodal points coin­
cide with those of the principal points if the lens is surrounded on both sides
by the same material {n\ = n3). This is not the case if the two surrounding

Lens / Interface

P P1

Figure 3.4: The nodal points and principal planes are shown for a system with
n i / ns.

Copyright 2014 by KMK Educational Services, LLC


158 3.7. STOPS, PUPILS, AND PORTS
media are different. In general, the positions are given by
P N ^f +f, (3.29)
where / and f are equivalent focal lengths and P N is the displacement of each
nodal point from the corresponding principal plane location. Note that each
nodal point is displaced the same distance from its respective principal plane.
For the simple case n% = we have / ——f and the nodal points coincide
with the principal planes.
- SECTION 3 . 7 -------------------------- --------------------------------------- ----------------------------------

Stops, Pupils, and Ports


We now introduce some important topics related to stops, pupils, and ports (5,
ch. 19) (4, ch. 5,6).

Two elements limit an optical system’s field of view:


A perture stop: The aperture stop is the physical entity that limits the amount
of light passing into an optical system when viewing an object. This may
be an actual aperture (e.g. in a camera), or it may merely be the edge
of a lens or some other physical element. Along with the field stop, the
aperture stop limits the field of view.

There may be many apertures, lenses, etc. comprising an optical


system, but by definition only one of these constituents will be the
aperture stop. The element that serves as the aperture stop will
vary for different object distances.

Field stop: The field stop limits the size of the object that can be imaged by
the system. It works with the aperture stop to limit field of view.

Pupils
Images of the aperture stop formed by lenses in front of or behind the stop
constitute pupils.
Copyright 2014 by KMK Educational Services, LLC
CHAPTER 3. GEOMETRICAL OPTICS 159

Entrance pupil: The entrance pupil is the image of the aperture stop formed
by lenses in front of it. If there are no lenses in front, the entrance pupil
itself is the aperture stop. The entrance pupil determines the size of the
cone of light that actually enters the system (4, ch. 5).

Finding the entrance pupil:


1 Go to the axial object point.
2 Look towards the front of the system (to the right, using typ­
ical convention).
3 The entrance pupil is the element or the image of an element
that subtends the smallest angle.
4 Because it subtends the smallest angle, it looks smallest when
seen from the object point.

Exit pupil: The exit pupil is the image of the aperture stop formed by lenses
behind it. If there are no lenses behind it, the exit pupil itself is the
aperture stop. The exit pupil determines the size of the cone of light that
actually exits the system (4, ch. 5).

If one knows the aperture stop of a system, the entrance and exit
pupils can be easily found by imaging the aperture stop through
lenses in front of or behind the aperture stop.

Exam ple 3 .9 : In m o s t telescopes used for distance view ing, th e o b je c tiv e lens is
th e a p e rtu re s to p . W h a t is th e e x it p u p il? W h a t is th e e n tra n c e p u p il? W h e re is
th e e x it p u p il lo c a te d fo r a G alilean telescope? A K eplerian telescope?

Solution 3.9: T h e o b je c tiv e lens is the a p e rtu re stop. T h e re a re no lenses on th e


o b je c t side o f th e o b je c tiv e lens, so i t is also th e e n tra n c e p u p il. T h e e x it p u p il
is th e image o f th e o b je c tiv e lens th ro u g h th e o cu la r lens. F o r a K e p le r telescope
(w ith a co n verg in g o c u la r lens), th e ex it p u p il is lo c a te d o u tsid e o f th e telescope
(clo s e r to th e p a t ie n t ’s eye), w h ile fo r a G alilean telescope (w ith a d iv e rg in g o c u la r
lens), th e e x it p u p il is lo c a te d inside o f th e telescope.

Copyright 2014 by KMK Educational Services, LLC


160 3.7. STOPS, PUPILS, AAD POFTS
Focal ratio: The focal ratio or /-number of a system is given by
// = I , (3-30 )

where / is the focal length and D the diameter of the entrance pupil. As
an example, a camera with an entrance pupil diameter of 30 mm and a
60 mm focal length will have an /-number of two, which we can write as
/ / 2 . As /-number decreases, more light can enter the system. Because
the focal ratio is closely related to the exposure time for a camera-, it is
conventionally referred to as the speed of the camera (4, ch. 5).

Images of the field stop formed by lenses in front of or behind the stop constitute
ports.
Entrance port: The entrance port is the image of the field stop formed by
lenses in front it.
Exit port: The exit port is the image of the field stop formed by lenses behind
it.

We review some related definitions (5, ch. 19).


Depth of focus: For the eye, the depth of focus is the interval surrounding
the retina in which an eye sees an object as in focus. That is, if light is
focused in this area, the object will be in focus.

Consider a patient with 20/40 Snellen acuity. With a pinhole oc­


cluder, her vision will improve (assuming her vision problem is
refractive in nature) because the pinhole will increase her depth
of focus. This can be useful for distinguishing between refractive
errors and other vision problems, particularly cataracts (see Equa­
tion 4.4).

Depth of field: Depth of field is the interval surrounding the fixation plane
in which an object can reside and still be in focus (with no accommoda­
tion). That is, if the object is located within this region, there will be no
perceivable blur on the retina (or screen).

Copyright 2014 by KMK Educational Services, LLC


CHAPTER 3. GEOMETRICAL OPTICS 161

General Trends:
A short focal length leads to a large depth of field.
An increase in aperture size leads to a decreased depth of field and
decreased depth of focus.

Field of View: The field of view is the extent of the object plane that is
imaged. It can be measured as an angle or as a linear distance in the
object plane. One common metric of measurement is the angle of half
illumination.

General TYends:
A minus lens will increase field of view.
A plus lens will decrease field of view.

Field of Fixation: The field of fixation is the angle made from the optical
axis by the entrance port as measured at the center of rotation of the
eye. Note the difference between field of fixation and field of view. In the
eye, the center of rotation is typically 14 mm from the cornea.
- SECTIO N 3.8 -----------------------------------------------------------------------------------------------------

Spherocylindrical Lenses
We now introduce concepts related to more general, nonspherical lenses (5,
ch. 15) (1, ch. 15,16) (3, ch. 2).
Stigmatic system: In a stigmatic system, a point source produces a point
image.
Astigmatic system: A simple astigmatic system is characterized not by a
single focal point, but by a pair of focal lines. In words, the power is dif­
ferent along different meridians. Consider a lens with principal meridians
at 180 and 90. The location of the vertical focal line is determined by
the power in the horizontal meridian of the lens, and the location of the
horizontal focal line is determined by the power in the vertical meridian
of the lens. Of course, this can be generalized to lenses with principal
meridians other than 180 and 90.
Copyright 2014 by KMK Educational Services, LLC
162 3.8. SPHEROCYLINDRICAL LENSES
Cylindrical lens: For intuition, consider a lens that is shaped like a cylinder.
The direction parallel to the axis of the cylinder is referred to as the
axis meridian. The direction perpendicular to the axis meridian will be
referred to as the power meridian. Collectively, these are known as the
principal meridians. We can describe a lens in terms of a power cross,
which shows the power along each of the principal axes.

Cylinder lenses always have zero power along one meridian. More
complex lenses may have different, nonzero powers along each prin­
cipal meridian. This is merely a generalization of the idea of a
cylinder lens. If you prefer, this more complex lens can be thought
of as two cylinder lenses crossed perpendicularly. A point object
will then produce one horizontal line image and one vertical line
image.

Spherical lens: Spherical lenses are rotationally symmetric, so the wavefront


will be altered in a meridian-independent manner. That is, the power in
each meridian is the same. Because of this symmetry, there is no need to
specify the orientation of a spherical lens.

Obviously, human eyes are not always optically spherical,


and corrective lenses must often account for this asymmetry
(astigmatism). As an optometrist, you must specify both a lens
and, if it is not spherical, also an orientation. One specifies orien­
tation using a contraocular view.

Contraocular view: This is the view that a doctor would have when looking
at the patient. The vertical meridian is labeled 8 = 90°, by convention.
The horizontal meridian is labeled 8 = 180°.
Labeling Cylindrical Lenses: One can specify cylinder power in several ways:
1 Specify the location of the axis using x.
2 Specify the power in a given meridian using the @ symbol.

Copyright 2014 by KMK Educational Services, LLC


CHAPTER 3. GEOMETRICAL OPTICS 163

The spherical equivalent for a lens is given by


= “y-
Fn
+ Esy Ese (3.31)
where Fq is the cylinder power and F$ the sphere power of the
lens.

Circle of least confusion (blur circle): The COLC is the point of best fo­
cus for a lens- It is located dioptrically halfway between the two line
images that are formed in the principal meridians.

Suppose we put an aperture against a lens with an asymmetric


power cross. Then we place a movable screen behind the aper­
ture. As the screen is moved back, we would see a series of images
including a vertical line, a horizontal line, and a circle. At some
location between the locations where horizontal and vertical lines
are formed, respectively, you will get an image that looks like a
circle. This is the point of best focus. In a sense, we are finding
the point where the image is distorted equally in each meridian (5,
pp. 295-99). When a clinician uses the spherical equivalent of
an astigmatic lens, he/she is assuring that the COLC will fall on
the retina.

Interval of Sturm: The loS is the distance between the two foci of the prin­
cipal meridians. It is the spatial distance between the locations of the
horizontal and vertical line formation.

Exam ple 3 .1 0 : C o nsider a lens w hose p o w e r is given b y +5.00 + 2.00 x 180. F in d th e


lo c a tio n o f th e circle o f least confusion. W h a t is th e In te rv a l o f S tu rm ?

Solution 3 .10 : T h e p o w e r cross shows pow ers o f + 5 . 0 0 a n d + 7 .0 0 , so th e C O L C is


lo c a te d a t + 6 . 0 0 ( o r a b o u t 1 7 c m ). T h e lo S is sim p ly th e region b etw e e n 1 4 .3 cm ( 1 / 7
m ) a n d 2 0 .0 cm ( 1 / 5 m ), so its le n g th is a b o u t 6 cm .

Copyright 2014 by KMK Educational Services, LLC


164 3.8. SPHEROCYLINDRICAL LENSES

It is absolutely critical that you can convert between power cross


notation and optometry “prescription” notation!

Exam ple 3 . 1 1 : A lens p rescription is given b y + 2 . 0 0 D - 3 .0 0 D x 180. D r a w th e p o w e r


cross for th e fens. W h ich m erid ian has th e m o s t plus p o w e r? I f y o u p lace this lens
o v e r a fogged em m e tro p ic eye a n d show th e p a tie n t a clock dial, w hich line w ill a p p e a r
sh arp est?

Solution 3 .1 1 : T h e p o w er cross w ill h ave + 2 . 0 0 D a t 1 8 0 a n d - 1 .0 0 D a t 90. So th e 1 8 0


m e rid ia n has th e m o st p lu s pow er. Obviously, th e axis alw ays has th e m o st plus p o w e r
w hen th e prescription is in m inus cyl form . I f th e p a tie n t is fogged, b o th fo cal lines fa ll on
th e cornea side o f th e re tin a . T h e m erid ian w ith th e least plus p o w e r alw ays corresponds
to th e lin e th a t is closest to th e re tin a u n d er these conditions. So th e line corresponding
to th e m erid ian 90 degrees fro m th e axis w ill a p p e a r th e s h a rp e s t because it is closest
to th e retina. In this case, th e line corresponding to th e 9 0 degree m erid ian w ill a p p e a r
s h a rp e s t H ow ever, th e 90 degree m erid ian creates a horizontal line. So th e 3 to 9 line
w ill a p p e a r sharpest.

[Equivalent ^ aiid ^ a n sp o sitio n ;


We can use power crosses to determine the power of lens combinations (or,
equivalently, two surfaces of the same lens). The rule is simple: add the pow­
ers along each principal meridian (assuming the lens is thin and the principle
meridians coincide). Of course, different combinations of lenses will give the
same power cross. This gives rise to the notion of transposition.
Transposition: Sometimes you want to go from a sphere-plus cylinder com­
bination to a sphere-minus cylinder combination with the same power
cross. There is an easy formula for accomplishing this transposition.
S + (C x 6) ++ (S + C) + ( - C x {8 ± 90)), (3.32)
where C is the original cylinder power, 8 is the location of the original
axis of the cylinder, and S is the original spherical power.
In words, the new sphere value is the sum of the old sphere plus the old
cylinder. The new cylinder value is the negative of the old cylinder value,
and the axis is rotated by 90°.
Copyright 2014 by KMK Educational Services, LLC
CHAPTER 3. GEOMETRICAL OPTICS 165

To find the power of a spherocylindrical lens in an oblique meridian, use the


so-called “sine squared” rule, which is given by
Fa = F$ 4- Fa sin20, (3.33)
where Fa is the power in the a meridian, Fs is the sphere power of the lens, Fq
is the cylinder power of the lens, and 8 is the difference between the meridian
a and the axis of the cylinder: 8 = a — axis.

Calculations on the exam will not involve trigonometric functions.


However, you should be familiar with the special case where you
want to find the power in a meridian exactly halfway between two
meridians of known powers (see example below).

Exam ple 3 .1 2 :What is the (approximate) power in the vertical meridian of the following
lens; +5.00 - 2.00 x 135?

Solution 3 .1 2 : The power cross shows +5.00 at 135° and +3.00 at 45°. Just by
looking, we might guess that the power would be +4.00 in the vertical meridian, because
it is halfway between the 135 and the 45 and +4.00 is the average of +5.00 and +3.00.
Indeed, if we take a = 90, 6 = 90 —135 = —45, we can confirm this intuition by using
Equation 3.33.
In general, one can find the power in the meridian exactly halfway between two other
meridians by averaging the powers; there is no need for trig functions. This trick is very
useful for vertical imbalance problems. While it is, strictly speaking, incorrect to use this
formula in combination with Prentice's rule, it can be used in cases where there is only
horizontal or only vertical decentering. We will come back to this point.

Copyright 2014 by KMK Educational Services, LLC


166 3.9. MIRRORS

Rotating a lens about the 180 meridian is called pantoscopic (or ret-
roscopic) tilt. Faceform tilt refers to rotation about the 90 merid­
ian. Rotating a lens will induce (minus) cylinder with the following
axes:
• Pantoscopic tilt of a minus lens —>Axis at 180
• Pantoscopic tilt of a plus lens -» Axis at 90
• Faceform tilt of a minus lens —>Axis at 90
• Faceform tilt of a plus lens —» Axis at 180

I - SECTION 3.9

M irrors
Mirror problems are usually fairly simple and in fact are very much like thin
lens and SSRI problems. We detail the specifics below (5, ch. 10) (4, ch. 5).

As before, we take the positive direction to be the direction that


outgoing light is traveling. However, for mirror problems, this di­
rection is typically to the left.

Indices of refraction in mirror problems: In mirror problems, light now


travels forward and backward through the same material (with index n ),
As a result, we can take n± — —n and ng = n. With this transformation,
all formulas stay the same as SSRJ formulas.
Radius of curvature: The radius of curvature r is negative for a convex mir­
ror. While this might initially appear different than before, one can see
that it is nevertheless consistent if we consider the positive direction to
be that associated with outgoing light.
Power: The power of a mirror is simply given by
F = 2n/r (3.34)
where n is the index of refraction (usually air) and r is the radius of
curvature. The sign of power can be assigned based on the shape of the
mirror (concave is positive, convex is negative).
Copyright 2014 by KMK Educational Services, LLC
CHAPTER 3. GEOMETRICAL OPTICS 167

Exam p le 3 .1 3 :Consider a real object placed 40 cm in front of a concave mirror with a


radius of curvature of 18 cm. Find the conjugate image and the magnification m.

Solution 3 .1 3 : The radius of curvature is 18 cm, so the power is


F = 2(1)/0.18 = 11.1 D. (3.35)
The power is positive, because the mirror is concave. The focal point is given by f =
1/F fh 9 cm. The object is located to the left of the interface, but rather than worrying
with signs, let usjust think about the problem. We know it is a real object, so light incident
on the mirror is diverging. Therefore, the incident vergence is negative (L = l/l ——2.50
D). Now, using the vergence equation, we get
L' = F + L,
l! = 11.1 + (-2.50) = 8.6 D. (3.36)
Hence the image is located at l' = 1/V = 11.6 cm. Finally, m = LjL‘ = —2.50/8.60 =
-0.29.

It is sometimes useful to consider a lens-mirror combination. For example,


consider a lens whose back surface is coated with silver. The power of this lens
is now the sum of two refractions and one reflection,
F = 2Fi + F2, (3.37)
where F\ is the power of the front surface of the lens and F2 is the power of
the mirror.

References
[1] C, Brooks and I. Borish (1996). S ystem fo r Ophthalmic D ispensing, 2nd E dition .
Blitter worth-Heinemann.
[2] F. Chang, D. Gerstman, P. Pietsch, and L. Locke (1994), M EPC: O ptom etry: E xam ination
Review, 4th Edition. McGraw-Hill Medical Publishing.
[3] T. Fannin and T. Grosvenor (1996). Clinical Optics, 2nd Edition. Butterworth-Heinemann.
[4] E. I-Iecht (2001). Optics, 4th E dition. Addison Wesley.
[5] M, Keating (1988). G eom etric, Physical, and Visual Optics. Butterworth-Heinemann,
[6] Laltshminarayanan, V. and E. Bennett (2006), Review Q uestions fo r the N B E O E xam ina­
tion, P art One, Butterworth Heinemann.7
[7] E. Stoner, P, Perkins, and R. Ferguson (2005). Optical Formulas Tutorial, 2nd E dition,
Butterworth Heinemann.

Copyright 2014 by KMI< Educational Services, LLC


( " I

( !

c
C
(
r
('
(

( \

c
(
(
(

(
(
(
(
(
(
(
(
(
(
(
(
Chapter 4
Physical Optics

Kevin B. Wood, Ph.D.


Sarah Dougherty Wood, O.D., F.A.A.O.

169
(
(
(■
c
r.
c
c
c
(
(
c
c
(
c
(
c
(
c
(
c
c
(
c
Copyright 2014 by KMK Educational Services, LLC (
c.
(.
(
CHAPTER 4 . PHYSICAL OPTICS 171

Physical optics has been greatly de-emphasized in the new exam format. We-
include only a few essential points.
1- SECTION 4,1

Electromagnetic Waves

Light is a spatiotemporal electromagnetic wave. Specifically, light consists of


electric and magnetic fields that oscillate in space and in time, with the oscil­
lations of the fields normal to the direction of propagation. Electromagnetic
waves propagate at many different frequencies. We characterize this spectrum
by frequency or wavelength (in a vacuum) (10, pp. 19-22) (5, ch. 3) (6 , ch. 1,21).

Note that the frequency of light is invariant across media. However,


the wavelength and velocity of light depend on the media.
c
n=
«n’ (4.1)
Ao
V
where c is the speed of light in a vacuum, vn is the speed in a
medium with index n, Ao is the wavelength of light in a vacuum,
and An is the wavelength in a medium with index n.

Spectrum Portion Vacuum A (nm) > (Hz)


Ultraviolet I - 400 1 0 14- 1 0 17
Visible 380 - 760 5.45 xlO 14
Infrared 700 - :io5 JO12 1014

Overall Spectrum (highest to lowest frequency)


1 Gamma (7 ) rays
2 X-rays
3 Ultraviolet (UV)
4 Visible
5 Infrared (IR)
Copyright 2014 by KMK Educational Services, LLC
172 4.2. LASERS
6 Microwaves
7 Radar waves
8 Television waves

9 Radio waves
UV light and the Eye
UV light is classified as either:
1 UV-A (near UV) is 400-320 nm
2 UV-B (middle UV) is 320-290 nm
3 UV-C (far UV) is 290-200 nm

Tears, cornea, and aqueous absorb all light with wavelengths below
290 nm (UV-C and below), so no UV-C light reaches the lens.
However, UV-A and UV-B will be partially transmitted to the lens.
The lens of a child will transmit some UV-A and UV-B to the
retina, while an elderly adult lens transmits only a small amount
of UV-A to the retina. This is why an aphakic patient is at a higher
risk for UV retinal damage (6 ).

A laser produces a highly intense, coherent beam of light. It results from a


chain reaction of photon emissions:
1 photon —>2 photons — > ... Avalanche (4.2)

The high intensity light associated with lasers is exploited clinically


during laser photocoagulation, a process in which a laser is used to
cauterize the leaky blood vessels associated with wet AMD. Unfor­
tunately, only a minority of patients benefit significantly from such
treatment (3), and it is no longer a common practice. Laser pho­
tocoagulation is more commonly used in the treatment of diabetic
retinopathy.

Copyright 2014 by KMK Educational Services, LLC


CHAPTER 4 . PHYSICAL OPTICS 173

Population inversion
To initiate this photon chain reaction, we need to achieve a situation where the
majority of atoms are in an excited state. This population inversion is achieved
by adding energy to the system (called pumping). Pumping could be from
UV light, chemical reactions, or any number of other sources.
M etastable states
Key ingredients to laser function are metastable states. These are long lived
intermediate energy states of the atoms.
Laser Functioning
1 Pump atoms to higher energy state to achieve population inversion.
2 Atoms will decay (non-radioactively) to the metastable energy states,
T 'm e f a •

3 Atoms will eventually decay to the ground state E q.


4 A photon will be released during this decay process.
5 The released photon stimulates the release of other photons (a process
termed stimulated emission).
Lasers in the Clinic
Laser technology is widely used in clinical settings. We briefly review three ex­
amples: confocal scanning laser tomography (CSLT), scanning laser polarime-
try (SLP), and optical coherence tomography (OCT) (11).
CSLT: In CSLT, a scanning laser ophthalmoscope is used to image the retinal
surface. Specifically, a narrow laser beam scans the retinal surface and is re­
flected backwards through a small diaphragm that is positioned such that only
light reflecting from the focal plane of the retina being imaged is collected. A
number of such planar confocal sections are collected and computationally ar­
ranged into a three dimensional representation of the retinal surface, including
the optic disk. This can be used to measure thickness and the cross sectional
area of the Retinal Nerve Fiber Layer (RNFL). The commercially available
CSLT instrument is the HRT (Heidelberg Retinal Tomograph).
SLP: SLP measures RNFL thickness using a scanning laser ophthalmoscope
combined with a polarimeter, which is a device used to measure the change
in polarization of light reflected back from the retina (see Section 4.6), The
naturally anisotropic RNFL produces a change in polarization of the laser light
Copyright 2014 by KMK Educational Services, LLC
174 4.2. LASERS
that passes through it, and this change is directly proportional to the thickness
of the RNFL. Thus, the polarization change gives information about the RNFL
thickness. Optically speaking, the RNFL acts as a phase retarder (Section 4.6).
A common commercial scanning laser polarimeter is the GDx VCC.
OCT: OCT is used to construct detailed two dimensional images of the
RNFL, macula, and optic disc. A near infrared laser beam is focused on the
retina, which is then reflected backwards. The temporal delay of the reflected
beams is measured and used to reconstruct an image of the retinal surface. This
is similar to an ultrasound, although it uses light rather than sound waves.

CSLT can precisely image the topography of the optic disc. In ad­
dition, SLP and OCT are excellent for measuring RNFL thickness;
the latter is also commonly used to diagnose macular disease.

Below we review some additional uses of specific laser types (9).


Argon:
• Used on the retina, specifically for pan retinal photo coagulation and
focal treatment of the macula for conditions such a neovascularization
and macular edema.
• Used on the trabecular meshwork in the angle of the eye to increase
outflow in patients with open angle glaucoma (argon laser trabeculoplasty
(ALT)).
• Can be used for an LPI (see below).
• Can be used to treat retinal tears.
Excimer and LASIK:
• Excimer lasers are used for LASIK and PRK refractive surgeries to ab­
late corneal tissue for reshaping.
• Measured 10P will decrease following LASIK because of the thinner
cornea.•
• For LASIK, one must consider four quantities: the flap thickness, the
amount of residual stroma, and the diameter and depth of ablation (8 ).
Copyright 2014 by KMK Educational Services, LLC
CHAPTER 4 . PHYSICAL OPTICS 175

• Flap thickness ranges from 160 microns (thin corneas and large correc­
tions) to about 20 0 microns (thick corneas and large corrections) (8 ).
• Approximately 250 microns of central stroma should be conserved follow­
ing LASIK (8 ).

The ablation depth per diopter for traditional LASIK is given by


Munnerlyn’s equation, which says
d2
l= (4-3)
Here l is the ablation depth (in microns) and d is the ablation
diameter in mm (also called “ablation zone”, this number is typi­
cally around 6 mm) (8 ). As a rough estimate, the ablation depth
is about 12 microns per diopter. The ablation depth is about 40
percent larger for wavefront-guided LASIK surgery.

Nd (Neodymium): YAG:
• Used to remove posterior capsular opacities that can occur months to
years after cataract extraction and lens implantation.
• Q-switched Nd:YAG laser is used for selective laser trabeculoplasty, a
variation of ALT that is applied to the trabecular meshwork to lower
IOP.•
• Used for LPI (laser peripheral iridotomy) for treatment of angle closure
glaucoma.
• Can also be used to make a laser sclerotomy in trabeculectomy surgery.
• Can be used for cyclodestructive procedures on the ciliary body to stop
aqueous production.
Helium neon:
• Used to illuminate the retina to view the fundus.
Krypton:
• Similar uses as the argon laser (photocoagulation of the retina).
Copyright 2014 by KMK Educational Services, LLC
176 4.3. DIFFRACTION
Holmium laser:
• Used to create a laser sclerotomy in trabeculectomy surgery.
• Also called infrared holmium YAG laser.

If one shines a coherent light beam on a wall, it will appear to have speckles as
a result of interference with other beams reflecting from the rough surface of
the wall (6 , pp. 465-68). These speckles remain clear, even when the wall itself
is blurred due to some optical limitation.
Speckle Effect in the Clinic: When a patient is asked to move his/her
head while observing the Speckle Effect, the speckles will either:
1 Appear stationary.
2 Move along with head motion.

3 Move opposite to head motion.


This gives insight into the refractive state of the patient’s eye.

• Emmetrope — >No motion


• Hyperope — >With motion
• Myope ■— y Against motion

- SECTION 4.3 ------------------------------------------------------------------------------------------

Diffraction *1
2

Diffraction refers to the bending of waves around obstacles (6 , ch. 22) (5,
ch. 9,10). It is classified as either
1 Fraunhofer: far field diffraction.
2 Fresnel: near field diffraction.
Copyright 2014 by KMK Educational Services, LLC
CHAPTER l PHYSICAL OPTICS 177

Resolution is limited by diffraction, even for a perfect optical system with ideal
focus and no aberrations. As light passes through a circular aperture, it creates
an Airy’s Disk. This disk is larger for smaller apertures. Two objects cannot
be distinguished when the two corresponding Airy’s disks overlap. Mathemat­
ically, this is known as Rayleigh’s Criteria and is given by sin0r = where
0r is the angle separating two just resolvable objects (6, ch. 21) (5, ch. 10).

Estimating Pinhole Acuity


Rayleigh’s Criteria can be used to estimate a patient’s expected visual acuity
when he/she looks through a small pinhole. In this case, d is the pinhole
diameter and 0 will correspond to the patient’s MAR, which is just (Snellen
Fraction)-1 (7). Because 0r is often very small, in practice, we can rewrite
Rayleigh’s Criteria in a form that is useful for optometrists (assuming A = 555
run) as
MAR= a (4.4)
In this case, MAR is the estimated minimum angle of resolution measured in
arc minutes and d is the diameter of the pinhole in mm. A pinhole of diameter
d = 2.3 mm corresponds to approximately 20/20 (diffraction limited) vision.

i - SECTIO N 4.4

Thin Films

An antireflective thin film can be used to create destructive interference between


two reflecting light waves (6, ch. 21) (5, pp. 425-31). This can be used, for
example, to reduce glare in sunglasses. Consider light traveling in air (n = 1)
and striking a thin film (n — n j) that coats a lens of a material with index
n = n/J. We choose the film so that n i > ri f > 1. The minimum film thickness
providing destructive interference can be found from
A (4.5)
inf '
where d is the thickness, rif is the index of refraction of the him itself, and A is
the wavelength of light (If wavelength is unspecified in a problem, one
should use 555 nm for visible light, because that is near the center
of the human sensitivity curve).
Optical Thickness: By convention, we define optical thickness (OT) as fol­
lows:
OT = dnj, (4.6)
where d is the thickness of the film.
Copyright 2014 by KMK Educational Services, LLC
178 4.5. SCATTERING
Minimum Thickness Version 2: Now we can write the minimum thickness
formula another way as
OT =
4 (4.7)
Ideal Thin Film: To choose a film material that minimizes reflection, use
the following simple formula (6, pp. 471):
n / = y/ni'nL, (4.8)
where n/ is the index of the film to be used, m is the index of the initial
medium (usually air), and til is the index of the lens material.

Exam ple 4 .1 : W h a t is th e m in im u m thickness n eed ed for an a n tire fle c tiv e film ( n = 1.8 )
u seful fo r in c id e n t lig h t o f w avelen gth 520 n m ?

Solution 4 .1 : F irs t fin d th e o p tic a l thickness needed.

OT = ~ = 130 nm (4.9)
4
S ince O T is d efin ed as d n , we have

, 130
d = — (4.10)
n
F o rn = 1.8 :
d = 72.2 nm . (4 .11)

I— SECTION 4.5

Scattering
We now review some topics related to light scattering (6, ch. 23) (5, ch. 4).
In general, light is scattered when the medium through which it travels is not
homogeneous.

Particles causing Rayleigh scattering are much smaller than the wavelength of
light. Rayleigh scattering is wavelength dependent:
I oc
1 (4.12)
A4’
where I is the intensity of scattered light and A is its wavelength.

Copyright 2014 by KMK Educational Services, LLC


CHAPTER l PHYSICAL OPTICS 179

Rayleigh scattering underlies the reddish-orange colors seen during


sunsets (see (6, pp. 500-01))

h": -'I '.-'h ;- c.'E' ;- ' V .?

Particles causing Tyndall scattering are much larger than the wavelength of
light. The scattering is due purely to geometric optics and is wavelength inde­
pendent.

Tyndall scattering underlies the white appearance of the clouds


(See (5, pp. 500-01))

r - SECTIO N 4.6

Polarization
We introduce some basic concepts of light polarization (6, ch. 23) (5, ch. 8). As
we stated earlier, light consists of oscillating electric (E) and magnetic (B) field
vectors. Both E and B vectors are in a plane perpendicular to the direction of
travel. Polarization allows us to talk about the shape that the E vector traces
out in that plane.

Light is often polarized in one of three ways (6, ch. 23) (5, ch. 8):
Linearly Polarized Light: Oscillations of the E vector (in time) all fall on
a line. The magnitude of E is changing, but its direction is constant.
Circularly Polarized Light: Oscillations of the E vector (in time) trace out
a circle. The E vector is rotating, but its magnitude is constant. Math­
ematically, this is due to the addition of two linearly polarized waves of
equal amplitude and a phase difference of 9 = tt/2.
Elliptically Polarized Light: Oscillations of the E vector (in time) trace
out an ellipse. The E vector is rotating and changing in magnitude.
Mathematically, this is due to the addition of two linearly polarized waves
of different amplitude and a phase difference of 0 = tt/2.
Copyright 2014 by KMK Educational Services, LLC
180 4.6. POLARIZATION

Reflection can change the state of polarization of a light wave (6, ch. 22 ) (5,
ch. 8), Imagine unpolarized light traveling through one medium (m) and into
a second medium (n^). Let the interface be a plane, and assume the light
strikes the plane at some angle 0. Brewster’s law then states that at some
angle of incidence <f>= the light ray reflected from the interface and the
ray refracted (passing through the interface) are perpendicular. This leads
to reflected light plane polarized in the plane of the surface. Mathematically,
Brewster’s angle is given by
tan (pB — —
m• (4.13)

Brewster’s Law is included for completeness, but we note that cal­


culations involving trigonometric functions will not be on the exam.

Polaroid sunglasses: Brewster’s Law is directly applied in the design of


Polaroid sunglasses. In particular, they are made to reduce glare arising from
horizontal surfaces. Specifically, they will NOT transmit the polarized light
(that is, the light polarized because it struck the surface at Brewster’s angle)
reflected from the surface.
Polarizers ' v• i....'
./ ,v.
ikes.-: fllsliflii! 1
A polarizer, as one might guess from its name, polarizes light.
Principles of Operation
When light hits a metal sheet, electrons move in response to the electric and
magnetic fields. That is, the electrons absorb energy from the light and use it to
oscillate. If the sheet is replaced with a vertical wire grid, the electrons can only
move up and down. As a result, electrons only absorb light that is vertically
polarized (that is, light whose electric field produces a vertical oscillation in the
electrons). As a result, horizontally polarized light passes through (6, ch. 22).
Absorption axis: The axis parallel to the wire grid in the example above is
the absorption axis. E field components parallel to the absorption axis
are NOT transmitted.
Transmission axis: The axis perpendicular to the absorption axis is the trans­
mission axis. E field components parallel to this axis are transmitted.
Copyright 2014 by KMK Educational Services, LLC
CHAPTER 4 . PHYSICAL OPTICS 181

When unpolarized light is incident on a polarizer, the intensity of


the light transmitted, /, is half that of the incident light, I q.
I=! (4.14)

Malus Law
When plane-polarized light is incident on a polarizer, the Malus Law gives the
intensity of the transmitted light as J —Jo cos2 0, where I is the intensity of
the transmitted light, Jo is the intensity of the incident light, and $ is the angle
between the incident polarization and the transmission axis of the polaroid.

Calculations involving trigonometric functions will not be on the


exam, but you should note that in the special case where the in­
cident polarization and the transmission axis of the polaroid are
separated by 45 degrees, the Malus law simplifies to J = I / 2 . q

Exam ple 4 .2 : U n p o la riz e d lig h t o f in te n s ity I q passes through a single p o la rize r. The
tra n s m itte d lig h t th e n passes th ro u g h a second p o la riz e r ro ta te d 45 degrees w ith resp ect
to th e firs t p o larizer. W h a t is th e in te n s ity o f th e lig h t w hich is tra n s m itte d th ro u g h th e
second p o la rize r?

Solution 4 .2 : L ig h t e x itin g th e first p o la riz e r has in te n s ity h = Jo/2. L ig h t tra n s m itte d


th ro u g h th e second p o la riz e r th e n has an in te n s ity given by M a lu s L a w in th e sp e c ia l
case $ = 45, w hich reduces in te n s ity b y an a d d itio n a l fa c to r o f two. T h erefore, th e fin a l
in te n s ity is I= I i ( 2 = /o/4, m e a n in g t h a t 2 5 p e rc e n t o f th e in c id e n t lig h t is tra n s m itte d
( 7 5 p e rc e n t has been a b so rb ed ).

References
[1] A. Carter (2001). Classical and Statistical Therm odynam ics. Prentice Hall.
[2] T, Fannin and T. Grosvenor (1996). Clinical Optics, find Edition. Butterworth-Heinemann.

Copyright 2014 by KMK Educational Services, LLC


182 . , POLARIZATION
4 6

[3] S. Fine, J. Berger, M. Maguire, A, Ho (2000), N Engl J Med, 342(7), 483-92.


[4] D. Griffiths (1999), Introduction to Electrodynam ics, 3rd Edition, Prentice Hall, 1999.
[5] E, Hecht (2002). Optics, J,th. Edition. Addison Wesley Longman.
[6] M. Keating (1988), Geometric, Physical, and Visual Optics. But 1erworth- Heinem an n.
[7] Lakshminarayanan, V, and E. Bennett (2006), R eview Q uestions fo r the N B E O E xam ina­
tion, P art One, Butterworth Heinemann,
[8] J, Machat, S. Slade, L, Probst (1998), The A rt of Lasik, Slack Incorporated.
[9] M. Misiuk-Hojlo, P, Krzyzanowska, A, Hill-Bator (2006), Proceedings on Sym posium of
P hotonics Technologies fo r 7th Framework Program , Wroclaw.
[10] S. Schwartz (1999). Visual P erception: A Clinical O rientation, 2nd Edition. McGraw-Hill.
[11] G. 'Prick, F, Calotti, and B. Skarf (2006), J. Neuro O pthalmal, 26(4), 284-95.

Copyright 2014 by KMK Educational Services, LLC


Chapter 5
Physiological Optics I

Kevin B, Wood, Ph.D.


Sarah Dougherty Wood, O.D., F.A.A.O,

183
(
(
(
(
c
(
(
(
(
(
(
(
(
(

(
(
(
(
(
(
Copyright 2014 by KMK Educational Services, LLC (
(
(
(
CHAPTER. 5. PHYSIOLOGICAL OPTICS I 185

SECTION 5.1

O phthalm ic Instrum ents


We first review some basics of common ophthalmic instruments (8) (5) (1) (10) (11) (2).
Ophthalmoscope
An ophthalmoscope allows the examiner to see the inner eye, including the
retina and optic nerve (10) (1) (2, pp. 481-82).
Types: There are two main types of ophthalmoscopes: direct and indirect
(BIO). The two types differ in optical properties and complexity. In
addition, the direct is a monocular device, while the BIO is a binocular
device.

1 Larger field of view


Indirect ophthalmoscope 2 Larger depth of focus
3 Smaller magnification
4 Inverted image

How it works: The patient’s retina is illuminated and then observed; opti­
cally speaking, the goal is to create a conjugate image of the patient’s
retina (or whatever structure is viewed) on the examiner’s retina. When
using the direct ophthalmoscope, a series of lenses is used to correct for
ametropia in both the patient and the examiner, thereby maintaining the
required conjugacy.
When using the BIO, a condensing lens is used to form an intermediate,
inverted, real image. To view the fundus, the examiner must focus on this
image. Optically, this means another lens (or accommodation) is needed
to see the intermediate image, which is located typically about an arm’s
length away from the examiner.

Lensometer
A lensometer measures properties of lenses, including the back vertex power
and the prismatic properties of the lens (5, pp. 65-74) (8, pp. 256) (2, pp. 482).
Copyright 2014 by KMK Educational Services, LLC
186 5. 1. OPHTHALMIC INSTR UMENTS

Standard Test Lens


Lens

Figure 5.1: The simplified optics setup for use of a lensometer involves a stan­
dard lens, a test lens, and a telescope system.

How it works: One views a target object (using a telescope system) through
both a standard lens (of some known power) and the lens of interest (test
lens). The target is typically a set of perpendicular lines. The setup is
shown in Figure 5.1:
1 To view the target (perpendicular lines) in focus, parallel light must
strike the viewer’s eye (and therefore parallel light must strike the
telescope).
2 This means parallel light is leaving the test lens. How do we make
this happen?
3 Light from the standard lens must converge at JF, the primary focal
point of the test lens.
4 Now we simply move the target (relative to the lenses) until this
happens.
M athematical description: The following equation comes from Newton’s
formula:
3 = f F vl (5.1)
where x is the distance that the target is moved, / is the focal length of
the standard lens, and Fv is the back vertex power of the test lens. Note
that when x is negative, Fv is negative.
Practical Use: The following usage instructions (3) describe the basic recipe
for standard (cross lined) lensometer use.*
* First, focus the eyepiece.
Copyright 2014 by KMK Educational Services, LLC
CHAPTERS. PHYSIOLOGICAL OPTICS I 187

• Blur the target by turning the power wheel in the plus direction;
then slowly step backwards (in the minus direction) until the sphere
lines are clear. Note that the sphere lines must clear first; if the
cylinder lines clear first, you must adjust the axis.
• Adjust the axis to further clear the sphere lines, if needed. Make
note of the sphere and axis.
• Continue turning the power wheel in the minus direction until the
cylinder lines are clear. The difference between this power and the
power at which the sphere lines are clear is the minus cylinder power.

A lensometer is also used to verify the amount and direction of


prism in a lens. To do so, one compares the location of the cross
hairs formed by the sphere and cylinder lines with the location of a
“bulIs-eye” image of concentric circles. We note here some generic
rules for verifying prism (3):
• If the center of the cross hairs falls to the left of the center
of the “bulls-eye”, the prism is base out (if the lens is OD) or
base in (if the lens is OS).
• If the center of the cross hairs falls to the right of the center
of the “bulls-eye”, the prism is base in (if the lens is OD) or
base out (if the lens is OS).
• If the center of the cross hairs falls above the center of the
“bulls-eye”, the prism is base up.
• If the center of the cross hairs falls below the center of the
“bulls-eye”, the prism is base down.

Exam ple 5 .1 : L e t's s ay w e a re using a + 2 5 .0 0 s ta n d a rd tens. To achieve a p p ro p ria te


focus, we m u s t m o ve th e ta r g e t 2.0 m m forw ard. W h a t is the back vertex p o w e r o f th e
lens b ein g te s te d ?

Solution 5 .1 : U sing E q u a tio n 5 .1 , w e have

0.002 m = ( 1 /2 5 ) 2.FI, (5.2)

S o lv in g fo r F v g ives F v = + 1 ,2 5 D .

Copyright 2014 by KMK Educational Services, LLC


188 5. i. OPHTHALMIC INSTR UMENTS

Exam ple 5 .2 : W h e n using a lensom eter, th e sphere p o w e r is fo u n d to be - 3 .0 0 D a n d


th e axis is 1 25. T h e p o w e r w heel is then ro ta te d in th e m in u s d ire c tio n u n til reach in g a
p o w e r o f - 4 .0 0 D , a t which p o in t th e c ylin d er lines b ec o m e clear. W h a t is th e p o w e r o f
th e lens?

Solution 5 .2 : T h e sphere p o w er is - 3 .0 0 D , a n d th e c y lin d e r p o w e r is th e d ifferen ce


b etw e e n th e tw o p o w er readings given, in th is case, th e ab s o lu te d ifference is 1 .0 0 D .
T h e p rescrip tio n w ou ld be given in m in us cy lin d er fo rm as - 3 .0 0 - 1 .0 0 x 125.

Exam ple 5 .3 : A p rescription calls for 3 prism d io p te rs base o u t prism O D . H o w sh o u ld


th e lens be c e n te re d In th e lenso m eter (b e fo re e d g in g ) fo r verifica tio n ?

Solution 5 .3 : T h e lens should have 3 prism d io p te rs base o u t, so w e do n o t w a n t to align


th e cross hairs fo rm ed b y the sphere a n d cylind er lines w ith th e c e n te r o f th e “b ulls-eye"
o f co n c e n tric circles. Instead, we p ositio n th e lens so t h a t th e cross hairs fa ll to th e le ft o f
th e c e n te r o f th e bullseye. Specifically, th e cross h airs sh o u ld be c e n te re d on th e 3 rd rin g
(in d ic a tin g 3 p rism d io p te rs ) a n d p o s itio n e d to th e le ft o f th e c e n te r o f th e "bulls-eye” ( 3 ).
T h e lens can th e n b e s p o tte d . N o te th a t base o u t p rism O S w o u ld call fo r th e cross hairs
to fa ll to th e rig h t o f th e c e n te r o f th e "bulls-eye."

Hand Neutralization
While it is not an instrument, the technique of hand neutralization is used for
a similar purpose as the lensometer (8, pp. 387-88) (5, ch. 9) (3, ch. 7). Hand
neutralization utilizes with or against motion to determine the power of an
unknown lens. Recall that lenses have prism properties. When we view an
object through a lens, its apparent position is shifted. Now move the lens. The
object will appear to move with or against the motion of the lens.
Minus lens: Like two prisms stacked apex to apex. Motion is with motion.
Plus lens: Like two prisms stacked base to base. Motion is against motion.
No power: No prismatic effect -> No apparent motion.

Copyright 2014 by KMK Educational Services, LLC


CHAPTER 5. PHYSIOLOGICAL OPTICS I 189

Thin lenses: To determine the power of an unknown lens, add


another lens of known power to neutralize the apparent mo­
tion effect. When motion is stopped, the power of the known
lens is equal in magnitude and opposite in sign to that of the
unknown lens.

Radiuscope
A radiuscope measures the radius of curvature of hard contact lenses (5, pp. 402-
404) (8, pp. 183-84) (2, pp. 483)
How it works: The scope forms a target image at some point P between the
viewer and the lens of interest. Light from this image will reflect off the
lens and form another image at some point Q. When will the viewer see
a clear image? There are two cases:
1 P is located at the surface of the lens. This is because the object
distance is zero, so the image distance is also zero. Hence P and Q
are at the same location.
2 P is located at the center of curvature of the lens. In this case, the
object distance is the radius of curvature, so the image distance is
also at the radius of curvature, and m = —1. P and Q are again at
the same location.
Take home summary: There are two locations at which the lens can sit
(along the optical axis) such that the viewer sees a clear image. We
simply need to move the lens from one such location to the other and
measure how far it had to be moved. This distance is the radius of
curvature.

The distance between the two positions of focus (the clear images)
corresponds to the radius of curvature.

Keratometer
A keratometer measures the radius of curvature (and therefore, the effective
refractive power) of the center of the cornea along certain axes (5, pp. 377-
79) (8, pp. 184-85) (2, pp. 483-84).
Copyright 2014 by KMI< Educational Services, LLC
190 5.1. OPHTHALMIC INSTR UMENTS
How i t works: The cornea acts as a convex mirror and creates an image of an
object (called a mire). One then measures the size of the reflected image.
We can use this to determine the radius of curvature of the cornea.

New keratometers give a readout in terms of power rather than ra­


dius of curvature. They do this by assuming n — 1.3375 and treat­
ing the cornea like a single spherical refracting interface (SSRI). So
to convert between power readings (units are indicated as “DK”,
with the <!K” indicating it is from keratometry) and radius of cur­
vature, we have
1.3375 - 1 0.3375
F =
r r (5 .3 )

where r is measured in m. Equivalently, we could write


F ^1 214 (5 .4 )

with r measured in mm.

Exam ple 5.4: A m ire is 8 cm from th e cornea a n d 5.5 cm ta ll. T h e re fle c te d im ag e is


2.3 m m . W h a t is th e rad ius o f c u rv a tu re fo r th e cornea? W h a t w ould
m e a s u re d to b e
th e re a d o u t on a n ew k e ra to m e te r be?

Solution 5.4 :
m = I/O = 0 .2 3 /5 .5 = 0.042. A lso, w e k n o w
m = 6.042 = L / L ' a n d
L = - 1 f l = —1/0 ,0 8 = -1 2 .5 0 D . So
m = 0.042 = —1 2 . 5 / 1 / , g iv in g
L* = -2 9 9 .0 4 D; N o w ' ' '
F = L' - L - -2 9 9 .0 4 - ( -1 2 .5 0 ) = -2 8 6 .5 4 D, so
F = —286.54 = 2n / r , g iv in g
r = 2 / ( —286.54) = - 7 . 0 m m .

O f coarse, th e k e ra to m e te r w ould re a d o f f a d iffe re n t value. I f we t r e a t th e cornea as an


S S R I, th e rad ius o f c u rv a tu re b ecom es positive.

F k = (1.3375 - l)/0 .0 0 7 « 48.21 D K . (5.6)

T h is is th e value th a t w o u ld be read o f f th e k e ra to m e te r.

Copyright 2014 by KMK Educational Services, LLC


CHAPTER 5. PHYSIOLOGICAL OPTICS I 191

The same principles can be used to determine curvature (effective


power) of a non-spherical cornea. In this case, the readout gives
power in the principal meridians.

Lens Clock
A lens clock is used to measure the sag of a, lens. In turn, this can be specified
in terms of a lens power using other properties of the lens (5, pp. 27-28) (8,
pp. 131-32).
How it works: A lens clock allows you to directly measure sag of a lens by
adjusting a movable pin. Based on the sag value and an assumed value of
n (the value used for calibration), some lens clocks give a power readout.
Note that if you use a lens clock calibrated with the correct n, the readout
for power is already correct. If not, you have to do a quick calculation.
M athematical Description: The following equation is useful to correct for
an incorrectly calibrated lens clock:
(5.7)
where ul is the index of refraction of the lens, n^c is the index of refrac­
tion value used to calibrate the lens clock, Fp, is the power of the lens,
and Flo is the power readout of the lens clock.

Exam ple 5.5 : You use a lens c lo c k c a lib ra te d w ith n = 1.5. You w a n t to m easu re th e
p o w e r o f a lens m a d e fro m m a te ria l n = 1.7 . T h e p o w e r re a d o u t o f th e lens c lo ck is
+ 6 .0 0 D . W h a t is th e a c tu a l p o w e r o f th e lens?

Solution 5 .5 : U sing e q u a tio n 5 .7 , w e have

1 . 7 - 1.0
(6.00) = + 8 .4 0 D . (5.8)

Slit Lamp Biomicroscope


A slit lamp biomicroscope is a compound microscope commonly used to view
the anterior segment of the eye (1).
How it works: The slit lamp consists of a number of components:
Copyright 2014 by KMI< Educational Services, LLC
192 5.2. OPHTHALMIC CHARACTERISTICS OF LENSES
1 A Keplerian telescope makes up the eyepiece.
2 An inverting prism corrects for the upside-down image produced by
the Keplerian telescope.
3 A Galilean telescope is used to further magnify the image.
4 An objective lens in the microscope works similarly to a very pow­
erful bifocal segment. That is, it allows a system optimized for dis­
tance viewing (in this case, the telescope combination) to be used
for near objects.
5 A complex illuminating system generates a slit-shaped beam whose
size and orientation can be adjusted to offer flexibility in viewing
different structures of the eye.
6 A binocular viewing system creates a stereoscopic image, creating a
three dimensional view of the optical structures.

A fundus lens is used in conjunction with a slit lamp to view the posterior
segment of the eye, including the retina, optic nerve, and macula. Typical
fundus lens powers are +78.00 and +90.00 D.
How it works: These high powered lenses create a real, inverted image of the
observed structure. This image becomes the object for the objective of
the slit lamp biomicroscope.

Higher powered fundus lens —>Lower magnification/increased field


of view.

“ SECTION 5.2 ------------------------------ ------------------------------------ —

Ophthalmic Characteristics of Lenses


We now introduce some fundamental properties and conventions related to
ophthalmic lenses (3, ch. 15,16) (5, ch. 2) (8, ch. 9).

'Standards;
Some ANSI standards are provided in Table 5.1, Table 5.2, and Table 5.3 (16).
Please keep in mind that ANSI standards are regularly updated.
Copyright 2014 by KMK Educational Services, LLC
CHAPTERS. PHYSIOLOGICAL OPTICS I 193

Sphere Power (D) Tolerance (D)


<±6.50 ± 0.13
> ±6.50 ±2% sphere power
Cyl Power Tolerance
< 2.00 ± 0.13
2.00-4.50 ±0.15
> 4.50 4% cyl power
Table 5.1: ANSI Z80.1-2010 Power Tolerances (16)

Cyl Power (D) Axis Tolerance (degrees)


< 0.25 ± 14
0.25-0.50 ±7
0.50-0.75 ±5
0.75-1.50 ±3
>1.50 ±2
Table 5.2: ANSI Z80.1-2010 Axis Tolerances (16)

Parameter Tolerance
Thickness ± 0.3 mm
Warpage 1.00 D
Base Curve ± 0.75 D
Impact Resists 5/8 in steel ball from 50 in.
Table 5.3: ANSI Z80.1-2010 Tolerances (16)

Safety Eyewear
As of this writing, the most current ANSI standards for protective eyewear
(Z87.1-20120) are organized by hazard type. Several relevant requirements are
given below (18):*
* Minimum coverage area: an ellipse of 40 mm by 33 mm centered on the
geometric center of the lens.
• Markings:
— “H” —>frames for smaller heads (minimum coverage area is reduced
to 34 mm by 28 mm)
Copyright 20.1.4 by KMK Educational Services, LLC
194 5.2. OPHTHALMIC CHARA CTERISTICS OF LENSES
— “H-’—>impact rating
— “W” shade number —> welding
— “U” scale number —>UV filters
— “R ” scale number —> IR filters
— “L” number visible filter
— “V” scale number —>variable tints
— Frames and lenses must carry manufacturer logos, as well as an
impact mark (e.g. Frames must also include a standard mark
(“Z87-2”).
• High mass impact (drop ball test): Pointed projectile, 500 g, dropped
from 50 inches.
• High velocity impact: Steel ball, 0.25 inches in diameter, fired at 150 feet
per second.

Lens form characterizes the relationship between the front and back surface
geometries of a lens.
1 Plano-convex and plano-concave: One surface is flat, the other is curved.
2 Biconvex and biconcave: Both surfaces are either convex or concave.
3 Equiconvex and equiconcave: Half of the total power is due to the front
surface and half is due to the back surface.
4 Meniscus: Convex front surface and concave back surface.
5 Plano Cylinder: One flat surface and one cylinder surface.
6 Toric: One toric surface and spherical surface.
Lens Transposition and Form:
Lenses can be made in several different physical forms and still have the same
effective power. For example, you can make an equivalent plus cylinder lens for
any given minus cylinder lens. In reality, most lenses used in the USA are minus
cylinder lenses (the toric surface is on the back). However, prescriptions are
written either way. They indicate the desired power and properties of the lens,
but are not necessarily indicative of the lens form that will, actually be given
to the patient. For example, a prescription might be written in plus cylinder
form, but the patient (in the USA) will probably be given an equivalent minus
cylinder lens.

Copyright 2014 by KMK Educational Services, LLC


CHAPTERS. PHYSIOLOGICAL OPTICS I 195

You should be intimately familiar with lens transposition.


_________________________________________________

-v;>v
Base curves are just a standard lens curve used by manufacturers (5, pp. 29,154-
55) (3, pp. 404-410). We specify the curve in terms of its power. The base
curve is always on the front surface for single vision lenses. Several common
conventions follow:

Base curves for spectacle lenses


• Spherical lens: Base curve is the front sphere curve.
• Plus Cylinder: Base curve is the flatter of the front (toric) surface curves.
The other front curve is called the cross curve. The back curve is called
the sphere curve.
• Minus Cylinder: Base curve is the front sphere curve. The back flatter
curve is called the toric base curve. The other back curve is called the
cross curve.

Base curves for contact lenses


• Base curves are typically on the back surface of contact lenses.

Exam ple 5.6: A contact lens has a base curve measured to be +42.00 DK. What is the
radius of curvature of this base curve, in mm?

Solution 5.6: This problem tests whether you understand DK notation, DK indicates
that this is the power readout corresponding to keratometry. To convert to a radius of
curvature, we have
F _ 3379}
r
337.5 (5-9)
r = ------ = 8.04 mm
42.00

Copyright 2014 by KMK Educational Services, LLC


196 5,2. OPHTHALMIC CHARACTERISTICS OF LENSES

.........
. : .
- ■
S
...... ."A ......V . :% .
■ * * 1 ‘
/Vv-Kvi

A minus lens is thicker in the periphery than in the center, while a plus lens
is thicker in the center than the periphery (5, pp.75-8) (8, pp. 153). For a lens
with two curved surfaces, we have

tc = Si —S2j (5.10)
where tE is the edge thickness, .si is the sag distance of the first surface of
the lens, S2 is the sag distance of the second surface of the lens, and tc is the
center thickness. We measure Si from arc to chord, with right being the positive
direction and left the negative direction, tc and tE are always positive. Note
that for a lens with one flat surface, si or S2 is zero.

When calculating edge thickness, it is good to draw a picture of


the lens. Then you do not have to worry about the sign convention
for Si but can instead just use geometry.

Recalling the equation for power of a curved interface, Equation 3.3, we can
get the following useful formula relating s to F for a surface:
2(n —l)s (5.11)
h2 ’
where F is the power of a lens surface, n is the index of refraction of the lens,
s is the sag, and h is half the chord length (half the diameter of the lens). We
can combine Equation 5.11 and Equation 5.10 to relate thickness to power for
a thin lens (5, pp. 76):
(F1 +F 2)hi (5.12)
tc - tE 2(n —1)

Copyright 2014 by KMK Educational Services, LLC


CHAPTER 5. PHYSIOLOGICAL OPTICS I 197

Rather than memorizing, the above equations can be conceptually-


summarized as follows:
ts tto o s i,s2 «+ r i,r 2 <+ FlyF2 (5.13)
The three arrows summarize the following intuitive steps (from left
to right):
1 Sags can be related t o t s and to by drawing a picture of the
lens.
2 Use s — ,2 to relate sag to the radius of curvature, h is the
semi-diameter of the lens.
3 Use F = to relate power to the radius of curvature.

Exam ple 5 .7 : A biconvex iens is clocked. The lens has the following properties: diameter
—44 mm, sag of the front surface = 4.5 mm, sag of the back surface is 3.2 mm, n ~
1.50. What is the power of each lens surface?
Solution 5 .7 : We use Equation 5.11 twice, once for each surface. The power of a lens
(assuming it is thin) is just the sum of the two. Since the lens is biconvex, both surfaces
are positive.
Fi = 2(0.50)0.0045/(0.022)2 = +9.30 D,
F2 = 2(0.50)0.0032/(0,022)2 = +6.61 D,

Curves drawn on a power cross to show the curves on which thickness is the
same (8, ch. 18), Figure 5.2. Any two regions lying on the same curve have
the same thickness. If the lines are close together, the thickness is changing
quickly. Note that the change in thickness is slower for lower (absolute value)
lens powers.
“ SECTION 5.3 -----------------------------------------------------------------------

Fram e and Lens Specifications


There are a number of systems for specifying frame geometry. We detail the
most commonly used system (5, ch. 4) (3, ch. 2).
Copyright 2014 by KMK Educational Services, LLC
198 5.3. FRAME AND LENS SPECIFICATIONS

- 2.00

- 5.00

Figure 5.2: Isothickness curves show regions of constant curvature. They are
similar to elevation contour maps: when lines are closer together, the curvature
changes very quickly in that region.

DBL

GCD (Frame PD)

Figure 5.3: The Boxing System for Frame Specification.

Copyright 2014 by KMK Educational Services, LLC


CHAPTERS. PHYSIOLOGICAL OPTICS I 199

The Boxing system augments the traditional Datum system (not covered here)
by adding vertical lines, essentially drawing a box around the lens (Figure 5.3).
Geometrical center (GC): Point on the datum line halfway between the
two vertical lines which are tangent to the edges.
Eye size or Lens size (A): Horizontal length of the box. Use the term eye
size when referring to the frame, lens size when referring to the lens itself.
B distance: Vertical length of the box.
Bridge size (DBL): Shortest horizontal distance between the lenses.
GCD or Frame PD: Horizontal distance between the geometrical centers of
the two lenses.
Effective Diameter: The longest diameter of the lens.
Minimum Blank Size: The smallest size lens blank (un-edged lens) needed
to make the lens. Mathematically, we can use
M = ED + 2(d) -f- 2mm, (5.15)
where M is minimum blank size, ED is the effective diameter, d is the
decentration per lens, and we include an additional 2 mm to account for
possible chipping.
Decentration per lens (d): Decentration of the lens is given by
, frame PD —wearer’s PD _.
d = -------------- g-------------- > (5*16)
where we have
W earer’s PD: Distance between the center of one pupil to the center
of the other pupil.
M ajor Reference Point (M RP): Point on the lens through which the line
of sight (visual axis) passes. Note that this would correspond to the optic
axis if no prism power were needed.

Sometimes a patient needs more plus power for near vision, which can be
accomplished by using a near add (Figure 5.4). An add is just a segment (a
mini lens) added on to the distance prescription (3, ch. 5,19,20) (5, ch. 8).
There are a variety of seg shapes and sizes, many with self-explanatory names
(flat top seg, curved top seg, etc). For some common segments, the distances
between the optical center (OC) of the seg and the seg edge are given below (3).
These numbers are critical for image jump problems.
Copyright 2014 by KMK Educational Services, LLC
200 5.3. FRAME AND LENS SPECIFICATIONS
• Flat top (28 mm or less) -+ 5 mm
• Flat top (35 mm) -+ 4,5 mm
• Flat top (larger than 35 mm) —> 0 mm (the OC is located at the seg line)
• Curve-top, Panoptic, and Ribbon-B —>4,5 mm
• Ribbon R-segs —>7 mm
• Franklin (Executive) segs —» 0 mm (the OC is located at the seg line)
• Round segs —>r, which is the radius of the seg.
Progressives
Hard Design: Short corridor and/or high add power.
Soft Design: Long corridor and/or low add power.
Trifocals
Trifocals have an additional intermediate add, usually one half the power of
the near add.

Exam ple 5.8: Consider the following prescription: 2,00—1.00 x 180, Add+2.00 D. What
is the total power through the near add portion of the lens? How might one choose an
intermediate add for this prescription?

Solution 5.8 : Through the near add portion of the lens, the power is 4.00 —1.00 x 180.
The add is like adding a spherical lens for near vision. If the lens also had an intermediate
add, the power of that add would likely be chosen to be +1.00 D, meaning the total
power of the lens through the intermediate add would be 3.00 —1.00 x 180.

WBmmmmm
Seg width: Longest horizontal dimension of the segment.
Seg depth: Longest vertical dimension of the segment,
Seg height: Distance from the lowest point on the lens to the top of the seg.
Seg drop: Vertical distance between the major reference point (MRP) and
the top of the seg.
Copyright 2014 by KMK Educational Services, LLC
CHAPTERS. PHYSIOLOGICAL OPTICS I 201

Total Inset
Inset j Seg Inset

MRP
Center of Seg (horizontal position}

Figure 5.4: Dimensions of a bifocal lens.

Inset: The inset we used for single vision lenses (which accounts for the OCs of
lenses in frames and wearer’s PD). Inset is the distance from the geometric
center (GC) to the MRP.
Seg inset: The inset accounting for near PD (PD is different for near vision).
The seg inset is the distance from the MRP to the center of the seg.
Total inset: The inset of the seg as measured from the OC of the lens. The
total inset is the distance from the geometric center (GC) to the center
of the seg (or the near reference point).

To find the total inset, use the following relation:


It = I a + I t (5.17)
with It the total inset, I s the seg inset, and I the inset.

Example 5.9: A frame measures 54 x 18. The patient's distance PD is 66 mm and the
near PD is 62 mm. What is the seg inset? What is the total seg inset?

Copyright 2014 by KMK Educational Services, LLC


202 5.3. FRAME AND LENS SPECIFICATIONS
Solution 5.9 : S eg inset a n d to ta l seg inset b o th m easure th e p o s itio n o f th e seg, b u t
th e y do so from d iffe re n t reference p oints. T he seg in set is sim p le to find. I t is h a lf th e
d ifferen ce b etw een th e distance a n d n e a r P D . So in th is case, th e seg inset is (6 6 - 6 2 ) / 2
= 2 mm.

T h e t o t a l seg in set is easiest to fin d by d ra w in g a p ic tu re . T h is is th e d is ta n c e b etw een


th e c e n te r o f th e d istan ce lens a n d th e seg lo cation . S in ce th e n e a r P D is 62, we p la c e
each seg 6 2 / 2 — 31 m m from th e c e n te r o f th e nose. B u t h o w fa r is th e o p tic a l c e n te r o f
each tens fro m th e c e n te r o f th e nose? B y loo king a t a p ic tu re , i t is easy to see t h a t th e
o p tic a l c e n te r o f each lens is lo c a te d ( 5 4 + 1 8 ) / 2 = 3 6 m m fro m th e nose. S o th e c e n te r
o f th e lens is 3 6 -3 1 — 5 m m from th e seg. Therefore, th e t o t a l seg in s e t is 5 m m in w a rd
fo r each eye.*•

Brooks and Borish provide the following recommendations for adjusting the
height of the segment in a bifocal (3). They recommend trying the following
adjustments, in the order given, until the problem is resolved.
• If seg appears too high to the patient:
—Increase pantoscopic tilt
— Decrease the vertex distance
— Spread the pads
— Move pads up by adjusting pad arms
— Stretch the bridge
• If seg appears too low to the patient:
— Narrow the pads
— Move pads down by adjusting pad arms
—Increase vertex distance
— Reduce pantoscopic tilt
— Shrink the bridge

One can use a lensometer to measure the power of a multifocal add (3).
• Turn the glasses around backwards in the lensometer.
* Measure the distance power (that is, the front vertex power); only the
sphere is required.
Copyright 2014 by KMK Educational Services, LLC
CHAPTER 5. PHYSIOLOGICAL OPTICS I 203

* Measure the near power (the front vertex power measured through the
near portion of the lens); only the sphere is required.
• The difference in the near and distance powers gives the add power.

In practice, one can typically estimate the power of the bifocal


add without turning the glasses around in the lensometer. When
the powers involved are small, the error is negligible. However, as
power increases, the error can be significant (3).

Exam ple 5 .10 : T h e fro n t v e rte x p o w e r th ro u g h th e distance p o rtio n o f a lens is m easu red
to b e - f 3 .4 5 -2 .0 0 x 90. T h e fro n t v e rte x p o w e r th ro u g h th e n e a r p o rtio n o f th e lens is
m easu red to be + 5 .4 5 - 2 .0 0 x 90. W h a t is th e a d d p ow er?

Solution 5 .10 : T h e a d d p o w e r is s im p ly th e d ifference in the sphere pow ers (3 ), w hich


in this e x a m p le is 5 .4 5 -3 .4 5 , o r 2 .0 0 D . •

Some typical patient complaints, along with possible solutions, are given below.
Detailed accounts of frame adjustment can be found, for example, in (3; 13).
• Glasses fall down nose —> pull in the temples, bend down temple tips,
and/or pull in nose pads to tighten the fit.
• One lens feels closer to the face than the other lens. —> straighten the
temples.
• Glasses touch the cheek —>reduce pantoscopic tilt, narrow the bridge or
pads to raise frame, and/or narrow the bridge or pads to increase the
vertex distance.
• Glasses too close to the face -» narrow the pads, shrink the bridge, or
decrease face form to move the lenses away from the face.
• Frames sit too low on the face —> narrow the bridge, add pads, or lower
the vertical position of the pads to move the frames up on the face.
Note that the opposite complaints/remedies are also possible. For example, if
the glasses are too far from the face, one can widen the pads, widen the bridge,
or increase face form to move the lenses closer (3).
Copyright 2014 by KMK Educational Services, LLC
204 5.1 ABERRATIONS AND LENS DESIGN

It is important to keep in mind some properties of the most commonly used


lens materials (5, pp.9).

M aterial n Comments Abbe


Ophthalmic 1.523 58.9
No longer commonly
crown glass used
Flint glass 1.58 - 1.69 30 - 40 Used for some fused
bifocal segs
Barium crown 1.54 - 1.62 55 - 59 Used for some fused ;
glass bifocal segs
OR-39 (Plastic) 1.498 58 Light, impact rests- :
taut, and amenable to
large range of optical
designs
Polycarbonate 1.586 30 Excellent impact
(Plastic) resistance, but high
chromatic ab erration
High Index Plas­ 1.54 - 1.66 Large range of prop­
to
-<F
CO
1

tic erties owing to large


ranges of n and Abbe
number,
Trivex 1.53 43 - 45 Relatively light and
impact-resistant

Material properties can be approximated using a six digit code


that contains information about both the index of refraction and
the Abbe value. For example, a particular type of barium crown
glass is 569-561, which means the index is n = 1.569 and Abbe
value is 56.1.

- SECTION 5.4 --------- --------------------------------------------------------------------------------

A berrations of Lenses and Corrective Curve Design


We will cover some important properties of aberrations (3, ch. 22) (5, ch. 6) (8,
ch. 20) (7, ch. 5). Aberrations create optical distortions in image quality or a
deformation of the image plane (8, ch. 20) (3, ch. 22) (7, ch. 6) (5, ch. 6).
Copyright 2014 by KMK Educational Services, LLC
CHAPTER 5. PHYSIOLOGICAL OPTICS I 205

We first discuss wavelength independent aberrations. These aberrations either


distort image quality (spherical, coma, radial astigmatism) or deform the image
plane (curvature of field and distortion).
Spherical Aberrations
Geometric optics is based on the paraxial approximation, which means that
incident rays are close to the optical axis. This yields point images for point
objects. In reality, the paraxial approximation is not always valid, and this is
the basis for spherical aberrations (8, pp. 433-36) (7, pp. 253-58).
Marginal rays: Rays on the periphery (not close to the optical axis) are called
marginal rays.
Longitudinal spherical aberration: The point of focus of a beam of light
depends on where the light strikes the optical system. Marginal rays
focus to a different location compared to paraxial rays. This is known as
longitudinal spherical aberration. Essentially, a point object is no longer
forming a point image. As a result of longitudinal spherical aberration,
the image is blurred. Longitudinal spherical aberration occurs for both
on and off axis points.
Coma;
Coma occurs only for off-axis point sources. It results from the fact that mag­
nification is varied as the height of incident rays above the axis is varied. The
result is an asymmetric comet-shaped patch (8, pp. 436-38) (7, pp. 258-61).

Spherical aberration and coma are not a big consideration in de­


signing ophthalmic lenses because the small pupil size only accepts
paraxial rays. However, in very high powered lenses (+10.00 or
more), it is necessary to compensate for spherical aberration by
using aspheric lenses, which modify the lens surfaces without
changing the power (5).

Aside: Aspheric Lenses


As mentioned above, aspheric lenses can optically correct for lens aberrations.
General uses for aspheric lenses include (3):*
* Correct for aberrations in high powered lenses (less than -23.00 D or
greater than +7.00 D).
Copyright 2014 by KMK Educational Services, LLC
206 5.4. ABERRATIONS AND LENS DESIGN
Flatten the lens, which is cosmetically appealing because it reduces mag­

nification (typically useful for lenses greater than +2.50).
• Reduce the weight of the lens.
• Progressive lenses.

Both coma and longitudinal spherical aberration (length of blur


circle) increase with the square of the system aperture. Lateral
spherical aberration (area size of blur circle) increases with the
cube of the aperture size. In summary, an increase in pupil size
typically leads to a decrease in image quality owing to increased
aberrations.

Radial Astigmatism:
Also called oblique or marginal astigmatism, radial astigmatism is due to rays
hitting the lens or interface obliquely. The power is altered by this “tilt” of
the lens. Specifically, the tangential rays and the sagittal rays are altered
asymmetrically. As a result, a flat object plane yields an asymmetrically warped
image plane (8, pp. 439-43) (7, pp. 261-64).

Radial astigmatism must be considered in designing ophthalmic


lenses. It can be reduced by picking the correct base curves. It
is associated with a “teacup and saucer” image characterizing the
tangential and sagittal foci (5).

Tscherning ellipse: The Tscherning ellipse is a collection of points on base


curve vs. Fv plot which shows the best value of the base curve (for a given
back vertex power) for eliminating oblique astigmatism. The specifics of
the elliptical set of points varies with viewing distance and material. The
ellipse is made of two curves (Figure 5.5):
1 Wollaston Curve
2 Ostwalt Curve (flatter curve)

Copyright 2014 by KMK Educational Services, LLC


CHAPTER 5, PHYSIOLOGICAL OPTICS I 207

F
{Base
Curve)

Figure 5.5: The Tscherning ellipse allows one to find the best base curve for
reducing oblique astigmatism (5, pp. 136-37).

The Ostwalt Curve is usually used to choose a base curve. Both cur­
vature of field and radial astigmatism can be minimized by choosing
the correct base curve. If the incorrect base curve is used, vision
will be reduced in the periphery of the lens (3).

Limits: The ellipse shows that there are limits to the powers for which RA can
be eliminated. Specifically, the upper limit usually falls around +7.50 D;
above that limit, one must use aspheric lenses to correct radial astigma­
tism. The lower limit falls around —22.00 D.

Curvature of Field (Power Error):


Consider an object that is a plane (e.g. a piece of paper). If for each point
on that paper, we find the image point, we will notice that the image points
formed by all points on the paper do not lie in a plane. That is, the image
plane is warped, even for a lens that is not tilted. This would mean the quality
of an image on a flat screen decreases for larger distances (where the effect is
exaggerated) (8, pp. 444-46) (7, pp. 264-66).
Relation to RA: Curvature of field is intertwined with radial astigmatism,
where there is a different warping along two principal axes. However, even
without radial astigmatism, you would still have warping of the plane due
solely to curvature of field.

Copyright 2014 by KMK Educational Services, LLC


208 54- ABERRATIONS AND LENS DESIGN

Petzval Surface The Petzval Surface is the image surface created


by a system with no radial astigmatism. It is still warped due to
curvature of field. For a thin lens in air, we can find the curvature:
K=~,
n
(5.18)
where K is the curvature of the image surface (for a distant ob­
ject), F is the power of the lens, and n is the index of the lens (8,
pp. 444) (7, pp. 265). Curvature of field will be present in an oph­
thalmic lens system any time the Petzval surface does not corre­
spond to the far point sphere of the eye.

Designing Lenses: Radial astigmatism vs. Curvature of field


Lenses that have been corrected for radial astigmatism, curvature of field, or
both fall under the heading “corrected curve” lenses (3).
Point focal lens: a lens corrected completely for radial astigmatism; curva­
ture of field is uncorrected.
Percival form lens: a lens corrected completely for curvature of field; radial
astigmatism is uncorrected.
In practice, lenses are typically designed as a compromise between point focal
and Percival form so that both radial astigmatism and curvature of field are
partially corrected.
Distortion:
Distortion does not cause blur or poor resolution. It results from the fact
that magnification of a point object depends on the object’s distance from the
optical axis. Straight fine objects form straight line images only if the line
passes through the optical axis. All other lines are curved. Examples, similar
to those in (8, pp. 446) (7, pp. 266) are shown in Figure 5.6. Distortion can be
minimized by correctly placing the stops in the system (5).

Barrel is typically associated with a minus lens, and pincushion is


associated with a plus lens. Distortion is minimized by a combi­
nation of lenses known as an orthoscopic doublet. Practically,
distortion is a problem for high powered lenses, such as those used
by an aphakic patient (5).

Copyright 2014 by KMK Educational Services, LLC


CHAPTERS. PHYSIOLOGICAL OPTICS I 209

Figure 5.6: Two types of distortion are barrel (bottom) and pincushion (top),
The pictures on the left represent the object; the image of that object is shown
on the right.

fL-1.7u\ :*s& ■ /^■c-1:


iH IS i
Chromatic aberration results from the fact that the refractive index n is slightly
dependent on wavelength (8, pp. 423-30) (7, pp. 268-73). Therefore, shorter
wavelengths (blues, violets) bend more as they pass through an interface or
optical system than do longer wavelengths. All wavelengths are not focused
at the exact same location, and thus a point object does not produce a point
image. The image will vary in location (longitudinal chromatic aberration) and
size (lateral chromatic aberration).

Patients wearing high-powered lenses may see colored fringes


around objects as a result of chromatic aberration (5).

Qualitative Features
1 Longitudinal chromatic aberration:
• Light comprised of many As from a point object results in a series
of point images along the axis.
Copyright 2014 by KMK Educational Services, LLC
210 5.1 ABERRATIONS AND LENS DESIGN
• Inversely related to p , called the Abbe value (see below).
2 Lateral (transverse) chromatic aberration:
• Lateral chromatic aberration produces different size images depend­
ing on A. It also results in different prismatic effects for different
wavelengths.
• Related to prismatic effect.
• More harmful to vision as prismatic effect increases.
• Also inversely related to v.

Chromatic aberration underlies the use of the Red-Green Spherical


test in clinical refraction. In particular, the discrepancies between
red and green stimuli yield a more precise estimate of the refractive
error of the eye than would a monochromatic stimulus.

Abbe Number: We can quantify chromatic aberration of a material by using


an Abbe value, p, which is inversely related to the magnitude of chromatic
aberration. For example, for longitudinal chromatic aberration, we have

v'
(5.19)
Similarly, lateral chromatic aberration, sometimes called chromatic power (3),
is given by
(5.20)
where d is the distance from the optical center of the lens and F is the lens
power. The d reminds us that lateral chromatic aberration increases as we
move to the periphery of the lens.
Achromatic Doublet We can combine a positive lens of one material with
a negative lens of another to eliminate CA. The resulting lens is called an
achromatic doublet. In equation form, we have
Ftotai —F\ + F<2,
(5.21)
CAtotai — C A i -f C A 2
where Ftotai is the total power, F\ is the power of material 1, A2 is the power of
material 2, and CAtotai Is the total chromatic aberration. To make an achro­
matic doublet, we want C'Atotai = 0- Therefore, we arrive at Equation 5.22.

Copyright 2014 by KMK Educational Services, LLC


CHAPTER 5. PHYSIOLOGICAL OPTICS I 211

The condition for zero GA is


0= Fi
(5.22)
^2 vi
where Fi is the power of lens 1, Fo is the power of lens 2, and v is
the Abbe value.

Exam ple 5 . 1 1 : M a k e a 5.00 D a c h ro m a tic d o u b ie t w ith O p h th a lm ic Crown Glass


(5 2 3 — 586) a n d H ig h lite Glass (701 — 310 ). T h a t is, choose the pow ers o f the tw o lenses
a p p ro p ria te ly to e lim in a te c h ro m a tic a b e rra tio n .

Solution 5 . 1 1 : U sing E q u a tio n s 5 .2 1 , 5 .2 2 , we have

P to ta i = 5.00 D = F \ +
0 = -F i /68.6 + F 2/ 3 1 . ^ '

W e h av e tw o e q u a tio n s a n d tw o unkno w ns. To s ta rt, solve th e first eq u atio n for F \ ,


g iv in g F i = 5.00 — F 2 . P lu g this v alu e o f F i in to th e secon d equ atio n, which gives

n 5.00 - F 2 , F 2
58.6" + 3 1
_ 5-00 _ F j _ Ft (5.24)
58.6 58.6 + 3 1
= 0 .0 15 F 2 + 0.085

S o lvin g fo r F 2 gives F 2 = —5.67 D . Since b o th p ow ers m u s t a d d up to 5 .0 0 D , we k n o w


Fi = 10.67 D . *•

Notice that to make an achromatic doublet, the ratio of the v values


equals the ratio of the powers (up to a sign).

Lens Materials with Low Abbe Values


When considering lenses made from a material with a low Abbe value, the
following considerations (provided by Brooks and Borish) may be helpful for
minimizing the effects of chromatic aberration (3):
• Use shorter vertex distances
Copyright 2014 by KMK Educational Services, LLC
212 5.5. PRISMS AND OPHTHALMIC LENSES
• Use monocular PDs
• Include sufficient pantoscopic tilt (less than 10 degrees for high powered
lenses)

The aberrations of most concern in ophthalmic lenses are (in order):


oblique astigmatism, curvature of field, and distortion.

- SECTIO N 5.5 --------- ---------------------------------------------------------------

Prism s and Ophthalmic Lenses


The concept of a prism appears frequently in vision-related optics. We intro­
duce some fundamental ideas below (8, ch. 17,18) (3, ch. 18) (7, ch. 5) (5,
ch. 4). When light passes through a prism, its direction is altered. Light bends
toward the base of the prism, so the image shifts toward the apex of the prism
(Figure 5.7).
jDpviat i on ppw cr ( Z o r 'A ) Lf',.'. ^ ■:■;u - , ®

Prism power (denoted by A) tells how far a light beam is diverted by a prism.

Figure 5.7: Light leaving the object is bent toward the base of the prism. As
a result, the viewer sees a virtual image of the object which is shifted in the
direction of the prism apex.

Copyright 2014 by KMK Educational Services, LLC


CHAPTER 5. PHYSIOLOGICAL OPTICS I 213

Units: The unit of prism power is the prism diopter (A). A prism with a
power of x A will shift a beam of incoming light x cm on a wall 1 m away. The
following ratio is always the same for a given prism:
A = -x, (5.25)
where A is the prism power, y is the linear distance the beam will be shifted
in cm> while x is the distance between the prism and the wall in m.

Prism Power and Apex angle


For thin prisms and small incident angles, we can find the deviation angle using:
d = A(n —1), (5.26)
where n is the refractive index of the prism, A is the apex angle (in degrees),
and d is the deviation angle in degrees.

Exam ple 5 .1 2 : W h a t is th e a p ic a l a n g le o f a glass p rism w ith p o w e r 5A ?

Solution 5 .1 2 : To use th e e q u a tio n abo ve, we n eed th e d eviation angle in degrees. I f


th e prism p o w e r is 5 p rism d io p te rs , t h a t m ean s i t deviates a b eam 5 cm on a w a ll 1 m
away. T his corresponds to an an g le d satis fy in g

d = 0 .0 5 /1; (5.27)

T h a t is, th e size o f th e an g le ( fo r s m a ll ang les) is a b o u t 0 .0 5 /1 , w hen m easured in radians.


T his corresponds to d = 0.05(180/7r) = 2,9 degrees. Therefore, i f n = 1.5, A is s im p ly
given b y ( 2 .9 / 0 .5 ), o r 5 .8 degrees.

Prism Power and Thickness


One can relate prism thickness to the prism power using
A = 100— (5.28)
where g is the difference in thickness between the apex and base, n is the index
of refraction, l is the apex to base length (same units as g), and A is the prism
power.

Exam ple 5 .1 3 : F in d th e p rism a t th e c e n te r o f th e follow in g lens: —6.00 D w ith n =


1.523. A ssum e th e lens has A = 50 m m , B = 30 m m , a nasal thickness o f 4.0 m m a n d
a te m p o ra l thickness o f 6.0 m m .

Copyright 2014 by KMK Educational Services, LLC


214 5.5. PRISMS AND OPHTHALMIC LENSES
Solution 5 .1 3 : T re a t this as a prism (on its side). F ro m th e setu p , we k n o w th e prism
wiH b e h o rizo n ta l. Now g = 6 — 4 = 2 m m . T h e A dim ension g ives us th e b as e -a p e x
len g th . So we have
A = 100(2) (0.523)/5 0 = 2 .1 D . (5.29)

T h e th ic k e r edge tells th e d irectio n o f the base. I t is B O .

Prisms used for eye purposes must have a specific orientation to function cor­
rectly. We can specify orientation using contraocular coordinates and the fol­
lowing symbols:

BU corresponds to base up
BD corresponds to base down
BI corresponds to base in (toward nose)
BO corresponds to base out (toward ear)
Alternatively, you could specify the angle at which the base points
(0 —360°) by using the @ symbol.

rentic-e’s I "issa
We can associate a prism power to each point on a lens. Intuitively, this means
that each point on the lens bends light a certain amount, which is given by
A = dF, (5.30)
where d is the distance from the optical center (in cm) at which light strikes a
lens, F is the lens power in diopters, and A is the prism power at that point
(given in A). Hence, one can use lens positioning to bend light in a certain
direction.
Decentering:
Often, we can take advantage of the prismatic effect by decentering a lens.
* Plus lens: For a plus lens, the decentering direction corresponds to the
base direction of the induced prism.•
• Minus lens: For a minus lens, the decentering direction corresponds to
the apex direction of the induced prism.
Copyright 2014 by KMK Educational Services, LLC
CHAPTER 5. PHYSIOLOGICAL OPTICS I 215

Exam ple 5 .1 4 : A p re s c rip tio n calls fo r a + 4 .0 0 D lens fo r the rig h t eye, a n d it also calls
fo r 3 A B O . B y h o w m u ch s h o u ld th e lens b e d ecen tered ?

Solution 5 .1 4 : Use P re n tic e 's R u le.

3 = 4.00 d,
(5.31)
d = 0.75 cm

This m eans w e w a n t th e lens to b e d ec e n te re d b y 7.5 m m to ind uce th e desired p ris m a tic


effect. T he lens is a p lu s lens (tw o prism s s ta c k e d base to base). W e w a n t base O U T , so
the d ec e n te rin g sh o u ld also b e O U T . W e can re la te th e d ecen terin g to th e shape o f th e
lens a n d th e spectacles, i f needed.

If given a lens that is decentered both horizontally and vertically,


simply use Prentice’s rule for each direction individually Then the
total prism is the vector sum of the two.

Oblique Meridians and Prentice’s Rule


Strictly speaking, one can not use Prentice’s rule with the so-called sine-squared
formula, which estimates the power in an oblique meridian. The results from
this approach will be incorrect when decentering (or looking) involves both a
horizontal and a vertical component. However, this combination can be used
when the decentering (or looking) is only vertical or only horizontal. This is
often helpful for problems of vertical imbalance.

Exam ple 5 .1 5 : F in d th e v e rtic a l prism in d u c e d in th e rig h t eye w hen a p a tie n t looks 1 0


m m b elo w th e o p tic a l c e n te r o f th e fo llo w in g lens:

OD + 2 .0 0 - 1.50 x 135.

Solution 5 .1 5 : T h e p o w e r cross shows a p o w e r o f + 2 . 0 0 a t 1 3 5 degrees a n d 1 -0 .5 0 a t 4 5


degrees. In th e 9 0 m erid ian , w hich is h a lfw a y b etw e e n th e 4 5 and th e 1 3 5 m eridians, we
k n o w t h a t th e p o w e r w ill b e th e ave ra g e o f 1 -2 .0 0 a n d + 0 .5 0 , w hich is + 1 . 2 5 D . So, b y
using P re n tic e 's rule, we k n o w t h a t th e p rism p o w e r is Z = (1,25) (1) = 1.2 5 A . L o o kin g
dow n is like d e c e n te rin g up, a n d we h ave a p lu s lens, so th e prism is base up.

Copyright 2014 by KMK Educational Services, LLC


216 5.5. PRISMS AND OPHTHALMIC LENSES

When combining two prisms, we use vector addition. If the prisms point in the
same direction (e.g. both are vertical), vector addition is equivalent to regular
addition. If the prisms are perpendicular, the total prism A is given by
A = v ^ + l? , (B.32)
where x is the prism power of prism 1 and y is the prism power of prism 2.
Exam ple 5 .1 6 : W h a t is th e to ta l prism th a t results fro m c o m b in in g 2A B U and 1 A
B O ? W h at about 2A B U and 1 A B U ?

Solution 5.16: In th e firs t case, w e use e q u a tio n 5 .3 2 , w hich g ives A2 = 22 4- l2, o r


A = 2.23 prism diopters. In th e second case, we ju s t use re g u la r a d d itio n : 2 B U + 1 B U
= 3 BU.

In optometry, one must often take into account the amount of vertical prism
induced in each eye (3, ch. 20) (5, ch. 10). If these amounts are different,
there is a so-called vertical imbalance or binocular prism included. Vertical
imbalance results from different vertical prismatic effects in each eye at a given
gaze position. This is due to a difference in power in the vertical meridian
between the left and right lenses.
Exam ple 5.17: A p a tie n t has a prescription callin g fo r O D +5.00 D and OS +3.00 D
s p h erical lenses. H o w m u ch v e rtic a l im b alance is in d u c e d w hen th e p a tie n t looks dow n
1 0 m m to read ?

Solution 5.17: W e can fin d th e prism in each eye using P r e n tic e ’s rule.

A = (1 cm)(+5.00) = 5.00a B U (r ig h t eye),


A = (1 cm)(+3.00) = 3.00aa B U ( le f t eye). (5.33)
N o te t h a t lo o k in g dow n is the sam e as d ecen terin g th e lens upward. S ince th is is a plus
lens, d ec e n te rin g is in th e directio n o f th e base, w hich is up. So th e n e t e ffe c t is 2.00A
B U O D or, equivalently, 2.00A B D OS.

To find vertical imbalance, we want the difference in the amount


of vertical prism in each eye.

Copyright 2014 by KMK Educational Services, LLC


CHAPTER 5. PHYSIOLOGICAL OPTICS I 217

Correcting Vertical Imbalance:


There are a number of methods for correcting vertical imbalance (5, pp. 298-
99) (3, pp. 460-63):
1 Slab off is a process in which one changes the shape of the more minus
lens so that the lower half has more base up to compensate for vertical
imbalance. This results in a horizontal line across the lens. The procedure
works well unless you need correction < 1.5A, in which case the line would
be too visible and prohibit clear vision. That said, most people will not
be bothered by a vertical imbalance that is this small.
2 Dissimilar Segs refers to placing the optical centers of the right and
left segs at different positions. This can be accomplished by choosing
different types of segs. Since bifocal segments are like miniature lenses
within a larger lens (see Section 5.3), looking through them (at points
away from the optical center) will produce a prismatic effect. If we place
the left and right seg optical centers at different locations, we will get
different prism effects in each eye. We can use this to compensate for
vertical imbalance at near distances.
3 Compensated R Segs are a more cosmetically appealing version of
dissimilar segs. The segs are optically compensated. This only works in
glass lenses.
4 M ultiple Spectacles can be used rather than multifocals. Sometimes
it is simply best to prescribe one pair for distance and one pair for near
vision (with the OCs lower than normal). The patient will not have a
problem with vertical imbalance because he/she can move his/her head
so that he/she always looks through the optical center.
5 Contact Lenses also provide a solution to vertical imbalance because
the optical center of the contact lens moves with eye, preventing vertical
imbalance.
6 Fresnel Prism can be attached to the lens to correct for vertical imbal­
ance. They are made from adherent, transparent material that one places
to the lens; they act a bit like a temporary slab-off procedure. One can
put a Fresnel prism anyplace on the lens and adjust as necessary.

Image jump refers to the sudden displacement of an image at the bifocal line.
This is due to added prismatic effect from crossing into the seg. When the eyes
are first lowered, the prismatic effect comes from the distance portion of the
lens. When the eyes cross the bifocal line, the prismatic effect is changed by
Copyright 2014 by KMK Educational Services, LLC
218 5.5. PRISMS AND OPHTHALMIC LENSES
an additional amount due to Prentice’s rule in the seg. To calculate the jump,
just find the prism power of the seg at its far edge (that is, at the seg line).

Exam ple 5 .18 : F in d th e im ag e ju m p in bifocals, given a p rescrip tio n + 2 .0 0 — 1.2 5 x 180,


Add = 2.00 D . In this case, th e seg is a fla t to p 2 8 seg. T h e seg d rop is 4 m m .

Solution 5 .1 8 : T h e ju m p is in d e p e n d e n t o f d istan ce p o w e r a n d seg lo c a tio n . W e o n ly


n eed th e a d d p o w e r a n d th e shap e o f th e seg. F o r a fla t to p 2 8 seg, th e o p tic a l c e n te r
is 5 m m fro m th e o p tic a l c e n te r o f th e lens. So using P re n tic e 's rule, we fin d t h a t th e
im a g e ju m p is s im p ly
A = 0.5 cm(2.00) = 1 A . (5.34)
Since the image jump results from looking above the optical center of a plus lens (the
add), it is base down.

mmmmBmmuBm&mmmsmmmm
To calculate the total prismatic effect from looking in a given location of a
bifocal lens, we must sum the prism induced from looking away from the seg
OC and the prism induced from looking away from the distance OC. Do not
confuse this calculation with vertical imbalance, where one is looking for the
difference in prism powers between the two eyes!

Exam ple 5 .19 : F in d th e v e rtic a l im b alan ce t h a t results w hen a p a tie n t loo ks d ow n 10


m m in th e fo llo w in g lenses.

OD: + 1 .5 0 - 1.00 x 90
OS: + 0 .5 0 D - 0.75 x 180
Add: + 1 .5 0 D
S e g d ro p : 4 mm
S e g ty p e : F la t to p 2 8
PD: 64/59 (d is ta n c e /n e a r)

Solution 5 .1 9 : F irst, re m e m b e r th a t v ertical im b a la n c e results fro m a d ifferen ce in prism


b etw e e n th e tw o eyes. I f th e adds are th e sam e p o w e r fo r each eye a n d are p la c e d in
th e s a m e lo c a tio n in each eye (w h ich is ty p ic a lly th e case), th e prism in d u c e d fro m th e
a d d w ill b e th e s a m e in each eye when lo o k in g d ow n by a g iven distance. S o fo r v e rtic a l
im b a la n c e , w e d o n o t n eed th e in fo rm atio n a b o u t th e a d d !

I t is h e lp fu l to d ra w a p o w e r cross o f each lens. We o n ly care a b o u t th e v e rtic a l c o m p o n e n ts


o f p ow er. T h e prism in d u ced fro m th e distance p o rtio n o f th e rig h t lens is

A = 1 x 1.50 - 1.5 a B U . ■ (5.35)

T h e prism is base up because th e p a tie n t looks b elo w c e n te r in a plus tens. F o r th e le ft


lens, we h ave
A = 1 x 0.25 = 0.25 a B D . (5.36)

Copyright 2014 by KMK Educational Services, LLC


CHAPTER 5. PHYSIOLOGICAL OPTICS I 219

T h e prism is base d ow n because th e p o w e r in th e v e rtic al m erid ian o f th e le ft lens is


n eg ative ( - 0 ,2 5 D ) . L o o k in g b e lo w th e o p tic a l c e n te r o f a m inus lens results in base down
prism . I t is clear, then , t h a t th e t o t a l im b a la n c e is 1 .7 5 A B U O D o r ( e q u ivalen tly) B D
OS.

- SECTION 5.6 ------------------------------- ------------------------------------------

Relative M agnification and Image Sizes


We present some terminology and techniques used for comparing image size
between corrected and uncorrected eyes (8, ch. 14) (5, ch. 10) (12).

Spectacle magnification compares the retinal image size in an uncorrected eye


with the retinal image size in a corrected eye. Specifically, we have
= iio SM (5.37)
where Ig is the retinal image size with glasses (or correction) and I q is the
retinal image size without glasses (correction).
SM for Thick Lenses
There are 2 contributions to SM for thick lenses: shape and power (5, pp. 302-
OS) (8, pp. 281-82). We have
S M — ( Shape factor ) x (Power factor), (5.38)
where
Shape factor = Ms — 1 —(tjn)Fi1 (5.39)

where t is the central thickness of the lens, Fi is the front surface power of the
spectacle lens, n is the index of refraction of the lens, and also,
Power factor = M p -- 1 —hFv ’ (5.40)
where h is the distance between the back surface of the lens and the entrance
pupil of the eye, and Fv is the back vertex power of the lens.

Copyright 2014 by KMK Educational Services, LLC


220 5.6. RELATIVE MA GNIFIGATION AND IMAGE SIZES

M'rend for Plus Lenses


Change Result v
Increase h + Increases SM
Increaset Increases SM
Increase base curve Increases SM (i',i t)
Increase n Decreases SM
Trend for Minus Lenses
Change Result
Increase h Decreases SM
Increase t Increases SM
Increase n Decreases SM

The clinician often orders spectacle lenses with the same front curve
(Ti) and the same center thickness (f) for anisometropic patients.
This will keep the shape factor about the same for each eye (6).

Exam ple 5.20 : Find SM for a patient who wears a + 10 .0 0 D lens (n = 1.5 , 11 mm
at
thick) 1 1 mm vertex distance. The front surface of the lens is + 16 .0 0 D.

Solution 5 .2 0 : If not specified, take the distance between the cornea and entrance pupil
to be 3 mm. This means h is 1 1 + 3 = 14 mm. Recall that prescriptions specify Ft,, so
Fb = 10.00 D . So we have

Mp
1 1.16 ,
(1 - 0.14)

M s
1 1 .1 3 , hence (5.41)
1 -0 .0 1 1 ( 1 6 .0 0 ) /( 1 .5 )
SM = 1 .1 6 ( 1 .1 3 ) = 1.3 1.
That is, the retinal image size is 3 1 % bigger v/ith spectacles.

RSM is used to compare the retinal image of a corrected eye with the retinal
image of a standard eye (8, pp. 288-92) (5, pp. 309-312). The standard eye is
defined to be a +60.00 D emmetrope. We have
R S M = y-y
■is
(5.42)
where Ia is the retinal image size in a corrected ametropic eye and I s is the
retinal image size in the standard eye.
Copyright 2014 by KMK Educational Services, LLC
CHAPTERS. PHYSIOLOGICAL OPTICS I 221

Deviations from Standard


For simplicity, one typically considers deviations from the standard eye to be
either purely refractive or purely axial. Of course, this is merely a rough ap­
proximation, but it offers some insight into some magnification-related effects
of optical correction.
1 Axial ametropes
• Hyperopia —*■ Axis too short
• Myopia — >Axis too long
• Best corrected with spectacles (Knapp’s Law).
Knapp’s Law: Knapp’s Law provides a rule for the R S M in an
axial ametrope. Specifically, it says that R S M — 1 if a thin
lens is placed at the primary focal point of the eye. For an axial
ametrope, the primary focal point is about 17 mm in front of
the eye (same as for an emmetrope). This provides intuition as
to why spectacles work better for axial ametropes.
2 Refractive ametropes
• Best corrected with contact lenses.
• Intuition: Uncorrected refractive myopes, hyperopes, and emmetropes
all have the same size image formed on the retina (Section 5.7). In­
tuitively, then, one could guess that we would like magnification
to remain approximately constant after correction of a refractive
ametrope. To accomplish this, make SM small by choosing contact
lenses (small h,t).

- SECTION 5 . 7 ------------------------------------------------------------------------------- ----------------

Com parative R etinal Image sizes


Uncorrected Axial Ametropes
Myope image size > Emmetrope image size > Hyperope image size
Corrected Axial Ametropes
As mentioned above, spectacles placed at the focal point of the eye give R S M ss
1. As such, contacts yield the same results as for uncorrected axial ametropes.
Uncorrected Refractive Ametropes
All retinal image sizes are the same (see Figure 5.8).
Copyright 2014 by KMK Educational Services, LLC
222 5.7: COMPARATIVE RETINAL IMA GE SIZES

P P' H,E,M

Figure 5.8: The relative sizes of retinal images in axial ametropes are given by
Im > h > Ih (top)- For refractive ametropes, Im —Ih — /e (bottom), where
Im is the retinal image size for a myope, I e for an emmetrope, and I}t for a
hyperope (8, pp.289).

Copyright 2014 by KMK Educational Services, LLC


CHAPTER 5. PHYSIOLOGICAL OPTICS I 223

Corrected Refractive Ametropes


As mentioned above, contacts give R S M ss 1. Spectacles, on the other hand,
result in a larger image for the hyperope and a smaller image for the myope.

Example 5.21: Which has larger retinal image size: a spectacle corrected refractive
hyperope or a spectacle corrected refractive myope?

Solution 5.21: Spectacle corrected refractive hyperope. Here is how the reasoning works.
Recall that contacts are best for refractive ametropes, so we expect that the spectacle
corrections will cause some unwanted magnification. Myopes need minus lenses, and
increasing vertex distance (and therefore h) for minus lenses decreases SM. We know
that the h for contacts should work well and give S M & 1, so S M for glasses must be
smaller. Thus, we expect spectacle corrected refractive myope will have a smaller retinal
image size (also see (8, ch. 14) for an excellent discussion).

Aniseikonia means there is a difference in the size or shape of the images seen
by the left and right eyes (8, pp. 292) (5, pp. 300-322). It can be anatomical or
induced.
Anatomical aniseikonia: Could arise from some anatomical asymmetry, such
as a discrepancy in the density of photoreceptors (6).
Induced aniseikonia: Due to the optics of the corrected eye, most notably a
difference in spectacle magnification (6).
Overall aniseikonia: Refers to a spherically symmetric difference in image
size (such as an overall increase in image size for one eye) (6).
Meridional aniseikonia: Due to differences in cylinder power between the
left and right eyes, meaning that the effect is prominent in one merid­
ian (6).

A vertical object might appear to be tilted to a patient with merid­


ional aniseikonia (6).

Copyright 2014 by KMK Educational Services, LLC


224 5.7. COMPARATIVE RETINAL IMAGE SIZES
Some rules of thumb
1 For small differences in R S M , prescribe equal base curves and equal
thickness lenses.
2 For large differences in RSM , prescribe a thin, flat lens for the eye with
the highest R SM . Prescribe a thicker, steeper lens for the eye with the
lowest R SM .
3 If a patient has a small amount of ametropia in one eye while the other
eye has more than 4.00 D of ametropia, the ametropia in the bad eye is
likely axial. Therefore, spectacles should minimize aniseikonia (6).
4 If a patient has a small amount of ametropia in one eye while the other
has has less than 4.00 D of ametropia, the ametropia in the bad eye
is likely refractive. Therefore, contact lens should be used to minimize
aniseikonia (6).
5 Aniseikonia due to large astigmatism in each eye can be optically best
corrected with contact lenses, because large amounts of astigmatism are
likely from the cornea (6),

A general rule says that for every 1.00 D power difference, there
will be approximately 1 percent aniseikonia (10)

Exam ple 5 .2 2 : A p a tie n t w ears O D - 2 .0 0 D a n d O S - f 1 .0 0 D . I f y o u s fig h tly increase th e


thickness o f th e O D lens, w ou ld yo u e x p ect an increase o r decrease in a niseikon ia?

Solution 5 .2 2 : T h e sp ectacle m ag n ific a tio n is larg er in th e O S lens th a n th e O D lens


because th e O S lens has m o re plus pow er, ( f we increase thickness o f a lens, we w ill alw ays
increase s p e c ta cle m a g n ific a tio n . S o increasing th e thickness o f th e O D lens s h o u ld m a k e
th e s p e c ta c le m a g n ific a tio n in th e O D closer to t h a t in th e O S , w h ic h s h o u ld red uce
aniseikonia.

fe. &*>ja&.Ji :-it ziCrX?'-.'.-7 Fwii -fiLif . - w b - S ' 1=>-V:-rv.i'.A d -IhEAMf. r.Ai a-?■-£-r k :L*£ -A
<; >;

Anisometropia refers to the condition where the refractive state of the left eye
differs from that of the right eye, usually by more than 1.00 D. To correct this
asymmetry, we use the same (simplified) rules that we used for correcting a
single ametrope while keeping RSM approximately 1. We list these rules along
with some general trends below.
Copyright 2014 by KMK Educational Services, LLC
CHAPTER 5. PHYSIOLOGICAL OPTICS I 225

• Patients with myopic anisometropia tend to have poor uncorrected dis­


tance acuity in both eyes, with the most myopic eye having the worst
acuity (6).
• Patients with hyperopic anisometropia tend to have relatively good uncor­
rected distance acuity in both eyes (because of accommodation, assuming
no amblyogenic factors), but they may have eyestrain (6).

You will recall that accommodation occurs equally in both eyes.


Therefore, there is no distance at which a hyperopic anisometrope
can see clearly with both eyes. In myopic anisometropia, a person
will typically learn to use the more myopic eye for near vision and
the less myopic eye for distance vision. By contrast, a hyperopic
anisometrope will likely use the same eye (the least hyperopic)
for all distances, putting him/her at risk for amblyopia in early
childhood.

Antimetropia
When one eye is hyperopic and the other eye is myopic, the ametropia is referred
to as antimetropia.
- SECTION 5.8 -----------------------------------------------------------------------------------------------------

A bsorptive Lenses
We cover some fundamental absorption properties of lenses (8, ch. 10) (5,
ch, 7) (7, pp. 119-21).
Loss of light when passing through lens
When light passes through a lens, it is reflected at both lens surfaces and also
absorbed by the lens material. We start with some definitions.
Transmittance (T): T measures the amount of light energy that gets through
an optical system. T ranges from 0 to 1.
When light falls upon a lens, it is lost in two ways:
1 Reflected at the front and rear surfaces: We use Fresnel’s Law, which
describes the reflection of light at the boundary between two media
Copyright 2014 by KMK Educational Services, LLC
226 5.8. ABSORPTIVE LENSES
of indices n 2 and n i:
= U2 ~ Hi
n2 + ni R (5.43)
This would occur at each surface. To convert to Transmittance, use
Ts = l - R , (5.44)
where Tg is the transmittance at each surface.
2 Absorbed by medium: Assume the amount transmitted (not ab­
sorbed) by the medium is Tm -
For a lens, we combine these 2 factors:
T = TSTSTM, (5.45)
where T is the total transmittance through the lens.

Exam p le 5 .2 3 : A lens ( n — 1.5 J 2.0 m m th ic k absorbs 3 0 % o f in c id e n t lig h t, ig n o rin g


re fle c tio n . F in d th e to t a l a m o u n t o f lig h t tra n s m itte d th ro u g h th e lens.

S olu tion 5 .2 3 : We n eed to f in d T s a n d T M -

= 0.202 = 0.04. (5.46)

So T s = 1 - 0.04 = 0.96.
Tm = (1 — 0.30) = 0.70, (5.47)

So th e to t a l tra n s m itta n c e is

T = (0.96) (0.96) (0,70) = 0.65. (5.48)

So th e to t a l tra n s m itta n c e is 65%.

Ideal Thin Film: To choose a film material that minimizes reflection, use
the following simple formula (8):
rif — (5.49)
where r i f is the index of the film to be used, n \ is the index of the initial
medium (usually air), and ng is the index of the lens material.
Copyright 2014 by KMK Educational Services, LLC
CHAPTERS. PHYSIOLOGICAL OPTICS I 227

1“ SECTION 5.9

C ontact Lenses
We will cover a number of important topics relating to contact lenses, including
the optical considerations of hard and soft contact lenses, as well as information
about the gas permeability of lens materials.

A contact lens should-sinc^ speaking-he treated as a thick lens. As a result,


one typically characterizes the power of a contact lens in terms of back vertex
power, FVy which is defined as usual to be
Jp*V —E2 +
Fi
1 - {ifn)F± (5.50)
where F2 is the back surface power, F\ the front surface power, t the center
thickness of the lens, and n the index of refraction. Although contact lenses
are quite thin, they typically have highly curved surfaces, meaning that Fv A
E2 + Ei j as it would for an infinitely thin lens. Nevertheless, it is common to
see contact lenses treated as infinitely thin in some applications. We offer the
following rough guideline for problem solving:

If center thickness is provided in the problem, always treat a contact


lens as a thick lens. Otherwise, you must approximate the lens as
a thin lens, even though this will result in a slight numerical error.

stance ip
ft
It is important to remember that the power of a lens needed in a given situation-
for example, to correct ametropia-will depend on where that lens is located.
Refractions are often performed at a vertex distance of approximately 12-14
mm, which means that the value of the refraction will not describe the power
needed at the cornea. Using the effective vergence equation from Geometric
Optics, it is easy to see that
Fe =
1— (5.51)
where Fg is the refraction measured at a vertex distance of x (x = 0 corresponds
to the cornea), and Fc is the refraction that would be measured at the cornea.
One will often need this equation to provide a starting point for a contact lens
prescription.
Copyright 2014 by KMI< Educational Services, LLC
228 5.9. CONTACT LENSES

This equation is an algebraic way of saying that the power of a


lens needed to correct ametropia can be determined by aligning
the secondary focal point of the correcting lens with the far point
of the eye.

We will first deal with rigid (hard) contact lenses. Unless stated otherwise, we
will assume the lens is sufficiently rigid that it maintains its shape when placed
on the cornea.

Tear Lens
The contact lens/tear lens/cornea optical system involves a number of refract­
ing surfaces. The essential idea to keep in mind is that the layer of tears formed
between a hard contact lens and the cornea has a power associated to it, and
that power must be accounted for when prescribing contact lenses.

An Optics Trick To study this system, it is convenient to consider the three


components in isolation:
1 The contact lens in air
2 The tear lens in air
3 The eye (including the cornea) in air
This is conceptually appealing, because once we understand the components
in isolation, we can add the powers together appropriately to get information
about the system as a whole.

Exploded System: In what follows, we will speak of the power of the tear
lens as being the power the tear lens would have if it were surrounded by air
on both sides. Obviously, in the exact physiological setting, the tear lens is
not surrounded by air on both sides (but by the cornea and the contact lens,
respectively). However, we will consider a thin layer of air to be present between
each surface. This does not change the overall power of the combined optical
system-henee this is an optics trick, of sorts-but it is intuitively appealing
(see (5) for an excellent discussion of this subtle point). This method is known
as the exploded system method.
Copyright 2014 by KMK Educational Services, LLC
CHAPTER 5. PHYSIOLOGICAL OPTICS I 229

Refraction and Hard Contact Lenses With this in mind, we will base
most of our calculations on the following equation (5):
Ftl + Fcl + For = Rc (5.52)
where Ftl is the power of the tear lens, Fcl is the power of the contact lens,
For is the power of the oyer refraction, and R c is the subjective refraction
obtained at the corneal plane. This equation must hold for a contact
lens to perfectly correct an ametropic eye. The equation reminds us
that we must account for the tear lens in prescribing hard contact
lenses!

Since most refractions are performed at a vertex distance of around


12 mm, one must first correct the refraction using Equation 5.51
to obtain the refraction that would be obtained at the cornea.

Tear Lens Power The power of the tear lens (in air), Ft l , is easy to find.
The back surface of the tear lens has a radius of curvature that is equal to
the radius of curvature of the cornea. The front surface of the tear lens has a
radius of curvature determined by the base curve of the contact lens. For each
surface, the power is given by (see Geometric Optics)
F = h i36-
r
1 = 2 ^r 5 (5.53)
This is just the power of an SSRI (in air) with n ~ 1.336, the index of the
tears, and r the radius of curvature of the surface, in meters.
Note that the front surface of the tear lens will be positive, and the back surface
will be negative. As a result, the total power of the tear lens (in air) will depend
on the relative sizes of the two surface powers.

The power of the tear lens (in air) would be:


• Positive if the base curve is steeper than the cornea.
• Negative if the base curve is flatter than the cornea.
This provides a conceptual basis for the common clinical sayings
steeper add minus and flatter add plusi or SAM-PAP. This rule
reminds you to compensate for the power of the tear lens. For
example, if one steepens the base curve, the tear lens becomes
more positive (less negative), so minus power must be added to the
prescription to compensate.

Copyright 2014 by KMI< Educational Services, LLC


230 5.9. CONTACT LENSES

Exam ple 5 .2 4 : T h e fo llo w in g in fo rm atio n is o b ta in e d fo r a p a tie n t:


C ornea c u rv a tu re (K -re a d in g s ): 7.5 m m o r + 4 5 . 0 0 D (sp h eric al)

S u b je c tiv e R e fra c tio n ( a t c o rn e a ): + 1 .0 0 D S O U


W h a t p o w e r o f rig id c o n ta c t fens should be chosen fo r an on-K fit? W h a t a b o u t i f th e
c o n ta c t lens base cu rve is chosen to b e 8 .0 m m ?

S olu tion 5 .2 4 : F o r th e o n -K fit, th e p o w er o f th e fro n t surface o f th e te a r lens, in air,


w o u ld b e (fro m eq u a tio n re ftlp e q n )

F i = 0.336/0.0075 = +44.8 D . (5.54)

T h e p o w e r o f th e b a c k surface w ou ld be

F2 = -0 .3 3 6 /0 .0 0 7 5 = -4 4 .8 D. (5.55)

N o te t h a t this is very n early th e sam e as th e K -re a d in g o f th e cornea ( b u t o p p o s ite in


s ig n ). W e w ill discuss th is m ore below.

S o th e te a r lens in question, assum ing it is in fin ite ly th in , has a to t a l p o w e r o f 0.00 D .


U s in g E q u a tio n 5 .5 2 , w e have

F Cl = 1.00 - 0.00 = 1.00 D . (5.56)

T h u s, th e c o n ta c t lens is p rescribed accordin g to th e re fra c tio n a t th e cornea.

O n th e o th e r h an d , i f th e base curve o f th e c o n ta c t lens w ere chosen to b e 8 m m , we


w o u ld have
(5.57)

S in ce F% w o u ld n o t change, th e te a r lens w o u ld have a to ta l p o w e r o f + 4 2 .0 0 — 44.8 =


—2.8 D . We th e re fo re have, using E q u a tio n 5 .5 2 ,

F cl = 1.00 - ( - 2 , 8 ) = 3 . 8 D . (5.58)

T h a t is, w e h a d to a d d plus p o w e r to th e c o n ta c t lens because th e base curve was f la t t e r


(la rg e r rad ius o f c u rv a tu re ) th a n th e cornea.

K eratom etry Recall that the curvature of the cornea is measured by leer-
atometry, and therefore the value is often given in terms of power (D)> not
radius of curvature (mm). Sometimes you will see the power indicated as
“DK”, which reminds you that the measurement is from a keratometer.

“DI<” is sometimes used to specify the base curve of a contact lens.


In this case, keep in mind that this is not the actual surface power
of the lens, because the reading is calibrated for n = 1.3375 rather
than for the lens index of refraction .

Copyright 2014 by KMK Educational Services, LLG


CHAPTER 5. PHYSIOLOGICAL OPTICS I 231

We know that the two readouts-power and (corneal) radius of curvature-are


related by
F —
1.3375-1 0.3375 (5.59)
r r
where r is the readout in meters and 1.3375 is the effective index of refraction
of the cornea. Sometimes this is written as
F r„ 337.5
n (5.60)
where ri is the readout in mm.

Since the effective n for the cornea (1.3375) is approximately equal


to the n for the tears (1.336), we can use keratometry readings
directly to calculate the surface powers of a tear lens! We have:
• K readings are equal (but opposite in sign) to the back surface
power of the tear lens (in air).
* If the base curve of the contact lens is given by a K-reading,
that K-reading power is equal to the power of the front surface
of the tear lens (in air).

Exam ple 5 .2 5 :Let's repeat the last question by using the K-readings directly. The
following information is obtained for a patient:
Cornea curvature (K-readings): 7.5 mm or +45.00 DK (spherical)
Subjective Refraction (at cornea): +1.00 DS OU
What power of rigid contact lens should be chosen for an on-K fit? What about if the
contact lens base curve is chosen to be 42.19 DK? Note: We did not round to nearest
0.25 D, because we want to compare the two answers.

Solution 5 .2 5 :We can use the K-readings directly. For the on-K fit, the power of the
front surface of the tear lens, in air, would be
Ft = +45.00 D. (5.61)
The power of the back surface would be
F2 = -45.0 D. (5.62)
Note that these values are very nearly the same as those obtained earlier, because the
tear n is very close to 1.3375 used for the K-readings.

Copyright 2014 by KMK Educational Services, LLC


232 5.9. CONTACT LENSES
So the tear lens in question, assuming it is infinitely thin, has a total power of 0.00 D.
Using Equation 5.52, we have
Fol = 1.00 - 0.00 = 1.00 D. (5.63)
Thus, the contact lens is prescribed according to the refraction at the cornea.
On the other hand, if the base curve of the contact were chosen to be 42.19 DK, we
would have
Fi = +42.19 D. (5.64)
Since would not change, the tear lens would have a total power of —3.81 D. We
therefore have, using Equation 5.52,
F Cl = 1.00 - (-2.81) 3.8 D.
rs (5.65)
That is, we had to add plus power to the contact lens because the base curve was flatter
(larger radius of curvature) than the cornea. Our answer agrees with the previous method.

You should be able to do problems using either radius of curvature or keratom-


etry readings for both the base curve and the central corneal curvature. A
useful approximate rule is given below:

The power of the tear lens (in air) is approximately 0.50 D in


magnitude for every 0.1 mm difference between the base curve and
the corneal curvature. Use this rule to compensate for tear lens
considerations in the prescription.

Exam ple 5 .2 6 : A contact lens with a base curve of 8 mm perfectly corrects a patient's
ametropia. The back vertex power of the contact lens is -3.00 D. If the base curve is
steepened by 0.1 mm to achieve a better fit, what is the new prescription?

Solution 5 .2 6 :Steepening a contact lens base curve will change the power of the tear
lens (in the exploded system). We must account for this by changing the power of the
contact lens. Since there are approximately 0.25 D of tear lens power for every 0.05 mm
of difference between the base curve and the cornea, we know that changing the base
curve by 0.1 mm changes the tear tens power by approximately 0.50 D. If we steepen the
base curve, the tear lens becomes more positive (less negative), so we must add minus to
the contact lens to compensate. In this case, we add -0.50 D, giving a final prescription
of -3.50 D.

Copyright 2014 by KMK Educational Services, LLC


CHAPTER 5. PHYSIOLOGICAL OPTICS I 233

Cornea Astigmatism and Rigid Contact Lenses


The tear lens formed when a (spherical) rigid contact lens is worn will effectively
eliminate the majority of corneal astigmatism. In terms of the exploded system,
this is a result of the fact that the astigmatism present in the tear lens (in air)
will be equal but opposite to the cornea astigmatism.
Residual Astigmatism
The term residual astigmatism can be defined (for hard contact lenses) by
—A r x ~ A c Ar (5.66)
where A r is the residual astigmatism, A r x is the astigmatism present in the
subjective refraction (the total astigmatism of the eye), and A c is the corneal
astigmatism.

Residual astigmatism is simply the amount of astigmatism not at­


tributable to the cornea. It is the amount of uncorrected astigma­
tism we would expect if the patient were fit with a spherical hard
contact lens (assuming no flexure).

Exam ple 5 .2 7 : T h e fo llo w in g in fo rm a tio n is o b ta in e d :


K -readin gs: 4 3 .0 0 D (h o riz o n ta l), 4 5 .0 0 D (v e r tic a l)

S u b je c tive R e fra c tio n : p ia n o -2 .0 0 x 1 8 0

W h a t is th e a m o u n t o f resid ual a s tig m a tis m ? W h a t w o u ld be th e base curve (in term s


o f radius o f c u rv a tu re ) fo r a s p h e ric a l h a rd c o n ta c t lens chosen fo r an o n - K fit? W h a t is
th e p o w e r o f the c o n ta c t lens p re s c rip tio n ?

Solution 5 .2 7 : F irs t l e t ’s fin d th e a m o u n t o f resid u al as tig m a tism . A c c o rd in g to th e


re fra c tio n , = —2.00 D x 180 o f a s tig m a tis m .
A rx T h e K -re a d in g s show t h a t th e re is
Ac = —2.00 D x 180 o f co rn eal a s tig m a tism .
Note th a t we are m easuring the astigm atism of the correction, not of the eye itself.
T h is is the source o f the extra negative sign in A c.
So using E q u a tio n 5 .6 6 w ith A r x — A c, we fin d t h a t th e residual a s tig m a tism is zero,
m e a n in g t h a t a spherical h a rd c o n ta c t lens w ill c o m p le te ly correct th e astig m atism .

F ittin g o n - K m eans we choose th e base cu rve p a ra lle l to th e fla tte s t corneal m eridian , so
we w ould choose it to be 4 3 .0 0 D , acc o rd in g to th e k e ra to m e te r. W e can co n vert this to
a radius o f c u rv a tu re b y using E q u a tio n 5 .5 9 , w hich gives

r = 337.5 / F = 7.8 m m . (5.67)

Copyright 2014 by KMK Educational Services, LLC


234 5.9. CONTACT LENSES
Because the spherical contact lens will correct cornea astigmatism, and all astigmatism
present in the refraction is from corneal astigmatism, we simply choose a contact lens
that is piano (the spherical portion of the refraction).
It is worth looking at this in detail to see why it works. The tear lens has a front surface
power of + 4 3 . 0 0 D. The back surface of the tear lens is astigmatic. In the vertical
meridian, the power is — 4 5 . 0 0 D, and in the horizontal meridian, the power is — 4 3 . 0 0 D.
So the total power of the tear lens Is given by
Ft l ,90 = + 4 3 . 0 0 - 4 5 .0 0 = - 2 . 0 0 D
D.
(5 .6 8 )
= + 4 3 . 0 0 - 4 3 .0 0 = 0 . 0 0

Using Equation 5.52, we see that if we choose the contact lens to be piano, we have in
the vertical meridian
F t l ,9o + 0 = —2.00 = R c,3o, (5 .6 9 )

and in the horizontal meridian, we have


F t l ,i 8q + 0 = 0 .0 0 = iic . is o * (5 .7 0 )

That is, the piano tens will perfectly correct the corneal astigmatism thanks to the prop­
erties of the tear lens!

Javal’s Rule
Javal’s rule allows one to predict the total astigmatism correction using the re­
sults from keratometry. This is an empirical measure determined by correlating
total astigmatism with experimental keratometry readings (6). .
1.25A c + (-0.50 D x 90),
Arx = (5.71)
where A r x is the total (refractive) astigmatism and A c is the corneal astig­
matism. The second term on the right hand side is simply saying to add 0.50
D of against the rule astigmatism. It is clear from this rule that the average
residual (non-corneal) astigmatism is roughly —0.50 D with an axis at 90.

Example 5.28: Let's consider a patient with the following keratometry findings.
K readings: 43.00 D (at 180), 46.00 D (at 90)
How much total astigmatism would you predict this patient to have?

Solution 5.28:
A rx = 1 . 2 5 ( — 3 .0 0 x 1 8 0 ) + ( - 0 . 5 0 x 9 0 )

= - 3 . 7 5 x 180 + (+ 0 .5 0 x 180) (5 .7 2 )

= -3 .2 5 X 18 0. .

Copyright 2014 by KMK Educational Services, LLC


CHAPTER 5. PHYSIOLOGICAL OPTICS I 235

Example 5.29: Based on the corneal findings, predict the amount of refractive astigma­
tism.
K readings: 43.75 D @180/ 47.00 D <§ 090

Solution 5.29: The corneal astigmatism found from keratometry shows 3.25 D of WTR
astigmatism (-3.25 x 180). Using Javal's rule, we can find the following:
Arx = 1.25(—3.25 X 180) + (-0.50 X 090) ^ -3.50 X 180 (5.73)

Over-refraction
An over-refraction allows the clinician to empirically measure the effect of a
given trial contact lens on the patient's ametropia. For example, an over­
refraction can be performed to determine whether there is any residual astig­
matism when the patient wears a spherical hard contact lens. While in prin­
ciple this will give the same answer as that predicted by Equation 5.66, an
over-refraction is a direct measure and is therefore sometimes preferred. See
also Equation 5.52.

Example 5.30: Given the information below, find the refraction that would be obtained
at the cornea.
K-readings: 43.00 (horizontal), 45.00 (vertical)
Trial lens base curve: 7.85 mm
Trial lens power: piano
Measured over-refraction: +1.00-1.00 x 180 D

Solution 5.30: The trial lens has a base curve of 7.85 mm, which corresponds to 43,00
D when converted to a keratometry reading. Therefore, the tear lens has a front surface
(spherical) power o f +43.00 D (in air). The back surface of the tear lens will have powers'
of —43.00 D (horizontal) and —45.00 D (vertical), so the overall tear lens power is
FT l = 0.00 - 2.00 D x 180 (5.74)
The trial lens power is piano, so we have F cl = 0, and the over-refraction is F or =
4-1.00 —1.00x180, so we have (by drawing out the power crosses)
180 meridian: Rc = 0.00 4- 0.00 + 1.00 = 4-1.00 D
90 meridian: Rc = -2.00 4- 0.00 4- 0.00 = -2.00 D.
That is, the subjective refraction at the cornea would be 4-1.00 D—3.00 D x 180. Note
that the residual astigmatism for this patient is
Ar = (-3.00 x 180) - (-2.00 x 180) = -1.00 Dx 180 = 4-1.00 D x 90. (5.76)

Copyright 2014 by KMK Educational Services, LLC


236 5.9. CONTACT LENSES
The patient has 1.00 D of WTR residual astigmatism. Of course, this is exactly what we
measured in the over-refraction. This eye might be a candidate for front toric lenses.

Front Toric Rigid Contacts


When residual astigmatism is sufficiently large, an astigmatic front surface can
be placed on a contact lens to compensate. In a front toric lens, the back surface
is spherical (and corneal astigmatism is therefore corrected). The power of the
lens as a whole is chosen to eliminate the residual astigmatism.

A patient can typically tolerate approximately 0.5 D of ATR astig­


matism or approximately 0.75 D of WTR astigmatism (5). A pa­
tient with residual astigmatism greater than these bounds is a can­
didate for toric contact lenses.

Given the information below, find the back vertex power of the front toric
Exam p le 5 .3 1 :
contact lenses needed. Assume that the base curve is chosen for an on-K fit. What is
the surface power of the front toric surface? You may assume an infinitely thin contact
lens with n = 1.49.
K-readings: 45.00 D (horizontal), 44.00 D (vertical)
Subjective: piano - 3.00 D x 90

We will choose a front toric lens, so the back surface is spherical and
Solution 5 .3 1 :
chosen to be 44.00 D, in agreement with the flattest corneal meridian (on-K fit). The
amount of residual astigmatism is
Ar = (-3.00 x 90) - (-1.00 x 90) = -2.00 D x 90. (5.77)
This is significantly more than a patient could tolerate, so we need to chose a front
toric lens that corrects this astigmatism. Thus, we choose a lens with back vertex power
Fv = 0.00 - 2.00 D x 90.
The surface powers are easy to find. We chose the lens to have a base curve of 44.00 D,
but this is according to the K-readingsf It does NOT correspond with a surface power
of 44.00 D because of the differences in index of refraction. A surface power of 44.00
corresponds to a radius of curvature of

(5.78)
Assuming the contact tens has an index of refraction ups 1.49, the power of a spherical
surface with radius 7.7 mm would be
(6.79)
Copyright 2014 by KMK Educational Services, LLC
CHAPTER 5. PHYSIOLOGICAL OPTICS I 237

w h e re we have a d d e d th e n eg a tiv e sign because i t is a concave surface re la tive to a ir


(o r yo u co u ld use th e sign c o n v e n tio n ). S o th is is th e p o w e r o f th e back surface o f th e
c o n ta c t lens. W h a t a b o u t th e fro n t surface?

In th e v ertical d ire c tio n , th e lens s h o u ld h a v e no p o w e r ( F v = 0 ). Since we w ill assum e


it is in fin ite ly th in , we have

Fv = 0 = F\ + F2 = — 63.63, (5.80)

so F i = + 6 3.63 D in th e v e rtic a l m e rid ia n . In th e h o riz o n ta l m erid ian , we have

Fv = -2.00 = F i + F 2 = F i - 63.63; (5.81)

so F i = + 6 1.6 3 D . So th e fro n t su rface o f th e lens w ould h ave a surface p o w e r o f


Fi = +6 3.6 3 D - 2.00 x 90.

B it o r ic R ig id C o n ta c t s

There are times when the corneal astigmatism is so great that a back toric
surface is needed to ensure a good physical fit between the lens and cornea.
In the simplest case where an on-K fit is chosen in both principal meridians,
the contact lens back vertex power should be chosen to match the subjective
refraction. Often times, this means choosing a front surface that is also toric,
meaning that the lens is bitoric.

When the base curves are chosen parallel to both corneal


meridians-that is, a back toric surface is cliosen-the power of the
tear lens (in the exploded system) is piano. As a result, one should
choose Fv for the contact lens to equal the subjective refraction.

Exam ple 5 .3 2 : A s p h erical c o n ta c t lens o f p o w e r —1.00 D (base curve 4 2 .0 0 D K ) w ill


correct the re fra c tiv e erro r fo r a g iven p a tie n t, b u t th e f it over th e cornea is poor. To
achieve a g o o d fit, o ne m u s t choose a b a c k to ric s u rface o f pow ers 4 2 .0 0 D K (1 8 0 ), 4 5 .0 0
D K (9 0 ), values w hich correspond to th e c o rn e a l curvatu res. H o w w o u ld you choose th e
a p p ro p ria te b ito ric lens? ( 6 )

Solution 5 .3 2 : T h e fa c t t h a t a s p h erical lens ( o n - K f it ) w ill correct th e refractive error


tells us t h a t th e a s tig m a tis m is a ll corneal. T h e p o w e r o f th e sp h erical lens gives th e sphere
p ow er o f th e sub jective re fra c tio n . W e can in fe r th e c o rn e a l a s tig m a tis m by assum ing th a t
th e back to ric surface is p a ra lle l to th e cornea in b o th m eridians; it is —3.00 x 180. So
th e p a t ie n t ’s sub jective re fra c tio n w o u ld be

Rs = - 1 . 0 0 0 - 3 . 0 0 x 180. (5.82)

Copyright 2014 by KMK Educational Services, LLC


238 5.9. CONTACT LENSES
This is the back vertex power of the bitoric lens that should be chosen.

We can summarize many ideas from the preceding discussion with the following
fitting guide. The number given in the table (2.50 D) is an estimate based on
several sources, but it is only a rule of thumb (14; 17). For corneal astigmatism
of greater than 2.50 D, it is difficult to achieve a good physical fit, which is
why these lenses must have a back toric surface.
RGF Generic Rules
Astigmatism Type Type of RGP
None spherical
Corneal (< 2.50 D) spherical
Corneal (> 2.50 D) back toric
Lenticular front toric
Corneal (<2.50 D) and lenticular front toric
Corneal (>2,50 D) and lenticular bitoric

wsmmmmsrnmmmmmm
Fluorescein pooling can be used to verify the relationship between the corneal
curvature and the base curves of a rigid contact lens. In particular, we briefly
discuss the staining patterns associated with on-K, flatter-than-K, steeper-
than-K, and astigmatic fits (6).
on-K fit: The pattern will show a green ring in the periphery of the lens and
no fluorescein in the optical zone.
Flatter-than-K fit: The center of the lens will touch the cornea (central
touch), meaning that there will be no fluorescein in the central region
but a broad green ring underneath the peripheral region of the lens.
Steeper-than-K fit: Fluorescein will pool in the apical region, forming a
bulls-eye pattern (concentric rings of pooling, touch, pooling, starting
from the center).
Fit over W TR Astigmatism: A dumbbell shaped pattern of touch will ap­
pear horizontally, with pooling vertically.
Fit over ATR Astigmatism: A dumbbell shaped pattern of touch will ap­
pear vertically, with pooling horizontally.
Flexure
We have assumed that rigid contact lenses maintain their curvature when
placed on the cornea. However, if the lens is too thin, a certain amount of
flexure will occur.
Copyright 2014 by KMI< Educational Services, LLC
CHAPTERS. PHYSIOLOGICAL OPTICS I 239

Keratometry and Flexure A keratometry reading can be performed while


a patient wears the contact lens to determine if the lens maintains its shape.
For example, for a rigid lens with a spherical back surface, we have (6)
• K-readings are spherical -> Lens maintains shape
• K-readings are toric -> Lens exhibits flexure

Plus lenses tend to maintain their shape, as do minus lenses thicker


than about 0.13 mm (6).

Interestingly, flexure can actually be helpful. For example, if a thin contact lens
is fit over a cornea with WTR astigmatism, the flexure of the lens counteracts
the typical internal (noncorneal) astigmatism (which, you will recall, averages
—0.50 D x 90) (6).
Warpage
In contrast to flexure, which occurs when the lens is on the eye, warpage can
occur even when the lens is not on the eye. Warpage can be measured using a
radiuscope. Because it modifies the base curve of the lens, warpage can alter
the magnification properties of the lens and also increase the effects of marginal
astigmatism (5).
Rotation of Toric Lenses
Toric lenses are fundamentally asymmetric and must be correctly aligned on a
patient’s eye. While back toric lenses are typically chosen based on the physical
fit itself and therefore are effectively self-orienting, front toric lenses are free
to rotate. To prevent rotation, the lens is typically prism-ballasted, meaning
base-down prism is ground into the lens to counter rotation. The lens can also
be cut so that the lower portion of the lens sits on the lower lid margin (termed
truncation) (6).

L-A-R-S If the prism-ballasted lens rotates following blinking, one must ac­
count for this slight rotation by altering the axis of the prescribed cylinder.
This is intuitively simple and can be summed up in the following generic rule:

Base-apex meridian rotates clockwise —>Add to axis


Base-apex meridian rotates counterclockwise —> Subtract from
axis (6),

Copyright 2014 by KMK Educational Services, LLC


240 5.9. CONTA CT LENSES
This rule is also known as LARS (left add, right subtract), which assumes the
marker of the base-apex meridian is on the bottom of the lens.
Exam ple 5 .3 3 : D u rin g c o n ta c t tens fittin g , th e lens is seen to r o ta te b y 1 0 degrees
co u n te rc lo c kw is e upon b lin kin g . T h a t is, th e m a rk a t th e b o tto m o f th e lens m o ves to
th e rig h t. T h e lens p rescrip tio n is -j-1 .0 0 D - 2 .0 0 D x 150. H o w w o u ld y o u choose th e
n e w axis?

Solution 5 .3 3 : S in ce th e lens ro ta te s by 1 0 degrees cou nterclo ckw ise, we m u s t choose


th e axis to b e 1 5 0 -1 0 = 1 4 0 . S o th e lens we w ould choose is - h i . 0 0 D - 2 .0 0 D x 140.

Gases such as oxygen can pass through a contact lens by dissolving into the
lens material and then diffusing through the lens. These two effects combine
to determine the permeability P of the lens (6), given by
P = Dk, (5.83)
where D is the diffusivity of the gas in the material and k is the solubility of
the gas in the lens material. D k values range from very high (approximately
150) for extended wear lenses to very low (approximately 10) for daily wear
lenses.
The total amount of gas that actually passes through a lens can be measured
in terms of transmissivity, T, which is given by (6)
T
P
t
Dk
t ’
(5.84)
where t is the thickness of the lens. The amount of oxygen transmitted to the
cornea when the contact lens is worn increases with the permeability of the
lens material and decreases with lens thickness.

As a rough guideline, one can consider P to depend strongly on


the water content of the lens:
• High water content —>High permeability (high Dk) (6).
This trend holds for traditional hydrogel lenses. On the other hand,
the trend for silicone hydrogel lenses is just'the opposite:
* High water content (silicone hydrogels) -» Low permeability
(low Dk).
The trend is reversed for silicone hydrogels because silicone is more
permeable to oxygen than water.

Copyright 2014 by KMK Educational Services, LLC


CHAPTER 5. PHYSIOLOGICAL OPTICS I 241

Corneal Edema
Corneal swelling can occur if the cornea is not receiving sufficient levels of
oxygen. Corneal edema can be remedied by (6):
• Reducing the contact lens wearing time,
• Fitting the patient with a looser lens.
• Choosing a higher Dk value.

Soft contact lenses are much easier to study, from an optics perspective, because
they do not retain their shape on the cornea. As a result, the tear lens has
no optical effect. The problems are therefore identical to those involving hard
contact lenses, except that Ftl = 0 always. Assuming the lens conforms to
the corneal topography, we have (6):
• Corneal astigmatism will NOT be corrected by spherical soft contact
lenses.
• Spherical soft contact lenses do NOT correct internal astigmatism (obvi­
ously).
• Fitting steeper or flatter than K will NOT change the prescription.
• One exception: For an aphakic patient with a very thick soft lens, it is
possible that a negative tear lens forms (so the contact lens power must
be slightly greater than that given by the subjective refraction).

Typically, patients can only be corrected with spherical soft contact


lenses if they have less than 1.00 D of WTR or 0.50-0.75 D ATR
or oblique refractive astigmatism (6).

Powers and Soft Contact Lenses


• Soft contact lenses are typically fit flatter than K readings.
• The term residual astigmatism, when soft contact lenses are used,
refers to the amount of astigmatism that is uncorrected. Since spherical
soft contact lenses do not correct corneal astigmatism, we can write
A r = A kx (5.85)
Copyright 2014 by KMK Educational Services, LLC
242 5.10. MAGNIFICATION AND LOW VISION
• Toric contact lenses can be chosen to correct astigmatism. The lens is
truncated or prism ballasted to prevent rotation. The front of the lens is
typically chosen as the toric surface (6).
|A(:eom m odaU on and
While we will not re-cover the optics of accommodation and magnification, we
will note some important concepts (5; 6):
1 Accommodation
• Accommodation required by a spectacle wearing hyperope is GREATER
than that required by an emmetrope.
• Accommodation required by a spectacle wearing myope is LESS
than that required by an emmetrope.
• Clinical Application: A presbyopic myope might need a reading add
with contact lenses, even if lie/she does NOT need an add with
spectacles (6).
2 Magnification
• Myopes -» Retinal image size larger with contact lenses than with
spectacles.
• Hyperopes —>■ Retinal image size smaller with contact lenses than
with spectacles.
• If the most ametropic eye shows axial ametropia —> Induced ani­
seikonia is minimal with spectacles (6).
• If the most ametropic eye shows refractive ametropia Induced
aniseikonia is minimal with contact lenses (6).
- SECTIO N 5.10 ---------------------------------------------------------------------

M agnification and Low Vision


Here we delve further into often-used concepts related to magnification (8 ,
ch. 13) (5, ch. 10).

Angular magnification is a way of comparing the retinal image sizes produced


during two situations. Specifically, it is defined as the ratio of the relevant
retinal image sizes under two different sets of conditions. Examples of angular
magnification include relative size and distance magnification. Note the dif­
ference between angular and lateral magnification (to — L jL '), the latter of
which is the ratio of image size to object size.
Copyright 2014 by KMK Educational Services, LLC
CHAPTER 5. PHYSIOLOGICAL OPTICS I 243

Relative Size Magnification: Angular magnification due to a change in ob­


ject size. That is, relative size magnification is the ratio h jh -, where fy
is the retinal image size of an object at one size, and h is the retinal
image size of the same object due to an increase in the size of the object.

Low vision patients are often encouraged to photocopy and enlarge


reading material. This is an obvious and direct use of relative size
magnification.

Relative Distance Magnification: Relative distance magnification is sim­


ilar to relative size magnification, but it is instead due to a change in
object distance. This leads to
M = U2 (5.86)
where u 1 is the original distance and is the new object distance.

For objects place inside J7, the primary focal point, a plus lens can act as
a magnifier (8, pp. 247-51) (5, pp. 366-70). Of course, when we place a lens
between the object and our eyes, we will have to move the object to see it clearly.
As a result, the overall magnification is a combination of lateral magnification
of the lens and Relative Distance Magnification, Specifically, for objects placed
inside F
Mt —m M , (5.87)
where M t is the total magnification, M is the relative distance magnification,
and m is the lateral magnification of the lens. A collimating magnifier
results from placing an object exactly at F. In this case we have
M t = |u[F, (5.88)
where u is the distance from the person’s eye to the (clearly viewed) object
without the magnifier and F is the power of the magnifying lens.. Note that
this is independent of the distance of the viewer from the magnifier. Do you
know why?

If not stated otherwise, we assume objects are placed at F.

Copyright 2014 by KMK Educational Services, LLC


244 5.10. MAGNIFICATION AND LOW VISION

Exam ple 5 .3 4 : Jon loo ks a t a s m a ll o b je c t 20 cm in fro n t o f his eye. H e th e n uses a


+ 1 5 .0 0 D lens lo c a te d 8 c m in fro n t o f his eye as a m agnifier. W h a t a n g u la r m a g n ific a tio n
(re la tiv e to th e in itia l v iew in g d is ta n c e ) is achieved b y m o v in g th e o b je c t to 4 cm in fro n t
o f th e m a g n ifie r?

Solution 5 .3 4 :
T h e s e tu p is show n in F ig u re 5 .9 . N o te t h a t w hen using th e tens, th e d is ta n c e b etw een Jon
a n d th e o b je c t m u s t chan ge so t h a t he can see it clearly. T his d ep en d s on th e p ro p e rtie s
o f his eye (w h ic h a re in h e re n tly g iven in this p ro b le m ). C o nsider th e fin a l s e tu p :

L = l / l ~ l / ( —0.04) = -2 5 .0 D,
iJ = F + L = - 1 0 . 0 D,
l‘ = 1jlJ = —10 cm, (5.89)
so
m = L /L r = 2.5,

a n d a v irtu a l im a g e is lo c a te d 10 cm to th e o b je c t side o f th e lens. T h is im a g e is th e


o b je c t for J o n 's eye. I t is lo c a te d 8 + 10 = 18 cm a w a y fro m Jon. So w e h ave

M = h / h = —2 0 / — 18 = 1 . 1 1 a n d hence
1 ! (5.90)
M t = 1 .1 1 ( 2 .5 ) = 2,8 X , v

w h e re M t is th e to t a l m a g n ific a tio n . U sing th e m a g n ifie r m ean s th e im a g e h e sees is 2.5


tim e s larger, b u t it is also closer. T h e to ta l resu lt is a m a g n ific a tio n ( o f re tin a l im a g e ) by
2.8 X .

20 cm

Object

G 8 cm

*
Object

V
Figure 5.9: Initially, Jon is located 20 cm from the object and views it clearly.
Subsequently, he is 8 cm from the lens, which is in turn 4 cm from the object.

Copyright 2014 by KMK Educational Services, LLC


CHAPTER 5. PHYSIOLOGICAL OPTICS I 245

Labeling a Magnifier
There is an inherent problem in our setup: different initial distances lead to
different values of M . The solution is simple: use a standard distance (u — 25
cm).

With the standard distance chosen to be 25 cm, magnification can


be calculated using Equation 5.88 as

4* (5.91)

Exam ple 5.3 5: W h a t is th e increase in re tin a l size fo r a + 1 5 .0 D th in lens used as a


c o llim a tin g m ag n ifier? A ssum e th e person in itia lly views th e o b je c t a t 140 cm . How
w o u ld this lens be la b e le d in a store?

Solution 5 .3 5 : T h e a n g u la r m a g n ific a tio n is

M = 1.4(15) = 21 X (5.92)

N o te th is is th e sam e no m a t t e r h o w fa r th e m a g n ifie r is fro m h is /h e r eye, as lon g as th e


o b je c t is a t P , th e p rim a ry fo c a l p o in t. T h e lens used w o u ld b e la b e le d as 1 5 /4 = 3.8 X .

Stand Magnifier
In stand magnifiers used for low vision patients, the object to lens distance
is constant and the image is formed a set distance from the magnifier. Often,
light leaving the magnifier is divergent (that is, the object is located within one
focal length of the magnifier) and therefore accommodation or an appropriate
add must be used. When both a stand magnifier and the add lens (or accom­
modation) are used together, one often specifies the equivalent power of the
magnifier-add system and uses that to find the total magnification (the change
in retinal image size). Specifically, the total magnification is simply Fequiv/i ,
In general, when a magnifier is non-collimating (such as a stand magnifier), the
maximum possible magnification is achieved when the eye is placed very close
to the magnifier. This requires a great deal of accommodation and/or an add.
In this case, this maximum magnification is given by
(5.93)
where F is the dioptric power of the magnifying lens.
Copyright 2014 by KMK Educational Services, LLC
246 5.10. MAGNIFICATION AND LOW VISION
Hand Held Magnifier
The magnification of a hand magnifier depends on how it is used. When used
as a collimating magnifier, the light leaving the lens is in the form of plane
waves, so no accommodation is needed. However, if the magnifier is held so
that the object is closer than the focal point, accommodation (or an add) must
again be used, and it is appropriate to talk about the equivalent power of the
magnifier-add system. In addition, the (linear) field of view can be found by (5,
pp. 368):
vi = d/(Fel), (5.9 4)

where d is the diameter of the lens, Fe is the equivalent power of the system,
and l is the distance between the magnifier and the spectacle plane.

Exam ple 5.36 : A p a tie n t uses a 4 0 m m h an d m a g n ifie r w ith p o w e r + 15 .0 0 D h e ld 8


cm fro m th e eye. A ssu m e th e m a g n ifie r is used as a c o llim a tin g m a g n ifie r a n d fin d th e
field o f view.

Solution 5 .3 6 : I f th e m a g n ifie r is used as a c o llim atin g m ag n ifier, no a c c o m m o d a tio n is


used so th e e q u iv a le n t p o w e r o f th e system is ju s t 15 .00 . T h e field o f view is th en

w= 0.040/(15.00 X 0.08) = 0.033, (5.95)


or 33 m m . I f th e s a m e m a g n ifie r were used d iffe re n tly so t h a t a c c o m m o d a tio n was
required, th e e q u iv a le n t p o w e r in this calcu latio n w ou ld change.

Telescopes deal with distant objects. We do not need to specify a reference


distance in this case because incoming waves can always be assumed to be
plane waves (8, pp. 260-65) (7, pp. 219-22).
Types of telescopes
There are two main types of telescopes:
1 Keplerian:
• Both lenses are positive (converging).
• Tube length d is given by
d - fobs + foe (5.96)
with f oc the focal length of the ocular lens and f 0bj the focal length
of the objective lens. Note that f oc and f 0bj are both positive for a
Keplerian telescope.
Copyright 2014 by KMK Educational Services, LLC
CHAPTER 5, PHYSIOLOGICAL OPTICS I 247

• Final image is magnified and inverted (but an inverting system is


typically included inside the telescope to compensate).
• Objective lens forms an inverted, real image inside the telescope.
• Ocular lens is used as a simple magnifier to view the image.
• Exit pupil is located outside of the telescope.
• Externally located exit pupil leads to a relatively large field of view,
because the exit pupil can be more easily aligned with the eye’s
entrance pupil.
• Available in higher magnifications than Galilean telescopes.
2 Galilean:
• Ocular lens is negative (diverging), objective lens is positive (con­
verging) .
• Tube length L is given by
d = fob, + foe ( 5 . 97 )

with f oc the focal length of the ocular lens (negative) and / 0^ the
focal length of the objective lens (positive).
• Lenses are separated by a distance equal to the difference in their
focal lengths.
• Objective lens forms a real image past the ocular lens; this image
serves as a virtual object for the ocular lens.
• Final image is magnified and erect.
• Exit pupil is located inside the telescope.
• Internally located exit pupil leads to a relatively small field of view.
• Only available in magnifications up to 4X,

Exam ple 5 .3 7 : A telescope is m a d e fro m an o b je c tiv e fens o f +20 .0 0 a n d an o cu lar


—50.00 D . H o w fa r a p a r t are th e lenses? W h e re is th e e x it p u p il? Finally, i f th e
lens o f
telescope h a d an ad ju s ta b le leng th so t h a t i t c o u ld b e used to view a n o b je c t 2 5 cm away,
h o w fa r a p a rt w ould th e lenses be se t?

Solution 5 .3 7 : T h is is a 2 .5 X G alilean telescope. T h e fo cal le n g th o f th e tw o lenses


are 5 cm a n d 2 cm, respectively. T h e re fo re , th e lenses m u s t be p la c e d 3 cm a p a rt (th e
difference in th e fo c a l le n g th s ) to form a n a fo c a l system . T h e e x it p u p il is th e im ag e o f
th e o bjective lens thro u g h th e o c u la r lens. T re a tin g th e o b je c tiv e lens as an object, we
fin d t h a t th e in c o m in g vergence ( L ) s tr ik in g th e o c u la r lens is —33.33 D , so th e exitin g
vergence ( L ’) a fte r re fra c tio n is —83.33 D . T h erefo re, th e e x it p u p il is lo c a te d 1 .2 cm to
th e o bjective side o f th e o c u la r lens. Finally, i f th e telescop e were used to view an o b je c t

Copyright 2014 by KMK Educational Services, LLC


248 5.10. MAGNIFICATION AND LOW VISION
2 5 cm away, w e m u s t a d d th e mergence fro m th a t o b je c t to th e p o w e r o f th e o b je c tiv e
lens ( 9 ) . H en ce, we a d d —4.00 D to th e objective power, g iv in g an e ffe c tiv e o b je c tiv e lens
pow er o f + 16 .0 0 D . T h e d is ta n c e b etw een a + 16 .0 0 D lens a n d a —50,00 D lens w o u ld
ag ain be th e d ifferen ce in th e ir fo c a l lengths, o r 4 .2 5 cm. T h is is th e tu b e le n g th t h a t
w o u ld be used to see th e n ea r o b je c t.

Magnification for a Telescope


Magnification for a telescope compares the retinal image size with the telescope
to the retinal image size without the telescope. It is given by
M = - (5.98)
where Foc is the power of the ocular lens, F0bj is the power of the objective
lens, and the minus sign reminds you that the image is inverted for a Keplerian
telescope and upright for a Galilean telescope. Also, the magnification of a
telescope is given by
M = dEnt/ds#, (5.99)
where dEnt Is the diameter of the entrance pupil and dgx is the diameter of the
exit pupil. Note that the entrance pupil of a telescope often is the objective
lens.

Telescopes are labeled A x B, where A is magnification and B is


the objective lens diameter in mm.

Exam ple 5 .38 : W h a t is th e size o f th e e x it p u p il for a 5 x 4 5 telescope?

Solution 5 .3 8 : T h e telescop e has 5 X m ag n ific a tio n a n d a 4 .5 cm o b je c tiv e lens. A s s u m in g


t h a t th e o b je c tiv e tens is th e e n tra n c e pupil, we can solve eq u atio n 5 .9 9 fo r d ex to g e t
th e size o f th e e x it pupil, w hich is = d E n t / M = 4 .5 /5 = 0.9 cm .

Copyright 2014 by KMK Educational Services, LLC


CHAPTER 5. PHYSIOLOGICAL OPTICS I 249

Spectacle Mounted Telescope


Tblescopes can be mounted in the upper portion of the carrier lens (for occa­
sional use) or directly aligned with the line of sight (for continuous use). In
the former case, the telescope is above the line of sight in what is known as the
bioptic position; in the latter case, the telescope is in the full diameter po­
sition. However, if these are used to view objects closer than optical infinity, a
problem arises because light from near objects is diverging, so light through the
telescope is very divergent. As a result, the accommodation demand is tremen­
dous. The solution is simple: put a collimating magnifier (reading cap) on the
front of the telescope. Now incoming divergent waves are made parallel. The
total magnification is just the product of the two individual magnifications (8,
pp. 287) (5, pp. 364):
M t = MCM*, (5.100)
with M t the total magnification, Mc the magnification of the cap (Ff 4), and
Mt the magnification of the telescope.

Note that a telescope with a reading cap is optically equivalent


to a microscope. It is sometimes called a telemicroscope. The
advantage of a telemicroscope (as opposed to a magnifier) is that
it can typically be used at a longer working distance.

Exam ple 5.39: A lo w vision a id has a 4 X telescop e a n d a -(-3.00 D cap . W h a t is th e


to ta l m a g n ific a tio n o f th e telescop e a n d lens cap ? W h a t re a d in g distance w ou ld this be
used for?

Solution 5.39: F irs t we fin d th e m a g n ific a tio n o f th e cap. I t is g iven by

M = 3 .0 0 /4 = 0.75 X . (5 .10 1)

So th e to t a l m a g n ific a tio n is s im p ly

M t = 0.75(4) = 3 X . (5.102)

T his m u s t be used a t a re a d in g d is ta n c e d of

(5.103)

Copyright 2014 by KMK Educational Services, LLG


250 5.11. CLINICAL LOW VISION

Some patients with retinitus pigmentosa might benefit from turning


the telescope around and using it backwards. This will obviously
decrease the patient’s acuity, but it drastically increases the field
of view.

Exam ple 5.40 : A p a tie n t w o u ld like to see letters on a d is ta n t sign. S e e in g these le tte rs
a t this d is ta n c e requires 2 0 / 2 0 acuity. I f th e p a tie n t's d istance a c u ity is m e a s u re d to be
2 0 /1 0 0 , w h a t p o w e r telescop e w ou ld h e need?

Solution 5.40 : T h e m a g n ific a tio n o f a telescope can be easily c o n n ected w ith acuity.
R e c a ll t h a t a c u ity is th e inverse o f M A R , th e m in im u m angle o f resolution. So in th is
p ro b lem , th e person w a n ts to g o fro m a M A R o f 5 a rc m in u te s ( 2 0 / 1 0 0 ) to an M A R o f
1 a rc m in u te ( 2 0 / 2 0 ) . T h is chan ge represents o f fa c to r o f 5 d ifference in M A R , so th e
p a tie n t w o u ld n eed a 5 X telescope.

SECTION 5.11

Clinical Low Vision


Low vision in the clinic is first assessed by careful refraction, followed by a
measurement of the patient’s corrected acuity. Often the Snellen chart is not
sufficient and modified charts, such as the ETDRS, are used at closer distances.
Below we review some other aspects of low vision (1).

Kestenbaum’s Rule: One can estimate the minimum power needed


in an add for a low vision patient who wants to read standard
print. The add power is simply 1/V, with V the distance visual
acuity (Snellen fraction).

Exam ple 5 ,4 1 : W h a t a d d w ou ld you prescribe a lo w vision p a tie n t whose visual a c u ity is


2 0 /1 0 0 ? A s s u m e th e p a tie n t w a n ts to re a d s tan d ard p rin t.

Solution 5 .4 1 : A c c o rd in g to K e s te n b a u m 's rule, th e p a tie n t m ig h t b e n e fit fro m a + 5 . 0 0


add. H o w ever, n o te t h a t this m ean s th e reading d istance (assu m in g no a c c o m m o d a tio n )
is a b o u t 2 0 cm .

Copyright 2014 by KMK Educational Services, LLC


CHAPTER 5. PHYSIOLOGICAL OPTICS I 251

The M unit is used to describe the size of print. One M unit is defined as the
size of a symbol subtending 5 arc minutes when viewed at 1 m. Most newspaper
print is 1 M. In terms of linear size, one M unit is 1.45 mm or about 1/16 of
an inch.

A Snellen fraction of 20/20 implies that MAR is 1 arcminute. Since


a just visible letter must subtend an angle equal to 5 times the
MAR, a patient with 20/20 vision could resolve a 1 M letter at 1
m.

Exam ple 5 .4 2 : A p a tie n t can ju s t see 2 M p rin t fro m a d istance o f 1 m . W h a t is his


ac u ity ?

Solution 5 .4 2 : A p a tie n t w ith 2 0 / 2 0 vision can see 1 M p rin t fro m 1 m . I f th e p a tie n t


can ju s t b arely see 2 M p rin t fro m th e s a m e d istance, his a c u ity is a b o u t 2 0 /4 0 .

Near Acuity and M notation


It is common to specify near acuity measurements using the Lighthouse Acu­
ity Card. Acuity can then be written as a distance (in meters) divided by
an M unit. For example, if the patient can see 1M print at 40 cm, one could
specify that as 0.40/1M. In doing so, we are essentially specifying an inverse
angle (a distance divided by a letter size, or ’’adjacent over opposite” if you pre­
fer to think of trigonometry). This acuity can be easily converted to Snellen
notation.

Exam ple 5.43: A p a tie n t can ju s t resolve 2 M p r in t fro m 5 0 cm. W h a t is h e r ac u ity ?

Solution 5.43: T h e a c u ity in M n o ta tio n is s im p ly 0 . 5 0 / 2 M . In term s o f a decim al, this


is 0.2 5. F o r a Snellen acuity, th is d e c im a l is th e s a m e as

20/x = 0,25 (5.104)


so x = 80. So the S nellen a c u ity w o u ld b e 2 0 /8 0 .

Copyright 2014 by KMK Educational Services, LLC


252 5.11. CLINICAL LOW VISION
CA >V", ■A.-''7 V';,s.i A Ai/AA :

The following statistics describe low vision in the United States (4):
• 1.1 million Americans are legally blind.
• 4.3 million Americans are visually impaired.

Vision loss can be broadly divided into central vision loss, peripheral vision
loss, and overall blur (4).
Central vision loss
• Generally due to macular disease.
• Patients will have trouble with near tasks, contrast issues, reading signs
and recognizing faces.
• Patients may have a scotoma.
• Patients generally have no trouble with mobility.
• Patients may need magnification, lighting and glare control, eccentric
viewing training, and contrast enhancement.
Peripheral vision loss
• Generally due to conditions like glaucoma and retinitis pigmentosa.
• Patients will have trouble with mobility, night vision, and glare.
• Patients may need held expansion and awareness, orientation and mobil­
ity, and tints.
General blur
• Can be due to a variety of conditions such as cataracts or corneal haze.
• Patients will have complaints of glare, blur at distance and near, poor
contrast, and may have mobility trouble.•
• Patients may need magnification, lighting and glare control, and contrast
enhancement.
Copyright 2014 by KMK Educational Services, LLC
CHAPTER 5. PHYSIOLOGICAL OPTICS I 253

Vision can be classified as follows based on the patient’s best corrected acu­
ity (1):
Normal Vision: Normal vision includes acuities from about 20/12 to 20/25.
Near-Normal Vision: Near-normal covers the range from 20/30 to 20/60.
Note that a 20/60 person could not resolve 1 M print at 1 m, but can
resolve it at a distance of 1/3 m.
M oderate Low Vision: Moderate low vision is characterized by acuities of
20/70 to 20/160. Because these patients must read (1 M print) at a
distance of less than 25 cm, convergence becomes a problem and prism
is often needed to maintain binocular viewing. Alternatively, the patient
can read binocularly with a hand held magnifier whose diameter is at
least as large as the patient’s PD.

When prescribing high powered readers, a rule of thumb is to apply


(X+2) prism diopters of base in prism for each eye, where X is the
sphere power of the lens used for reading.

Severe Low Vision: Severe low vision covers acuities ranging from 20/200
to 20/400, meaning 1 M reading distance must be 10 cm or less. This
close proximity precludes binocular reading, and these patient’s must
read by occluding one eye. Their reading is typically much slower, but
still possible. Note that those with severe low vision would be considered
legally blind in the United States.
Legal Blindness: Legal blindness in the United States corresponds to one or
more of the following (15):
1 Patient cannot read any letters on the 20/100 line in the better eye.
2 A visual field diameter of 20 degrees or less in the better eye.
Profound Low Vision: Profound low vision includes acuities from 20/500 to
20/1000, meaning that the 1 M reading distance is considerably less than
5 cm. Reading is very difficult in these cases.
Near Blindness: Near blindness includes acuities that are worse than 20/1000.
Blindness: Blindness means that a patient has no light perception.
Copyright 2014 by KMK Educational Services, LLC
254 5.11. CLINICAL LOW VISION

Reduced contrast sensitivity is seen in most low vision patients. Contrast can
be measured in a number of ways (1):
Pelli-Robson: This chart consists of large letters with uniform size. The rows
of the chart show progressively decreasing contrast.
Vistech Contrast Test System: This chart uses a series of sine wave grat­
ings of various contrasts and frequencies to assess contrast sensitivity.

Visual field testing in the low vision patient is done in several ways (1):
Peripheral Field Loss: Loss of peripheral field can be assessed using Gold-
mann (kinetic) or Humphrey (static) perimetry or Tangent screens.
Central field loss: Central scotoma is most effectively detected with a scan­
ning laser ophthalmoscope, but Goldmann perimetry and the Amsler grid
can also be used. The latter two are less effective at detecting small cen­
tral scotomas because of perceptual filling in by higher order cortical
processing.

References

[1] B asic and Gtincial Science Course, Section 3: Optics, Refraction, and C ontact Lenses.
Foundation of the American Academy of Ophthalmology, 2002.
[2] The B erkeley Guide, 8th E dition (2002). University of California, Berkeley School of Op­
tometry, UC Optometric Student Association,
[3] C. Brooks and I. Borisli (1996). System fo r Ophthalmic D ispensing, 2nd Edition.
Butte rwor th-Heinemann.
[4] E. Faye, D. Albert, B. Freed, K. Seidman, and M. Fischer (2000). The Lighthouse Ophthal­
m ology R esident Training M anual, Lightouse International.
[5] T. Fannin and T. Grosvenor (1996), Clinical Optics, 2nd Edition. Butterworth-Heinemann.
[6] T, Grosvenor (2002). Primary Care O ptometry, 4th Edition. Butterworth-Heinemann.
[7] E, Hecht (2002). Optics, 4th Edition. Addison Wesley Longman,
[8] M. Keating (1988). G eom etric, Physical, and Visual Optics. Butterworth-Heinemann.
[9] Lakshminarayanan, V. and E. Bennett (2006). Review Q uestions fo r the N B E O E xa m in a ­
tion, P ari O ne , Butterworth Heinemann.
[10] J. Parel, G. Crock, L, Pericik (1980). J. Phys, E: Sci. Instrum., 13, 1242-53.
[11] Schwartz, S (1999). Visual Perception: A Clinical O rientation, 2nd Edition. McGraw-Hill.

Copyright 2014 by KMK Educational Services, LLC


CHAPTERS. PHYSIOLOGICAL OPTICS I 255

[12] E. Stoner, P. Perkins, and R. Ferguson (2005). Optical Formulas Tutorial, 2nd Edition.
Butterwortli Heineniann.
[13] D. Wooten (2003). Optical Training: Skills and Procedures, Elsevier Health Sciences.
[14] http ://www. clspectrunt.com/art ic le ,aspx?article=12047
[15] www.lighthouse.org
[16] www.opticampus.com/tools/anBi.php
[17] http://www.psyduck.nat/school/Bitorics.pdf
[18] See, lor example, Uvcx Rx ANSI Advisory Bulletin 2010, http://www.uvexrx.com

Copyright 2014 by KMI< Educational Services, LLC


c-c .
((
(
('
(
( · ..
'.
j
f

((
;

((
(f
((

((_
((
((
((
((
(
(
(
((
((
(f' .. I
.

(_(
(
(_
((
(c
(
{_ /

(
(_
(
(_.
(( .
Chapter

Physiological Optics II

Kevin B. Wood, Ph.D.

257
c
(
(
(
(
(
(
(
(
c.
(
(
(
(
(
(
(.
(
(
(
(
(
(
Copyright 2014 by KMK Educational Services, LLC (
(
(
(
CHAPTER 6. PHYSIOLOGICAL OPTICS II 259

- SECTION 6.1 -------------------------------------------------------

Models of the Eye


We can build mathematical models of the eye which are accurate to varying
degrees. The more complex the model, the better it describes the actual eye.
On the other hand, simple models can be used to gain a great deal of insight
about the optics of vision. We review some of the major concepts related to
some traditional eye models (8, ch. 5). Specifically we introduce, in decreasing
order of complexity:
1 Gullstrand’s Exact Model
2 Reduced Eye Model

In this highly detailed model, the eye is made up of the cornea (which has both
a front and back surface), the humor, and the lens (which has both a core and
an outer layer). In total, there are 6 refractive surfaces (front and back cornea,
front and back inner lens, front and back outer lens) and 4 chambers (each with
a different n). In general, calculations with this model are very tedious and
unnecessarily complicated. Some typical parameters of the unaccommodated
Gullstrand exact eye are given in the following table (8, pp. 110). Note that
Gullstrand also developed a somewhat simpler version of the eye, but we do
not cover those details here.

Gullstrand^ Exact Eye:


Component Property
Cornea ' n = 1.376
Aqueous/Vitreous n — 1.336
Lens inner core ' n ~ 1.406
Lens outer core n — 1.386
Axial eye length 24.00 mm
Cornea to anterior focal point 15.70 mm
Cornea to posterior focal point 24.38 mm
Cornea power 43.05 D
Lens power 19.11 D
Total eye power 58.64 D

samer"*
The reduced eye model simply treats the eye as a single spherical refracting
interface at some axial length from the retina. This is very handy for quick
Copyright 2014 by KMI< Educational Services, LLC
260 6.2. REFRACTIVE ANOMALIES: AMETROPIA
calculations, and we will use a similar model below to understand the basics of
visual optics.

Reduced Eye:
Component Property
Interior eye n - 1.33 ;
Cornea to anterior focal point 16,67 mm
Cornea to posterior focal point 22.22 nun
Total eye power 60.00 D

Exam ple 6 .1 : C o n s id e r a red uced eye m o d e l w ith vitreous o f n = 1.3, a single lens (se rv in g
as th e c o rn e a /le n s c o m b in a tio n ) o f p o w e r + 61.0 0 , a n d a 3 m m (d ia m e te r) a p e rtu re s to p
lo c a te d 7 m m b e h in d th e lens (in th e vitreous). W h a t is th e size o f th e e x it p u p il? W h a t
a b o u t th e e n tra n c e p u p il?

Solution 6 .1 : T h e e n tra n c e a n d e x it p up ils are ju s t th e im ages ( i f th e y e x is t) o f th e


a p e rtu re stop. In th is eye, th e re are no lenses b eh in d th e a p e rtu re stop, so th e a p e rtu re
sto p is its e lf th e e x it p u p il, m e a n in g th a t the e x it p u p il is 3 m m in d ia m e te r ( 9 ) . The
+61.00 D lens is th e en tra n c e pupil. T h e a p e rtu re
im ag e o f th e a p e rtu re s to p th ro u g h th e
s to p is lo c a te d 7 m m fro m th e lens , so we have l = —0 .0 0 7 m . S in ce th e w a v e fro n t is
lo c a te d in th e vitreo u s ( n = 1 .3 ), th e verge nee is L = 1,3/(— 0,007) = —185.71 D . T h e
o u tg o in g vergence is th en

L' = F + L = 61.00 - 18 5 .7 1 = - 1 2 4 . 7 1 . (6 .1)

T h e e n tra n c e p u p il is th e re fo re lo c a te d a t l f = l / ( —12 4 ,7 1) = —0.008 m , w hich is 8 m m


to th e vitre o u s side o f th e lens. T h e m ag n ificatio n ( m ) is

m = L(L!« 1.5. (6.2)

H ence, using m = d ent j d e x , th e size o f th e entran ce p u p il is 1 .5 ( 3 ) , o r a b o u t 4 .5 m m .

- SECTION 6.2 -------------------------------------------------------------------

Refractive Anomalies: A m etropia


To gain an intuitive understanding of some important concepts, we will now
utilize a thin lens (reduced) model of the eye which is similar to those in Keat­
ing (1, ch. 6) and Fannin (8, ch. 5). This model is of course an oversimplification
of the biological complexity, but it nonetheless gives insight into some aspects
of visual optics.
Copyright 2014 by KMK Educational Services, LLC
CHAPTER 6. PHYSIOLOGICAL OPTICS II 261

:.V + ■ ■ ■ t - v / C '■'■

The eye is considered to be a thin lens (corresponding to the cornea-lens com­


bination) separated from a screen (corresponding to the retina) by material of
index n. We take the thin lens to be +60.00 D for the unaccommodated eye
and use this reduced eye model to study some simple properties of ametropia.
Far point: The far point is the point conjugate to the axial retinal point.
Another equivalent definition is as follows: a point object at the far point
will result in the formation of a point image at the retina (and vice versa).
This definition, of course, is not limited to a thin lens model of the eye.
Far point sphere: When the eye rotates, the far point is no longer just a
point, but traces out a spherical surface. This surface is known as the far
point sphere, and its center of curvature is the center of rotation of the
eye (about 27 mm behind the typical spectacle plane) (8).

The far point of an ametropic eye can be found from a correcting


prescription. The far point is always located an absolute distance of
I f F from the correcting lens, where F is the power of the correcting
lens.

An emmetrope is a person who can see a distant object clearly, which means
that incoming plane waves should converge to a point at the retina. In the
reduced eye model, the retina should be located at F ', the secondary focal
point of the lens, which must account for the index of refraction used for the
interior eye. For a reduced eye with power +60.00 D and an internal index
n=1.3., the retina should be located at 1.3/60 m, or about 22 mm from the
lens, to be emmetropic.

The far point for an emmetrope is located at optical infinity.

A myope is a person that is near-sighted. Optically speaking, light from in­


coming plane waves will converge to a point in front of the retina.

Copyright 2014 by KMK Educational Services, LLC


262 6.2. REFRACTIVE ANOMALIES: AMETROPIA

The far point for a myope will be located between the eye and
infinity; that is, it will sit somewhere in front of the retina.

Exam ple 6 .2: W h e re is th e fa r p o in t fo r a m yo pe w ith an ax ial eye le n g th x = 2 7 mm a n d


F = 60.00 D ? W h a t p o w e r o f glasses w ith vertex d istance o f 1 4 m m w o u ld b e re q u ire d ?
You m a y use a re d u c e d eye m odel.

Solution 6.2: O n e easy w ay to fin d th e fa r p o in t is to reverse th e system a n d co n sid er


lig h t le a vin g a p o in t o n th e re tin a . T hen we have

l = —2 7 m m ,
L = L 3 3 / ( —0.027) - -4 9 .2 6 D ,
L' = F + L = 10.74 D , ' ' }
l' = 1 / L ' = 9.31 cm.

T his is th e lo c a tio n o f th e fa r p o in t. A n o b je c t p la c e d here w ill form a c le a r im a g e on th e


re tin a , in tu itiv e ly , th is m akes sense since we k n o w th e p a tie n t is n ear-sig hted, a n d 9.31
cm is less th a n in fin ity (w h ic h is th e lo c a tio n o f an o b je c t fo rm in g a clear re tin a l im a g e
for an e m m e tro p e ). T h e p a tie n t needs a —10.74 D lens ( a t th e co rn e a ) fo r c o rrectio n
(a n d o f course y o u w o u ld ro u n d to th e nearest q u a rte r D ) . H ow ever, we do n o t g iv e h e r
a —10.74 D c o n ta c t lens. W h y ? R e c a ll t h a t fo r a x ia l myopes, S p e c ta c le M a g n ific a tio n
con sideration s d ic ta te t h a t we use spectacles (see S ection 5 .6 ). S o we m u s t c o m p e n s ate
fo r th e v e rte x d is ta n c e (see S e c tio n 3 .5 ). W e k n o w we n eed a —10.74 D lens a t th e eye.
S p ectacles w ill s it a b o u t 1.4 cm fro m th e cornea. T h e d istance b etw een th e spectacles
a n d th e fa r p o in t w ill be 9.31 —1.4 = 7.94 cm . So th e p o w er o f th e c o rre c tin g lens a t th is
d istance s h o u ld b e - 1 /0 .0 7 9 4 , o r - 1 2 .5 9 D . T h e fin a l answ er w ou ld be ro u n d e d to - 1 2 .5 0
D

Night myopia: Often, patients will become more myopic under low light lev­
els. This is due to a combination of increased spherical aberration, as
well as light levels which are too small to fully relax accommodation
when viewing distant objects (11).
Myopia Trends
We note here some important epidemiological and age-related trends associated
with myopia (11). For more in-depth coverage, we recommend (11).
• Myopia prevalence at birth has been estimated at between 25 and 50
percent, depending on the study (11).
Copyright 2014 by KMK Educational Services, LLC
CHAPTER 6. PHYSIOLOGICAL OPTICS II 263

• By age 1 year, few children are myopic (11).


• By age 6, myopia of -0.50 D or more is found in only 2 percent of chil­
dren (11).
• Myopia of -0.50 D or more occurs with increasing prevalence between ages
6 and 20, with the number reaching about 20 percent by age 20 (11).
• Myopia of -0.50 D or more occurs in about 30 percent of those aged 20-40
years, and that number then begins to decline slightly (11).
• The prevalence of myopia tends to increase somewhat in later years be­
cause of nuclear lens changes (11).

Hyperopic individuals are far-sighted, meaning that light from plane waves
converges to a point behind the retina. Again, both axial distance and refractive
power play a role.

The far point of a hyperope will be located behind the retina and
is considered virtual.

Example 6.3: Where is the far point for a patient who needs +3.00 D correcting tenses
at the cornea?

Solution 6.3: Again, consider light leaving the retina. We know that the light must be
diverging as it leaves the cornea, and its vergence must be -3.00 (this is why the T3.00
D correcting lens neutralizes it). So the far point is 33.3 cm or 3.00H behind the retina.

Optically speaking, a correcting lens should be placed so that its


secondary focal point coincides with the far point of the ametropic
eye.

Latent hyperopia: Often young hyperopes will accommodate to neutralize


some or all of their hyperopia. As a result, a patient may show a lower
amount of hyperopia in subjective refraction than during objective re­
fraction (11).
Copyright 2014 by KMK Educational Services, LLC
264 6.2. REFRACTIVE ANOMALIES: AMETROPIA
Manifest hyperopia: The manifest hyperopia is the amount present in the
subjective refraction (XI).

Exam p le 6.4: R e tin o sc o p y m easures 4 .0 0 D o f h yp ero pia in a p a tie n t, b u t s u b jec­


tiv e re fra c tio n d e te rm in e s th e re to b e 3 .0 0 D o f h yp ero pia. W h a t a re th e la te n t
a n d m a n ife s t h yp e ro p ia ?

Solution 6.4: T h e m a n ife s t h yp ero pia is 3 .0 0 D , as fo u n d in th e s u b je c tiv e re­


fra c tio n . T h e re m a in in g 1 .0 0 D o f ac c o m m o d a tio n , as m easu red w ith retinoscopy,
rep resents th e la te n t h yp e ro p ia (1 1 ).

Absolute hyperopia: When hyperopia is too large to be neutralized by ac­


commodation, the amount of hyperopia that can not be corrected is called
the absolute hyperopia (11).
Facultative hyperopia: The amount of hyperopia that can be neutralized
by accommodation is called facultative hyperopia (11).

Exam ple 6 .5: C o nsider a p a tie n t w ith 1 .5 0 D o f a c c o m m o d a tiv e ability. The


p a tie n t's d is ta n c e re fra c tio n a t th e cornea is -f 4 .0 0 D . F in d th e a b so lu te a n d fac­
u lta tiv e h yp ero p ia.

S olu tion 6.5: B ecause th e p a tie n t has 4 .0 0 D o f h yp ero pia b u t o n ly 1 .5 0 D o f a c ­


c o m m o d a tio n , th e a b s o lu te h yp ero p ia is 2 .5 0 D (th e difference) a n d th e fa c u lta tiv e
h yp e ro p ia is 1 .5 0 D (1 1 ). *•

Hyperopia Trends
We note here some important epidemiological and age-related trends associated
with hyperopia (11). For more in-depth coverage, we recommend (11). The
statistics regarding hyperopia are a bit confounded by the fact that accommo­
dation ability also changes with age, and the two are naturally linked.
• The prevalence of hyperopia is approximately 6 percent in children aged
6 to 15 years (11). Unlike with myopia, this prevalence does not change
with age for this age range.
• Despite the nearly constant prevalance of hyperopia, one can predict that
a hyperopic child, approximately 5 years old, will likely be (11):
- Hyperopic at age 14 if original hyperopia is greater than 1.5 D.
Copyright 2014 by KMK Educational Services, LLC
CHAPTER 6. PHYSIOLOGICAL OPTICS II 265

- Emmetropic at age 14 if original hyperopia is beteween 0.5 and 1.25


D.
- Myopic at age 14 if original hyperopia is less than 0.5 D.
• As amplitude of accommodation decreases, near work may suddenly be­
come a problem for a hyperopic child.
• Between the ages of 20 and 40, hyperopia in a patient is expected to be
relatively constant (perhaps increasing slightly) (11). However, the de­
crease in accommodation ability during this time may functionally high­
light otherwise “non-burdensome” hyperopia.
• Hyperopes and emmetropes older than 45 tend to show an increase in
hyperopia with age (11).
- SECTION 6.3 -----------------------------------------------------------------------------------------------------

Ametropia: Clinical Considerations


There are a host of clinical skills needed to diagnose and treat ametropia. In
what follows, we outline some techniques for measuring variables related to
ametropia and assessing refractive errors.

Inter pupillary Distance


Interpupillary distance can be measured using a PD ruler or a pupilometer.
In the former case, one measures from one edge of one pupil to the same edge
of another (rather than trying to estimate the pupil center). One can measure
both distance and near PD using this method (2).
1 Examiner sits 40 cm from the patient and closes his/her right eye.
2 Patient is instructed to fixate on the examiner’s left eye. PD is measured
(left pupil edge to right pupil edge). This is the Near PD.
3 With ruler in place, examiner closes his/her left eye and opens his/her
right eye, instructing the patient to fixate now on his/her right eye.
4 The measurement is again made. Because the ruler remained stationary,
the new measurement corresponds to the Distance PD.
Alternatively, one can use a pupilometer, which is designed for such measure­
ments.
Copyright 2014 by KMK Educational Services, LLC
266 6.3. AMETROPIA: CLINICAL CONSIDERATIONS
1 Examiner sets pupilometer to infinity or 40 cm distance, This specifies
the location of the visual stimulus the patient will see.
2 Patient places pupilometer against the bridge of the nose (at the spectacle
plane).
3 Pupilometer light produces a corneal reflex visible to the examiner.
4 Examiner aligns the vertical hairline within the device so that it coincides
with the location of the corneal reflexes.
5 Both binocular and monocular PD can be read directly from the instru­
ment.

Using a pupilometer reduces errors resulting from parallax.

Visual Acuity
Distance acuity is traditionally measured by showing the patient high contrast
letters on a chart 20 feet away. To resolve a letter, (e.g. E), the patient must
be able to distinguish the dark bars making up the letter from the white space
separating different portions of the letter. By determining the smallest letter
that a patient can see, one obtains a measure of his/her acuity.
There are a number of different types of acuity used in a clinical setting (4,
p p . 2 0 0 -2 0 2 ):

Resolution Acuity: RA is determined by asking a patient to distinguish a


pattern from a uniform patch of equal luminance. A typical young adult
has a cutoff at 40 —60 cycles per degree (MAR—0.75).
Recognition Acuity: Snellen acuity is an example of recognition acuity. Recog­
nition acuity only gives information about our ability to resolve high
frequencies; note that this measure is therefore not useful for patients
who show problems at other frequencies (as is the case in patients with
cataracts).
Minimum Detectable Acuity: Minimum detectable acuity asks, essentially,
what is the thinnest possible wire that is visible? The average person has
much better MDA (about 1 arcsec) than Resolution and Recognition
Acuity.
Hyperacuity: Hyperacuity deals with a person’s ability to sense directional
relationships. For example, hyperacuity helps us determine whether two
lines are parallel, and our ability tp do so is very good (the threshold is
on the order of several arcsecs). Hyperacuity is believed to be a result of
higher cortical processing.
Copyright 2014 by KMI< Educational Services, LLC
CHAPTER 6. PHYSIOLOGICAL OPTICS II 267

MAR: MAR is the minimum angle of resolution. It is typically measured in


arcminutes (1/60 of a degree).
Snellen Fraction: Snellen fraction is simply 1/MAR, with MAR measured in
arcminutes. The fraction is typically multiplied by 20/20 to obtain the
standard optometry form.
LogMAR: The LogMAR is simply log(MAR).

20/20 — 1 MAR (minimum angle of resolution) = 0 logMAR

Each letter on the 20/20 line of the Snellen chart subtends an angle
of 5 arcminutes when viewed at 20 feet. Each distinct bar making
up the letter subtends 1 arcminute (for example, it is easy to see
that the E is made of five bars).

Example 6.6: When a patient is asked to read the Snellen chart from a distance of 10
feet, he can just read the 20/20 line. What is his acuity?

Solution 6.6: The 20/20 fine indicates 20/20 acuity at a distance of 20 feet. However,
because distance has decreased by a factor of 2, the acuity must be reduced by the same
factor. Hence his acuity is 20/40.

There are a number of different charts for measuring acuity, Aside from the
Snellen chart, a commonly used chart is the ETDRS, which is commonly used
in clinical research because the geometrical progression of letters is based on
logMAR scaling (3).
M easuring acuity in children
Acuity in children can be measured using the following tests discussed be­
low (11). We recommend (11) (Chapter 7) for an excellent discussion.
Birthday Cake slide: Rather than letters, this chart uses familiar shapes,
such as a birthday cake. It is appropriate for children who do not know
the alphabet.
Copyright 2014 by KMK Educational Services, LLC
268 6.3. AMETROPIA: CLINICAL CONSIDERATIONS
Tumbling E Chart: Appropriate for ages 3-6, this chart shows the letter E
in different orientations.
Landolt C Chart: Similar to the tumbling E chart, this chart shows the letter
C in different orientations.
Allen vision test: Appropriate for ages 2 and older, this test involves a child
naming a series of images on cards while viewing them at close range.
Then, the examiner occludes one eye of the patient and determines the
longest distance at which the child can resolve the images. The acuity
is then expressed as x/30, where x is the distance (in feet) at which the
child can read.

Exam ple 6 .7: A c h ild resolves im ag es a t th e follow in g distances w hen e x a m in e d


w ith th e A lle n vision t e s t W h a t a re th e a cuities fo r each eye?

O D : 1 2 fe e t

O S : 2 0 fee t

Solution 6.7: T h e a c u itie s w o u ld b e recorded as 1 2 / 3 0 ( O D ) a n d 2 0 / 3 0 ( O S ) . In


te rm s o f S n ellen fra c tio n r th is is 2 0 / 5 0 ( O D ) a n d 2 0 / 3 0 (O S ).

STYCAR: This test involves showing the child the page of a booklet that
contains a single letter. The child holds a so-called key card that contains
all the possible letters on it, and the child indicates which letter on the
key card corresponds to the letter on the booklet. The examiner also
performs a similar test by showing small utensils to the child at 10 or
20 feet and asking the child to indicate which utensil is being held. The
Sherxdan-Gardiner test is similar, but also includes near vision testing
via a reduced Snellen chart (11).
HOTV letter chart: This test is similar to the STYCAR and provides the
child with a key card. The child then views an acuity chart at 10 or 20
feet and indicates letters by pointing to the key card (also called a lap
card).
Lea Symbols: Very similar to HOTV.
Corneal Curvature
Corneal curvature is typically measured using a keratometer or a corneal to­
pographer. The former provides only the curvatures associated with the central
3 mm of the cornea, while a corneal topographer provides an elevation map of
the entire cornea (4).
Copyright 2014 by KMK Educational Services, LLC
CHAPTER 6. PHYSIOLOGICAL OPTICS II 269

Corneal Thickness
Corneal thickness is measured using a pachymeter. Traditional methods in­
volve measuring the distance between the Purkinje images formed by the an­
terior and posterior cornea, while more modern methods use ultrasound tech­
niques which directly measure the time it takes sound waves to travel through
the cornea.

Pachymeter use has increased substantially with the popularity of


corneal refractive surgery, and it also plays an increasingly impor­
tant role in glaucoma risk assessment.

Objective refraction is performed without the subjective input from the patient.
This can be performed using an autorefractor or a retinoscope. Often, objective
refraction is used to obtain a baseline refractive estimate which is then refined
using subjective techniques.
Static Retinoscopy
The retinoscope allows the examiner to determine a patient’s spherocylindrical
refractive error (4, Ch IV). The static procedure is performed while the patient
fixates through the phoropter on a distant object.
Optical Principles: The examiner shines a streak of light through the pupil,
and the streak is reflected from a patient’s retina, refracted as it passes
through the cornea and lens, and focused to a point (the far point of
the eye). This image is known as the retinal reflex. The examiner
observes both the retinal reflex as well as the light reflected from the
exterior of the patient’s eye (the streak). Depending on the refractive
state of the patient’s eye, the images from these two sources-as seen by
the examiner-will move together (with motion) or in opposite directions
(against motion). The goal of retinoscopy is then to neutralize this mo­
tion with a properly chosen lens. In turn, the optical properties of this
lens give indirect verification of the refractive state of the patient’s eye.

With a working distance lens in place (at typical examination dis­


tance) :
Myope => Examiner sees against motion.
Hyperope Examiner sees with motion.

Copyright 2014 by KMK Educational Services, LLC


270 6.3. AMETROPIA: CLINICAL CONSIDERATIONS
Treatment:
R X = F - d w, (6.4)
where R X is the patient’s prescription, F is the power of the neutralizing
lens, and dw is the dioptric distance (typically 1.50 D) between the patient
and the examiner (4, pp.482).

Exam p le 6.8: W h e n e x a m in in g a p a tie n t, yo u need to use a + 3 .0 0 D lens to s to p


a p p a re n t m o tio n b etw e e n th e s tre a k a n d th e reflex. T h e d istance b etw e e n you a n d
th e p a tie n t is 50 cm . F in d th e re fra c tiv e s ta te o f th e p a tie n t's eye a n d g iv e h im
th e p ro p e r p rescriptio n.

Solution 6.8: T h e R X is given b y

R X = + 3 .0 0 - 2.00 = + 1 .0 0 D (6.5)

Astigmatism: When the streak is not aligned with one of the eye’s principle
meridians, the streak and the reflex will not be parallel. This can be used
to find the principle meridians of an astigmatic eye. Once the principle
axes are found, they can be neutralized independently with two sphere
lenses to obtain the power cross. Alternatively, one can use a sphere lens
to neutralize the most positive (with motion) axis, and then neutralize
the perpendicular axis using a minus cylinder lens.

Exam ple 6.9: W h e n u sin g retino scop y on a p a tie n t w ith a s tig m a tism , y o u can
n e u tra liz e m o tio n o f th e v e rtic al streak b y using a + 3 . 0 0 D lens. You n e u tra liz e
m o tio n o f th e h o riz o n ta l s tre a k b y using a + 1 . 0 0 D lens. T h e w o rk in g d is ta n c e is
4 0 cm . W h a t p rescrip tio n is need ed fo r this p a tie n t?

Solution 6.9: T h e v e rtic a l s tre a k gives in fo rm atio n a b o u t th e h o riz o n ta l m e rid ia n


o f th e eye. T h e p o w e r need ed in th e h o rizo n ta l m eridian w o u ld be

F = + 3 .0 0 - ^ = + 0 .5 0 D . (6.6)

Sim ilarly, th e p o w e r need ed in th e v ertical m erid ian is given by

F = + 1 .0 0 - - i - = —1.50 D . (6.7)

T h e p rescrip tio n w o u ld th e n be + 0 ,5 0 - 2.00 x 180.

Copyright 2014 by KMK Educational Services, LLC


CHAPTER 6. PHYSIOLOGICAL OPTICS II 271

Dynamic Retinoscopy
Dynamic retinoscopy, for example MEM retinoscopy, is used to measure the
refractive state of the accommodated eye. Typically the patient is asked to
focus on a near object while retinoscopy is performed. If plus lenses are needed
to neutralize motion, the power of those lenses-above and beyond that needed
for static retinoscopy-represents the lag of accommodation. Several types
of dynamic retinoscopy are listed below.

Example 6.10: A patient's static retinoscopy findings are identical to his dynamic retinoscopy
findings. That is, the same power of lens is used to neutralize motion in both cases. What
is the patients lag of accommodation?

Solution 6.10: If the static and dynamic findings are identical, the patient has been able
to fully accommodate to see the near object So the lag is 0 (11).

MEM: In MEM, the fixation target is located at a typical reading distance.


See Binocular Vision for more details on uses of MEM.
N ott’s Method: Nott’s method relies on moving the retinoscope backwards
(away from the patient) instead of using lenses. The distance at which
neutrality is achieved, along with knowledge of the vergence of light strik­
ing the spectacle plane, can be used to determine the lag of accommoda.-
tion (11).

Example 6.11: Retinoscopy (Nott's method) is performed using a near point card
40 cm from the spectacle plane. When the retinoscope is moved to a distance of
60 cm, neutrality is achieved. What is the lag of accommodation?

Solution 6.11: The near point card is 40 cm or 2.50 D from the spectacle plane.
Neutrality is achieved at 60 cm, which is 1.67 D. So the lag in accommodation is
the difference in these numbers: 2.50-1.67 — 0.83 D.*•

Mohindra: Also called “near retinoscopy”, this technique can be used to de­
termine the refractive state of children and infants. Some differences be­
tween this technique and other dynamic retinoscopy techniques are given
below (11):
• Performed in the dark at 50 cm,
• Unexamined eye is occluded (monocular technique).
Copyright 2014 by KMK Educational Services, LLC
272 6.3. AMETROPIA: CLINICAL CONSIDERA TIONS
• An “adjustment factor” (1.25 D) must be subtracted from the sphere
component of the lens powers (this is basically the lag of accommo­
dation) (11).
• Recent studies (Maino et al.) suggest it can not identify children
with more than 3.00 D of hyperopia or more than 1.00 D of astig­
matism (11).
• Not as effective as cycloplegic retinoscopy.
Autorefractor
Most autorefraction techniques are based on the following principle. A lens
within the machine is placed at a distance equal to its focal length from the
spectacle plane of the patient. A target sits on the other side of the lens and
can be moved back and forth, thereby continuously changing the vergence of
light in the spectacle plane. This is effectively the same as keeping the target
stationary and using a trial lens whose power can change continuously (4); as
a result, one can measure the refractive error with higher precision.

Subjective RcfrAction T L-y;/ .; v. *

Subjective refraction involves a patient’s perception of a stimulus. It is typically


used following objective refraction to refine the refractive error measurement.
Several common techniques are reviewed below (3; 4). These methods typically
make use of fogging, which refers to the practice of adding sufficiently strong
plus sphere that accommodation is inactivated (any astigmatism becomes com­
pound myopic astigmatism).

A patient’s Just Noticeable Difference (JND) is approximately


equal to the denominator of the Snellen fraction divided by 100,

Exam ple 6 .1 2 : I f a p a tie n t's Sn ellen a c u ity is 2 0 /1 5 0 , w h a t is th e J N D ?

Solution 6 .1 2 : T h e J N D is a p p ro x im a te ly equal to 1 5 0 /1 0 0 = 1 .5 0 D , w hich represents


th e ran g e t h a t sh o u ld b e covered by th e tw o lens o ptions shown to th e p a tie n t. In th is
case, one should choose -T 0 .7 5 D a n d - 0 .7 5 D lenses.

Copyright 2014 by KMK Educational Services, LLC


CHAPTER 6. PHYSIOLOGICAL OPTICS II 273

Stenopaic Slit
The stenopaic slit is a device rarely used in common optometric practice, but
it offers a tool for conceptual understanding. It consists of an opaque disk
with a narrow 1 mm rectangular slit. The slit can be placed before the eye
and used to decrease the entrance pupil diameter of the eye in the meridian
perpendicular to the slit (3). The slit can be rotated until a “best” position is
found, meaning that the stimulus appears most clear; in this position, the slit
will be perpendicular to the major axis. Subjective refraction techniques with
spherical lenses can be used with the stenopaic slit to glean information about
the astigmatism.

Exam ple 6 .13 : C o nsider a p a tie n t w h o sees m o s t c learly w ith th e s lit a lo n g th e 1 8 0


degree m eridian . W ith th e s lit in p la c e a t 9 0 degrees, su b je c tiv e re fra c tio n d eterm ines
t h a t th e b est sphere is a b o u t - 1 .2 5 D . T h e s lit is th e n r o ta te d 9 0 degrees a n d th e best
sphere is fo u n d to be - 0 .2 5 D . W h a t is th e p a tie n t's re fra c tiv e error?

Solution 6 .13 : T h e re fra c tiv e e rro r is a p p ro x im a te ly - 0 .2 5 - 1 .0 0 x 1 8 0 ( 3 ) .

Jackson Cross Cylinder Technique


This is the most common method for measuring astigmatic correction. The
cross cylinder (JCC) is a lens whose principle meridians have equal power but
are opposite in sign. Colored dots mark the positions of the axes: red is the
positive meridian (or negative axis), white is the negative meridian (or positive
axis). The steps involved are (4):

1 Refine the sphere power so that the circle of least confusion falls on the
retina. This corresponds to fogging and then decreasing plus power until
the best acuity is achieved.
2 Add the cross cylinder and refine the axis. First, place the cross cylin­
der with the dots straddling the prescription axis, as determined by
retinoscopy. After a few seconds, flip the lens so that the dot positions are
interchanged. Determine which case provides the patient with a clearer
image. If the prescription lens axis corresponds to their true axis, both
sides of the flip will appear the same. If not, find the preferred flip choice
and chase the red, meaning rotate the cylinder of the prescription in
the direction of the red dot. That is, rotate the minus cylinder axis
of the prescription toward the minus axis of the Jackson cross cylinder.
Continue until there is no flip preference.
Copyright 2014 by KMK Educational Services, LLC
274 6.3. AMETROPIA: CLINICAL CONSIDERATIONS
3 Refine the power. First move the JCC so that it aligns with the principle
axes of the prescription. Interchange the position of the dots and deter­
mine which orientation provides the patient with the clearest image. If
the red dot (negative JCC axis) aligns with the minus cylinder axis of the
prescription in the preferred orientation, add -0.25D cylinder. Otherwise,
remove -0.25D cylinder (effectively add plus power). Continue until there
is no preference between the two orientations.
4 With each ±0.5D change of cylinder power, one must change the sphere
power (±0.25D) to maintain a constant spherical equivalent.

A ±0.25 D JCC is typically used for patients with acuity of 20/30


or better, while higher powers (± 0.50 D or ± 1.00 D) are used for
patient’s with poorer vision (4).

Astigmatic Dial
An astigmatic dial is a spoke-like pattern used to diagnose astigmatic refractive
errors. In an astigmatic eye, the images formed by the lines are located different
distances from the retina, depending on the powers of the principle meridians
of the eye. The steps involved with using the dial technique are (4):
1 Fog the eye to an acuity of approximately 20/50.
2 Find the lines of the dial that appear sharpest and darkest to the patient.
3 Add minus cylinder (with power in the direction of the darkest lines)
until all lines appear equal.
4 Add minus sphere until the best acuity is obtained.

The rule of 30 is useful with the astigmatic dial. It says that


the axis of the correcting prescription can be found by asking the
patient which dial appears most clear. If the answer is given in
terms of lines on a clock, the axis (in minus cyl form) is simply the
lowest number given times 30.

Exam ple 6 .14 : A fo g g ed p a tie n t sees th e 3 o ’clock to 9 o ’clock line m o s t d e a r ly o n th e


clock dial. W h a t is th e axis o f th e n eed ed p rescriptio n?

Copyright 2014 by KMK Educational Services, LLC


CHAPTER 6. PHYSIOLOGICAL OPTICS II 275

Solution 6 .14 : U sing th e rule of 30, th e axis w o u ld b e 3 x 30, o r 9 0 degrees. D o es this


m a k e sense? L e t's check. F or a fogged p a tie n t, th e lin e t h a t a p p ears m o s t c lear is th e one
co rresp on din g to th e eye's m erid ian w ith th e le a st p lu s p ow er. So this w ill correspond to
th e p rescriptio n m erid ian w ith th e m o s t p lu s p ow er. So in this exam p le, th e p rescription
m e rid ia n w ith th e m o s t plus p o w e r is th e o ne asso ciated w ith th e h o riz o n ta l ( 3 to 9
o 'c lo c k ) line. T his is th e v e rtic a l m erid ian , because th e h o riz o n ta l lin e corresponds to
p o w e r in th e v e rtic a l m erid ian . So th e v e rtic a l m e rid ia n o f th e p rescrip tio n m u s t have th e
m o s t plus pow er. B u t in m in u s cyl form , this is j u s t th e axis!

Refining the Spherical Component


Following subjective determination of the astigmatic portion of the prescription
(using JCC or Astigmatic Dial), the examiner must refine the sphere portion
of the prescription. It is important to not over minus the patient; that is, the
goal is to provide the most plus power (least minus power) that yields the best
acuity. Accomplish this by:
1 Following JCC, add +0.25 D increments until acuity is compromised.
2 Now, subtract 0.25 D increments from each eye, checking acuity after
each step until maximum acuity is reached.
3 If an astigmatic dial is used for the astigmatic portion, the patient should
be sufficiently fogged that only step 2 (stepping down) is necessary.
4 Verify spherical endpoint using the duochrome (or bichrome) test.

If too much minus power is prescribed, the patient can accommo­


date to offset the excess minus sphere. This combination of a minus
lens and a plus lens is optically similar to a (backwards) Galilean
telescope, and hence the patient will see images that appeal*1234in­
creasingly smaller as minus is added.

The duochrome test utilizes chromatic aberration to verify the final sphere
component in a refraction. An acuity chart is shown on a split background
that is half red and half green. Chromatic aberration results in the green
stimulus focusing before the red (the difference is about 0.50 D for the typical
filters used), and the goal is to ensure that the retina falls about half way
between the two foci (0.25 D from each). That is, the green should fall slightly
in front of the retina, the red slightly behind. In this case, the red and green
images will appear equally clear.

Copyright 20J.4 by KMK Educational Services, LLC


276 6.3. AMETROPIA: CLINICAL CONSIDERATIONS

A fogged patient will perceive the red image to be clearer, while


a patient that is overminused will perceive the green image to be
clearer.

For the mnemonically-inclined: some prefer to remember RAM-GAP, or red


add minus} green add plus (4).

Chromatic aberration underlies the duochrome test. It is inde­


pendent of color perception and therefore works with color vision
impaired individuals. However, it cannot be used until acuity has
been corrected to at least 20/30 or better (4).

Equalization Techniques
Pre-presbyopic patients may accommodate different amounts in each eye when
the final sphere check is done. Therefore, following subjective refraction, one
must ensure accommodation is balanced. The following techniques can be
used (4):
Fogging: Fog both eyes to about +2.00 D, which should reduce acuity to
around 20/200-20/100. Place a -0.25 D lens in front of one eye at a
time, switching eyes in rapid succession. The patient should report a
clearer image in the eye with the -0.25 D lens cover, and acuity in this
eye should improve similarly for each eye. Note that patients with acu­
ity approaching 20/70 or better with fogging lens in place are probably
overminused (4).
Prism Techniques: Use prism (called Risley prism and located in the phoropter)
before one eye so that the images seen by each eye are dissociated. Fog
both eyes and look for differences in the images seen by each eye. When
the eyes are balanced, the patient will report equal clarity between the
two images.
Prism Dissociated Bi-ocular Balance: The previous two techniques work
best when the patient has equal best corrected acuity in the two eyes. If
the patient has different acuities, the dissociated bi-ocular balance can
be used. In this case, the patient does not compare the two eyes at any
point during the testing.
Copyright 2014 by KMK Educational Services, LLC
CHAPTER 6. PHYSIOLOGICAL OPTICS II 277

Cycloplegic Techniques
Cycloplegic agents can be used with subjective refraction to ensure that ac­
commodation is fully relaxed. This is sometimes useful for refractions where
complete relaxation of accommodation is difficult to achieve by fogging, for
example, in young children (4).
W hen to use cycloplegic refraction The following are general guidelines
suggested by Grosvenor. The following list of patients might require cycloplegic
refraction (11):
* Child with convergent strabismus (must determine if accommodation is
contributing).
* Child with significant esophoria at near (40 cm).
* Young adult complaining of headaches but with no uncorrected hyperopia
(in this case, there could be latent hyperopia).

We have so far neglected accommodation in our treatment of the eye. We can


modify our model to account for accommodation by adding power to the thin
lens; we can thereby introduce some fundamental concepts related to accom­
modation (1, ch. 6) (8, ch. 5).
Remember, an object located closer to the eye than the far point results in rays
striking the eye that are too divergent. To correct this, the lens gains power,
and this allows the person to focus on the near object. To say this another way,
accommodation must offset the change in vergence which results from moving
an object closer to the eye.

A person uses only about 50% of his/her accommodation ability


comfortably.

Ocular accommodative demand: The ocular accommodative demand is the


amount of accommodation needed to see an object clearly.
Ocular accommodation: Ocular accommodation refers to the amount of ac­
commodation that the eye actually provides. This does not always match
the demand.
Amplitude of accommodation (AoA): The amplitude of accommodation
is the maximum amount of accommodation possible for a given eye.
Copyright 2014 by KMK Educational Services, LLC
278 6.3. AMETROPIA: CLINICAL CONSIDERATIONS
Near point: The near point is the point conjugate to the retina when the eye
is achieving its maximum accommodation. This is exactly like the far
point, except now the eye is fully accommodated
Range of clear vision: The range of clear vision refers to all other points
analogous to the near and far points, but at varying degrees of accom­
modation. It covers all points between the near point and the far point.
This makes sense - if a point is conjugate to the retina with some level
of accommodation, the person can see it when he/she accommodates ap­
propriately.

Exam ple 6 .1 5 : C ra ig req uires e x a c tly —5.38 D spectacle lenses a t a v e rte x d is ta n c e o f 1 4


m m . H e can m a x im a lly a c c o m m o d a te 2.00 D . T h a t is, his a m p litu d e o f a c c o m m o d a tio n
is 2.00 D . F in d his (u n c o rre c te d ) ran ge o f c lear vision.

Solution 6 .1 5 : W e n eed to fin d his fa r p o in t a n d his near p o in t. S ince he requires


—5.38 D lenses, his fa r p o in t is 1 8 .6 cm in fro n t o f his spectacles. T his is a t o t a l o f 2 0
cm in fro n t o f th e cornea. S o a t th e cornea, his re fra c tiv e error is 5 .0 0 D ( to o m u ch
plus). N o w con sider th e n e a r p o in t. W ith m a x im a l acc o m m o d a tio n , his eye is essen tially
+ 7 . 0 0 D re la tiv e to a n o rm a l e m m e tro p ic eye. So th e n ear p o in t is s im p ly 1 / 7 o r 1 4 .3
cm. H ence his ran ge o f c le a r vision is 14.3 — 20.0 cm.

Exam ple 6 .16 : J o e y req uires a —3.00 D lens a t th e cornea. W h a t is his (u n c o rre c te d )
23 cm from his eye?
a c c o m m o d a tiv e d e m a n d w hen he looks a t an o b je c t th a t is

Solution 6 .16 : F irs t we n eed his fa r p o in t. A g ain , th e fa r p o in t is a t 3 .0 0 d io pters, o r


3 3 cm in fro n t o f th e eye. N o w consider h o w m uch a c c o m m o d a tio n is needed to see
th e o b ject. T h e o b je c t is 23 cm in fro n t o f th e eye, which is 4.35 D in fro n t o f th e
eye. T h e d istan ce b etw een th e o b je c t a n d th e fa r p o in t (in d io p ters) tells us h o w m u c h
a c c o m m o d a tio n we need.
A = 4.34 - 3.00 = 1.3 4 D . (6.8)

In o th e r words , to see th e o b ject, J o e y m u s t a d d 1.34 D o f p o w er to his lens.

Exam ple 6 .1 7 : H o w w o u ld y o u r an sw er to th e previous question change i f J o e y were an


e m m e tro p e ?

Solution 6 .1 7 : N o w , th e fa r p o in t is d io p tric a lly lo c a te d a t 0.00 D . To see an o b je c t


t h a t is 4.34 D away, he m u s t a c c o m m o d a te 4.34 D . T his m akes sense. M y o p e s a re

Copyright 2014 by KMK Educational Services, LLC


CHAPTER 6. PHYSIOLOGICAL OPTICS II 279

"n e a r-s ig h te d ", so th e y should h ave to a c c o m m o d a te less th a n e m m e tro p e s o r hyperopes.

Exam p le 6 .18 : F in d th e (u n c o rre c te d ) a c c o m m o d a tiv e d e m a n d fo r M ary , a 3.00 D hy-


23 cm , as befo re?
p ero p e, to see a d is ta n t o b je c t. W h a t i f th e o b je c t w ere p la c e d a t

Solution 6 .18 : H yp ero p es do N O T h ave enough re fra c tiv e p ow er, even to see a d is ta n t
o b je c t (lo c a te d a t in fin ity ). W ith a c c o m m o d a tio n , s o m etim es h yp ero pes can see very
—3.00 D
d is ta n t objects, a t least u n til h is /h e r c ilia ry m uscle tires. M a r y 's fa r p o in t is a t
or —33.3 cm. W ith ac c o m m o d a tio n , th e fa r p o in t app ro aches n eg a tiv e in fin ity ; or 0.00 D .
T his is ju s t w h a t we need to see an o b je c t a t infinity. T h e o b je c t is 0.00 D away, a n d th e
fa r p o in t is —3.00 D , so
A = 0.00 - ( -3 .0 0 ) = 3.00 D . (6.9)
i f th e o b je c t were a t 23 cm ( 4 .3 4 D ) , she w o u ld n eed

A = 4.34 - ( -3 .0 0 ) = 7.34 D . (6.10)

In o th e r words, she w ou ld a c c o m m o d a te by th e a m o u n t n eed ed to see th e o b je c t a t in fin ity


a n d then a d d itio n a lly to see an o b je c t a t 23 cm .

Presbyopia refers to the loss of accommodation ability with age. In particular,


it is typical to see a decrease in amplitude of accommodation (AoA) due to
hardening of the crystalline lens (8) (12) (3) (4).
Accommodation and Age
Age AoA
10 14.00 1)
20 11.00 D
50 < 2.00 D
Estimate: AoA~ 18.50 D - 0.3 age

Presbyopia: Clinical Considerations


There are a number of clinical techniques for diagnosing and characterizing
presbyopia and accommodative ability. Several of these techniques are dis­
cussed below (see (3), (4), (5)).

Copyright 2014 by KMI< Educational Services, LLC


280 6.3. AMETROPIA: CLINICAL CONSIDERATIONS

As a general rule of thumb, the theoretical add power, a, can be


calculated using:
a = W d — A a /2 (6.11)
where Wd is the working distance (in D) and A a is the AoA.

Exam ple 6 .19 : W h a t a d d w o u ld b e prescribed fo r a p a tie n t whose n ea r p o in t is m easu red


to be 3 3 cm w hen a + 1 . 0 0 D assisting lens is used? A ssum e th a t th e w o rk in g d is ta n c e is
4 0 cm .

Solution 6 .19 : T h e n e a r p o in t is 1 / 3 3 — 3 .0 0 D , b u t we m u s t s u b tra c t o f f th e assisting


lens = —> 2 .0 0 D . T h e w o rk in g d istan ce is 1 / 4 0 — 2 .5 0 D, m e a n in g t h a t th e a d d w o u ld
b e 2 .5 0 - 2 .0 0 / 2 = 1 .5 0 D .

Aphakia means literally without a lens. Sometimes patients with cataracts must
have the lens removed. In a pseudophakic eye, the lens has been replaced
with an implant. In both cases, the patient can no longer accommodate (though
accommodating,.multifocal intraocular lenses are gaining popularity).
Aphakia
Optically speaking, aphakia is similar to having a large minus error. This leads
to significant aniseikonia and diplopia (in the case of monocular correction) or
a reduced visual field and pincushion distortion (binocular correction). Cor­
rection is also associated with a ring scotoma. The ring scotoma results in
visual stimuli appearing and disappearing briefly as they pass in and out of
the scotoma (the “jumping” effect of the image is sometimes called the jack-in-
the-box effect). Finally, corrected aphakic patients face additional convergence
demands at near owing to the large horizontal base out prismatic effects.

To calculate magnification effects in aphakic patients, use the for­


mula from telescopes: M = —F1 /F 2

Copyright 2014 by I<MK Educational Services, LLC


CHAPTERS. PHYSIOLOGICAL OPTICS II 281

Clinical Note: An aphakic patient can wear a contact lens of power P or


can alternatively be treated with spectacles of power F. The magnification
in each case is given by the ratio of the error lens power (not given) and the
prescription (M — —PeriPr x )- If we only want the change in magnification
in going from spectacles to glasses, the answer is simply one ratio divided by
the other, and the unknown error lens power drops out. We therefore have a
magnification of P/F in going from contact lenses to glasses.

Exam ple 6.20: A n a p h a k ic p a tie n t w ith an a x ia l le n g th o f 24 m m has an in te rn a l in d ex


o f re fra c tio n o f 1.3, I f the k e ra to m e try read ing s fo r th is p a tie n t are 4 4 .0 0 a t 9 0 a n d 4 1 .0 0
a t 1 80, w h a t is th e p a tie n t's re fra c tiv e error?

Solution 6.20: F o r an a p h a k ic p a tie n t, th e re is no o c u la r lens, so th e p o w e r com es e n tire ly


fro m th e cornea ( 9 ) . W e w ill fin d th e fa r p o in t o f th e eye in each m e rid ia n . F o r th e 9 0
degree m erid ian , we firs t p lace an o b je c t on th e re tin a a n d fin d th e im age, w hich is (b y
d e fin itio n ) lo c a te d a t th e fa r p o in t. T h e a x ia l le n g th is 0 .0 2 4 m a n d th e in d ex is 1 .3 , so
L is —64.16 D s trik in g th e cornea. W e th e re fo re have

L1= F + L = 44.00 - 54.16 = - 1 0 . 1 6 D (6.12)

S o th e fa r p o in t is lo c a te d a t l 1 = 1 / ( —10 .16 ) = —0.098 m , o r ~ 9 .8 cm fro m th e cornea.


In this m eridian , th e p a tie n t is h yp e ro p ic b y 10 .16 D . U s in g a s im ila r c a lcu la tio n (b u t w ith
F = + 4 1.0 0 , one can show t h a t in th e 1 8 0 deg ree m e rid ia n , th e p a tie n t is h yp ero p ic b y
13 .16 D.

Pseudophakia
After the lens has been removed, an IOL (intraocular lens) can be placed at the
anterior chamber, iris plane, or posterior chamber, the last of which is by far
the most common. The power of the chosen lens is dependent on the position
of the lens, the axial length of the eye, and the power of the cornea.
SECTION 6.4

Astigm atism
Astigmatic eyes cannot form a point retinal image from a point object. This is
typically due to some asymmetry in the lens or the cornea, rendering them no
longer akin to spherical lenses. Such asymmetry results in a Conoid of Sturm
inside the eye. We review some basic ideas relating to visual astigmatism (1,
ch. 15) (8, ch. 2).
Copyright 2014 by KMK Educational Services, LLC
282 6.1 ASTIGMATISM

■C] assitying A:sti g ii)at ism

Classifying Astigmatism by Power


Recall that the image of a point source through an astigmatic system results
in two line foci. We classify astigmatism of the eye using these lines.
Simple astigmat: If one line falls on the retina and one line falls in front of
(myopic) or behind (hyperopic) the retina, we have a simple astigmat
(SMA or SHA, for short).
Compound astigmat: If both lines fall in front of (myopic) or behind (hy­
peropic) the retina, we have a compound astigmat (CMA or CHA, for
short).
Mixed astigmat: If one line falls in front of the retina and one line falls
behind the retina, we have a mixed astigmat.
Equally mixed astigmat (EM A): An EMA is a special case of mixed astig­
mat where the circle of least confusion falls on the retina.
Classifying Astigmatism by Orientation
We can also classify astigmatism by orientation. To do so, we first need to
define the eye’s major meridian.
M ajor meridian of the eye: The major meridian is the principal meridian
of the eye with the most plus power. Since the cornea is responsible for
the majority of the eye’s refractive power, the eye’s major meridian will
typically correspond to the steepest corneal meridian.
Now we can define astigmatism based on the location of the eye’s major merid­
ian.
W ith-the-rule (W TR): WTR, corresponds to an astigmatism where the cornea’
major meridian is vertical {$ = 90 ± 30°). This is the most common type
of astigmatism, hence the name.
Against-the-rule (ATR): ATR corresponds to an astigmatism where the
cornea’s major meridian is horizontal (6 = 0 ± 30°). This is the second
most common type of astigmatism.
Oblique: An oblique astigmatism corresponds to an eye whose major meridian
is in the remaining zone not covered by WTR and ATR,.

Copyright 2014 by KMK Educational Services, LLC


CHAPTER 6. PHYSIOLOGICAL OPTICS II 283

In terms of a lens prescription in minus cyl form, WTR astigmatism


corresponds to an axis that is approximately horizontal, and ATR
corresponds to an axis that is approximately vertical.

Exam p le 6 .2 1: A p a tie n t is corrected b y th e fo llo w in g p rescriptio n; + 2 . 0 0 + 1 .0 0 x 180.


W h a t ty p e o f a s tig m a tism does she h a v e ?

Solution 6 .2 1 : In m inus cyl form , this is + 3 . 0 0 - 1 .0 0 x 90. B ecause th e axis is vertical,


th is is A T R astig m atism .

SECTION 6.5

Optical Effects in Vision


We summarize some common optical effects related to vision.

Light reflects off the front and back surfaces of the lens and cornea and forms
unintended “ghost” images which we call Purldnje images. The optics are com­
plicated, but some important properties are listed below (8):

Purkinje Images:
Image Source Properties Brightness
I Anterior cornea Virtual, upright, Very bright
small
II Posterior cornea Virtual, upright, Bright
very small
III Anterior lens Virtual, upright, Dim
large
IV Posterior lens Real, inverted, Very dim
smallest

Copyright 2014 by KMK Educational Services, LLC


284 6.5. OPTICAL EFFECTS IN VISION

Images I-III have similar characteristics, largely because they are all
created from convex surfaces. The curvature of the surface leading
to image IV is concave.

Im portant Characteristics
• Image I is formed only by reflection.
• Images II-IV are formed by combination of reflection and refraction.
• Image II, I, and IV are located in this order just to the retina side of the
anterior lens. Image III is located behind the posterior lens.
• Following accommodation, the size ordering becomes I > II> III> IV.
Image III moves forward and IV moves backward (toward the retina).

Haidinger’s brush is an optical brush-like effect due to the birefringence of the


macular retina (1, pp. 515-17). When a patient stares at a blue object through a
rotating polaroid, a rotating yellow brush (like a propeller) will be seen against
the background.

Birefringence
Birefringence is a polarization-related phenomenon that is due to the spatial
anisotropy of a material. The macular region has radial nerve fibers (called
the Layer of Henle). These fibers are birefringent and lead to the optical effect
known as Haidinger’s Brush.

Clinical Uses
We can use a blue filter and a linear polarizer over a bulb, and this serves to keep
only blue light that is linearly polarized. When a patient looks at the apparatus,
the birefringence results in blue or yellow brushlike shapes radiating from the
fixation point. Quickly, this image becomes invisible (because it is stable on
the retina). If we then change the polarization angle, the image reappears
momentarily. Furthermore, if we rotate the Polaroid, the image rotates. This
can be used to diagnose eccentric (non-macular) fixation. An eccentric
fixator will see the brush centered on a point other than the fixation target (1,
pp. 515-17).
Copyright 2014 by KMK Educational Services, LLC
CHAPTER 6. PHYSIOLOGICAL OPTICS II 285

Suppose we shine purple light on a patient’s retina. The patient will see a red
spot (typically at the point of fixation). This is because blue is absorbed by the
macular pigments (not the photoreceptors). This effect can also be exploited
to diagnose eccentric fixation (12, pp, 520).

Moore’s lightning streaks are vertical flashes of light sometimes seen in the
peripheral visual field. These indicate a potential problem in the peripheral
retina (12).

References
[1] M. Keating (1988). Geom etric, Physical, and Visual Optics, Butter wort li-Hei n eman n.
[2] G. Brooks and I. Borish (1996). System fo r O phthalm ic D ispensing, 2nd Edition.
Butterworth-Heinemann.
[3] Ft. Rabbetts (2007). B ennett and R abbetts’ Clinical Visual Optics, 4th Edition.
Butter wort h-Heinemann,
[4] Basic and Clincial Science Course, Section 3: O ptics, R efraction, and C ontact Lenses.
Foundation of the American Academy of Ophthalmology, 2002.
[5] D. Miller (1991). Optics and Refraction. Gower Medical Publishing.
[6] Casser, L,, Chang, F., Gerstman, D., Pietsch, P.f and Bradley, A (1994). Optometry E xam ­
ination Review, 4th Edition. Appleton and Lange.
[7] D. Goss (1995). Ocular Accom m odation, Convergence, and F ixation D isparity, 2nd E di­
tion. Butterworth-Heinemann.
[8] T. Fannin and T. Grosvenor (1996), Clinical O ptics, 2nd Edition. Butterworth-Heinemann.
[9] Lakshin in arayan an, V. and E. Bennett (2006). Review Q uestions fo r the N B E O E xam ina­
tion, P art One, Butterworth Heinemann,
[10] Bennett, E. and V. Lakshminarayanan (2006). R eview Q uestions fo r the N B E O E xam ina­
tion, P art II. Butterworth Heinemann.
[11] T. Grosvenor (2002), Primary Care O ptom etry, 4th Edition. Butterworth-Heinemann.
[12] The B erkeley Guide, 8th E dition (2002), University of California, Berkeley School of Op­
tometry, UC Optometric Student Association.

Copyright 2014 by KMK Educational Services, LLC


c{ '
(c--,
(( :
((
(
( - i

((
((
((
((
((
((
((
((
((
((
((
((
((
((
((
(c
((
(_L /

(C /

L
(_,
t
(_,
((
Chapter 7
Ocular Motility

by Kyle M. Cheatham, O.D., F.A.A.O.,


Sarah Dougherty Wood, O.D., F.A.A.O.

287
(
c
(
(
(
(
c
(
c
(
(
(
(
(
(
(
(
(
(
(
(
(
(
Copyright 2014 by KMK Educational Services, LLC (
(
c
(
CHAPTER 7. OCULAR MOTILITY 289

SECTION 7.1

Ocular R otations
In what follows, we will summarize the mechanics behind eye rotations and
the role of extraocular muscles (14, ch. 13), (13). In order to understand
eye rotations, we must first be aware of certain definitions pertaining to eye
movements:
• Listing’s Plane: The plane normal to the visual axis and passing through
the equator of the globe and the center of rotation of the eye when it is
in primary position of gaze. Listing’s plane is described by a horizon­
tal (x) and a vertical (z) axis. Rotations about these axes are termed
secondary positions of gaze. Tertiary positions of gaze involve looking
in an oblique direction. The y axis is the axis through the line of sight,
normal to Listing’s plane (18).
• Muscle plane: The plane that describes the direction of pull of an
individual extraocular muscle. The plane passes through the center of
rotation of the eye and is determined by the origin and insertion sites of
the extraocular muscle (18).
• Axis of rotation: The axis perpendicular to the muscle plane around
which the eye rotates when acted on by an extraocular muscle (18).
• Tangential point: The point where the muscle tendon first makes con­
tact with the globe of the eye (18).
• Arc of contact: The area between the tangential point and the point
of insertion of the muscle on the globe of the eye; this is the area where
the muscle exerts its action on the eye (18).

A duction is a monocular rotation.


Abduction: A rotation about the z axis, away from the midline.
Adduction: A rotation about the z axis, toward the midline.
Elevation: An upward rotation about the x axis.
Depression: A downward rotation about the x axis.
Incyclo duct ion: A rotation about the y axis such that the upper portion of
the eye tilts inward (toward the nose).
Excycloduction: A rotation about the y axis such that the upper portion of
the eye tilts outward (away from the nose).
Copyright 2014 by KMK Educational Services, LLC
290 7.1. OCULAR ROTATIONS

Versions are binocular eye movements that result in the visual axes of both
eyes moving in the same direction (conjugate movements). Their purpose is
to enlarge the field of view and to move the fovea of each eye to an object for
fixation. They may be voluntary or involuntary (18).
Dextroversion: Both eyes rotating about the z axis to the right.
Levoversion: Both eyes rotating about the z axis to the left.
Dextrocycloversion: Rotations about the y axis such that the upper portion
of both eyes tilts to the patient’s right.
Levocy clovers ion: Rotations about the y axis such that the upper portion
of both eyes tilts to the patient’s left.

B iiilS lIlli
Vergences align the visual axes of both eyes to obtain binocular fixation and
fusion. They are disconjugate eye movements, meaning that the visual axes of
the two eyes move iu opposite directions (18).
Convergence: Both eyes rotate about the z axis toward the midline.
Divergence: Both eyes rotate about the z axis away from the midline.
Incyclovergence: Both eyes rotate about the y axis such that the upper por­
tion of each eye tilts toward the midline.
Excyclovergence: Both eyes rotating about the y axis such that the upper
portion of each eye tilts away from the midline.

There are 6 EOMs and 6 duction movements; each EOM has one of these
movements as its primary action (20).
Lateral Rectus: The lateral rectus inserts on the lateral side of the eye and
is responsible for abduction.
Medial Rectus: The medial rectus inserts on the medial side of the eye and
is responsible for adduction.
Superior Rectus: The superior rectus inserts on the top of the eye anterior
to the equator and 23 degrees temporal to the line of sight (with the eye
in primary position of gaze). Its primary action is elevation, secondary
action is incyclotorsion, and tertiary action is adduction.
Copyright 2014 by KMI< Educational Services, LLC
CHAPTER 7. OCULAR MOTILITY 291

Inferior Rectus: The inferior rectus inserts on the bottom of the eye anterior
to the equator and 23 degrees temporal to the line of sight (with the eye
in primary position of gaze). Its primary action is depression, secondary
action is excyclotorsion, and tertiary action is adduction.
Superior Oblique; The superior oblique passes through the trochlea and
travels diagonally to insert on the upper temporal region of the eye, pos­
terior to the equator and at an angle of 54 degrees medial to the line
of sight (with the eye in primary position of gaze). Its primary action
is incyclotorsion, secondary action is depression, and tertiary action is
abduction.
Inferior Oblique: The inferior oblique inserts on the lower temporal region of
the eye, posterior to the equator at an angle of 51 degrees medial to the
line of sight (with the eye in primary position of gaze). Its primary action
is excyclotorsion, secondary action is elevation, and tertiary action is
abduction.
Knowing where the muscles attach to the globe of the eye aids in the under­
standing of how they function. As an example, let’s look at how the superior
oblique inserts into the eye, resulting in its actions of intorsion, depression, and
abduction. Remember, the superior oblique will leave the trochlea, traveling
diagonally to insert posterior to the equator on the lateral side of the eye. As
the muscle contracts, it is easy to visualize that it will primarily rotate the
eye around the y axis towards the nose, while also pulling it downward and
turning it away from the midline (it will rotate the posterior-lateral side of the
eye medially, resulting in abduction).

The Spiral of Tillaux describes the line of insertion of the recti


muscles on the globe of the eye. The medial rectus (MR) inserts
closest to the limbus, while the superior rectus (SR) inserts furthest
away from the limbus. Because the MR has the most anterior
insertion site, it has the strongest effect on the globe of the eye
when it contracts (as compared to the other EOMs) (18).

- SECTION 7.2 -----------------------------------------------------------------------------


Laws of Eye M ovement Dynamics
Many prominent features of ocular motility are illustrated by the following
laws.
Copyright 2014 by KMK Educational Services, LLC
292 7. 3. EYE MOVEMENT DYNAMICS

Bonder’s law says for any position of gaze, the eye has a unique orientation in
3 dimensions in space. The orientation of the eye for a particular gaze is always
the same, regardless of where the eye was initially positioned before moving to
that gaze (8). In other words, the starting location of the eye and the path
taken to a unique position of gaze does not influence the orientation of the eye
at the final position of gaze.

vBaw
Listing found that the eye must rotate around axes to achieve a given direction
of gaze. These axes are located in a single plane called Listing’s Plane (see
chapter introduction).

In order for the eyes to produce smooth, controlled eye movements and maintain
single vision, synergistic muscles (the yoke muscles) of the two eyes must
receive equal innervation. The yoke muscles (one from each eye) are the LR
and MR, SR and IO, and SO and IR.

Sherrington’s law says that agonist and antagonist EOMs of the same eye are
reciprocally innervated, meaning that when the agonist is excited, the antago­
nist is inhibited, and vice versa (14). Paired muscles (of the same eye) are the
SR and IR, IO and SO, and LR and MR.
- SECTION 7.3 -----------------------------------------------------------------------------------------------------

Eye Movement Dynamics *•

Eye movements serve two primary purposes (20) (18):


1 Hold images in place on the retina
• The fixation system holds an image on the fovea while the head is
motionless.
• The vestibular system (i.e. VOR) holds an image on the fovea during
short motions of the head.
Copyright 2014 by KMK Educational Services, LLC
CHAPTER 7. OCULAR MOTILITY 293

• The optokinetic system holds an image on the fovea during pro­


longed head movements.
2 Move the eye so that the fovea aligns with an object of interest
• The saccade system moves the eye very rapidly to quickly align the
fovea with an object.
• The smooth pursuit system moves the eye at the same speed as a
moving target in order to keep the target on the fovea.
• The vergence system aligns the eyes through convergence and di­
vergence eye movements to ensure bifoveal fixation of an object in
order to maintain fusion and binocular vision.

The eye is constantly moving during fixation. These involuntary eye move­
ments continuously shift an image onto neighboring photoreceptors, preventing
bleaching of the retina, fatigue, and subsequent fading or smearing of an image.

The Troxler effect describes the fading of peripheral images when


the eye is fixated on a central object. The small, involuntary move­
ments that occur during fixation help to minimize this effect (10).

There are three types of eye movements associated with fixation:


Microsaccades: These are intentional conjugate eye movements with a mod­
erately high velocity (2-10 degrees/sec) and amplitude (6 arc minutes) (7).
Microsaccades move the fovea back on an object of interest after mi­
crodrifts and microtremors have caused the fovea to shift from the tar­
get (18).
Microtremors: These are unintentional, disconjugate eye movements with a
high frequency (65-75 Hz) and amplitudes of about 10 arc minutes (7).
Microtremors are the fastest of the three types of eye movements associ­
ated with fixation. Because these movements are disconjugate, they are
thought to be a result of neural noise within the brainstem.
Microdrifts: Microdrifts are disconjugate, unintentional eye movements that
are larger and slower than microtremors, with velocities of only 1 ar-
cminute per second and amplitudes of 6 arc/min (7). Similar to mi­
crotremors, they are likely secondary to neural noise in the brainstem (18).

Copyright 2014 by KMI< Educational Services, LLC


294 7.S. EYE MOVEMENT DYNAMICS

Microsaccades counteract errors in monocular fixation produced


by spurious microdrifts and microtremors (7) (18).

The vestibulo-ocular reflex (VOR) stabilizes images on the fovea during brief
head movements by producing an eye movement of equal magnitude to the
head movement, but in the opposite direction. The VOR occurs very rapidly
(300 degrees/sec) and with a very small latency (15 msec) (8).
The VOR is stimulated by movement of endolymph within the semicircular
canals of the ear that occurs during head movement. The VOR does not
require a visual stimulus - it will occur in response to head movement even if
the eyes are closed.
Although the VOR compensates well for fast eye movements, the reflex begins
to fade with sustained head movements over 30 seconds in duration; the optoki­
netic system is then responsible for continuing the eye movements in response
to continuous head movements (1).
Abnormalities of the vestibulo-ocular system
Damage to the vestibular system, whether central (vestibular nuclei in the brain
or vestibular connections to the brainstem/cerebellum) or peripheral (damage
to the labyrinth or vestibular nerve of the inner ear), can result in an imbalance
of output and a resulting horizontal nystagmus (15).
• Acute vestibular lesions typically cause nystagmus, while slow growing
lesions do not (12).
Central lesions are commonly accompanied by other neurological symptoms.
Damage to the inner ear (from degeneration/trauma/infection/ischemia/drug

Table 7.1: The numeric values in this chart were found in Adler (8).
Eye Movements Latency (msec) Velocity (deg/see)
Saceades 200 1000
Smooth Pursuit 125 50
Vestibular Reflex ' 15 300
Vergence 100 10

Copyright 2014 by KMK Educational Services, LLC


CHAPTER 7. OCULAR MOTILITY 295

toxicity) is often accompanied by symptoms of nausea, oscillopsia, and ver­


tigo (15).

Oscillopsia is characterized by the sensation of objects moving up


and down in the visual field. In contrast, vertigo is the sensation
of the body moving around in the environment, even though it is
still.

If vestibular dysfunction is suspected, oculocephalic testing, caloric testing,


and/or rotational testing is indicated.
• In oculocephalic testing (doll’s head maneuver), the patient is asked
to fixate on a distant target while the head is moved from side to side or
up and down. A normal response is seen as a conjugate eye movement in
the direction opposite to the head rotation (11).
• In caloric testing, the patient is positioned so that the head is elevated
60 degrees. The head is" turned to one side and water is slowly poured
in the inner ear. In a patient with an intact vestibular system, warm
water poured in the right ear will result in a slow conjugate movement to
the left, with a resulting fast movement back towards the right (caloric
nystagmus). Cold water poured in the right ear will result in nystagmus
with the fast phase towards the left (15).

COWS (cold-opposite, warm-same) summarizes the normal re­


sponse of a healthy individual to caloric testing (16).

• In rotational testing, a patient is slowly rotated in a chair for about 20


seconds as the examiner watches the eye movements. A normal response
involves a slow conjugate eye movement in the direction of rotation, fol­
lowed by a fast eye movement in the opposite direction (15).

Nystagmus is an involuntary back and forth movement of one or both eyes that
disrupts fixation and may be horizontal, vertical, or torsional in direction. It
may occur secondary to pathology of the afferent visual pathway, disruption of
ocular motor control, or due to abnormalities of the eye movements necessary
for stable fixation.
Copyright 2014 by KMK Educational Services, LLC
296 7.3. EYE MOVEMENT DYNAMICS
• A jerk nystagmus is characterized by a slow and fast phase. The slow
phase (drift) represents the abnormality in fixation, while the fast phase is
a correcting saccade to bring the fovea back on the target. The nystagmus
is named for the direction of the fast phase (11) (15).
• A pendular nystagmus is characterized by an even back and forth
movement of the eyes (11) (15).

The null point is the direction of gaze where the nystagmus has
the lowest amplitude. The neutral point is the direction of gaze
where the nystagmus changes direction (15).

Nystagmus may be physiologic or pathologic in nature. Physiologic nystagmus


is often a conjugate jerk nystagmus without associated symptoms or decreased
vision. Pathologic nystagmus is characterized by dissociated (disconjugate) eye
movements with excessive drift, causing decreased visual acuity and oscillop-
sia (11). Examples of physiologic nystagmus include:
End-point nystagmus: Small, intermittent, conjugate jerk nystagmus ap­
parent in extreme (>30 degrees'from midline) horizontal positions of gaze
(less commonly vertical). The nystagmus is often worse when the patient
is tired (15).
Optokinetic nystagmus: A conjugate jerk nystagmus responsible for main­
taining the image of a moving object on the fovea when the head is
stationary (see OKN section for further details).
Caloric nystagmus: A conjugate jerk nystagmus produced during caloric
testing of the vestibular system (see caloric testing for further details).
Rotational nystagmus: A conjugate jerk nystagmus produced in response
to head/body rotation (see rotational testing for further details).
Examples of pathologic nystagmus include:
Congenital (infantile) nystagmus: Nystagmus present at birth or before
the age of 6; affects males 2X more than females. The nystagmus is
often horizontal and conjugate and may be a pendular or jerk waveform.
Approximately 40% of cases of congenital nystagmus are secondary to a
defect in the afferent visual pathway, resulting in poor image formation
on the fovea and inadequate feedback to the oculomotor system to control
fixation. The remaining 60% of cases are due to defects in the efferent
motor pathway (18).

Copyright 2014 by KMK Educational Services, LLC


CHAPTER 7. OCULAR MOTILITY 297

Conditions such as aniridia, albinism, achromatopsia, optic nerve


hypoplasia, optic atrophy, and congenital cataracts may lead to
poor image formation on the fovea with subsequent nystagmus (18).

Latent nystagmus is a congenital, conjugate jerk nystagmus that in­


creases in velocity and amplitude when one eye is occluded; it is typi­
cally horizontal in direction, with the fast phase towards the fixating eye.
It is commonly associated with essential infantile esotropia and ambly-
opia (3) (11) (15).
Spasmus nutans: Nystagmus that develops within 4-12 months after birth
and resolves after 2-8 years. Multiple family members are usually af­
fected (no sex predilection). The nystagmus is characterized by a discon-
jugate, high frequency, low amplitude, pendular waveform that often has
a horizontal and vertical component. It is accompanied by compensatory
head nodding and, in approximately 50% of patients, head turns. Spas­
mus nutans is usually not associated with other ocular abnormalities and
rarely has long term consequences (15).
Convergence-retraction syndrome: Characterized by an intermittent jerk
nystagmus with a fast phase that causes convergence or retraction of the
eyes. When the patient looks in upgaze, the eyes will move slowly down
(slow phase), followed by a fast phase causing convergence and/or retrac­
tion (replacing the quick movement towards upgaze). This condition is
often associated with dorsal midbrain syndromes (11).
Gaze-evoked nystagmus: Jerk nystagmus occurring only at extreme gazes
(usually horizontal or upgazes); may be conjugate or disconjugate. Unlike
physiologic end-point nystagmus, gaze-evoked nystagmus occurs in the
presence of other ocular motor abnormalities. It may develop secondary
to drug use (alcohol, sedatives, anti-convulsants) or posterior fossa dis­
ease (11).
See-saw nystagmus: Characterized by elevation and intorsion of one eye
with depression and extorsion of the fellow eye in a pendular or jerk wave­
form. Although traditionally associated with parasellar lesions (e.g. pi­
tuitary tumor), see-saw nystagmus may also be congenital in nature (11).

Remember, pathologic nystagmus is often associated with other


ocular motor abnormalities, while physiologic nystagmus is not.
Always evaluate binocular vision, the OKN response, the VOR
response, saccades, and pursuits to help differentiate pathologic
from physiologic etiologies of nystagmus.

Copyright 2014 by KMK Educational Services, LLC


298 7.3. EYE MOVEMENT DYNAMICS
Optokinetic Nystagmus (OKN)
Optokinetic nystagmus maintains a stable image of a moving object on the
fovea when the head is still. Because it requires input from the visual system,
it has a longer latency than the VOR response (which occurs in response to
head movement and does not require a visual stimulus).

Note the difference between optokinetic nystagmus and the optoki­


netic reflex:
• The optokinetic reflex is responsible for maintaining an
image on the fovea with prolonged head movements (>30
seconds duration) after the VOR response has faded (1).
• Optokinetic nystagmus maintains the image of a moving
object on the fovea when the head is still.

The cortical regions of the brain responsible for the OKN response are not fully
developed at birth. As a result, the nasal to temporal optokinetic nystagmus
is absent in infants until 3-4 months of age (8).
Clinical Note: An OKN drum contains alternating black and white stripes.
When the drum is rotated slowly and watched by the patient, optokinetic nys­
tagmus will occur with the slow phase in the direction of rotation of the drum.

The OKN drum can be used as a gross method of measuring visual


acuity in infants. A positive OKN response denotes a visual acuity
equal to or better than the corresponding VA size of the drum
stripes. Although a negative response is inconclusive, decreased
VA and/or a parietal lobe lesion should be considered (1).

Saccades
Saccades are very rapid, yoked eye movements that move the fovea to an ob­
ject of interest in the visual field. They occur with velocities reaching 1000
degrees/second (dependent on the magnitude of the saccade), and a latency
of approximately 200 msec (this time can decrease with learning) (8) (12).
Copyright 2014 by KMK Educational Services, LLC
CHAPTER 7. OCULAR MOTILITY 299

Although most saccades are voluntary, sudden visual, auditory, or peripheral


stimuli may elicit involuntary saccades (1). Micros accades, characterized by a
smaller amplitude and velocity, are used for reading and occur approximately
5X per minute.

Once a saccadic movement has started, the saccade system can­


not change the amplitude of the planned saccade in response to a
change in object position, resulting in errors in fixation. The most
common saccadic error is undershooting (15).

Visual information regarding target position is sent from the eyes to the supe­
rior colliculus, which then sends information to the cortex. Most saccades are
initiated by the contralateral frontal eye fields (the right contralateral eye
field initiates a saccade towards the left); the occipitoparietal junction also
helps to initiate saccades in response to visual stimuli (15).
• Damage to the right frontal eye field results in impaired saccades towards
the left (the side OPPOSITE the lesion), resulting in the eyes turning to
the right (TOWARDS the side of the lesion). These effects are usually
temporary and disappear within weeks (1) (8) (12).
* Damage to the superior colliculus causes a change in the velocity and
accuracy of saccades; these effects are usually transient as well.

In order to prevent blurring of the image during a saccade, vision


is suppressed by the cortex; this is known as saccadic suppres­
sion (8).

Abnormalities of Saccades
Abnormal saccades often result in symptoms associated with reading, includ­
ing loss of place in the text, skipping lines, excessive head movements, slow
reading speed, poor comprehension, and a short attention span to reading
material. Children with saccadic dysfunction may also have difficulty copy­
ing information from the chalkboard or solving math problems involving large
columns of numbers (15). If saccadic dysfunction is suspected, the following
tests should be considered:
Copyright 2014 by KMK Educational Services, LLC
300 7.3. EYE MOVEMENT DYNAMICS
NSUCO oculomotor test: The examiner directly observes the patient per­
forming 5 rounds of saccades at near, rating the patient on a scale of 1 to
5 in relation to head movement, body movement, ability, and accuracy
of saccades. This is the only standardized, direct observation test with
normative data for comparison (15).
Developmental Eye Movement (DEM) test: The patient is asked to call
off a series of numbers as fast as he/she can without using a finger to
help with tracking. The first series of numbers are arranged in vertical
columns and test the patient’s ability to automatically call off numbers
(random automaticity of naming (RAN)). The second series of numbers
are arranged in horizontal columns and assess RAN and saccades. The
patient’s response time and number of errors for each series are compared
to normative values to determine if saccades are impaired. Similar tests
include the King-Devick and Pierce saccade tests (15)
Readalyzer: An objective test assessing the patient’s comprehension and read­
ing ability (and thus saccades) by tracking eye movements as the patient
reads a short excerpt. A score below the 15th percentile is considered ev­
idence of saccadic dysfunction. Similar tests include the Visagraph (15).
Saccadic dysfunction may involve impairment of the initiation, velocity, and/or
accuracy of saccades or inappropriate saccades. Examples of inappropriate
saccades include the following:
Square-wave jerk: Rare, uncontrollable saccades that occur at random and
interfere with fixation. A corrective saccade is necessary to restore foveal
fixation of the object of interest. Movements are termed macrosquare
waves if the amplitude is larger than 10 degrees.
Ocular flutter: Characterized by multiple, spontaneous, conjugate horizontal
saccades (referred to as “spring-like”) that decrease in amplitude over time
and occur after a series of small saccades or during fixation (15).
Opsoclonus: An advanced form of ocular flutter that appears as an almost
constant series of involuntary conjugate saccades in multiple directions
that occurs only while awake (9) (15).

Ocular flutter and opsoclonus are typically associated with cere­


bellar disease. Myasthenia gravis, Parkinson’s, Alzheimer’s, ocu­
lar motor apraxia, progressive supranuclear palsy, and internuclear
ophthalmoplegia are additional disorders that are characterized by
abnormal saccades (non-exhaustive) (11) (15).

Copyright 2014 by KMK Educational Services, LLC


CHAPTER 7. OCULAR MOTILITY 301

Pursuits
Smooth pursuits are slow, tracking movements that allow continuous fixation of
the fovea on a moving object. Although pursuits are voluntary eye movements,
very few people are able to perform smooth pursuits without a moving stimulus.
Pursuits occur with a latency of approximately 125 msec and a velocity of 50
degrees/second (8) (15).

If an object moves at a velocity greater than 50 degrees/second,


the eyes can no longer maintain a smooth tracking movement. A
voluntary saccade will be necessary to place the fovea on the tar­
get again, resulting in a continuous pursuit-saccade-pursuit process
until the target velocity slows back down to the pursuit threshold
velocity (1).
----------------------------------------------------------------------------------------------

Pursuits are controlled primarily by the parietal lobe, although the frontal
eye fields may also be involved. A parietal lobe lesion will cause impaired
pursuits towards the IPSILATERAL side. Clinically this can be seen using an
OKN drum - the patient will not be able to make smooth pursuits when the
drum is rotated toward the side of the lesion (1) (17) (12).
Abnormalities of Pursuits
Poor pursuits often leads to poor performance in sports and excessive head
movements while tracking moving objects. Lesions of the parietal lobe, the
occipitoparietal junction, the brainstem, and the cerebellum may all lead to
abnormal smooth tracking eye movements,
• The most common abnormality of pursuits is cogwheeling, a series of
step-like eye movements that are used to follow a moving object, rather
than smooth pursuits.
• Low pursuit gain (asymmetry between eye movement velocity and the
target velocity) may also occur secondary to lesions of the CNS or due
to medications (e.g. tranquilizers, barbituates, anti-convulsants, anti­
anxiety medications, alcohol, marijuana, and lithium) (15).
There are few standardized techniques available for assessing the patient’s abil­
ity to make smooth pursuits. The most common tests used include:
NSUCO oculomotor test: A direct observation test where the examiner as­
sesses the patient’s ability to make two clockwise and two counterclock­
wise pursuit rotations. The patient is graded on ability, accuracy, and
degree of head/body movement compared to age-expected norms (15).
Copyright 2014 by KMK Educational Services, LLC
302 7.3. EYE MOVEMENT DYNAMICS
Groffman tracings: The patient is asked to visually trace a line without
using any guides and is scored on his/her ability to accurately reach the
correct end point for each line. Although this test has been used for many
years to assess pursuits, there are no studies demonstrating its validity
or reliability in diagnosing pursuit dysfunctions (15).

Oculomotor dysfunction refers to abnormalities in fixation, sac-


cades, and pursuits. Patients with OMD should undergo a com­
prehensive binocular, accommodative, and visual perception eval­
uation as these systems are often abnormal as well (15).

jVefgence M oycment s'


Vergence movements align the eyes so an object is fixated by both foveas si­
multaneously, allowing for fusion and binocular vision. In contrast to other
eye movements, vergences are disconjugate movements; the two eyes move
in OPPOSITE directions, resulting in convergence or divergence of the line of
sight. These movements occur with a latency of about 160 msec (similar to
other eye movements), with a slow velocity of only 10 degrees/second (12) (8).
Several different types of vergence movements are detailed below (8) (5):
Tonic: Tonic vergence describes the eyes' position of rest at distance without
a stimulus to convergence or accommodation. Clinically it is measured
as the best-corrected distance phoria.
• A phoria describes the position of the lines of sight of each eye when
there is no stimulus for fusion (the eyes are dissociated) and repre­
sents the physiological position of rest of the eyes (6). The following
prefixes are used to describe a phoria or tropia (a misalignment of
the line of sight of the eyes under associated conditions):
Ortho: Perfect alignment of the visual axes.
Eso: Over-convergence of the visual axes.
Exo: Under-convergence of the visual axes.
Hyper: An upward deviation of one eye.
Hypo: A downward deviation of one eye.
Proximal: Proximal convergence is associated with a person’s awareness of a
near target. Because the degree of proximal vergence is sufficiently small,
it is usually neglected when measuring vergence movements.
Fusional: Fusional vergence is initiated by retinal image disparity and helps
the eyes compensate for a phoria to obtain bifoveal fixation and binocular
vision; it is analogous to motor fusion.
Copyright 2014 by KMK Educational Services, LLC
CHAPTER 7. OCULAR MOTILITY 303

Accommodative: Accommodative convergence is initiated by blur and oc­


curs in concert with changes in accommodation.
• Accommodation and convergence are linked to one another in a way
that is characterized by the AC/A ratio (5) (see BV chapter for
additional details).

The near reflex triad of convergence, accommodation and pupil­


lary constriction occurs when fixation is shifted from a distance
object to a near object.

- SECTION 7.4 --------------------------------------------- — ------- -----------------------------

Diagnostics for extraocular muscle dysfunction


Binocular diplopia occurs when the extraocular muscles of the two eyes are
no longer working as a team in one, many, or all directions of gaze. Differential
diagnoses for new onset diplopia that must be considered include a decompen­
sated phoria, an anatomical muscle restriction, or an innervation abnormality
to the extraocular muscles.

A decompensated phoria describes a large phoria that the pa­


tient has previously compensated for, but is 'now unable to do so
as he/she tires over time, resulting in a tropia and symptoms of
diplopia.

1 Establish comitancy by performing versions (extraocular motilities), a


cover test, and subjective testing (e.g. Maddox rod, the red lens test or
the Hess-Lancaster test).
Comitant deviation: A misalignment of the visual axis of each eye
that is the same in all positions of gaze. This finding indicates a
decompensated phoria.
Noncomitant deviation: A misalignment of the visual axis of each eye
that is not the same for all positions of gaze. This finding indicates
an anatomical muscle restriction or a muscle palsy.
Copyright 2014 by KMK Educational Services, LLC
304 7.Jh DIAGNOSTICS FOR EOM DYSFUNCTION
2 If the deviation is comitant, it is likely secondary to a decompensated pho-
ria and no further testing is necessary. If the deviation is non-comitant,
forced ductions (see description below) should be performed to differ­
entiate between an anatomical muscle restriction and a muscle palsy.
* Comparing monocular ductions to versions may also help to differ­
entiate a muscle restriction from a cranial nerve palsy, as it may
be easier for the muscle to overcome a weakness caused by a palsy
during ductions (due to strong innervation) compared to versions
(due to shared innervation of yoked muscles).
3 If further testing indicates a muscle palsy, isolate the EOM that is in­
volved (see Figure 7.1). Park's 3 step will help to isolate the involved
EOM when a vertical deviation is present (see description below).

During motility testing, the patient must keep the head still while
following a moving target with his/her eyes in the 9 diagnostic
directions of gaze, allowing for isolation of the involved EOMs.

Clinical note: To isolate the oblique muscles, have the patient look towards
his/her nose (o ’s to the nose). Elevation of the eyes will isolate the inferior
oblique and depression will isolate the superior oblique. To isolate the
rectus muscles, have the patient look away from the midline (ABduct);
elevation of the eyes will isolate the superior rectus and depression will
isolate the inferior rectus.

Clinical note: If the patient has a non-comitant deviation, the deviation will
be worse when looking in the direction of the underperforming muscle.
For example, a CNIV palsy (affecting the SO) is characterized by a hyper­
deviation that increases with ADduction and depression of the affected
eye (see Figure 7.1).

Copyright 2014 by KMK Educational Services, LLC


CHAPTER 7. OCULAR MOTILITY 305

Figure 7.1: Summary diagram of EOMs. Note how testing the patient’s
right inferior rectus would require the patient to look down and to his/her
right.

As a general rule, in a patient with an abnormal head turn sec­


ondary to a non-comitant deviation, the face will be pointing in
the direction that corresponds to the field of action of the affected
muscle. The direction of a compensatory head tilt is more variable,
depending on the muscles involved; the head will tilt AWAY from
the side of the lesion in an SO muscle palsy. The head will tilt
TOWARDS the side of the lesion in an 10 muscle palsy. Vertical
rectus muscle palsies may be accompanied by a head tilt in either
direction (less predictable) (3) (18).

Clinical note: The magnitude of a non-comitant deviation will vary depend­


ing on which eye is used for fixation.
• Prim ary deviation: The deviation of the paretic eye when the
normal eye is fixating.•
• Secondary deviation: The deviation of the normal eye when the
paretic eye is fixating. In a non-comitant deviation, the secondary
deviation will ALWAYS be greater than the primary deviation due
to Herring’s law of equal innervation (greater stimulus is necessary
to maintain fixation of the paretic eye (compared to when the normal
eye is used for fixation), resulting in a greater stimulus to the yoked
muscle in the normal eye and a larger deviation).
Copyright 2014 by KMK Educational Services, LLC
306 74- DIAGNOSTICS FOR EOM DYSFUNCTION

Note that cover test and Maddox rod testing are detailed in the BV chapter.
Additional tests for comitancy and muscle restrictions are covered below.
Red lens testing:
A subjective test of comitancy the involves dissociating the two eyes by placing
a red lens in front of one eye; the fellow eye will view a white penlight or muscle
light on the projector screen. The patient is asked to describe the relative
positions of the red light and white light in all 9 directions of gaze.
Hess-Lancaster test:
This is the most selective subjective test for comitancy. The patient views a
large grid while wearing red-green glasses with the red lens placed in front of
the normally fixating eye (if measuring the primary deviation). The examiner
holds a red flashlight (viewed by the normal eye of the patient) and the patient
is given a green flashlight (viewed with the paretic eye). The examiner places
the red light on a specific point on the screen and the patient is asked to line
up his/her green light with the examiner’s red light. The relative position of
the two lights are recorded on the grid for all 9 positions of gaze. In order
to measure the secondary deviation, the filters are switched so the red filter is
now in front of the paretic eye.
Forced Ductions
The forced duction test differentiates an anatomical muscle restriction from a
cranial nerve muscle palsy. The test is performed as follows (9):
1 The patient is asked to look in the direction of restricted movement.
2 After anesthetizing the eye, forceps are used to grasp the conjunctiva to
rotate the eye in the direction of restricted movement. Comparing the
degree of movement of each eye in the direction of interest will aid in the
diagnosis and interpretation of the test.

A positive forced duction test occurs when the eye DOES NOT
move in the direction of restriction, indicating an anatomical mus­
cle restriction. A negative force duction test results when the eye
CAN be moved and indicates a muscle palsy (3).

Copyright 2014 by KMK Educational Services, LLC


CHAPTER 7. OCULAR MOTILITY 307

Differential diagnoses for anatomical muscle restrictions include Graves’ oph­


thalmopathy, an orbital tumor, or entrapment of an EOM following trauma
(see the ocular disease chapter for additional details). Additional diagnoses
include:
D uane’s Retraction Syndrome: A congenital condition most commonly noted
in the left eye in females. The underlying etiology for Duane’s retraction
syndrome is not well understood - theories include structural abnormal­
ities of the muscles or an innervational issue arising from abnormalities
of the abducens and oculomotor nuclei within the brainstem. There are
three types:
• Type I: Limited ABduction (most common type that must be dif­
ferentiated from a GN 6 muscle palsy)
• Type II: Limited ADduction (least common)
• Type III: Limited AB and ADduction.
All three types are often associated with globe retraction and narrowing
of the palpebral fissure with ADduction. Patients commonly present with
an esotropia in primary gaze (18).

Brown’s Syndrome (aka SO tendon sheath syndrome): Brown’s syndrome


involves abnormality of the SO muscle and tendon or abnormality of the
trochlea. Brown’s syndrome may be acquired (iatrogenic damage or scar­
ring or inflammation of the trochlea) or congenital (the SO muscle is too
short or the tendon is inelastic). It is typically unilateral and is character­
ized by a small hypotropia in primary gaze and limited elevation during
ADduction. Differential diagnoses include 10 paralysis, overaction of the
SO, and an orbital floor fracture (20) (3) (18).

Park’s 3 step test is used to isolate a single muscle responsible for a vertical
deviation. Note that it will not be helpful if multiple muscles are involved.
The test involves the following questions (4):
1 Which eye is highest in primary position?
2 Is the hyper worse when the person looks towards the left or right?
3 Is the hyper worse when the person tilts his/her head towards the left or
right (Bielchoswky’s head tilt test)?
Consider the following cases where the right eye is hyper:
• If hyper is worse on gaze right and tilt right —i left inferior oblique
Copyright 2014 by KMK Educational Services, LLC
308 14- DIAGNOSTICS FOR EOM DYSFUNCTION
• If hyper is worse on gaze right and tilt left —>right inferior rectus
• If hyper is worse on gaze left and tilt right —>right superior oblique
• If hyper is worse on gaze left and tilt left — left superior rectus

Exam ple 7 .1 : A p a tie n t show s a h y p e rtro p ia in th e le ft eye in p rim a ry p ositio n a n d th e


h yp e rtro p ia b eco m es m o re p ro n o u n c e d w hen he looks rig h t a n d tilts his h ea d left. W h ic h
E O M is lik e ly involved?

Solution 7 .1 : T h e h y p e r becom es worse w hen he looks rig h t a n d tilts left, so th e an sw er


(u s in g P a rk 's m e th o d ) is th e le ft s u p e rio r oblique.

References
[1] Bajandas, Frank J., Kline, Lanning B. Neuro-Ophthalmology Review Manual, 3rd ed. Tho-
rofare: SLACK, 1988.
[2] Becker W: Metrics. In Wurtz RH, Goldberg MB (eds): The neurobiology of saccadic eye
movements, New York, 1989, Elsevier.
[3] Bennett, E, and V. Lakshminarayanan (2006). R eview Q uestions fo r the N B EO E xam ina­
tion, P art II. Butter worth Heinemann.
[4] Gasser, L., Fineret, M., and H. Woodcome (1997). A tlas o f P rim ary Eyecare Procedures ,
2nd Edition. Appleton and Lange.
[5] Goss, David A. Ocular Accommodation, Convergence, and Fixation Disparity, 2nd ed.,
Boston: Butterworth-Heinemann, 1995.
[6] Grosvenor, T, (2007). P rim ary Care O ptom etry, 5th edition Butterworth.
[7] Horner, Douglas G, V 665 Physiological Optics III. Ocular Motility. Indiana University Class
Notes, 2004.
[8] Kaufman, Paul. Aim, Albert. Adler’s Physiology of the Eye, 10th ed. St, Louis: Mosby, 2003.
[9] Kline, L. and Bajandas, F. (2004). N euro-ophthalm ology Review M anual, 5th Edition. Slack
Incorporated.
[10] Martinez-Conde S, Macknik SL, Hubei DH. The role of fixational eye movements in visual
perception. Nature Reviews Neuroscience 2004;5:229-240.
[11] Miller NR, Newman NJ, Biousse V, Kerrison JB. Walsh and Hoyt’s Clinical Neuro-
Ophthalmology: The essentials. 2nd ed, Philadelphia: Lipincott, Williams and Wilkins, 2008.
[12] Purves, D. Augustine, G., Fitzpatrick, D., Katz, L., LaMantia, A, , McNamara, J, Williams,
S.. Neuroscience, 2nd Ed, Sinauer Associates, Inc, 2001.
[13] Rabbetts, R. (2007). B en n ett and R abbetts’ Clinical Visual Optics, 4th E dition.
Butte rwor t h- He in eman n.

Copyright 2014 by KMK Educational Services, LLC


CHAPTER 7. OCULAR MOTILITY 309

[14] Remington, Lee Ann. Clinical Anatomy of the Visual System, Boston: Butterworth Heine-
mann, 1988.
[16] Scheiman M, Wick B. Clinical management of binocular vision: Heterophoric, accommoda­
tive, and eye movement disorders. 3rd ed, Philadelphia: Lipincott, Williams and Wilkins,
2008.
[16] Smith, JL. Optokinetic Nystagmus: Its use in Topical Neuvoophthalmologic Diagnosis.
Springfield, CC: Thomas, 1963.
[17] Smith, J.L. The Pupil. The J. Lawton Smith Lecture Series. The University of Miami, 1974.
[18] Von Noorden GK, Campos EC. Binocular vision and ocular motility: Theory and manage­
ment of strabismus. Gth ed. St. Louis: Mosby, Inc., 2006,
[19] Williams. Neuroscience, 2nd edition. Sinauer Associates, 2001.
[20] Wong, A. M. F. (2008). Eye M ovem ent Disorders. Oxford University Press.

Copyright 2014 by KMK Educational Services, LLC


(( '

((
((
f(
((
((
((
(c·
((
((
((
((
((
(( I

(( I
l

(( I

((
((
{(
( ( . !

((
((
((
(( /

(( /

c
(
(
(
Chapter 8
Binocular Vision

Sarah Dougherty Wood, O.D., F.A.A.O.,


Kevin B. Wood, Ph.D,

311
(
f
c
(
f
(
(
(
(
(
(
(
(
(
(
(
(
(
(
(
(
(
(
Copyright 2014 by KMK Educational Services, LLC (
(
(
(
CHAPTER 8. BINOCULAR VISION 313

SECTIO N 8.1

Introduction
Binocular vision involves the motor coordination of the two eyes to align the
foveas on an object of interest, allowing for fusion and sensory integration of
information from both eyes in the visual cortex. There are distinct advantages
to binocular vision, including increased visual acuity, increased contrast sensi­
tivity, a larger field of vision, stereopsis, and improved performance on visual
tasks (9).

M otor fusion is a vergence eye movement stimulated by retinal


disparity that moves the eyes to align each fovea with an object.
Sensory fusion involves combining two images from each fovea
into a single percept in the visual cortex (24).

For an object to be perceived as single with binocular vision, the image on


each retina must be clear, the same size, and fall on corresponding retinal
points.
• Corresponding retinal points are points in each eye that have the
same visual direction and project to the same area in the visual cortex.
In other words, an image falling on corresponding lines of sight in each
eye will be viewed as in the same visual direction by each eye.
• The primary visual direction is the line of sight going through the
fovea. Secondary visual directions are lines of sight through all other
retinal points.•
• Under binocular conditions, the foveas of each eye share a common
subjective principle visual direction. All other corresponding retinal
points in the two eyes have a secondary common subjective visual
direction.

Primary/secondary visual directions of each eye are in reference


to the eye position (oculocentric) under monocular conditions.
Common subjective visual directions are in reference to the head
(egocentric) under binocular conditions (24).

Copyright 2014 by KMK Educational Services, LLC


314 8.2. NON-STRABISMIC B V DISORDERS
The horopter is a spacial representation of all points in space that are imaged
on corresponding retinal points in the two eyes. Any object falling on the
horopter will be seen as single.

Panum’s fusional area describes the area immediately around


the horopter where objects are still seen as single and in depth (even
though they are off the horopter). Objects outside of Panum’s area
will be seen as diplopic (24).

Binocular vision disorders may be classified into two categories:


1 Non-strabismic disorders where single vision is present under most cir­
cumstances but is a stress to the visual system, leading to patient symp­
toms.
2 Disorders where binocular vision is absent, resulting in possible strabis­
mus, diplopia, and confusion.
Compensatory mechanisms to avoid diplopia and confusion in the visual system
include suppression, covering an eye, an abnormal head tilt, and anomalous
retinal correspondence (10).
- SECTION 8.2 ------------------------------ --------------- -------------------------------------------------

Non-strabismic binocular vision disorders•

1 Magnitude and direction of the phoria at distance and near.


• Tests: Cover test, Von Graefe phoria, Maddox rod, modified Thor-
ington card, fixation disparity.
2 Magnitude and direction of positive and negative fusional vergence ranges
at distance and near.
• Direct tests: Smooth vergence, step vergence, vergence facility.
• Indirect tests: NRA/PRA, fused cross cylinder (FCC), binocular
accommodative facility, MEM retinoscopy.
3 Convergence amplitude.
• Test: Near point of convergence (NPC).
4 Sensory status (suppression and stereopsis).
• Tests: Randot, Worth 4 dot.
Copyright 2014 by KMK Educational Services, LLC
CHAPTER 8. BINOCULAR VISION 315

M S m sm
1 All testing should be done with the refractive error corrected, as an
uncorrected refractive error can change the accommodative response and
lead to a phoria, a shift in the range of fusional vergence, and/or poor
fusional ability secondary to blur.
2 Distance testing - use the 20/30 line or two lines above their BCVA (best
corrected visual acuity).
3 Near testing - unless otherwise noted, objects should be held at 40 cm;
remind the patient to keep the words and/or object clear in order to
control accommodation.
We will now describe tests used in the assessment of non-strabismic binocular
vision disorders in more detail.

Purpose: Cover test is a dissociated test (fusion is broken) that allows for the
objective determination of the presence, direction, and magnitude of a phoria
or tropia.
Unilateral Cover Test: The unilateral cover test allows the examiner to
determine the presence of a phoria or tropia. The patient fixates on an object
at a particular distance while the examiner covers and uncovers one eye while
observing the fellow eye. Rabbetts offers a concise summary of the conclusions
that may be drawn from this test (19):
1 Cover the right eye while viewing the left eye.
• If the left eye moves to assume fixation, there is a tropia of the left
eye. The left eye moving out indicates an esotropia; the left eye
moving in indicates an exotropia.
• If the left eye does not move, there is no tropia. However, a phoria
may still be present.
2 Uncover the right eye while still viewing the left eye.
• If movement of the left eye was seen in step 1, both eyes should
make a version movement as the right eye regains fixation (again
indicating a left tropia).
• If movement of the left eye was not seen in step 1, watch for move­
ment of the right eye as it is uncovered; movement indicates a het-
erophoria.3
3 Cover the left eye and watch the right eye.
Copyright 2014 by KMK Educational Services, LLC
316 8.2. NON-STRABISMIC BV DISORDERS
• If no movement of the left eye was seen in step 1 and the right eye
moves to obtain fixation as the left eye is covered, there is a right
tropia.
• If the left eye moved in step 1 and now the right eye also moves to
obtain fixation as the left eye is covered, the patient has an alter­
nating strabismus (either eye is used for fixation).
4 Uncover the left eye while viewing the right eye.
• If movement of the right eye was seen in step 3, look for a version
movement of both eyes as the left eye regains fixation of the target
(again indicating a right tropia or an alternating strabismus if the
left eye also moved when the right eye was covered).
• If the right eye did not move in step 3 (and the left eye did not
move in step 1), look for movement of the left eye as it is uncovered,
indicating a heterophoria.
Alternating Cover Test: The alternating cover test allows the examiner to
measure the amount of phoria or tropia that is present by using prism to neu­
tralize the deviation. The patient is asked to fixate on an object at a particular
distance as the examiner quickly alternates between covering the right eye and
the left eyes, viewing the movement of each eye just as it is uncovered. An
exophoria/tropia is neutralized with base in prism; an esophoria/tropia
is neutralized with base out prism.
Expected findings for non-presbyopic patients:
• At distance: 0-2 exophoria
* At near: 0-6 exophoria

The phi phenomenon (subjective cover test) describes the pa­


tient’s perception of movement opposite to the direction of devia­
tion of an eye as it is uncovered. For example, a patient with an
esophoria will perceive an image moving to the left (opposite the
inward turn of the eye) as the paddle is switched from covering the
left to the right eye (motion opposite to the direction of move­
ment of the paddle). A patient with an exophoria will perceive the
image moving to the right (opposite the outward turn of the eye)
as the paddle is moved from the left to the right eye (motion in the
same direction as the movement of the paddle) (19).

Copyright 2014 by KMK Educational Services, LLC


CHAPTER 8. BINOCULAR VISION 317

Purpose: Von Graefe phoria is a dissociated test that allows for the subjec­
tive determination of the presence, direction, and magnitude of an eye
deviation. However, the examiner will not be able to determine whether
the deviation is a phoria or a tropia (unlike cover test).
How to: The test is performed in the phoropter with Risley prisms. 12 BI
is placed over the right eye and 6 BU is placed in front of the left eye.
The patient should report one target down and to the left and the other
target up and to the right (uncrossed diplopia with the left eye’s image
to the left and the right eye’s image to the right). If the patient does not
report uncrossed diplopia, increase the amount of BI OD until the lower
image is to the left of the upper image.

Patients should initially view uncrossed diplopia before measur­


ing Von Graefe phorias in order to avoid a stimulus to convergence
that could alter the phoria.

• To test the horizontal phoria, the patient is instructed to look at the


lower left target while the examiner moves the upper right target by
decreasing the amount of BI prism until the patient reports the
targets are lined up vertically (buttons on a shirt).
* To test the vertical phoria, the patient is instructed to look at the
upper right target while the examiner moves the lower left target
by decreasing the amount of BU prism until the patient reports the
targets are lined up horizontally (headlights on a car).

Vertical deviations are recorded as the eye with the hyper deviation.

Expected findings: The same findings as cover test.

K iliiii
The Maddox rod is a series of stacked cylinders (seen as multiple grooves) that
forms a line image on the retina when a patient views a point of light (19). It is
a dissociated test that allows for the subjective determination of the magnitude
and direction of a deviation, but it cannot differentiate a phoria from a tropia.
Copyright 2014 by KMK Educational Services, LLC
318 8.2. NON-STR.ABISMIG B V DISORDERS
* To test for a horizontal deviation, the Maddox rod striations are oriented
horizontally, producing a vertical line image for one eye while the fellow
eye views a point light source.
• To test for a vertical deviation, the Maddox rod striations are oriented
vertically, producing a horizontal line image for one eye while the fellow
eye views a point light source.
Interpretation of the test (assuming the Maddox rod is in front of the right
eye and oriented horizontally, producing a vertical line image to measure
a horizontal deviation):
• If the vertical line seen by the right eye passes directly through
the center of the point source seen by the left eye, the patient is
orthophoric.
• If the vertical line seen by the right eye is to the right of the light
source seen by the left eye, the patient has uncrossed diplopia,
indicating an eso deviation.
• If the vertical line seen by the right eye is to the left of the light
source seen by the right eye (crosses the midline), the patient has
crossed diplopia, indicating an exo deviation.
Now assume the Maddox rod is oriented vertically in front of the right eye
(producing a horizontal line image to measure a vertical deviation):
• If the horizontal line seen by the right eye is below the light source
seen by the left eye, the patient has a right hyper deviation.
• If the horizontal line seen by the right eye is above the light source
seen by the left eye, the patient has a left hyper deviation.
After determining the direction of the deviation, prism may be used to measure
the magnitude by neutralizing the deviation.

The double Maddox rod test involves one Maddox rod over each
eye and is used to detect a torsional misalignment of the eyes.

Purpose: A subjective, dissociated test that measures the magnitude and


direction of an eye deviation (similar to subjective cover test, Maddox rod,
and Von Graefe phorias). Studies have shown the modified Thorington
card is the most repeatable method for measuring phorias (21).
Copyright 2014 by KMK Educational Services, LLC
CHAPTER 8. BINOCULAR VISION 319

How to: The test is performed outside the phoropter with the Thorington
card held at 40 cm. The patient holds a Maddox rod over the right eye
while viewing a light shone through the center of the Thorington card
with the fellow eye.
• To measure a horizontal deviation, the Maddox rod is oriented hor­
izontally, producing a red vertical line image OD. As the patient
looks at the center light, he/she reports whether the red line is to
the right or left of the zero, and which number the red line passes
through on the Thorington card. The number indicates the magni­
tude of the phoria.

Remember, uncrossed diplopia (the red line is seen to the right of


the zero) indicates esophoria; crossed diplopia (the red line is seen
to the left of the zero) indicates exophoria.

• To measure a vertical deviation, the Maddox rod is oriented ver­


tically, producing a horizontal line image OD while the fellow eye
views a Thorington card with vertically oriented numbers. As the
patient looks at the center light, he/she reports whether the red line
is above or below the zero, and which number the red line passes
through on the card.
Expected findings: similar findings as cover test at near

For all tests used to measure the magnitude and direction of a


phoria/tropia, remind patients to keep the letters clear in order to
control for accommodation (21).

' AT v A A • .V. •.....V-';' ' 7: . ;


" - - 7 -’ - . - A " v •'">

Fixation disparity is a very small misalignment of the visual axes (minutes


of arc) that is not observed with standard tests for ocular alignment.
Because the object falls within Panum ’s fusional area for the corre­
sponding retinal points in each eye, it will still be seen as single, despite
the slight deviation of the eyes.
Fixation disparity is measured under associated conditions (images from the
two eyes are fused). Devices that may be used to measure FD include:
Copyright 2014 by KMK Educational Services, LLC
320 8.2. NON-STRABISMIC BV DISORDERS
• Mallet unit
• AO vectographic slide
• Bernell lantern
• Wesson fixation card
• Sheedy disparometer
In general, the patient will be viewing two separate images with the right and
left eye in the presence of fusion locks. The patient notes the alignment
of the images of the right and left eyes. Let’s look at the Wesson card,
the most common and inexpensive clinical test for measuring FD, as an
example:
• The patient holds the Wesson card at 40 cm while wearing polarized
glasses. The Wesson card has a black arrow below and colored lines
(with the red line in the center) above a horizontal line. The left
eye sees the black arrow and the right eye sees the colored lines (aka
nonius lines).
• The patient reports which colored line the black arrow is pointing
to. The amount of fixation disparity can then be read directly off
the card.
• Note that the black arrow on the Wesson card will always point
to the center red line. However, the black arrow APPEARS DIS­
PLACED to a patient with fixation disparity (21).
The Sheedy disparometer uses another approach to measure fixation disparity.
The patient is shown two vertical lines (one upper and one lower line seen
by each eye) that are deliberately misaligned. The examiner gradually
reduces the amount of prism offsetting the vertical lines until the patient
reports a single vertical line. In contrast to the Wesson card, the two
vertical lines will actually remain separated, although they APPEAR
ALIGNED to the patient (21).

The amount of prism required to neutralize fixation disparity is


termed the associated phoria. The associated and dissociated
phorias are typically in the same direction. Paradoxical FD oc­
curs when the associated and dissociated phorias are in opposite
directions.

There are four types of fixation disparity curves that may be plotted where the
y-axis is the amount of fixation disparity and the x-axis is the magnitude
of the associated phoria:
Copyright 2014 by KMK Educational Services, LLG
CHAPTER 8. BINOCULAR VISION 321

• Type 1 curve is the most common with a sigmoidal shape. Patients


are asymptomatic.
• Type 2 curve occurs with an eso disparity.
• Type 3 curve occurs with an exo disparity.
• Type 4 curve occurs in an unstable binocular system. Patients may
not have an associated phoria and may have symptoms of aneisoko-
nia and poor sensory fusion.

mmmmammmmmmmmma
The AC/A ratio represents the amount of accommodative convergence (diver­
gence) that occurs in response to an increase (decrease) in accommodation and
is measured in units of prism diopters/diopter. There are two different methods
for determining the AC/A ratio:
Calculated AC/A: The patient’s phoria is measured through his/her best
corrected prescription at distance and near, leading to the following for­
mula:
AC /A = PD + NFD (Pn - Pd) (8.1)
• PD = interpupillary distance in centimeters
• N F D = near fixation distance in meters
• Pn = near phoria (eso is +, exo is -)
• Pd = distance phoria (eso is +, exo is -)

Exam ple 8 .1: A p a tie n t w ith a P D o f 6 4 m m has 1.5A esophoria a t distance a n d


3 a exophoria a t 4 0 cm . W h a t is h is /h e r A C / A ra tio ?

Solution 8 .1: In th is case, c a lc u la te d A C / A = 6 .4 cm -j-.4 m ( - 3 - ( + 1 . 5 ) ) = 6 .4 -1 .8


= 4 . 6 A / D *•

Gradient AC/A: The patient’s phoria is measured at the same distance but
with different lenses to change the stimulus to accommodation. Typically
the phoria is measured at 40 cm, first through the patient’s subjective
refraction and then with +1.00D over the subjective refraction. The
following formula is used to determine the gradient AC/A:
A C /A = (Pi - P2 )/(S a i - SA2 ) (8.2)
• P±t2 = phoria under the 1 st and 2 nd condition
• S a i ,A2 — accommodative stimulus under the 1st and 2 nd condition
Copyright 2014 by KMK Educational Services, LLC
322 8.2. NON-STRABISMIG B V DISORDERS

Example 8.2: A patient with a PD of 64 mm is 2A esophoric at 40 cm through


their subjective refraction and 10A esophoric at 40 cm through -l.OOD over the
subjective refraction. What is his/her A C / A rath?

Solution 8.2: Since we measured phorias at the same distance but with different
stimuli to accommodation, we find the gradient A C / A = (2 - 10) / (2.5 - 3.5) =
— = 8A/D •

The calculated AC/A will be greater than the gradient AC/A due
to the added proximal convergence when the phoria is measured at
near.

Expected findings: 4 A /D

The gradient and calculated AC/A ratios are stimulus AC/A ra­
tios - we assume the patient is accommodating equal to the given
stimulus. In reality, most patients have a lag of a accommodation
(they accommodate less than the given stimulus). The response
AC/A ratio is determined by measuring the patient’s actual ac­
commodative response to a given stimulus and is usually larger
than the stimulus AC/A (21).

Determination of the AC/A ratio is imperative for reaching the correct


diagnosis and for developing the most appropriate management strategy
for patients with non-strabismic binocular vision disorders (21).
• Patients with high AC/A ratios will have a significant change
in accommodative convergence, and thus their phoria, with small
changes in spherical lenses, For example, a patient with a high
AC/A ratio and moderate esophoria at near will have a large change
in accommodative divergence with just a small addition of plus,
resulting in less esophoria and reduced patient symptoms.
* Patients with low AC/A ratios respond better to prism or vision
therapy as a change in lens power would not significantly alter
accommodative convergence (and thus the patient’s phoria).
Copyright 2014 by KMK Educational Services, LLC
CHAPTER 8. BINOCULAR VISION 323

Exam ple 8.3: F o r a p a tie n t w ith convergence excess (h ig h esophoria a t n ea r) a n d an


A C / A ra tio o f 1 0 A / D , an a d d itio n o f + 1 . 5 0 D a t n ea r o v e r the sub jective refraction w ill
decrease convergence b y h o w m u ch ?

Solution 8.3: T h e a d d w ilt decrease convergence b y 1 5 prism diopters. F o r exam p le, i f


th e p a tie n t were 1 0 p rism d io p te rs esophoria a t n e a r throu gh th e sub jective refractio n ,
h e /s h e w ould b e 5 prism d io p te rs exop ho ria a t n ea r th ro u g h + 1 . 5 0 D o ve r th e su b jective
refraction . •

Purpose: Determines the amplitude and recovery of the fusional vergence


system, which causes convergence/divergence of the eyes in response to
retinal image disparity in order to maintain sensory fusion.
• Positive fusional vergence (PVF) causes convergence of the eyes
and is tested by adding BO prism (the image will move closer,
forcing the eyes to converge).
• Negative fusional vergence (NVF) causes divergence of the eyes
and is tested by adding BI prism (the image will move further
away, forcing the eyes to diverge).

• Plus lenses indirectly test PVF: Plus lenses will decrease


accommodation and result in divergence of the eyes. PVF is
required to converge and re-align the eyes with the object to
avoid diplopia.
• Minus lenses indirectly test NVF: Minus lenses increase
accommodation and convergence. NVF is required to diverge
the eyes to place them back on the target.

Some forms of nystagmus can be treated by inducing fusional con­


vergence through the addition of BO prism or plus lenses (3).

Copyright 2014 by KMK Educational Services, LLC


324 8.2, NON-STRABISMIC BV DISORDERS
How to: Smooth vergences are performed in the phoropter using Risley prisms
(rate of 2 PD/second).
• To test horizontal ranges, the Risley prisms should be orientated
so the zeros are facing up. Determine PFV ranges first by adding
BI OU. The patient should report when the target becomes blurry
(blur) and splits into two (break). Slowly decrease BI OU until
the patient reports the image is single again (recovery). The same
procedure is followed with BO OU to measure NVF ranges. Note
that if the patient blurs at 6 PD OD and 6 PD OS, the blur is
recorded as 12 PD (add the prism in each eye).
—The blur point represents the limit of fusional vergence. At
this point, accommodative vergence kicks in to prevent diplopia,
causing the image to blur.
—The break point represents the limit of fusional AND accom­
modative vergence, resulting in a blurred and double image.
—The recovery is a qualitative assessment of the flexibility of
the binocular system to regain fusion after diplopia occurs. The
recovery should be at least half of the break.

Negative fusional vergence (BI) ranges should always be performed


before positive fusional vergence (BO) ranges as convergence of the
eyes may interfere with the test results.

• To test vertical ranges, the Risley prisms are oriented so the zeros
are horizontal. Add BU OD only until the patient reports diplopia
of the image (break), followed by a decrease in BU OD until the
patient reports the image is single again (recovery). Repeat the
procedure with BD OD.
Expected findings (M organ’s norms) (8):
Distance BI X/7/4
Distance BO 9/19/10
Near (40 cm) BI 13/21/13
Near BO 17/21/11
Sheard’s and Percival’s criteria allow the clinician to determine whether the
vergence ranges are sufficient to account for the patient’s phoria, allowing for
comfortable and clear binocular vision.
Copyright 2014 by KMK Educational Services, LLC
CHAPTER 8. BINOCULAR VISION 325

Although limited, research suggests that Sheard’s criterion is most


effective when prescribing prism for exophorias while PercivaFs
criterion is most effective when prescribing prism for esopho-
rias (21).

Sheard’s criterion states the compensatory fusional vergence reserve (blur


point) should be at least twice the demand of the phoria. The prism
needed (S ) is determined by the following equation:
(8.3)
• D —phoria (demand)
• R = compensating fusional vergence (reserve)
• Remember, BI = EXO, BO = ESO

Example 8.4: If a patient has 5A of exophoria and the base-out blur finding is 4A,
what is the amount and direction of the prism that should be prescribed based on
Sheard's criterion?

Solution 8.4: S —§ (5) - | (4) — 2A BI

Percival’s criterion states the smaller fusional vergence reserve should be at


least half of the greater fusional vergence reserve; this places the demand
line within the middle third of the zone of clear, single, binocular vision,
preventing patient symptoms. Percival’s criterion DOES NOT take into
account the phoria of the patient. The prism needed (P ) is determined
by the following equation:
P = \ G~ \ L (8-4)
• G — greater of the two vergence ranges
• L = lesser of the two vergence ranges
• IfP is zero or a negative number, no prism is necessary.

Example 8.5: If a patient has 10A esophoria with BI vergence 6/10/4 and BO vergence
21/28/22, what is the amount and direction of prism that should be prescribed based on
Percival's criterion?

Solution 8.5: P= | (21) - § (6) = 3A BO

Copyright 2014 by KMK Educational Services, LLC


326 8.2. NON-STRABISMIC B V DISORDERS
testing
Purpose: Determines the amplitude and recovery of the fusional vergence
system (similar to smooth vergences) but is performed OUTSIDE the
phoropter with a prism bar.
How to: The patient views a letter target at distance or near. BI prism is
introduced in front of the right eye (increasing at a rate of 2A/second)
until the patient reports the image is blurred and then double. After the
break, continue adding an additional 5A before decreasing BI prism until
the patient reports the image is single again. The same procedure is then
repeated with BO prism OD (21).
Expected findings are the same as smooth vergence testing.
v t.
Purpose: Determines the patient’s ability to make large, rapid, and sustained
changes in fusional vergence over a period of time (5).
How to: Using a 12 BO/3 BI combination flipper, have the patient view a near
card and introduce 12 BO in front of his/her eyes, causing the letters to
double. As soon as the patient reports clear single vision again, rotate
the flipper to introduce 3 BI in front of the eyes. The examiner counts
the number of cycles (clearing BI and BO) the patient is able to complete
within 60 seconds.
Expected findings: 15 cycles/minute (25).

Remember to encourage the patient to always keep the letters clear


on the near card in order to control accommodation (21).

Purpose: Determines the patient’s ability to converge his/her eyes while main­
taining fusion.
How to: Show the patient an isolated target at near just below his/her line
of sight on the midline. Slowly bring the target towards the nose of the
patient until lie/she reports diplopia or an eye drifts out (indicating loss
of fusion). Once fusion is lost, slowly move the target away from the
patient until he/she reports a single image again (21).
Expected findings: 5 cm break, 7 cm recovery
Copyright 2014 by KMK Educational Services, LLC
CHAPTER 8. BINOCULAR VISION 327

NPC may also be performed with a penlight while the patient wears
red-green glasses. The break point for convergence occurs when
the patient reports two colored lights instead of a single light. The
patient may have a more receded NPC under dissociated conditions
than with standard testing (21).

- SECTION 8.3 ---------------------------------------------------------------------------

Assessment of Accom m odative Disorders


Between 70-80% of all binocular vision disorders are secondary to accommoda­
tive dysfunction (12) (13). Accommodative disorders are also thought to be
initially responsible for the majority of non-accommodative binocular vision
abnormalities (18).

When investigating for accommodative disorders, we need to answer 3 main


questions:
1 Plow much accommodation is present?
• Amplitude of accommodation: Push-up test, pull-away test, minus
lens test
2 How accurate is the accommodative response?
• Tests: Near retinoscopy (MEM or Nott’s), FCC/BCC, NRA/PRA
3 How flexible is the accommodative system?
• Tests: Monocular and binocular accommodative facility
Now we will describe the tests in more detail.
(dip (test
Purpose: A subjective measure of the amplitude of accommodation.
How to: A near target (20/30 equivalent) is brought towards the patient un­
der monocular conditions until he/she reports the first sustained blur of
the target. This distance is measured from the spectacle plane in cm.
The amplitude of accommodation in diopters is 100/cm.
Copyright 2014 by KMK Educational Services, LLC
328 8 .8 . ASSESSMENT OF ACCOMMODATIVE DISORDERS
Expected findings: The minimum and average amplitude of accommodation
for any age may be calculated using Hofstet ten’s formulas (21):
• Average amplitude of accommodation:
(8.5)
• Minimum amplitude of accommodation:
( 8 . 6)

The push-up method overestimates the amplitude of accommo­


dation compared to other methods because of relative distance
magnification; as the target is brought towards the eye, it sub­
tends a larger angle on the retina, making it more difficult for the
patient to detect blurring of the retinal image (21).

jPull-away test u s:■A; '. . d A •: a A"tefjyDp |


Purpose: A subjective measure of the amplitude of accommodation.
How to: A near target is held very close to the patient under monocular
conditions. The target is slowly pulled away from the eye until the patient
reports they can JUST read the letter. As in the push-up method, this
distance is measured from the spectacle plane and is recorded in cm.

The pull-away method is thought to minimize the variability in the


subjective interpretation of “first sustained blur” and avoids the
effect of relative distance magnification with the push-up test (6).

Expected findings: Direct comparison studies have found no significant dif­


ferences in the amplitude of accommodation measured with the pull-away
test compared to the push-up test (21).
Copyright 2014 by KMK Educational Services, LLC
CHAPTER 8, BINOCULAR VISION 329

Purpose: Subjective measurement of the amplitude of accommodation.


How to: The patient sits behind the phoropter viewing a near target at 40
cm under monocular conditions. Minus lenses are added until the patient
reports the first sustained blur of the target.

The amplitude of accommodation is the amount of minus


added over the patient’s prescription PLUS 2.SOD (to account for
the working distance of 40 cm).

Expected findings: The amplitude of accommodation measured with the mi­


nus lens test is 2.00D less than the amplitude obtained with the push-up
test. This underestimation is due to minification of the image as minus
lenses are added (as opposed to the increase in retinal distance magnifi­
cation with the push-up test).

To avoid the effects of minification with the minus lens test, the
near target may be placed at 33 cm, but 2.SOD is still used as
the working distance to determine the amplitude of accommoda­
tion (21).

In order for the patient to be comfortable, half of the accom­


modative amplitude should be kept in reserve.

Purpose: Determines the flexibility and endurance of the accommodative re­


sponse.
How to: As the patient views a near card at 40 cm, +2.00D/-2.00D lenses are
alternately introduced in front of the patient, forcing him/her to clear
the image. The clinician counts the number of cycles (clearing -f2.00D
and -2.00D lenses) the patient can successfully clear within GOseconds.
Copyright 2014 by KMI< Educational Services, LLC
330 8.3. ASSESSMENT OF A GCOMMODAFIVE DISORDERS
• The test is performed first under binocular conditions. If the patient
fails to clear the expected number of cycles, the test is performed
again under monocular conditions to differentiate a binocular ver~
gence issue from an accommodative problem.
Interpretation of results :
• Inadequate clearing of plus and minus lenses —>poor accommoda­
tive facility.
• Inadequate clearing of plus only under binocular and monocular
conditions over accommodation.
• Inadequate clearing of minus only under binocular and monocular
conditions —» deficient accommodation.
• Inadequate clearing of binocular plus or minus, normal clearing of
monocular plus or minus —» vergence issue.
Expected findings (ages 13-30): 8 cpm (binocular), 11 cpm (monocular).

Purpose: Objectively determines the accuracy of the accommodative response.


How to: The patient sits outside of the phoropter and is asked to read the
words or print on a near card attached to the retinoscope at eye level
with both eyes open. Using a vertical beam, the examiner observes the
motion of the reflex while scoping at the patient’s working distance or
Harmon’s distance. After estimating the degree of with or against
motion, the examiner quickly drops lenses in front of the eyes until the
motion is neutralized.•

Lenses should be held in front of the eye as briefly as possible in


order to avoid changing the patient’s accommodative response (21).

• If plus lenses neutralize the reflex, the patient has a lag of accom­
modation (they accommodate LESS than the stimulus demand).
* If minus lenses neutralize the reflex, the patient has a lead of ac­
commodation (they accommodate MORE than the stimulus de­
mand).

Copyright 2014 by KMK Educational Services, LLC


CHAPTER 8. BINOCULAR VISION 331

The near card chosen for MEM retinoscopy should be age appropri­
ate and the room illumination should be normal illumination used
for reading in order to avoid influencing the patient’s accommoda­
tive response (21).

Expected findings; H-0.25D to +0.50D (lag of accommodation)


• Plano, any minus, and > -f0.75D is considered an abnormal accom­
modative response.
• Note that because MEM is performed under binocular conditions,
an abnormal finding may reflect an accommodative OR a binocular
disorder.

Purpose: Subjectively determines the accuracy of the accommodative response.


FCC may also be used to determine the add for a presbyopic patient.
How to: The patient views a card with a set of horizontal and vertical lines
at 40 cm in the phoropter through JCC lenses with the axis set at 90
(red dots oriented vertically). The room illumination should be as dim as
possible while still allowing the patient to see the lines in order to obtain
the most accurate response.
• The horizontal lines should be sharper at the start of the test.
If the vertical lines are sharper, add minus or reduce illumination
until the horizontal lines are sharper.
• Once the horizontal lines are sharper, add plus lenses until the ver­
tical lines are sharper. Then reduce plus until the patient reports
the horizontal and vertical lines appear equally sharp.
Expected findings (non-presbyope): +0.25D to +0.75D lag of accommoda­
tion.

|NegativiEtftr clative accotnm odation/positivc yelativd accom­


m odation ( N R A / P R A )
Purpose: Determines the patient’s maximum ability to relax/stimulate ac­
commodation (NRA/PRA) while maintaining clear, single vision (thus
indirectly testing PFV/NFV). NRA/PRA testing may also be used to
determine a near add or to determine if the patient has been overmi-
nused in his/her prescription.
Copyright 2014 by KMK Educational Services, LLC
332 8.3. ASSESSMENT OF ACCOMMODATIVE DISORDERS
How to: The patient views a line of letters at 40 cm through the phoropter
under binocular conditions. For NRA testing, plus lenses are slowly
added in +0.25D steps until the patient reports the first sustained blur
or diplopia. For PRA testing, minus lenses are slowly added in -0.25D
steps until the same endpoint is reached. NRA should be performed
before PRA as PRA stimulates accommodation, which could alter the
accommodative response during NRA testing.

NRA/PRA indirectly tests PFV/NVF ranges.


• The addition of PLUS lenses (NRA) causes a relaxation of
the accommodative response with a corresponding decrease
in accommodative convergence. In order to keep the target
single, the patient must use PFV to converge the eyes back to
the target.
• The addition of MINUS lenses (PRA) causes an increase in
accommodation and accommodative convergence. In order to
keep the target single, the patient must use NVF to diverge
the eyes back to the target.

The endpoint in NRA/PRA testing represents the maximum amount of


relaxation/stimulation of accommodation OR the limits of PFV/NFV
ranges.
• If NRA/PRA results are abnormal under binocular AND monocular
conditions, the accommodative system was the limiting factor.
• If NRA/PRA results are abnormal with binocular testing but NOR­
MAL with monocular testing, the PFV/NFV ranges were the lim­
iting factor.

On NRA testing, a patient reaches the endpoint at +1.50D un­


der binocular conditions and +2.SOD under monocular conditions.
What is the cause of the low binocular NRA? Because the NRA
is normal under monocular conditions, we know the patient is ca­
pable of maximal relaxation of accommodation. The endpoint un­
der binocular conditions was therefore limited by low PFV ranges,
causing the target to become diplopic.

Expected findings: +2.50D NRA/-2.50D PRA (NRA/PRA should be bal­


anced)
Copyright 2014 by KMK Educational Services, LLC
CHAPTER 8. BINOCULAR VISION 333

• An NRA > +2.SOD indicates the patient is overminused in his/her


prescription.
• Note that although the PRA may be > -2. SOD, there is no reason
to continue testing beyond this point as the NRA/PRA ranges are
balanced.

Minimum ranges for BY and accommodative testing (21)


Measurement Range
Distance phoria 1 eso - 3 exo
Near phoria ortho: - 6 exo
AC/ A ratio 4/1
Distance Smooth 1.11 ranges X/7/4
Distance Smooth BO ranges 9/19/10
inear oiiiuotu oi ranges 1l tQ
)-| « xj/ ll o9
Near Smooth BO ranges 17/21/11
Vergence facility 15 cpm
: Near point of convergence : 5 cm break/7 cm recovery
Monocular accomm facility 11 cpm (13 - 3Q years of age)
Binocular accomm facility 8 cpm (13 - 30 years of age)
Amp of accommodation (mini­ 15 - 0.25(age)
mum)
Amp of. accommodation (aver- 18 - 0.33(age)
age)
Lag of accommodation -0.25D to -| 0.751)
(MEM/FCC)
NRA/PRA +2.50D/-2.50D

- SECTION 8.4 -----------------------------------------------------------------------------------------

Common non-strabism ic binocular vision and


accom m odative disorders
•• •
Asthenopia is a group of non-specific symptoms including blurred vision, in­
termittent diplopia, eye pain, eye strain or fatigue, words moving/swimming
on the page, and headaches that occur during or after the completion of vi­
sual tasks. Any of the following binocular vision or accommodative disorders
discussed below may be characterized by asthenopia.

Copyright 2014 by KMK Educational Services, LLC


84- COMMON NON-STRABISMIC BV / ACCOMMODATIVE
334 DISORDERS
Patients with convergence and accommodative disorders will be
most symptomatic at near, especially after periods of prolonged
near work. If patients with signs of binocular vision disorders DO
NOT report symptoms, ask about avoidance of near tasks.

Remember that patients may have normal binocular and accommoda­


tive testing with significant symptoms, while others may have abnormal
testing and no symptoms. After making the appropriate diagnosis, the goal
of therapy focuses on relieving symptoms rather than bringing the results of
testing into the normal range.

Convergence insufficiency is the most common non-accommodative binocular


vision disorder and affects 3-5% of the population (16).
Symptoms: Eye strain, headaches, blurred vision, diplopia, poor concentre
tion, poor reading comprehension, sleepiness, movement of print, and
a pulling sensation of the eyes. Symptoms are usually longstanding in
nature (21).
Signs: Larger exophoria at near than distance, low AC/A, receded
NPC, reduced positive fusional vergence ranges, and a low lag
or lead of accommodation.
• Because of reduced PFV ranges, patients will also have a low NRA
and will NOT be able to clear plus lenses on binocular accommoda­
tive facility testing (monocular accommodative facility testing will
be NORMAL).

Remember that plus lenses relax accommodation and convergence.


In order to maintain single vision, the patient must use PFV to
converge the eyes back to the target.

Pseudo-convergence insufficiency is an important differential diagnosis for


true Cl. Pseudo-CI is characterised by accommodative insufficiency with
borderline PFV ranges and near exophoria. The decrease in the accommodative
response leads to a decrease in convergence. Unlike true Cl, a patient with
pseudo-CI will have decreased amplitude of accommodation, low PRA,
and will respond well to low plus at near (+0.75D to +1.00D) (21).
Copyright 2014 by KMK Educational Services, LLC
CHAPTER 8. BINOCULAR VISION 335

Example 8.6: You suspect a 21 yo has Cl due to an 8A exophoria at near and a receded
NPC, but a +2.00D lens at near improves the NPC and the patient also has a reduced
amplitude of accommodation. What is the diagnosis?

Solution 8.6: Pseudo-CI. The patient has reduced accommodation, leading to reduced
convergence with a moderate exophoria at near and reduced NPC that will improve with
the addition of low plus at near.

If patients present with sudden symptoms characteristic of Cl,


multiple sclerosis and myasthenia gravis must be considered as dif­
ferentials as they often present with similar symptoms. Remember
that patients with true Cl often have longstanding symptoms.

Additional differentials for Cl include basic exophoria and divergence excess.

Divergence insufficiency is the least common non-strabismic binocular vision


disorder.
Symptoms: Intermittent diplopia at distance that is worse at the end of the
day, headaches, fatigue, blurred vision, difficulty focusing from far to
near, light sensitivity, motion sickness, and nausea (22).
Signs: Greater esophoria at distance than near, low AC/A, and re­
duced NFV at distance (21) (11).

An important differential for DI is a CN VI palsy. Both con­


ditions will be characterized by sudden onset diplopia at distance
with a large eso deviation. DI is characterized by a large comitant
esophoria, while a CN VI palsy will be seen as a non-comitant
eso deviation.

Additional differentials for DI include basic esophoria, convergence excess, and


divergence palsy.
Copyright 203.4 by KMK Educational Services, LLC
8.1 COMMON NON-STRABISMIC BV / ACCOMMODATIVE
336 DISORDERS
■.VSPV Zf.-v

Although Cl is the most common non-strabismic binocular vision disorder, CE


often causes greater symptoms (13).
Symptoms: Headaches, eye strain, blurred vision, diplopia, poor concentra­
tion, and poor reading comprehension. Patients may be asymptomatic if
they suppress an eye or avoid near tasks. Patients may also hold reading
material closer than normal.
Signs: G reater esophoria at near than at distance, reduced NFV ranges,
high AC/A, and a larger lag of accommodation. CE may be accom­
panied by accommodative excess or latent hyperopia.
• Because of reduced NFV ranges, patients will also have a low PRA
and an inability to clear minus lenses with binocular accommoda­
tive facility testing (monocular accommodative facility will be NOR­
MAL unless the patient has an accompanying accommodative ex­
cess) .

Remember that minus lenses stimulate accommodation and 'con­


vergence. The patient must use NFV to diverge the eyes back to
the target.

CE must be differentiated from pathologic causes of accommodative/convergence


spasms such as:
1 Ocular inflammation: Uveitis, scleritis
2 CNS disease: Tertiary syphilis, sympathetic paralysis
3 Pharmacological: Parasympathomimetic drugs (physostigmine, pilocarpine),
high doses of J3i, sulfonamides (21).
Additional differentials include basic esophoria, divergence insufficiency, and
accommodative excess.

Symptoms: The most common symptoms are cosmetic concerns because an


eye turns out or the patient covers an eye in bright light. The most
common visual symptom is diplopia (21) (20).
Copyright 2014 by KMK Educational Services, LLC
CHAPTER 8. BINOCULAR VISION 337

Signs: Greater exophoria at distance than near that becomes more pro­
nounced with fatigue (may lead to an interm ittent exotropia), and a
high AC/A. PFV ranges at distance and near are usually normal. The
patient may have a V pattern exo deviation.

Intermittent exotropia with DE rarely leads to symptoms of


diplopia, suggesting patients suppress an eye or develop anomar-
lous retinal correspondence to avoid diplopia and confusion.

Differential diagnoses include basic exophoria and convergence insufficiency.

Symptoms: Eye strain, headaches, blurred vision at distance and near, diplopia
at distance and near, poor concentration, and poor reading comprehen­
sion.
Signs: Exophoria that is equal at distance and near, normal AC/A, reduced
PPV ranges at distance and near, reduced NRA, low lag or lead
of accommodation, inability to fuse BO with vergence facility testing,
and inability to clear plus lenses with binocular accommodative facility
testing.
Differential diagnoses include divergence excess or convergence insufficiency.

P ^ sh c e so p h o riA ^ ,>•. 7 -A:/---: .

Symptoms: Eye strain, headaches, blurred vision at distance and near, diplopia
at distance and near, poor concentration, poor reading comprehension,
and avoidance of near work.
Signs: Esophoria that is equal at distance and near, normal AC/A, reduced
NVF ranges at distance and near, reduced PRA, high lag of accom­
modation, inability to fuse BI with vergence facility testing, and inability
to clear minus lenses with binocular accommodative facility testing.
Differential diagnoses include divergence insufficiency, convergence excess, CN
VI palsy, and divergence paralysis.
Copyright 2014 by I<MK Educational Services, LLC
8.1 COMMON NON-STRABISMIC BV / ACCOMMODATIVE
338 DISORDERS

Symptoms: Blurred vision at distance and near, headaches, diplopia, losing


place or skipping lines when reading, poor concentration, motion
sickness, nausea, and sleepiness (21).
Signs: Small hyperphoria at distance and near, head tilt, reduced PPV
and NFV ranges, and reduced vergence facility with BO and BI. Vertical
vergences may be constricted (if recent onset vertical deviation) or larger
than normal (if the vertical deviation is longstanding).
* Note that vertical dysfunction may contribute to the development of
horizontal non-strabismic binocular vision disorders. Treating the
vertical disorder first will often result in resolution of the horizontal
binocular disorder(s).

A patient with a recent onset, large, non-comitant vertical de­


viation should be investigated for underlying systemic patholo­
gies (21).

Fiisional vergence dy sfunction (FV13) V.■/.


Fusional vergence dysfunction is the third most common non-accommodative
binocular vision disorder (after Cl and CE) (21).
Symptoms: Eye strain, headaches, blurred vision, diplopia, poor concentre
tion, and poor reading comprehension.
Signs: Normal phoria measurement at distance and near, normal AC/A, nor­
mal accommodative function, and reduced PFV and NFV ranges at
distance and near.

An important differential diagnosis for FVD is accommodative infa­


cility. Patients with FVD will have abnormal results with binocular
accommodative facility testing, but NORMAL results with monoc­
ular testing. In contrast, patients with accommodative infacility
will fail binocular AND monocular facility testing.

Copyright 2014 by KMK Educational Services, LLC


CHAPTER 8. BINOCULAR VISION 339

ic ro in i
Symptoms: Blurred vision, headaches, eye strain, poor reading ability and
comprehension, movement of print at near, and pulling sensation around
the eyes.
Signs: Reduced amplitude of accommodation (AoA), reduced PRA,
high lag of accommodation on MEM and/or FCC, and inability to clear
minus lenses with binocular AND monocular accommodative facility
testing.
Differential diagnoses include accommodative excess, accommodative infacil­
ity, pseudo Cl, basic exophoria.

The following conditions are thought to be subsets within accom­


modative insufficiency:
• Ill-sustained accommodation - normal accommodative
testing that fatigues with repetition.
• Accommodative paralysis ~ pathological or pharmacolog­
ical causes for reduced accommodation.

: '-'i ^ - :

Symptoms: Eye strain, headaches, interm ittent distance blur after near
activities, difficulty shifting focus from far to near, and photophobia.
Signs: Normal to high AoA, reduced NRA, low lag or lead of accommo­
dation on MEM/FCC, and inability to clear plus lenses with binocular
AND monocular accommodative facility testing. Patients may also have
pseudo-myopia.

A cycloplegic refraction should be performed if pseudo-myopia


is suspected due to accommodative excess or convergence insuffi­
ciency.

Differential diagnoses include accommodative infacility, accommodative in­


sufficiency, basic esophoria, convergence excess, and accommodative
spasm.
Copyright 2014 by KMK Educational Services, LLC
340 8.5. SENSORY ANOMALIES OF BV/STRABISMUS

Accommodative excess is when the patient over-accommodates


in response to any stimulus to accommodation. Accommodative
spasm is usually a result of fatigue due to over-stimulation of the
accommodative system. Plus lenses will reduce symptoms of ac­
commodative spasm but will NOT work well for accommodative
excess.

Symptoms: Eye strain, headaches, blurred vision when shifting focus from
distance to near, pulling sensation around the eyes, movement of print,
and poor reading ability and comprehension.
Signs: Reduced NRA and PRA and difficulty clearing plus and minus
lenses on binocular AND monocular accommodative facility testing. The
AoA and MEM/FCC accommodative response may be normal or abnor­
mal in either direction, depending on the degree and direction of accom­
modative inflexibility.
Differential diagnoses include accommodative insufficiency, accommodative
excess, basic esophoria, convergence excess.

- SECTION 8.5 -----------------------------------------------------------------------------------------

Sensory Anomalies of Binocular V ision/Strabism us


Remember that in order for an object to be perceived as single and in depth, the
retinal image in each eye must be clear, the same size, and fall on corresponding
retinal points. If the retinal images appear significantly different or are localized
in different retinal areas, the visual cortex will be unable to fuse the two retinal
images into a single percept, resulting in confusion and diplopia. Suppression
(with resulting amblyopia), eccentric fixation, and anomalous retinal
correspondence are mechanisms employed by the visual cortex to avoid the
effects of poor sensory fusion (10).

iA itib ly o p in

Amblyopia is defined as a reduction in vision (20/30 or worse) in one eye or a


difference of 2 or more lines of visual acuity between the two eyes that is not
correctable with refraction and is not explained by structural abnormalities or
pathology. Amblyopia occurs at the level of the visual cortex (21) (19).
Copyright 2014 by KMK Educational Services, LLC
CHAPTER 8. BINOCULAR VISION
Copyright 2014 by KMK Educational

Condition CT A C/A NPC Vergence Amplitude


Basic XP XP D—N wnl wnl low BO D/N
Basic EP EP D=N wnl wnl low BI D/N
Cl XP at N low receded low BO
DI EP at D low wnl low BI at D
CE - EP at N high wnl low BI
DE XP at D high wnl low BO at D, low BI at N
Accom Insufficiency wnl receded BO blur at N may be low
111 sustained accom wnl wnl BO blur at N may be low
Accom Excess wnl wnl BI blur at N may be low
Accom Infacility wnl wnl BO, BI at N may be low
® Table 8.1: Non-strabismic binocular disorder characteristics adapted from (8) (21)
t-1
fo

CO
tfs-
8.5. SENSORY ANOMALIES OF BV/STRABISMUS

Copyright 2014 by KMK Educational Services, LLC


Condition N R A /PR A Accom Amp Accom Facility MEM
Basic XP low NRA wnl bino fails +, mono wnl low
Basic EP low PRA wnl bino fails mono wnl high
Cl low NRA wnl bino fails +, mono wnl low
DI wnl wnl wnl wnl
CE low PRA wnl bino fails mono wnl high
DE wnl wnl wnl wnl
Accom Insufficiency low PRA low fails - high
111 sustained accom low PRA wnl fails - high
Accom Excess low NRA wnl fails + low
Accom Infacility low NRA/PRA wnl fails +/- wnl
Table 8.2: Non-strabismic binocular disorder accommodative characteristics adapted from (8) (21)

05
CO
CHAPTER 8. BINOCULAR VISION 343

• Animal studies have shown a loss of binocular cells in the visual cortex
in amblyopia. Remember that binocular neurons develop in response to
competition between the two eyes. If one eye has a blurred or distorted
image compared to the fellow eye, it will not send as many signals to the
cortex, causing the binocular cell to become a monocular cell driven by
the fellow “good” eye.
• Over time, the eye with the blurred image will have fewer cortical con­
nections compared to the “good” eye with a resulting decrease in visual
acuity (see psychology chapter for further details) (24).

Amblyopia will only develop from dissimilar retinal images that oc­
cur during the critical period of development of the visual cortex
- from birth to 7-9 years of age. The first 2-3 years of life are the
MOST sensitive for development of the visual cortex. Any decrease
in vision in one eye compared to the other that occurs outside of
the critical period WILL NOT result in amblyopia (21) (24).

The plastic period describes the time frame when amblyopia can be success­
fully treated. Although the visual system is most amenable to treatment during
childhood and the teen years, studies have shown the visual cortex retains some
degree of plasticity throughout adulthood (21).

Patients with amblyopia often experience a crowding phe­


nomenon, characterized by difficulty distinguishing between let­
ters or words that are close together. Visual acuity will be better
when a single letter is isolated on the chart compared to when an
entire line or several rows of letters are presented.

The degree of amblyopia is classified as follows (varies between studies) (27):


• Moderate: 20/40 - 20/80
• Severe (deep): 20/100 - 20/400
There are three main types of amblyopia that will be discussed below: refrac­
tive, form deprivation (occlusion) and strabismic ambylopia. An amblyo-
genic factor MUST be present in order to make a diagnosis of amblyopia.
Copyright 2014 by KMI< Educational Services, LLC
344 8.5. SENSORY ANOMALIES OF BY/STRABISMUS

Form deprivation, or occlusion amblyopia, occurs when there is an obstruc­


tion that blocks the formation of a clear retinal image in one eye. Common
causes of obstruction include congenital cataracts, ptosis, and corneal opacities
(non-exhaustive list). Early diagnosis during the sensitive period is critical so
the obstruction can be medically/surgically treated to avoid the development
of severe amblyopia.

Refractive amblyopia occurs when there is a large amount of uncorrected


refractive error in one or both eyes, leading to a blurred retinal image(s) and
underdevelopment of connections between the eye(s) and the visual cortex.
Refractive amblyopia includes (19):
Anisometropic amblyopia: Due to a significant difference in uncorrected
refractive error between the two eyes. The risk of developing amblyopia
increases with greater degrees of anisometropia.
• When there is a large difference in uncorrected refractive error be­
tween the two eyes, one eye has a clearer retinal image than the
fellow eye. The eye with the blurred retinal image will develop less
cortical connections compared to the eye with the clearer retinal
image, resulting in amblyopia.

Anisometropic hyperopia is more likely to lead to amblyopia


compared to anisometropic myopia. In anisometropic myopia, each
eye will have a clear retinal image at some distance, reducing the
risk of developing amblyopia (although stereopsis will still be de­
creased as the eyes never learn to work together). In contrast, in
anisometropic hyperopia, the eyes will accommodate the smallest
amount possible in order to obtain a clear image, resulting in the
less hyperopic eye having a clear retinal image while the more
hyperopic eye ALWAYS has a blurred retinal image.

Isometropic amblyopia: Due to a high refractive error (myopia/hyperopia/astigmatism)


in BOTH eyes. Because the refractive error is so high, neither eye receives
a clear retinal image, resulting in poor connections between each eye and
the visual cortex.
Copyright 2014 by KMK Educational Services, LLG
CHAPTER 8. BINOCULAR VISION 345

Meridional amblyopia is characterized by reduced acuity for stimuli in a


certain orientation and is due to uncorrected astigmatism during the sen-
sitive period. The reduced acuity persists even after correction of the
astigmatism.

• Uncorrected astigmatism results in two lines images near the retina.


The line image furthest away from the retina will be blurred com­
pared to the line image close to the retina, resulting in decreased
cortical connections for stimuli with the same orientation as the line
image furthest from the retina.

Potentially Amblyogenic Refractive Errors (25)


Ametropia Anisometropia Isornetropia
Myopia > 3.00 D > 8.00 D
Hyperopia > 1.00 D > 5.00 D
Astigmatism > 1.50 D > 2.50 D

Strabismic amblyopia occurs when there is a binocular misalignment of


the visual axes of the eyes that is larger than Panum’s fusional area, resulting
in diplopia and confusion and eventual suppression of the retinal image
of the deviated eye. Strabismic amblyopia will only occur if the strabismus
is constant and unilateral. An alternating strabismus DOES NOT result in
amblyopia because each eye is used for fixation at some point time (although
stereopsis will be reduced because both eyes are never viewing the object at
the same time). Strabismus is also known as a tropia or “squint.”

• Suppression occurs when the image of one eye is filtered out at the level
of the visual cortex.•

• If suppression occurs long enough during the sensitive period, the patient
will have reduced visual acuity in the strabismic eye (amblyopia).

Copyright 2014 by KMK Educational Services, LLC


346 8.5. SENSORY ANOMALIES OF BV/STRABISMUS

• Diplopia occurs when an object falls on non-corresponding


retinal points (because the fovea of one eye is deviated), caus­
ing the object to appear in two different visual directions.
Suppression in the peripheral retina eliminates diplopia.
• Confusion occurs when each macula views a different object
(again because one eye is deviated compared to the other),
causing the dissimilar objects to appear in the same visual
direction. Suppression at the fovea eliminates confusion (3).

Although suppression may occur in response to strabismus, there are other sen­
sory adaptations the visual cortex may employ to avoid diplopia and confusion:
eccentric fixation and anomalous retinal correspondence.
Eccentric fixation
Eccentric fixation occurs when a non-foveal point is used for fixation in the
strabismic eye. It occurs under monocular AND binocular conditions, but is
best diagnosed under monocular conditions.
• Eccentric fixation will only develop in an eye with a stable, constant,
unilateral, micro-strabismus of longstanding duration during the devel­
opmental sensitive period.

Esotropes will develop a nasal eccentric fixation point and ex­


otropes will develop a temporal eccentric fixation point. Para­
doxical eccentric fixation is when the point used for fixation
is OPPOSITE to the direction of the deviation (e.g. an esotrope
develops a temporal eccentric fixation point).

• Because a non-foveal point is used for fixation, visual acuity will be re­
duced. The further away the fixation point is from the fovea, the worse
the visual acuity will be.•
• Note that the objective measurement of the deviation on cover test will
be less than the true objective deviation by the amount of the eccentric
fixation. The patient will fixate with the eccentric fixation point rather
than going to the fovea for fixation.

Copyright 2014 by KMK Educational Services, LLC


CHAPTER 8. BINOCULAR VISION 347

Eccentric viewing occurs in older patients that have lost visual


acuity due to macular disease. Under monocular conditions, the
patient will first move to the fovea for fixation before moving to
the eccentric viewing point to obtain better visual acuity.

Anomalous retinal correspondence (ARC)


Normal retinal correspondence (NRC) occurs when the fovea in each eye
has the same visual direction. If the fovea of one eye is deviated compared to the
fellow eye due to strabismus, diplopia and confusion will occur. If the foveal
misalignment develops before the age of 5, the visual cortex will respond by
developing anomalous retinal correspondence to eliminate diplopia and
confusion.
In anomalous retinal correspondence, the non-foveal point (fn) in the
deviated eye that is viewing an object of interest becomes “linked” to
the fovea (f) of the fellow eye. The new anomalous correspondence en­
sures that the object will be perceived in the same direction by each eye,
eliminating diplopia and confusion.

Similar to eccentric fixation, esotropes will have a nasal f n and


exotropes will have a tem poral f n in relation to f of the deviated
eye.

Anomalous retinal correspondence will only occur under binocular condi­


tions. Under monocular conditions, the deviated eye will use the fovea
to fixate the target (unless eccentric fixation is present).

The angle of anomaly is the difference between the objective and


subjective angle of deviation (aka tropia).

Harmonious ARC is when the angle of anomaly equals the objective angle
of deviation. In other words, f n of the deviated eye corresponds with f of
the fellow eye so that the subjective angle of deviation is 0. The patient
WILL NOT have symptoms of diplopia or confusion. Harmonious ARC
is the most common type of ARC.
Copyright 2014 by KMK Educational Services, LLC
348 8.5. SENSORY ANOMALIES OF BV/STRABISMUS
• Note that in harmonious ARC, f n of the deviated eye is constantly
changing depending on where the patient is looking.
• At any point in time, whatever point of the retina that is hit by an
object will become f n and will be linked to f of the fellow eye under
binocular conditions.
Unharmonious ARC is when the angle of anomaly is less than the objective
angle of deviation. In other words, a point (/4) in between f and f n
(where the object hits the retina) of the strabismic eye becomes linked to
the fovea of the fellow eye. The subjective angle is NOT equal to 0, and
the patient WILL have diplopia and confusion (although less than with
NRC) because there is not perfect correspondence between the two eyes.
• Unharmonious ARC usually occurs for 2-3 weeks after strabismus
surgery as the visual cortex transitions to harmonious ARC,
Paradoxical ARC is when f n moves in the direction OPPOSITE to the de­
viation, causing the subjective and objective angles of deviation to be
in opposite directions. Patients will have worse diplopia and confusion
than if they had NR.C.
• Paradoxical ARC occurs in an esotrope when / n is TEMPORAL,
rather than nasal, to f in the deviated eye.
• Paradoxical ARC occurs in an exotrope when f n is NASAL, rather
than temporal, to f in the deviated eye.

Covariance describes a phenomenon where the type of correspon­


dence shifts depending on which eye is fixating. In most patients
with covariance, harmonious ARC is used when the normally fix­
ating eye is fixating, and NRC is used when the strabismic eye is
fixating.

Harmonious ARC: angle of anomaly = objective angle of devi­


ation =4* No diplopia
Unharmonious ARC: angle of anomaly < objective angle of de­
viation =£> Diplopia present but less than with NRC
Paradoxical ARC: angle of anomaly > objective angle of devia­
tion => Greater diplopia than with NRC

Copyright 2014 by KMK Educational Services, LLC


CHAPTER 8. BINOCULAR VISION 349

Exam ple 8.7: A p a tie n t has 1 2 ^ esotro pia on o b je c tiv e cover te s t a n d o rtho ph oria on
sub jective M a d d o x ro d testing. W h a t ty p e o f a n o m a lo u s correspondence does th e p a tie n t
have?

Solution 8.7: T h e p a tie n t has h arm o n io u s A R C . T h e esotropia m easu red w ith cover
te s t is o ffset by th e chan ge in co rresp on din g re tin a l p oints, re fle c te d in th e p a tie n t's
o rth o p h o ric su b jective response to th e M a d d o x rod.

Exam ple 8.8: A p a tie n t has 12A esotro pia on o b je c tiv e cover te s t a n d SA esotropia on
sub jective M a d d o x ro d testing . W h a t ty p e o f a n o m a lo u s correspondence does th e p a tie n t
have?

Solution 8.8: T h e p a tie n t has u n h arm o n io u s A R C b ecause the su b jective d eviatio n is n ot


e q u a l to zero ( 3 ).

jStcps.iri assessment of sensory anomalies in bihoculai>yi-


sion
Tests for ocular deviations and sensory anomalies include the following:
Ocular alignment: Hirschberg/Krimsky test, major amblyoscope, Bruckner
test, 4 BO test
Eccentric fixation: Visuoscopy, Haidinger’s brush, Maxwell’s spot
Anomalous retinal correspondence: After image test, Bagolini lenses
Sensory status: Worth 4 dot, stereopsis
Now we will describe the tests in more detail.
jH irschb^g/K rim .; y■A/A . A."A :.v-A.; vvAh- ' ; k •
Purpose: The Hirschberg and Krimslcy tests allow for the gross determina­
tion of ocular misalignment.
How to for Hirschberg: The patient views a point light source held at 50
cm on his/her midline. The examiner observes the corneal reflex of
each eye compared to the pupillary axis.
Copyright 2014 by KMK Educational Services, LLC
350 8.5. SENSORY ANOMALIES OF BV/STRABISMUS
• Nasal displacement of the corneal reflex in one eye compared to
the other indicates an exotropia.
• Temporal displacement of the corneal reflex indicates an es­
otropia.
• Upwards displacement of the corneal reflex indicates a hypotropia.
• Downwards displacement of the corneal reflex indicates a hy-
pertropia.
To obtain the best estimate of ocular misalignment, the examiner should mea­
sure angle lambda, the angle between the pupillary axis (through the center
of the pupil) and the line of sight (through the fovea). Angle lambda is mea­
sured under monocular conditions and is 0.5 mm nasal (2-3 degrees) in
each eye.
• If the corneal reflex is displaced temporally (indicating esotropia), angle
lambda must be ADDED to the gross eso deviation to obtain the true
deviation.
• If the corneal reflex is displaced nasally (indicating exotropia), angle
lambda must be SUBTRACTED to the gross exo deviation to obtain
the true deviation.

A 1 mm shift of the corneal light reflex corresponds to 22A (19).

Additional angles (under monocular conditions) include:


• Angle kappa: The angle between the pupillary axis and the
visual axis (line passing from the fovea through the nodal
point of the eye). In most cases, angle kappa closely approx­
imates angle lambda.
• Angle alpha: The angle between the pupillary axis and
the optical axis (line passing through the nodal point that is
normal to the cornea).
• Angle gamma: The angle between the optical axis and the
fixation axis (line extending from the fixation point through
the center of rotation of the eye).
Angle lambda is the only angle we can clinically measure (24).

Copyright 2014 by KMK Educational Services, LLC


CHAPTER 8. BINOCULAR VISION 351

How to for Krimsky test: The Krimsky test measures the magnitude of the
deviation using prism. The patient views a point light source on his/her
midline at 50 cm.
• If the corneal reflex is displaced nasally in one eye, add BI prism in
front of the normal eye until the corneal reflex in the deviated eye
is in the same position as the original position of the reflex in the
normal eye.
• If the corneal reflex is displaced temporally in one eye, add BO
prism until the same endpoint is reached as described above.
• If the corneal reflex is displaced upwards in one eye, add BD prism
in front of the normal eye until the same endpoint is reached.
• If the corneal reflex is displaced downwards in one eye, add BU in
front of the normal eye until the same endpoint is reached.
Note the examiner does NOT have to measure angle lambda prior to the
Krimsky test as it is already taken into account when prism is used to
bring the deviated eye’s corneal reflex in the same position as the normal
eye’s corneal reflex.

• • ' .5; • /
, :• .

Purpose: To detect strabismus, anisometropia, and/or media opacities in in­


fants.
How to: The examiner uses a direct ophthalmoscope 80-100 cm away from the
patient on his/her midline to view the red reflexes of the two eyes as the
patient views the light. The brightness of the red reflexes are compared
to each other.
• Equal red reflexes between the two eyes indicates the patient is
binocularly viewing the light.
• If the red reflexes are unequal, the darker reflex corresponds to the
eye with the media opacity, the higher uncorrected refractive error,
or the fixating eye if the patient has strabismus (the deviated eye
will have a brighter reflex).

Purpose: To detect a small central suppression scotoma secondary to a mx-


crostrabismus. Patients often have mildly reduced visual acuity with­
out an apparent underlying ocular etiology.

Copyright 2014 by KMK Educational Services, LLC


352 8.5. SENSORY ANOMALIES OF B V/STRABISMUS

A microstrabismus is defined as a deviation of less than 10A that


is not visible on cover test or other standard tests of ocular mis­
alignment (24).

How to: As the patient views a distant target, a loose 4 BO prism is intro­
duced in front of one eye while the examiner observes the movement of
both eyes.
Result interpretation (assuming 4 BO is placed in front of OD):
* If OS makes an outward movement and then re-fixates on the target
No suppression of OD or OS.
* If OS makes an outward movement but DOES NOT re-fixate ==>
Suppression of OS.
- As 4 BO is placed in front of OD, OD will move in and OS will
move out due to Hering’s law (yoked eye movements). However,
because the image still falls within the suppression scotoma, the
small movement goes undetected and OS will NOT re-fixate.
* If OS does NOT make an outward movement or re-fixate Sup­
pression of OD.
—When 4 BO is placed in front of the suppressing eye (OD),
the image moves such a small amount that it still falls within
the suppression scotoma. Because OD cannot detect the image
movement, it will not move when the lens is placed in front of
the eye, and therefore OS will not move.

Purpose: To detect eccentric fixation.


How to: The patient is asked to look in the center of the grid target in the
ophthalmoscope under monocular conditions. The examiner views the
location of the foveal light reflex (FLR) in relation to the center of the
grid.
Result interpretation :
* If the FLR is centered within the grid, there is no eccentric fix­
ation.
* If the center of the grid is superior to the FLR: superior eccentric
fixation.
* If the center of the grid is inferior to the FLR: inferior eccentric
fixation.
Copyright 2014 by KMK Educational Services, LLC
CHAPTER 8. BINOCULAR VISION 353

• If the center of the grid is tem poral to the FLR: temporal eccen­
tric fixation.
• If the center of the grid is nasal to the FLR: nasal eccentric
fixation.
Note that visual acuity decreases as the degree of eccentric fixation increases.

Additional tests for determining eccentric fixation include


Haidinger’s brush, Maxwell’s spot, and monocular
Hirschberg test.

Purpose: To determine the type of retinal correspondence used by the patient.


How to: The fixating eye is flashed with a horizontal line image and the devi­
ated eye is flashed with a vertical line image. Because each eye is flashed
monocularly, each fovea will have an after image. The patient reports the
relative positions of the horizontal and vertical line images on a screen.
Results interpretation: After images are a cortical phenomenon. The
location of the images is independent of the positions of the eyes once the
after images have been created (24).
• If the patient has NRC, he/she will see a perfect cross. The fovea
of each eye will have the same visual direction so the patient will
see both lines in the same location (regardless of the direction of the
ocular misalignment).•

The patient will also see a perfect cross if the patient has eccen­
tric fixation, and e is in the same location as f n under binocular
conditions with ARC (eccentric fixation = objective angle of devia­
tion) . E will pick up the after image (under monocular conditions)
and will project to the same location as the after image of f in the
fellow eye.

• If the patient has ARC, the horizontal and vertical line images will
be misaligned.
Copyright 2014 by KMK Educational Services, LLC
354 8.5. SENSORY ANOMALIES OF BV/STRABISMUS
— Ifthe patient has an OD esotropia with a nasal f n relative to
the fovea, under binocular conditions the vertical line image will
appear displaced towards the left of center of the horizontal
line.
—If the patient has an OD exotropia with a temporal f n rel­
ative to the fovea, under binocular conditions the vertical line
image will appear displaced towards the right of center of the
horizontal line image.

Bagolini lenses are the most sensitive vest for retinal correspondence. They
are piano, clear, striated lenses that produce a line image 90 degrees from the
orientation of the striations when the patient views a point light source (similar
to Maddox rod). Bagolini lenses may also be used to detect suppression.
How to: The lenses are oriented so the right eye sees a line oriented as / (stri­
ations at 45 degrees) and the left eye sees a line oriented as \ (striations
oriented at 135 degrees). The patient reports the relative positions of the
lines.

Interpretation of Bagolini lenses (as seen by the patient):


/ = suppression OS
\ — suppression OD
X = NRC if cover test shows no tropia
X = ARC if cover test shows tropia
V = Esotropia with NRC
A = Exotropia with NRC

Key to interpretation :
• If the patient has an OD esotropia with NRC, the line image will
hit a point nasal to the fovea, resulting in the image being seen to
the right of the left eye’s image (uncrossed diplopia) => V.
• If the patient has an OD exotropia with NRC, the line image will
hit a point temporal to the fovea, resulting in the image being seen
to the left of the left eye’s image (crossed diplopia) => A.
• If patient has an eye deviation with ARC, the line image will hit f n
in the deviated eye. Because f n is linked to f of the fellow normal
eye, the two line images will be seen in the same direction => X
(with tropia on CT).
Copyright 2014 by KMK Educational Services, LLC
CHAPTER 8. BINOCULAR VISION 355

There are four levels of sensory fusion per Worth:


Zero-degree: No fusion, monocular/suppression.
First-degree (superimposition targets): Testing uses two very
dissimilar targets. Patients will have diplopia as the targets
are difficult to suppress; used in anti-suppression therapy.
Second-degree (flat fusion targets): Testing uses identical tar­
gets with suppression checks. Patients do NOT have diplopia.
The image is single but is NOT in stereo. Requires motor fu­
sion.
Third-degree: Ultimate sensory fusion characterized by stere-
opsis. Requires motor and sensory fusion.

Purpose: Worth 4 dot detects suppression and flat fusion ability. Testing is
indicated if stereopsis is below 40 seconds of arc.
How to: The patient wears red/green glasses (red over OD) while viewing a
flashlight with four dots (1 white, 1 red, and 2 green) at distance and
near. The patient reports the number and color of the lights he/she sees.
• OD will see two vertical red lights.
• OS will see three green lights.
Interpretation: Based on the subjective response of the patient.
• If two red dots are seen => OS suppression.
• If three green dots are seen => OD suppression.
• If four dots are seen Flat fusion without suppression.
• If five dots are seen =>- Diplopia.

Patients who suppress only at distance and only with the room
lights dim have a shallow and small suppression scotoma. If
the patient suppresses at near and with the room lights on, he/she
has a large and deep suppression scotoma.

Copyright 2014 by KMK Educational Services, LLC


356 8.6. STRABISMIC DISORDERS

There are two types of targets used to assess stereopsis: 1) Contour (local) and
2) Global (stereo) targets.

The patient’s habitual glasses should always be worn under the


stereo glasses because blur can negatively affect stereopsis.

Contour testing uses laterally displaced targets with monocular cues. Pa­
tients without stereopsis will be able to guess correctly. Contour testing
is better at detecting peripheral stereopsis (>60 seconds of arc).
• Examples include W irt circles, Titmus fly, animals.
Global testing uses random dot targets that have NO monocular clues
(the patient must have bifoveal fixation to detect shapes within random
dot targets). These targets are good for detecting constant strabismus.

Stereopsis can also be evaluated using anaglyphs and Polaroid tar­


gets (21).

Expected results: 20 seconds of arc with contour testing and an appreciation


for gross random dot targets (21),

Horror fusionis describes when patients with a heterotropia are


unable to obtain fusion even with the use of prism. As the images
are brought closer together with prism, they eventually jump over
each other rather than fusing.

I— SECTIO N 8.6

Strabism ic disorders

Esotropia describes an ocular misalignment that is convergent in nature.


There are four main categories of esotropia:
Copyright 2014 by KMK Educational Services, LLC
CHAPTER 8. BINOCULAR VISION 357

1 Infantile ET (aka congenital ET) is characterized by a large angle (40 —


60a ), constant ET that occurs prior to 6 months of age and is usually
idiopathic in nature. Additional associations include:
• Overacting inferior oblique (OIO) - hyperdeviation when ADducting.
• Dissociated vertical deviation (DVD) - excyclotorsion with an
upward movement and incyclotorsion with a downward movement.
• Latent nystagmus - nystagmus that is manifest with one eye cov­
ered.
Because of the large angle of deviation within the critical period (birth -
2 years), non-accommodative infantile ET is often treated with surgery.
2 Acquired ET occurs after 6 months of age. There are three types of
acquired ET: 1) Accommodative, 2) Acute, and 3) Mechanical.
• Accommodative esotropia is secondary to accommodation and
is due to either a high amount of uncorrected hyperopia and/or
a high AC/A ratio. Initially the deviation is intermittent, but it
may become constant if not promptly treated. Treatment is typi­
cally with corrective lenses (bifocals may be necessary). Prism, VT,
and/or surgery may be considered if glasses do not help to avoid
suppression, amblyopia, and ARC.
— Accommodative ET may be diagnosed by performing CT with
and without a +2.00D lens. The +2.00D lens should relax ac­
commodation and decrease the magnitude of the esotropia.
• Acute ET has a sudden onset and is secondary to a neurological
problem (i.e. CN 6 palsy) or a decompensated phoria. Although
some cases resolve over time, treatment with patching or surgery
may be necessary.
• Mechanical ET is secondary to a physical restriction of an EOM
(i.e. Duane's syndrome type 1).

Duane’s Syndrome is characterized by limited ABduction (type 1),


limited ADduction (type 2), or limited AB and ADduction (type
3), as well as globe retraction and narrowing of the palpebral fissure
on adduction.

3 Secondary ET is due to either sensory deprivation or consecutive ET


after strabismus surgery.•
• Sensory deprivation occurs after the age of 5 and is a result of
reduced visual acuity in one eye from trauma, disease, etc.
Copyright 2014 by KMK Educational Services, LLC
358 8.6. STRABISMIC DISORDERS
• Consecutive ET is iatrogenic in nature and is usually due to an over­
correction of an XT during strabismus surgery. Treatment includes
prism, VT, or lenses.
4 Micro ET (aka monofixation syndrome, microsquint, small angle strabis­
mus) is a constant, unilateral eso deviation of less than 10A that develops
before 3 years of age (1). It is associated with a small central suppression
scotoma that can usually only be confirmed with the 4 BO test rather
than conventional cover test. Although vision is only mildly reduced,
micro ET is highly like to lead to intractable diplopia.

Exotropia describes an ocular misalignment that is divergent in nature.


There are 4 main categories of exotropia:
1 Infantile or congenital XT is usually a large deviation of 30 —80A that
occurs prior to 6 months of age. Because it is usually characterized by
an alternating pattern, infantile XT usually does not lead to amblyopia,
but it can lead to a reduction in stereopsis. Patients may have associated
neurological problems, especially in cases of constant infantile XT.
2 Acquired XT occurs after 6 months of age. There are 2 types of acquired
XT: 1) Acute and 2) Mechanical.
• Acute acquired XT is characterized by a sudden onset, constant
exo deviation. Etiologies include neurological issues, trauma, or a
decompensated phoria.
• Mechanical acquired XT is due to a physical restriction of an
EOM (i.e. Duane’s syndrome type 2, thyroid disease, etc.). Patients
will NOT have full versions due to the EOM restriction.
3 Secondary XT:
• Sensory XT occurs after the age of 5; it is much more common
in adults with acquired vision loss in one eye, resulting in a loss of a
stimulus to fuse and a resulting exo deviation. Treatment involves
correcting the cause of vision loss, if possible.
• Consecutive XT is iatrogenic in nature and is often secondary to
an over-correction of an esotropia during surgery.
4 Micro XT is an exo deviation of less than 10A that is usually unde­
tectable with cover test. Similar to micro ET, it is associated with a
small suppression scotoma that may be confirmed with the 4 BO test.
Micro XT is NOT common (1).
Copyright 2014 by KMK Educational Services, LLC
CHAPTER 8. BINOCULAR VISION 359

H ypertropia describes an ocular misalignment where one eye is deviated


above the fixating eye. Hypotropia describes an ocular misalignment where
one eye is deviated below the fixating eye. The most common causes of verti­
cal diplopia are CN IV palsy and thyroid-related ophthalmopathy (23).
Other causes include:
• Ocular surgery
• Orbital fracture
• Neurosurgery
• Childhood strabismus
• Myasthenia gravis
• Decompensated hyperphoria
• CN III palsy

Eye deviations may be paralytic (e.g. CN palsy) or non-paralytic


(e.g. childhood strabismus, decompensated phoria, mechanical
EOM restriction). Remember to check comitancy and forced
ductions to differentiate paralytic strabismus from other causes
(see ocular motility chapter for further details).

- SECTION 8 . 7 ----------------------------------------------------------------------

Acquired Brain Injury (ABI)


An acquired brain injury is characterized by sudden neurological damage and
is often due to a traumatic brain injury (TBI) and/or stroke. The prevalence
of TBI in the United States is 2.5 - 6.5 million individuals. Teenage boys and
young children are at the greatest risk for TBIs; motor vehicle accidents are the
most common cause of TBIs (7). Stroke is the most common cause of chronic
disability in adults in the United States, as about 80% of stroke victims survive
and must adapt to varying degrees of neurological impairment (7) (21).

Copyright 2014 by KMK Educational Services, LLC


360 8.7. ACQUIRED BRAIN INJURY (ABI)

TBIs cause sudden neurological damage due to shearing forces


within the brain that lead to injury and death of axons. CVAs
cause neurological damage due to ischemia.

Every lobe of the brain has some role in visual processing. If patients experience
an acquired brain injury, they may develop difficulties in multiple areas includ­
ing psychological, cognitive, motor, perceptual, and sensory anomalies (21).
Visual problems may include accommodative dysfunction, version deficits,
vergence dysfunction, visual field deficits, and photophobia. Visual difficulties
often occur due to an inability to match visual information with proprioceptive,
kinesthetic, and vestibular stimuli.

The prevalence of oculomotor dysfunction in TBI patients


ranges from 40-85%, depending on the source (14) (2).

Most patients with TBIs experience a spontaneous recovery in symptoms an


estimated 6 months after the initial injury. However, higher order functions, in­
cluding vision, may take longer to recover (7). Most of the BV techniques that
have been previously discussed are applicable to patients with acquired brain
injuries, but these patients may have unique needs that must be recognized
and appropriately managed.
Bymptoms
Diplopia, photophobia, asthenopia, blurred vision, skipping words, headache,
avoidance of near tasks, poor visual attention, decreased reading speed, poor
hand-eye coordination, perception of movement of stationary objects, loss of
comprehension when reading, memory problems, abnormal posture, nausea and
vomiting, dizziness, clumsiness.

1 Binocular vision testing should be performed in free space rather than


in the phoropter in order to obtain better patient responses and more
accurate measurements.•
• Convergence insufficiency is found in approximately 40% of pa­
tients with acquired brain injury and is the most common binoc­
ular dysfunction. Patients will have a large exophoria at near
with a reduced NPC (21).
Copyright 2014 by KMI< Educational Services, LLC
CHAPTER 8. BINOCULAR VISION 361

• Remember to perform phoria measurements and vergences to de­


termine if the phorias are compensated. Vergence ranges may be
overestimated due to slow subjective responses.
2 Perform stereopsis tests to evaluate fusion as binocular dysfunction may
cause decreased depth perception and poor balance.
3 Diplopia assessment: Check for comitancy of ocular deviations at distance
and near (see neuro-ophthalmic disease chapter for additional details).
* The most common non-comitant deviation after trauma is a CN 4
palsy (7), followed by CN 3, CN 6, multiple CN palsies, and restric­
tive ophthalmopathy. Patients may have a head turn and/or tilt
to compensate for the deviation(s).

If a vertical deviation is present, perform Park’s 3 step to de­


termine the involved EOM. The patient will often have a head tilt
away from the affected eye to decrease diplopia.

• Another BV disorder than may present with ABI is a cyclovertical


heterophoria, leading to diplopia and horror fusionis (21). Double
Maddox rod testing will assist in the diagnosis.
4 Accommodation testing is indicated in patients with ABIs as 10-41% of
patients with TBI have a loss of accommodation (21). Both amplitude
and accommodation facility should be tested.

Remember the expected average amplitude of accommodation is


18 - 0.33(age).

5 Fixation testing: Patients should be able to fixate on a target for 10


seconds.
6 Saccades and pursuits may be impaired due to dysfunction of the supranu­
clear control center. Testing includes NSUCO saccades and pursuits and
DEM (see ocular motility chapter).•
• Possible oculomotor deficits following a CVA include saccadic over­
shooting or undershooting, inability to sustain fixation and main­
tain attention, and difficulty shifting attention between multiple ob­
jects (21).
Copyright 2014 by KMK Educational Services, LLC
362 8,7. ACQUIRED BRAIN INJURY (ABI)
7 Visual-perceptual processing tests may be indicated and include testing
in two major aras (see psychology chapter for further details):
• Visual-spacial dysfunction: Standing angels in the snow (bilateral
integration), Piaget test of left-right concepts (laterality), Jordan
left-right reversal test (directionality).
• Visual perception (includes visual discrimination, visual closure, form
constancy, figure-ground discrimination, visual-spatial relations, and
visual memory): TVPS (Test of Visual-Perception Skills), M FVP
(Motor Free Vision Perceptual test), DTVP (Developmental Test
of Visual Perception).
8 Bilateral visual field loss occurs in an estimated 14% of patients with
severe TBIs. Homonymous hemianopsia is the most common visual
field defect.
• Patients with ABIs may have difficulty with testing on automated
perimeters. Goldmann visual fields or tangent screen may be more
appropriate tests for field loss.
9 Additional testing to consider may include contrast sensitivity, gonioscopy
(to rule out angle recession due to trauma), cranial nerve testing, and slit
lamp examination (look for reduced blink rates).

Bp^cific conditions in ABI ■£;; A ; 1


Midline shift syndrome: Following a TBI/CVA, patients may have a dis­
connect in the feedback loop between sensory (e.g. visual processes) and
motor systems. This disruption leads to a distortion of space, with an
expansion of space on the “good” side and a shift away from the neglected
side (17).•
• Associated neuro-motor characteristics include hemiparesis,
hemiplegia, flexion, extension, and neglect (17).
• Signs include leaning, balance issues, dizziness, spatial disorienta­
tion, perception of the floor being tilted, staying on one side of the
room, and bumping into things.
• Testing: Move a target in front of the patient from left to right,
and then right to left. The patient should indicate each time when
he/she perceives the object at the midline. If the object is not at
the true midline, note whether the shift is to the right or left (17)
• Treatment: Yoked prism with the base towards the neglected
side.

Copyright 2014 by KMK Educational Services, LLC


CHAPTER 8. BINOCULAR VISION 363

If a patient has a stroke affecting the left side of the brain, he/she
will have right-sided hemiparesis. The visual midline shift will be
away from the affected side (towards the left). Yoked prism with
the base towards the right should be used for treatment (17).

Neglect (aka unilateral inattention (one component of midline shift syndrome)):


• Characterized by a perceptual deficit where the patient is unaware
of and does not attend to an entire hemifield. The left hemifield
is most often neglected, as neglect is commonly due to damage to
the right front parietal lobe.
Signs: Patient is unaware of one side, often bumping into objects, ignor­
ing that side of the body (grooming), turning head/body away from
side of neglect, and missing words/letters on the affected side.
Testing: Clock-dial test or house test - a patient with neglect will have
asymmetry in his/her drawings.
Treatment: 1) Increase awareness on the neglected side by forcing pa­
tients to perform activities on that side (present stimuli on the ne­
glected side) 2) Scanning - remind patients to scan into the neglected
side using dynamic and static targets.
Specific considerations for patients suffering from strokes (4):
• Aphasia: Difficulty expressing language (expressive) or difficulty
understanding the speech of others (receptive) due to neurological
damage.
— For expressive aphasia, consider using a number chart, tumbling
Es, or having the patient match distance symbols with symbols
on a near card. The examiner should establish a signal for yes
and no, such as blinking or finger tapping.
— For receptive aphasia, objective testing is mainly used as the
patient is unable to understand directions.
• Apraxia: Motor planning deficit resulting in difficulty initiating
motor responses.
• Acuity is typically minimally affected by homonymous visual field
defects.
Post Trauma Vision Syndrome (17) After traumatic injury or ischemia
(CVA) to the brain, there can be a disruption in the ambient visual pro­
cess responsible for peripheral fusion and spatial organization/orientation,
resulting in an unstable visual processing system. Patients are often over­
whelmed in crowded environments with lots of movement. They may also
have binocular vision and accommodative insufficiencies.
Copyright 2014 by KMK Educational Services, LLC
364 8.7. ACQ UIRED BRAIN INJUR Y (ABI)
• Binasal occlusion or base in prism (typically 2 BI over each eye)
often helps to reduce patient symptoms by decreasing the degree of
binocular processing.
• Patients frequently have an increase in myopia.
• Visual evoked potential (VEP) testing is necessary to make the
diagnosis.

• Lenses: Small amounts of ametropia are important to correct as patients


may be more symptomatic than usual. Bifocals or reading glasses may
be needed for non-presbyopic patients with accommodative insufficiency.
• Prism
—Yoked prism (base towards the defect) and/or reverse telescopes may
be used for field expansion.
- Prism should be prescribed for patients with BV disorders to elim­
inate diplopia and aid in fusion. BI prism is prescribed for exo de­
viations (including convergence insufficiency) and BO prism is pre­
scribed for eso deviations. See BV section for determining how much
prism to prescribe (Sheard’s and Percival’s criteria).

Use the least amount of prism necessary to obtain fusion. If pre­


scribing a low amount, place in front of one eye (typically the eye
with worse BCVA). Larger amounts of prism should be split be­
tween the two eyes (4).

- Nystagmus may be dampened by using prism to move the eyes to


the null point.
— Homonymous hemianopsias may be treated with prism (most com­
monly the Peli lenses).
* Older prism systems used to treat hemianopsia include INWAVE
prism (12a ground into the lens) or the VFAS prism (aka button
prism)).
• Mirrors
• Eccentric viewing techniques and scanning techniques for field defects.
• Occlusion:
Copyright 2014 by KMK Educational Services, LLC
CHAPTER. 8. BINOCULAR VISION 365

- Bitemporal occlusion eliminates distracting peripheral stimuli


when reading.
- Binasal occlusion is used in the treatment of post trauma vision
syndrome.
—Total occlusion may be necessary in cases of diplopia.
• Improve lighting and contrast through the use of tints and glare control.
—Typically 35% blocking tints are used indoors and 85-90% are used
for outdoors (15).
• Orientation and mobility training (O&M).
• Vision therapy (for binocular, accommodative, and oculomotor dysfunc­
tion).
• Ergonomic considerations.
• Increase print size.
• Neuropsychiatric counseling.
• Patient and family education.

The rehabilitation team is key to improving the patient’s visual


symptoms and overall quality of life and often includes an ophthal­
mologist, optometrist, occupational therapist (OT), vision training
technician, counselor, social working, and physical therapist (PT).

References
[1] AOA Optometric Clinical Practice Guidelines, Care of the Patient with Strabismus: Es­
otropia and Exotropia, AOA website, updated 2004.
[2] Ashley, M, (2004), Traumatic B rain Injury, R ehabilitative T reatm ent and Case M anage­
m ent, 2nd edition. CRC Press.
[3] Bennett, E. and V. Lakshminarayanan (2006), R eview Q uestions fo r the N B E O E xam ina­
tion, P art IS. Butterworth Ilememann.
[4] Brilliant R. Essentials of Low Vision Practice. Butterworth and Hcineman, Boston, 1999.
[5] Carlson, N. , Kurtz D,, Heath, D., Hines, C. (1996). Clinical Procedures fo r O cular E xam ­
ination, 2nd edition Appleton and Lange.
[6] Elliott DB. Clinical procedures in primary eye care. 4th ed. Philadelphia: Elsevier, 2014.

Copyright 2014 by KMI< Educational Services, LLC


366 8.7. ACQUIRED BRAIN INJURY (ABI)
[7] Falk N, Aksionoff, E. The primary care optometric evaluation of the traumatic brain injury
patient. Journal of the American Optometric Association, Vol. 63, Number 8, 1992.
[8] Goss, David A. Ocular Accomodation, Convergence, and Fixation Disparity, 2nd ed. Boston:
Butter worth-He inem arm, 1995,
[9] Griffin JR. Grisham JD, B inocular Anom alies; Diagnosis and V ision Therapy.
Butterworth-Heinemann, 1996.
[10] Grosvenor, T. (2007). P rim ary Care O ptom etry, 5th edition Butterworth.
[11] Hermann JS, Surgical therapy for convergence insufficiency. J Ped Ophthalmol Strab.
1981;18:28-31.
[12] Hoffman L, Cohen A, Feuer G. Effectiveness of non-strabismic optometric vision training in
a private practice. Am J Optom Arch Am Cad Opt, 1973;50:813-816.
[13] Hokoda SC. General binocular dysfunctions in an urban optometry clinic. J Am Optom Assoc
1985;56:560-562.
[14] Kapoor N, Ciufffeda K, Han Y. Oculomotor Rehabilitatation in Acquired Brain Injury: A
case series. Arch Phys Med Rehabil Vol 85, October 2004.
[15] Kapoor N, Ciuffreda K. Vision disturbances following traumatic brain injury. Current Treat­
ment Options in Neurology, 2002.
[16] Norn MS. Convergence insufficiency: Incidence in ophthalmic practice. Results of Orthoptics
treatment. Acta Ophthalmol 1966;44:132-138.
[17] Padula W, Neuro-optometric Rehabilitation, 3rd edition. Optometric Extension Program,
Santa Ana, CA, 1988.
[18] Press LJ. Applied Concepts in Vision Therapy. Mosby, 1997.
[19] R. Rabbetts (2007). B en n ett and R abbetts’ Clinical Visual Optics, 4 th E dition.
Butterworth-Heinemann.
[20] Rutstein R, Daum K. Anomalies of Binocular Vision: Diagnosis and Management. Mosby,
1998.
[21] Scheiman M., Wick B. (2002). Clinical M anagem ent o f B inocular Vision, 2nd edition.
Lippincott Williams and Wilkins,
[22] Scheiman M. Divergence insufficiency: Characteristics, diagnosis and treatment. Am J Op-
tom Physiol Opt. 1986;63(6);425-431.
[23] Tamilankar M, Kim J, Ying G, et al. Adult, hypertropia: a guide to diagnostic evaluation
based on review of 300 patients. Eye (Lond). 2011 January; 25(1): 91aA§96.
[24] Von Noorden GK, Campos EC. Binocular vision and ocular motility: Theory and manage­
ment of strabismus. 6th ed. St. Louis: Mosby, Inc., 2006.
[25] Weissberg E. (2004). Essentials of Clinical Binocular Vision. Butterworth,
[26] Wong AMF. (2008). E ye M ovem ent Disorders. Oxford University Press.
[27] Yen KG. 2014 Mar 20, Amblyopia treatment and management. Medscape.
<http://emedicine.medscape.com/article/1214603-treatment>. Accessed 2014 May
10.

Copyright 2014 by KMK Educational Services, LLC


Chapter

Visual Perception

Kevin B. Wood, Ph.D.

367
C
(
f
308
c
(
(
(
(
(
(
(
(
(
(
(
(
(
(
(
(
(
(
(
Copyright 2014 by KMK Educational Services, LLC
(
(
i
(
CHAPTER 9. VISUAL PERCEPTION 369

I - SECTION 9.1

Psychophysics
Psychophysics allows us to make quantitative statements about perception. For
example, psychophysics makes it possible to measure and analyze luminance,
which is a perceptual entity. Since we deal with perception, the measure­
ment fundamentally depends on a human intermediary. Before dealing with
Visual Perception in detail, we briefly discuss some common methods in psy­
chophysics (4, ch. 11).

We need some way to quantify a given perceptual quantity of interest. For


example, how do we find, in a systematic way, the minimum level of light
intensity that is detectable by a human? Quantifying such thresholds is difficult
because the quantity of interest is one of perception, and therefore results will
vary both person to person and trial to trial. In addition, different individuals
may have different criteria for defining threshold. To untangle these ideas and
put perception on solid quantitative footing, we have to be very careful with
how we define these quantities.
Psychometric Functions
A psychometric function is a plot of the percent correct (or percent detected)
by an observer against the variable of interest, for example light intensity or
brightness. This will be a smooth curve rather than a step function which
abruptly changes from 0 % correct to 1 0 0 % correct at some value of the variable.
Typical Psychometric Functions

(FO
Intensity o f Variable
(e.g. Light Intensity)

Figure 9.1: Two typical psychometric functions.

Copyright 2014 by KMK Educational Services, LLC


370 9.1. PSYCHOPHYSICS
Threshold is usually taken to be the 50% correct line (halfway between highest
and lowest performance) (4, pp. 249-56).
Figure 9.1 shows two typical psychometric functions (4, pp.255). For a forced
choice test in which the subject must choose between two alternatives, the
percent correct ranges between 50% —100%. For a minimum detection type
test, the range varies from 0 % —1 0 0 %.
M ethod of Ascending or Descending Limits: In the method of ascend­
ing (descending) limits, the experimenter increases (decreases) the stim­
ulus intensity incrementally from presentation to presentation and plots
the resulting psychometric function. This technique is useful when dark
adaptation (visual acuity) is being measured (4, pp. 252).
M ethod of Constant Stimuli: In this method, the experimenter varies the
intensity randomly from presentation to presentation. This ensures that
the observer will not anticipate the next stimulus (such anticipation could
theoretically affect the results), though it often takes a great deal of
time (4, pp. 253).
Stairstep M ethod: In this method, the experimenter will increase the stim­
ulus intensity in a staircase manner (gradually increasing one step at a
time), followed by a decrease in a staircase manner. This reversal (up,
down, up, down) continues for several periods, allowing the threshold to
be refined from both directions. It provides a time-efficient method for
threshold detection (4, pp. 252).

Automated visual field testing often exploits The Stairstep


Method (4, pp. 252).

Adjustment M ethod: In this method, the experimenter allows the subject


to adjust the stimulus intensity until it is just perceivable.

All of the above methods suffer from one drawback: the effect
of individual (varied) threshold criteria may skew the results (4,
pp. 254).

Forced Choice M ethod: The forced choice method minimizes the poten­
tial damage done by variable threshold criteria (4, pp. 254-56). In this
method, the experimenter forces the subject to make a choice between
Copyright 2014 by KMK Educational Services, LLC
CHAPTER 9. VISUAL PERCEPTION 371

two alternatives, and this limits the effects of threshold criteria. As we


mentioned earlier, the psychometric function varies between 50% (chance)
and 100% (perfect). The threshold is typically defined as halfway between
chance and perfect (75 percent for a two alternative test). We note that
there can of course be more than two alternatives, and in this case the
lower bound obviously changes.

Exam ple 9 .1: W h ich psychophysical m e th o d s h o u ld be used fo r d e te rm in in g th e d ark


a d a p ta tio n curve? W h a t are th e d raw b acks o f this m e th o d ? Is th e re a m e th o d th a t
m in im ize s these draw backs?

Solution 9 .1: T h e m e th o d o f ascen din g lim its sh o u ld be used fo r te s tin g d ark a d a p ta tio n ,
as i t involves s ta rtin g a t a very lo w in te n s ity a n d g ra d u a lly increasing th e intensity. U n like
a forced choice test, this m e th o d suffers fro m a s e n s itivity to in d iv id u a l th resh o ld criteria.

The visual system is noisy, meaning that there is some inherent uncertainty
involved in the transmission of signals. As an observer, how do we select the
signal (the important information) from the noise (the background fluctua­
tions)? Signal detection theory provides a mathematical basis for answering
this question (4, pp. 256-62). We start by considering a signal and an idealized
receiver. The receiver receives some input, and from this input we must decide
if a stimulus (the signal) was present.

D e t a ils o f S D T

1 Noise is assumed random and Gaussian distributed, like a bell curve.


This noise corrupts the signal.
2 The receiver does not get the pure signal, but instead receives a combi­
nation of signal and noise (S + N). If the signal is very weak, one can
imagine that it might be difficult to tell if we are seeing just noise (N) or
S + N.

When the distributions of S + N and N substantially overlap, distinguishing


the two is difficult.
Copyright 2014 by KMK Educational Services, LLC
372 9,1. PSYCHOPHYSICS

Sensory Response

Figure 9.2: Distributions for S and S -j- N are shown, with detectability d
specified.

Detectability (d): Detectability characterizes the distance between the means


of the S p N distribution and the N distribution. Figure 9.2 show distributions
for both the signal + noise and the noise. Any time the receiver gets some type
of input, this input will activate the receiver in some way. For example, the
receiver might give a readout in terms of a series of numbers or sounds, or
alternatively the activation may be thought of as the score on some medical
test. We use the generic term “activation” to mean the response of the receiver.
With this in mind, Figure 9.2 shows the probability of a given response (activa­
tion) by the receiver. In the “easily detected” case, the average response of the
system to noise, N, is very different than the average response of the system
to signal and noise, S + N. Thus it is easy to distinguish between the two.

Criteria: This framework allows us to study the effect of threshold criteria


on stimulus detection. Suppose the N and S P N distributions look as below.
We must define some criteria for deciding whether or not the signal is present
or not based on the activation of the receiver. That is, by looking at the level
of activation, we must define some rule for guessing whether or not the signal
is there (see Figure 9.3).•
• Lax criteria: We may choose very lax criteria by using the following rule:
if the activation is above the Lax line on the plot above, we will guess
that the signal is present. While this will lead to many positives (we will
think the signal is present often), many of these will be false positives.
On the bright side, we will almost never miss a signal that is actually
present.
Copyright 2014 by KMK Educational Services, LLC
CHAPTER 9. VISUAL PERCEPTION 373

Figure 9.3: Criteria

• Strict criteria: If we choose a criteria line that is much farther to the


right, we will have fewer hits, but also fewer false positives. We may
guess that there is no signal when there actually is one (false negative),
but we will very rarely guess that there is a signal when there is not one.
Sensitivity, Specificity, and Predictive Value: The terms sensitivity and
specificity are often used to quantify medical tests. Sensitivity refers to the
probability of a test giving a positive result given that the signal (e.g. a disease)
is present. In other words, it measures how sensitive a given test is to the
presence of a signal. Specificity refers to the probability of a test giving a
negative result given that the signal (e.g. a disease) is absent.
On the other hand, it is often more useful to know the positive (negative)
predictive value of a test. The positive (negative) predictive value is the
probability of a disease being present (absent) given that the test is positive
(negative). These terms can be thought of in the following framework.•
• There are two possibilities for a test result (positive or negative), and
two possibilities for the signal or disease (present or absent). As a result,
there are four possible outcomes. It is sometimes useful to draw a Purmett
square with all possible outcomes.
1 Positive test and present signal —^ True positive
2 Positive test and absent signal —>False positive (Type I Error)

3 Negative test and present signal False negative (Type II Error)


4 Negative test and absent signal —> True negative
Copyright 2014 by KMK Educational Services, LLC
374 9.1. PSYCHOPHYSICS
• The number of true positives plus the number of false negatives must
equal the total incidence number (that is, the total number of people
with the disease in the population),
• The sum of the number of false positives and true negatives must equal
the total number of people without the disease in the population.
• Sensitivity is, mathematically, the number of true positives divided by
the total incidence number.
• The false negative rate is given by 1 -sensitivity.
• Specificity is, mathematically, the number of true negatives divided by
the total number of people without the disease.
• The false positive rate is given by 1-specificity.
• Sensitivity and specificity are closely connected to the idea of criteria
discussed above.
• Positive predictive value is given by the number of true positives
divided by the sum of the number of true positives and false positives.
• Negative predictive value is given by the number of true negatives
divided by the sum of the number of true negatives and false negatives.

Receiver Operating Characteristic Curve (ROC): ROCs (Figure 9.4)


help us determine how criteria affects detectability (4, pp. 259-62). We con­
struct an ROC as follows: plot the hit probability as a function of the false
positive probability. That is, the vertical axis shows the probability that the
observer correctly detects the signal, while the horizontal axis tracks the prob­
ability of the observer falsely believing that the signal is there when it is not.
Notice how the curve changes for different values of d. No matter what crite­
ria one uses, the ratio of hit probability to false positive probability will fall
somewhere on the ROC curve. Lax criteria fall to the upper right part of the
curve (high false positive, high hit rates), while strict criteria will force you to
the lower left part of the curve (low false positive, low hit rates).

Exam ple 9.2: C o nsider ag ain a s tim u lu s given by a w eak sign al In a very noisy e n viro n ­
m e n t. D e s c rib e th e a p p e a ra n c e o f th e R O C curve likely e licited by such a stim ulus. H o w
w ill th e R O C cu rve chan ge as noise is g ra d u a lly red uced to 0 ?

Solution 9.2: S ee F ig u re 9 .4 , w hich shows h o w th e curve changes as d e te c ta b ility


changes.

Copyright 2014 by KMK Educational Services, LLC


CHAPTER. 9. VISUAL PERCEPTION 375

Hits

Figure 9.4: An ROC curve is plotted. The dark star to the left indicates strict
criteria. The lighter star to the right indicates lax criteria.
- SECTION 9.2 --------------- -------------------------------------------------------------------------------------

Radiom etry vs. P hotom etry


We detail the differences between radiometry and photometry (4, ch. 4).
A ; - : : . : ' . ... :&r-r-'Xy

Radiometry deals with energy per time produced by a source of electromag­


netic radiation (light). It deals with physical properties which we can easily
measure (4, pp. 69).
Radiant power: Radiant power is the energy per second produced by a light
source (unit is watts, W).
Radiant intensity: Radiant intensity is the energy per second (or radiant
power) in a given direction (unit is watts per solid angle, Wjll), We use
this for dealing with point sources.
Radiance: Radiance is the radiant intensity per projected area of source. We
use this for dealing with spatially extended sources, where we need some
Copyright 2014 by KMK Educational Services, LLC
376 9.2. RADIOMETRY VS. PHOTOMETRY
way of accounting for the fact that the source itself has some area. The
intuition is as follows: how much light (power) does a source of a certain
size produce?
Irradiance: Irradiance is the radiant power per unit area of surface. Irradi-
ance deals with the amount of radiant power that hits or fall upon some
object. That is, irradiance deals with the light that reaches a target.

Photometry deals with how our visual system responds to this electromagnetic
radiation. This is a property of perception, not physics. We must measure
these properties using psychophysics, which involves a person’s response to
light, not the light itself.
Luminous power: Luminous power is analogous to radiant power, but it
deals with the visual system’s response to light rather than the phys­
ical energy of light. The basic unit of luminous power is the lumen.
Roughly speaking, if a source elicits a stronger reaction by the visual sys­
tem, it has higher value of luminous power (higher number of lumens) (4,
pp. 66 -6 8 ).
• We can define a function 0 < V(X) < 1 called a Luminous Effi­
ciency Function. V is related to brightness, with V — 1 being
most bright and V = 0 being least bright. V takes on its maximum
value at A —555 nm.
• We say by convention that there are 680 lumens per watt at A = 555
nm. (This number may seem arbitrary. The logic behind it is not
particularly important: just remember that we are defining lumens
so that there are 680 of them at the wavelength A = 555 nm).
• For every watt of power (which is a physical or radiometric quantity)
from a light source at A = 555 nm, the visual system responds with

Figure 9.5: Scotopic and photopic sensitivity functions.

Copyright 2014 by KMK Educational Services, LLC


CHAPTER 9. VISUAL PERCEPTION 377

680 lumens (a unit of photometry). This provides a method for


converting a physical property to a psychophysical property. Light
of other wavelengths will have a different number of lumens per watt
associated to them. We use V (A) to determine lumens per watt at
other wavelengths. Of course, V (A) is measured psychophysically,
and it should be given (we cannot calculate it without doing the
experiment ourselves).
Luminous intensity: Luminous intensity is the luminous power in a given
direction (lumens per solid angle = candela, 1/91). It deals with point
sources (4, pp. 68-69).
Luminance: Luminance is the Luminous intensity per projected area of source
(candelas per square meter, c/m 2, sometimes called nits). It deals with
spatially extended sources (4, pp. 70). Luminance can also be measured
in foot-lamberts, with 1 foot lambert equal to 3.43 nits.
Illuminance: Illuminance is the luminous power per unit area of surface (lu­
mens per square meter or lux, l / m 2). As with irradiance, illuminance
deals with light striking or falling upon a target object (4, pp. 70-71).
Illuminance can also be measured in foot-candles, which are lumens per
square foot. One foot-candle is equal to 10.8 lux.
Scotopic Lumens (4, pp. 75-76)
There is an analogous system for dimly lit circumstances when rods play a
large role in vision. In that case, we talk about scotopic lumens and V is called
V f. There are still 680 scotopic lumens per W at 555 nm. Now, however,
the maximum V/nax — 1 now occurs at A = 507 nm and corresponds to 1700
lum ens/watt. Remember that V and V' must be determined with a psy­
chophysical experiment. As Figure 9.5 shows, under scotopic conditions, the
visual system is most sensitive to light at A507 * Under photopic conditions, the
visual system is most sensitive to light at A555 .

Exam ple 9.3: First, consider th e fo llo w in g in fo rm a tio n re a d fro m a cu rve o f V { A);

W 60 0 ) = 0.5
(9 11
U(500) = 0.2 v ' '

A ssum e th e re are tw o sources, each 10 W pow er. S o u rc e o ne e m its tig h t a t A = 600 nm .


S ource tw o e m its lig h t a t A = 500 nm . F in d th e lu m in o u s p o w e r fo r each. W h ich lig h t
source elicits th e stro n g est re a c tio n b y th e visual system ?

Solution 9 .3: R e m e m b e r, th e m a x im u m value o f lu m in o u s p o w e r is 680, a n d t h a t occurs


a t w hen V = 1. A low er value o f V in d ic a te s a lo w e r value o f lu m in o u s p ow er. F irst, for
source one w e have
(0,5) (680) — 340 lu m en s / w a tt (9-2)

Copyright 2014 by KMK Educational Services, LLC


378 9.2. RADIOMETRY VS. PHOTOMETRY
Since th e source is 10 W , we h ave t h a t lum ino us p o w e r = (10)(340) = 3400 lum ens. F o r
source tw o, w e have
(0.2) (680) = 136 lum ens / w a t t (9.3)

S in ce th e source is 10 W , we h av e lu m in o u s p o w e r = (10) (136) = 1360 lum ens. S o urce


o ne has h ig h e r lu m in o u s p o w e r and, intu itiv e ly, p roduces a stro n g er re actio n b y th e visual
system . N o te t h a t i f w e w ere d ea lin g w ith th e scotop ic system a n d th erefo re V ’, we w o u ld
m u ltip ly V ' b y 17 0 0 l / W r a th e r th a n 6 8 0 l / W , as we d id here.

If multiple wavelengths are present, you can just add the contribu­
tion (lumens) from each wavelength. This is known as Abney’s law
of additivity. In the example, if both sources above were combined
into a single light source and turned on at the same time, the total
luminous power of the new source would be 3400 + 1360 = 4760
lumens.

Filters
Filters absorb some of the light incident upon them. This means that light
leaving the filter has a different spectral composition (different wavelengths
malm up the light) than the light that strikes the filter. We introduce some
basic concepts related to filters (4, pp. 84-90).

Narrow band filters: Filters that only allow a small range of wavelengths
to pass through are narrow band filters. One would specify a narrow band filter
by two quantities:
1 Location of peak: Tells which A range most easily passes through the
filter.
2 Half height bandwidth: Tells the selectivity of the filter. A higher half­
height bandwidth corresponds to a less sensitive filter.

Interference filters: Interference filters are highly sensitive (low half-height


bandwidth) and essentially transmit only a single wavelength.

Broad band filters: Filters which transmit a large range of As are called
broadband filters.
Copyright 2014 by I<MK Educational Services, LLC
CHAPTER 9. VISUAL PERCEPTION 379

Long-pass filters: Filters which transmit only long wavelengths are called
long-pass filters.

To block the potentially cataract-inducing effects of UV rays, sun­


glasses are often coated with a substance acting as a long-pass
filter.

N eutral density filters: Neutral density filters transmit all wavelengths


equally. Neutral density filters will slow the neural response to a stimulus by
effectively decreasing the magnitude of the stimulus. As a result, if one places
a neutral density filter over one eye, the motion of a planar pendulum will
appear elliptical as a result of the processing delay in the covered eye. If the
filter covers the left eye, the motion will appear clockwise (and counterclockwise
if the filter covers the right eye).

In reducing the amount of visible light transmitted, sunglasses act


as neutral density filters. Distortion of color is minimized when all
As ar e equally blocked.

Related Concepts
Lambert surfaces (or Cosine Diffusers): A Lambert surface is a surface
that shows the same luminance from every viewing angle (e.g. paper
with matte finish). To say this another way, it scatters light equally in
all directions. The opposite of a Lambert surface is a specular surface
(e.g. a shiny piece of silver). In equation form, we have
L —RE (9.4)
where L is luminance (measured in foot-lamberts), R is a reflectance
factor (dimensionless), E is illumination (measured in foot-candles) (4,
pp. 73-74). The reflectance factor R measures the relationship between
incoming light and outgoing (reflected) light. It is always a good idea to
remember the general relationship, which (in this case) is linear. That
is, if E is doubled, L is also doubled. If E is tripled, L is also tripled,
etc. As you can see, the value of A-which is specific to a given type of
Lambert surface-is not crucially important for understanding this general
behavior.
Copyright 2014 by KMK Educational Services, LLC
380 9.2. RADIOMETRY VS. PHOTOMETRY

Figure 9.6: Illumination varies with both distance d and tilt <f>.

Retinal Illumination: Retinal illumination describes the amount of light


which falls on the retina. We can relate this quantity to the Luminance
of the viewed surface and the area of the pupil. We have
T = LA. (9.5)

where T is the retinal illumination (units are trolands ), L is the lumi­


nance of the viewed surface, and A is the area of the pupil (4, pp. 74-75).

Exam ple 9.4: W h e n a person loo ks a t a s m a ll re d cloth, th e illu m in a tio n fa llin g on


h e r re tin a is g iv e n b y t tro lan ds. S h e then places drops in h e r eyes w hich increase
h e r re tin a l d ia m e te r b y 2 0 0 % . F in d th e re tin a l illu m in a tio n w hen she ag a in stares
a t th e sam e re d c lo th a ft e r p u ttin g th e drops in h e r eyes.

Solution 9.4: T h e d ia m e te r o f th e p u p il is increased b y 3 fold. F o r a c irc u lar


pupil,

(9.6)

w here r is radius a n d d d ia m e te r. T h a t is, area goes like th e square o f th e d ia m e te r.


So i f th e d ia m e te r triples, A w ill increase by a fa c to r o f 3 * 3 = 9. I f area increases
by a fa c to r o f 9, T w ill also increase by a fa c to r o f 9. So th e n ew illu m in a tio n is
91. N o te t h a t this la rg e r p u p il w ill also lead to increased sp h erical ab e rra tio n .

Copyright 2014 by KMK Educational Services, LLC


CHAPTER 9. VISUAL PERCEPTION 381

General Illumination: Inverse Square and Cosine Laws: We can relate


the general illumination (falling on a tilted surface) to the intensity of the
source as follows (Figure 9.6):
p 1 c°s(<£) (9.7)
e =—¥ ~ '
where E is the illumination falling on the surface, I is the intensity of
the point source, d is the distance from the point source to the surface,
and <fi is the angle of tilt of the surface. As the distance from the source
doubles, E changes by a factor of 1/4. If the surface is tilted, E changes
by a factor of cos(</>) (4, pp. 74-75).
SECTION 9.3

Light Perception
We introduce some fundamentals of human light perception (4, ch. 3,4).

Scotopic Vision: Vision under dim lighting (due primarily to rods) is known
as scotopic vision. It is characterized by poor acuity and a lack of color
discrimination ability, but very good sensitivity.
Photopic Vision: Vision under bright conditions (mediated primarily by cones
is known as photopic vision. It is characterized by excellent acuity and
color discrimination but poor sensitivity.
The two facets-scotopic and photopic vision-work largely under mutually
exclusive circumstances (a phenomenon which leads some to refer to the
duplex nature of the retina). However, under some conditions-known as
mesopic conditions~both facets work together. This might happen in a
dimly lit setting where small amounts of very bright light are present
(for example, Schwartz points out that both systems are active at sunset,
where the setting sun still provides small amounts of very bright light) (4,
pp. 26-27).
Principle of Univariance (4, pp. 30-32)
The principle of univariance states that a photopigment responds to the ab­
sorption of a quantum of light (photon) in a manner that is completely inde­
pendent of the wavelength of the photon. The response of the corresponding
rod or cone is not determined by the wavelength of the photon; instead, the
wavelength only determines the probability of absorption (not the effect of an
absorption). As a result, once a photon is absorbed, all information about
Copyright 2014 by KMK Educational Services, LLC
382 9.3. LIGHT PERCEPTION
the wavelength of that photon is lost. This leads to ambiguities between,
for example, color and intensity.
Photo chromatic interval: The difference in scotopic and photopic sensitiv­
ity at given wavelength is known as the photochromatic interval. Recall,
of course, that sensitivity is inversely related to threshold (4, pp. 33).
Point of equal sensitivity: The photochromatic interval is zero at 650 nm,
meaning that scotopic and photopic sensitivities are equal at this wave­
length.
Purkinje Shift: As illumination (the amount of light falling on a surface) in­
creases, longer wavelengths appear brighter. This is known as the Purk­
inje Shift. It is due to a shift from largely scotopic vision (which has a
peak sensitivity at 507 nm) to largely photopic vision (which has a peak
sensitivity at 555 nm) (4, pp. 35-36).

Exam ple 9.5: C o nsider a s im p lifie d visual system t h a t con tain s o nly rods. D escrib e
th e e ffe c t th e absence o f c o n e -m e d ia te d vision has on th e P u rk in je s h ift fo r th is
visual system .

Solution 9 .5: T h e P u rk in je s h ift is d ue to th e tra n s itio n fro m larg ely s co to p ic to


larg ely p h o to p ic vision. A s a resu lt o f th is s hift, th e p e a k o f th e visual sy s te m 's
s e n s itiv ity shifts to a d iffe re n t w avelen g th . In o u r s im p lified exam ple, no such s h ift
w o u ld ta k e p lace, a n d th e P u rk in je s h ift w ould be absent.

^ark Adaptaibion •-- m Ad v -: r\r -ti ilb v id A-,Ov.A.^rV;.-b>T-1

Dark adaptation deals with how our threshold changes as we spend more time
in the dark (4, pp. 36-45). Keep in mind that threshold and sensitivity are in­
versely related. That is, if we have a high threshold, we have a low sensitivity.
Said another way, if we can only detect a very bright light, we have low sensi­
tivity and a high threshold. We can put a person in the dark and then measure
his/her threshold (what is the smallest amount of light he/she can perceive?).
Then we can plot this against the amount of time spent in the dark. We will
get a difference curve for each wavelength of stimulus we use (Figure 9.7).
In Figure 9.7, it is important to note the rod-cone break, which is a small blip
in the curve. This break will go away if we use light that is 650 nm. Note that
the decay in threshold due to cones is very fast, while the rod-induced decay is
much slower.

Copyright 2014 by KMK Educational Services, LLC


CHAPTER 9. VISUAL PERCEPTION 383

Time in Dark

Figure 9.7: A typical dark adaptation curve is shown. Note that the rod-cone
break would not be present for A —650 nm (4, pp. 38,40,42).

Exam ple 9.6: W h y is th e re no ro d -c o n e b re a k on th e d ark a d a p ta tio n curve fo r A = 650


nm ?

Solution 9.6: T h e b reak is d ue to th e chan ge b etw e e n cone d o m in a te d vision a n d ro d


d o m in a te d vision. H ow ever, fo r tig h t t h a t is 650 n m , th e re is no d ifferen ce b etw een th e
s e n s itivity o f ro d a n d cone vision (th e p h o to c h ro m a tic in te rv a l is 0 ). Therefore, th e re is
no c h an g e in th e fu n c tio n a l form o f th e d ec a y as we g o from o ne ty p e o f vision to th e
o th er.

Biophysical Explanation
Photopigments absorb photons. However, they must be regenerated after an
absorption (that is, they are bleached, or used up once they absorb a photon).
As they are regenerated, the probability to absorb a photon rises because there
are more unbleached pigments, and therefore one can detect even very small
amounts of light. As a consequence, the visual threshold is lowered. When we
first flip off the lights, many photopigments are “used up” (due to the light that
was on), and so sensitivity is initially poor. However, with time the sensitivity
increases (and threshold lowers).

Regeneration of photopigments is not the only factor contributing


to the time scale of dark adaptation.

Copyright 2014 by KMK Educational Services, LLC


384 9.3. LIGHT PERCEPTION
Dowling-Rushton Equation: We can describe dark adaptation mathemati­
cally using the following empirical equation. It is important to note that
if photopigment regeneration were the sole cause of dark adaptation, the
mathematical relationship would not take this form.
log ( E ) = cP (9.8)
where It is the threshold intensity for a flash, I q is the dark-adapted
threshold intensity, P is the fraction of pigment in the bleached state,
and c is an unimportant constant.

Light adaptation deals with how well one can pick out a spot of light from
a background of slightly different intensity (4, pp. 45-47). To measure light
adaptation, we would show a person a spot of light on a background of light
at slightly different intensity. We would find the person’s threshold at a given
background intensity; that is, we would adjust the intensity of the spot until
it was just barely visible, and then record the difference between the intensity
of the spot and the intensity of the background. We call this difference A I (or
Just Noticeable Difference (JN D )). Then we would increase the background

Figure 9.8: A light adaptation curve is characterized by five regions, labeled A


through E (4, pp. 45).

Copyright 2014 by KMK Educational Services, LLC


CHAPTER 9. VISUAL PERCEPTION 385

intensity and repeat the measurement. Figure 9.8 shows a light adaptation
curve.
Characteristics of the curve
The light adaptation curve has five characteristic regions. Regions A-D corre­
spond to the Scotopic Portion, while region E corresponds to the Photopic
Portion of the curve.
1 Region A;
Here, the slope is 0. In this region, vision is dominated by the fluctua­
tions and corresponding uncertainty in the response of individual neurons.
Sometimes this is called “neural noise,”
2 Region B:
Here, the slope is 1/2. In this region, vision is dominated by fluctuations
in the background light. This noise is due primarily to statistical uncer­
tainties in the background photon bath (fluctuations in the number of
photons in the background). We quantify this with the DeVries-Rose
Law:
(9.9)
As background I doubles, I increases by a factor of 21/2.
3 Region C:
Here, the slope is 1. This is known as W eber’s Law.
AI ot I q (9.10)
4 Region D:
Here, the slope is infinite. This is due to saturation of rods.

It only takes bleaching of 10% of all rhodopsin molecules to satu­


rate. This is because saturation results from the closing of some
critical number of Na+ channels in the photoreceptors, which oc­
curs when 10% of rhodopsin molecules are bleached.

5 Region E:
Here the slope is 1. This is also Weber’s Law.

Copyright 2014 by KMK Educational Services, LLC


386 9.3. LIGHT PERCEPTION

There is no saturation effect for photopic vision.

This shows that simultaneous contrast plays a big role is determining an


object’s appearance. This is just an academic way of saying what we all know:
stimuli look different against different backgrounds, and in fact an individual’s
threshold will vary depending on background intensity. It is instructive to
see the way the psychophysical concept of JN D allows us to quantify this
perceptual behavior.

Now consider the following: how does the intensity of light sensation grow far
above threshold?
Fechner’s Log Law
Fechner’s log law simply assumes that Weber’s law holds above threshold (4,
pp. 266-68). This leads to the following equation:
S — k log(J) (9.11)
where S is the magnitude of sensation (perceived brightness, for example), I is
stimulus intensity, and k is a constant (related to Weber’s constant).

In fact, Fechner’s Log Law does NOT match with experimental


evidence.

Stevens’ Power Law


Stevens’ empirically-derived law gives the correct formula. Note that it is
different than what Fechner’s Law predicts:
S =rk' (9.12)
where S is the magnitude of sensation, I is the stimulus intensity, a is the
scaling exponent, and k' is a constant (4, pp. 266-68).

Sensation scales as a power law with stimulus intensity rather than


scaling logarithmically, as Weber’s law would predict.

Copyright 2014 by KMK Educational Services, LLC


CHAPTER 9. VISUAL PERCEPTION 387

- SECTION 9.4 -------------------------------------------------------------------

Spatial and Temporal Summation

The way that the visual system combines inputs over time and space underlies
some of the differences in Scotopic and Photopic Vision. We review some of the
fundamental concepts here (4, ch. 3). The basic idea to keep in mind is that of
a “pixel,” or the smallest division of space or time that the visual system sees.
These divisions are not unlike pixels on a computer monitor or television screen.
Everything in the world around us is divided into small “pixels” by the visual
system. Everything within a given pixel is added together, and stimuli smaller
than a single pixel cannot be clearly resolved. These “pixels” do not have to
be purely spatial. In addition, we can think of all visual events that happen in
some very small amount of time as having occurred during one “pixel” of time.
We will speak more about this below.

We say that spatial summation is higher in scotopic vision than it is in photopic


vision. This explains the higher sensitivity and lower acuity associated with
scotopic vision (4, pp. 47-51).
The visual system divides space up into pixel-like regions. Within each pixel,
all visual input is added together. This summation takes place over a larger
area in the scotopic system; thus, we are saying that the “pixels” are larger
in scotopic vision than photopic vision. Consequently, the scotopic system is
characterized by poor acuity (as we cannot see things smaller than one pixel)
but great sensitivity (the pixel covers a large region of space, and we add up
input at every point in space, so we get more input overall contributing to each
pixel).
Biophysical explanation
On average, many rods communicate with a given ganglion cell. By contrast,
relatively few cones communicate with a given ganglion cell. This is the basis
for higher spatial summation in the Scotopic system (see Toy Example below).

Spatial Summation: A Toy Example


Let us consider a simple toy model similar to that in Schwartz (4, pp.49).
Suppose that for a ganglion cell to signal an event, it must receive input from 5
photoreceptors (either 5 rods or 5 cones, depending on whether we are dealing
with the scotopic or photopic system). Let us also say that for a photoreceptor
to be activated, it must absorb 1 quantum of light. Now shine two flashes of
light on our simple toy visual system, with each flash containing enough photons
Copyright 2014 by KMI< Educational Services, LLC
388 94- SPATIAL AND TEMPORAL SUMMATION
to activate exactly three photoreceptors (3 rods or 3 cones). Obviously we are
sacrificing biological validity for the sake of simplicity.
Figure 9.9 represents our toy system. The ovals represent flashes of light, the
small rectangles represent the photoreceptors, and the large rectangles repre­
sent retinal ganglion cells. In the left picture, each retinal ganglion cell gets
input from 5 photoreceptors, so this is similar to the photopic system (few in­
puts per retinal ganglion cell). The right picture shows a retinal ganglion cell
receiving input from 10 photoreceptors, so this is more like a scotopic system
(many inputs per retinal ganglion cell). As we said above, it takes five “on”
photoreceptors to elicit a response in the ganglion cell. In the photopic picture,
the total input is 3, so the ganglion cell is “off.” In the scotopic picture, the
total input is 6, so the ganglion is active. As this illustrates, the stimulus can
be detected by the scotopic system, but not by the photopic system. However,
note that in this example, the scotopic system would only see one flash of light
(because all of the photons from the two flashes are summed together into one
“pixel”).

• The photopic system has high resolution but poor sensitivity.


• The scotopic system has poor resolution but high sensitivity.

Ricco’s Law: An Equation of Spatial Summation


Consider the following experiment: present to an observer a spot of area A.
Next, allow the observer to change the intensity of the spot until it becomes
visible. Record this intensity (the intensity at which the spot is visible). Repeat
this experiment for different spot sizes. Then plot the intensity of the spot at

Figure 9.9: A toy model for understanding spatial summation (4, pp.49).

Copyright 2014 by KMK Educational Services, LLC


CHAPTER 9. VISUAL PERCEPTION 389

threshold (in terms of number of photons that make up the spot) vs. spot
diameter. A typical plot is shown in figure 9.10.
Critical diameter: There is some diameter below which the critical intensity
is constant (Figure 9.10). This means everything smaller than this is
being summed together. In other words, this is the diameter of the “pixel.”
Ricco’s law: Ricco’s law provides a rule for stimuli smaller than the spatial
summation threshold. It essentially says that the number of quanta in a
subthreshold stimulus is constant.
IA = C, (9.13)
where I is stimulus intensity (quanta per area), A is the stimulus area,
and C is an unimportant constant.

Exam ple 9.7: C onsider a stim u lu s w hich is a s m a ll circle o f rad ius r . T h e corre­
s p o n d in g in te n s ity o f th e s tim u lu s a t th res h o ld is fo u n d to be I . N o w con sider a
s tim u lu s t h a t is id e n tic a l b u t fo r a 3 fo ld increase in radius. W h a t does R ic c o ’s la w
p re d ic t fo r th e change in s tim u lu s in te n s ity a t th re s h o ld fo r this n e w s tim u lu s ?

Solution 9 .7: T h e p ro d u c t ( I A ) m u s t re m a in c o n s ta n t. A s rad ius increases b y 3


fold, area increases b y 9 fold. T h erefore, I changes b y a fa c to r o f 1 /9 .

Figure 9.10: Spatial Summation, Temporal Summation, and Pixel Size (4,
pp. 51,54). To save space and highlight the analogy between spatial and tempo­
ral summation, we include both plots in one figure. The dashed lines represent
the locations of the critical diameter and critical duration.
Copyright 2014 by KMK Educational Services, LLC
390 9.j. SPATIAL AND TEMPORAL SUMMATION

Exam ple 9.8: A ssum e K y le stares a t tw o o bjects separated by a distance c /2 ,


w here c is th e c ritic a l d ia m e te r. H o w fa r a p a rt do th e o bjects a p p e a r to K y le ?

Solution 9.8: T h e c ritic a l d ia m e te r represents th e “p ix e l” size o f K y le 's visual


system . I f tw o o b jects are b o th c o n ta in e d w ith in one “p ix e l,” th e y c a n n o t be d is tin ­
guished. Thus, K y le w ill o n ly see one o b je c t-a s s u m in g th a t the c o m b in e d in te n s ity
is s u fficie n t to eclipse visual th re s h o ld -a n d so i t is inap p ro p riate to ta lk a b o u t
distance o f sep aratio n .

The scotopic system has a larger critical diameter (larger pixels).

Temporal summation is really the same idea as above, but now we deal with
time rather than space (4, pp. 51-57). That is, the visual system divides time
up into chunks and adds up all the information within a given chunk of time.
Thus the “pixels” are now chunks of time rather than chunks of visual space.

Temporal Summation: A Toy Example


Let us consider a light which flashes at random times, as shown in Figure 9.11.
The size of the time windows or “pixels” for a toy visual system are given by
the vertical lines. Now we consider three different trials using the randomly
flashing light. In this example, we will suppose that the model visual system
has a threshold of two flashes: that is, at least two flashes must occur in one
time window to be detected. In the top frame, the model system perceives one
bright flash followed by two intervals with no flash. In the middle frame, two
flashes are perceived, and in the bottom frame, three flashes are “seen.” Note
that the temporal spacing of the stimuli determines what is seen; all examples
consist of six total light flashes, but the perception of these stimuli varies. A
similar toy example is given in Schwartz (4, pp, 52).
Copyright 2014 by KMK Educational Services, LLC
CHAPTER 9. VISUAL PERCEPTION 391

time
>

Figure 9.11: A toy model for understanding temporal summation (4, pp. 52-53).

Bloch’s Law: An Equation for Temporal Summation


Consider the following experiment: present to an observer a flash of light of
given duration and intensity. Allow the subject to adjust the intensity of the
flash until it becomes visible. Then record this intensity (intensity at which the
spot is visible). Repeat for different flash durations. Finally, plot the intensity
(given by number of photons in the spot) vs. flash duration. See Figure 9.10.

Critical duration: There is some duration r below which everything is added


together. Everything on a smaller timescale is just summed. The scotopic
system has a larger critical duration (r « 100 ms) than the photopic
system (r « 10 ms), meaning that the time “pixels” are larger in the
scotopic system.
Copyright 2014 by KMK Educational Services, LLC
392 9.5. COLOR PERCEPTION
Bloch’s Law: Bloch’s law is similar to Ricco’s, but for temporal summation.
It = Ch, (9.14)
where I is the stimulus intensity (quanta per time), t is the stimulus
duration, and C\ is an unimportant constant.
Stiles-Crawford Effect
The angle at which light strikes cones strongly affects the perception of the
light. Rays that strike orthogonal to the surface of the cones appear much
brighter. This effect is absent in rod-dominated vision.
- SECTION 9.5 ------------------------------------------------------------------- ------------------------

Color Perception

We review the fundamentals of color perception (4, ch. 5,6) (2, ch. 8), which
is a critical topic for board exam success. The theory of color perception is
a combination of several interrelated ideas. The two main components are
trichromatic theory and color opponent theory (4, pp. 93-94,114-20).
^Trichromatic theory - c
Trichromatic theory says that there are three types of cone photoreceptors,
each characterized by the peak wavelength in its absorption spectrum: short
(S), middle (M), and long (L). Note that there are also rod photoreceptors, but
this does not factor largely into trichromatic theory. To say this another way,
each type of photoreceptor has a characteristic absorption spectrum (which
measures the probability of absorbing a photon at a given wavelength). Each
peaks at different wavelengths. Recall that Univariance says that the response
to a photon absorption is universal, so these different absorption probabilities
must underlie the ability of the visual system to distinguish different stimuli.

Photoreceptor Pigment t
S cones cyanolabe 440 nm
M cones chlorolabe 534 nm
L cones erythrolabe 564 nm
Rods rhodopsin 498 nm

Copyright 2014 by KMK Educational Services, LLC


CHAPTER 9. VISUAL PERCEPTION 393

Evidence suggests that the S cone system is less robust against


pathological damage. Early-stage glaucoma patients show S-cone
visual field loss, even when complete visual field loss is not readily
apparent (4, pp. 132-33).

Color Ambiguity and Metamers


Two visual stimuli will appear identical if viewing them results in the same
number of photon absorptions by each photoreceptor. As a simplified example,
consider a visual system made up of only one S cone. A stimulus that consists
of a small number of photons of short wavelength (say A = 440 nm) light may
look similar or identical to another stimulus containing many more photons of
longer wavelength light (perhaps A = 460 nm). Even though the probability
of absorbing a single short wavelength (440 nm) photon is much higher than
the probability of absorbing a single 460 nm photon, there are many more 460
nm photons and therefore a similar number of total absorptions take place,
on average. Essentially, we are saying that there is some interplay between
intensity (number of photons) and wavelength of a stimulus, and therefore two
stimuli that are physically different may look identical.
Metamers: Two visual stimuli which are physically different but appear iden­
tical are known as metamers. The idea of a metamer underlies the reason
for limits in color discrimination (4, pp. 102-04).
In a visual system with all 3 photopigments, the situation is more complex
than in the illustrative S cone only example above, but the idea is the
same. That is, the overlap in the absorption spectra (combined with
the principle of univariance) limits the ability of the visual system to
distinguish between physically different stimuli.

Exam ple 9.9: C o nsider a s im p lifie d visual system c o n ta in in g o n ly L cones. T w o


s tim u li are presented: one o f w avelen g th 564 n m a n d th e o th e r o f w avelen gth
580 n m . L e t us assum e th a t th e s im p lifie d visual system sees th e tw o s tim u li as
m etam ers. W h a t can we say a b o u t th e re la tiv e in te n s ity o f th e tw o s tim u li?

Solution 9.9: T h e L cone absorbs m a x im a lly a t 5 6 4 n m . R e c a ll t h a t u n ivarian ce


im p lies t h a t th e n u m b e r o f q u a n ta l ab so rp tio n s d e te rm in e s w h a t w ill b e seen. T h e
tw o s tim u li are p hysically d iffe re n t , b u t th e n u m b e r o f q u a n ta l ab so rp tio n s w ill
b e th e sa m e (on a v erag e) i f th e 5 8 0 n m source is m o re intense. T h a t is, th e
a d d e d in te n s ity (a d d itio n a l p h o to n s ) w ill co m p e n s ate fo r th e decreased abso rp tio n
p ro b a b ility o f p h o to n s a t 5 8 0 nm .

Copyright 2014 by KMK Educational Services, LLC


394 9.5. COLOR PERCEPTION
Grassman’s Laws: These axe mathematical implications of the principle of
univariance. They give general rules for dealing with metamers.
1 Suppose we add radiation (perhaps light of another wavelength) to
two metamers. They remain metamers if the radiation applied to
each is the same.
2 Suppose we increase or decrease the intensity of two metamers by
the same amount. They remain metamers (but will look brighter or
dimmer than before).
3 If stimuli A and B are metamers and B and C are metamers, then
A and C are metamers.

Color opponency theory posits the existence of three channels for color vision.
The channels, which are listed below, treat certain pairs of traits as “oppo­
nents/5in some sense. Take the first channel as an example. Color opponency
theory says-loosely spealdng-that the componentry used for seeing red works
against the componentry used for seeing green. That is, cells that respond
with excitement to red may respond with inhibition to green, and therefore the
colors work against each other. The three color opponent channels are:
1 Red-Green
2 Blue-Yellow
3 Brightness
Sensitivity Effects of Opponency
The effects of color opponency are seen when measuring the sensitivity of an
individual to various wavelengths. One finds that a normal human has three
peaks in his/her sensitivity spectrum, and they fall at roughly 440 nm, 520 nm,
and 620 nm. That is, the person is most sensitive to light at these wavelengths.
However, these do NOT correspond exactly to the peaks in the S, M, and L
cone absorption spectra (which were roughly 440 nm, 534 nm, and 564 nm,
respectively). This is a result of color opponent processing of the M and L
cone inputs, which shifts the location of the peaks.
Supportive electrophysiological evidence
Electrophysiological studies confirm the existence of color opponent channels,
in the following sense (4, pp. 116-18):
1 Color opponent cells:
Copyright 2014 by KMK Educational Services, LLC
CHAPTER 9. VISUAL PERCEPTION 395

Studies that record from neurons in the visual system show that certain
cells, termed color opponent cells, respond to part of the electromagnetic
spectrum with excitation and to part of the spectrum with inhibition.
Therefore, the response of these cells to a stimulus gives information
about the color of the stimulus.
2 Noncolor opponent cells
In contrast to color opponent cells, some cells are found to be always
excited or always inhibited by a stimuli. These cells do not respond
differently to different parts of the spectrum and therefore do not provide
useful information about the wavelength (color) of a stimulus.

Color can be described by three characteristics (4, pp. 108-10):


1 Hue:
Essentially, hue is the perception we attribute to wavelength. As such,
hue has a meaning that is most closely linked to our intuitive sense of
color.
2 Saturation:
Intuitively, it’s easier to think of desaturation than saturation. If some­
thing is more desaturated, it appears to contain more white. As an
example, pastel blue is more desaturated than deep navy blue.
Quantifying Saturation: Colorimetric purity (p): Colorimetric pu­
rity is used to quantify saturation. A high value of p indicates a
highly saturated color.

L\ + Lw (9.15)
where L \ is the luminance of the pure wavelength light comprising
the stimulus in question and Lw is the luminance of white light
comprising the stimulus in question.
3 Brightness: Brightness is closely related to the V function defined in
Section 9.2. It matches well with our intuitive notion of brightness.
A system related to these concepts is the Munsell Color Appearance Sys­
tem. The Munsell system is a system for specifying any color (it makes match­
ing paint colors easier). We can specify any color with three numbers:
1 Hue (related to wavelength, as above)
2 Chroma (related to saturation)
Copyright 2014 by KMK Educational Services, LLC
396 9.5. COLOR PERCEPTION
3 Value (reflectance of sample; related to brightness)
If we specify a value for all three characteristics above, we have completely
specified the color.

Our perception of color depends on more than simply the wavelength of the
light (4, pp. 112-14),
Bezold-Brucke Phenomenon
Wavelengths show a slight change in hue as the intensity of light is increased.
That is, our perception of hue is not totally independent of our perception of
brightness.
Color Constancy
Colors (on a fabric, for example) appear the same even under very different
lighting conditions. For example, the stripes on a flag appear red and white
whether we see the flag at a baseball game on a summer day or in a dimly
lit hallway at school. This is a cool phenomenon because the reflected wave­
lengths which actually hit our eyes ARE very different under different lighting
conditions. The take home message is that our brains somehow process these
higher level contextual clues to maintain the illusion of color constancy.
!CIE Goior <r - . - v : : ■/
. • •••"
lima
We can combine Red (645 nm), Green (526 nm), and Blue (444 nm) light to
yield a stimulus that matches with any other (sample) wavelength. We would
like to quantify this so we can speak about how much of each primary color we
need to create another color.
CIE: Arriving at x, y (4, pp. 121-32)
Consider the following experiment. Present to an observer a stimulus of arbi­
trary color. The subject is asked to mix red, blue, and green so that the mixture
looks identical to the given sample. For the most part this is straightforward,
but a problem arises: sometimes the person must actually add a primary color
(Red, Green, or Blue) to the sample (rather than the mixture) to achieve a
perfect match. We refer to this as adding a negative amount of that color.
Changing the coordinates: To avoid these negative values, we change the
coordinate system in color space. This is just a mathematical transfor­
mation which says that rather than describing everything in terms of
Copyright 2014 by KMK Educational Services, LLC
CHAPTER 9. VISUAL PERCEPTION 397

Figure 9.12: An example demonstrates use of the CIE system.

building blocks R, G, and B, we can use different building blocks (we’ll


call them X, Y, and Z) so that the amounts used (of X, Y, and Z) are all
positive.
Imaginary primaries: X, Y, and Z are called imaginary primaries. Now we
can specify any color by giving the (X,Y,Z) coordinates.
Tristimulus values: The coordinates (or amounts of X, Y, and Z used) are
called tristimulus values. We make one further change to simplify
things. Instead of (X,Y,Z), we use (x,y,z) coordinates, where
X
XVY + Z
Y
X + Y + Z
(9.16)
Z
X + Y + Z

This is better because now x + y T z = 1, so we only need to specify two


of them.

Copyright 2014 by KMK Educational Services, LLC


398 9.6. COLOR VISION ANOMALIES

It is not important to understand the fine details of what x and y


are, but it is crucial to understand that they are two descriptive
numbers which can be used to specify any color stimulus. This
system is very helpful because now we can plot colors in the two
dimensional (x,y) plane.

Using CIE: Consider an example similar to that in Schwartz (4, pp. 127-28).
Let us mix 2 “unit amounts” of 570 nm light with 1 “unit amount” of 500
am light. We can use our (x,y) system to find the dominant wavelength
of the mixture as well as the excitation purity, which is related to
saturation. First notice that the pure wavelengths trace out a curve in
the x-y plane. Any mixture will fall somewhere in this x-y plane as well.
As shown in Figure 9.12, we start by locating the positions of the two
pure wavelengths on the CIE diagram. From here, we can:
1 Draw a line between the two wavelengths being mixed.
2 Find the point on this line where the mixture in question lies. In
our example, the mixture contains twice as much 570 and 500, so
we expect the distance from the mixture point (on the line) to 570
to be half of the distance from the mixture point to 500. That is,
we are closer to 570 than to 500.
3 Draw a line originating at white and passing through the mixture
point. The intersection of this line with the CIE pure wavelength
arc gives the dominant wavelength.
4 The excitation purity, which is related to saturation, is then given
by
Pe =
d\
d\ + d2 ’
(9.17)
Large Pe indicates a high level of saturation (that is, the color is
located far from white on the diagram).
S E C T I O N 9.6

C o lo r V i s i o n A n o m a lie s

We cover the fundamentals of some typical anomalies in color vision (4, ch. 6) (1).

A dichromat is missing one of the three retinal photopigments. The three types
of dichromats are listed below:
Copyright 2014 by KMK Educational Services, LLC
CHAPTER 9. VISUAL PERCEPTION 399

Protanope: Protanopes are missing erythrolabe and readily confuse red, or­
ange, yellow, and green. In addition, they tend to see reddish hues as
very dim and almost gray. Also, colors that involve a mixture of another
color with red tend to look indistinguishable from the other color because
the dimming can be so pronounced. For example, purple-a mixture of
blue and red-will look almost identical to blue.
Deuteranope: Deuteranopes are missing chlorolabe. They show the same
problems with color confusion as do protanopes, except there is NOT the
noticeable dimming of reds to gray, as in protanopes.
Tritanope: Tritanopes are missing cyanolabe and tend to confuse blues and
yellows.

General Trends of Dichromacy


• Red-green defects (as in protanopes and deuteranopes) are usually inher­
ited.
• Blue-yellow defects (as in tritanopes) are usually acquired.

K §® SH
Anomalous trichromats have all 3 photopigments, but the absorption spectrum
of one of the pigments is displaced.
Protanomalous trichromat: The L cone (erythrolabe) spectrum is shifted
to shorter wavelengths. As a result, red, orange and green all look similar
and all look greener than they might to a normal person. Reds may again
be dimmer (though the effect is less pronounced than in a protanope).
Conceptually, one may think of these individuals as having a weakness
when it comes to seeing reds.
Deuteranomalous trichromat: The M cone (chlorolabe) spectrum is shifted
to longer wavelengths. As a result, red, orange and green all look similar,
but this time they look more red than they might to a normal person.
One may think of these individuals as having weak green perception.

X-chrom Contact Lenses: X-chrom contact lenses are red-tinted lenses


which act as long pass filters, thereby shifting the effective absorption peak
associated with a given eye to longer As (4, pp. 134-36).

Copyright 2014 by KMK Educational Services, LLC


400 9.6. COLOR VISION ANOMALIES

500 600

Mnm)

Figure 9.13: Color discrimination curves for dichromats.

Figure 9.14: Color confusion lines for protanopes and deuteranopes.

Copyright 2014 by KMK Educational Services, LLC


CHAPTER 9. VISUAL PERCEPTION 401

Wearing an X-chrom contact lens in one eye can help a patient


with mild color vision difficulties. By shifting the absorption peak
associated with one eye, it essentially creates a new spectral sen­
sitivity curve for that eye. Now each eye is sensitive to different
wavelengths, meaning that more discrimination is possible, in prin­
ciple (4, pp. 134-36).

We can use several quantitative tests to gain further insight into color vision
anomalies.
Chromatic discrimination
By how much must we change wavelength to get a perceived change in hue?
We can quantify this by showing an observer a spot of a certain wavelength
on a background of the same wavelength. We then ask the subject to adjust
the wavelength of the spot until it looks different from the background, and
we record this change in wavelength (which we call AA). Figure 9.13 shows a
typical example (4, pp. 145).

Protanopes and deuteranopes have poor color discrimination at


wavelengths A > 540 nm or so. Tritanopes have poor color dis­
crimination at wavelengths A ~ 495.

Color Confusion Lines


Color Confusion Lines on a CIE diagram are lines in the x-y plane along which
all colors are indistinguishable. All lines will originate from one point, which
is called the copunctal point. Figure 9.14 shows typical color confusion lines
for common dichromats (4, pp. 148-49).
Pseudoisochromatic Plate Tests
To test color vision, pseudoisochromatic plates are typically presented to the
patient in a booklet form and consist of many images whose complexities cannot
be fully discerned without proper color vision. For example, one image may
be embedded in another in such a way that color discrimination is required to
see both images (4, pp. 159). To be valid, the test should be conducted in
daylight illumination or using a M acbeth Illuminant C lamp (3).

Copyright 2014 by KMK Educational Services, LLC


402 9,6. COLOR VISION ANOMALIES

Traditional pseudoisochromatic plates-called Ishihara plates-


typically cannot be used to diagnose blue-yellow defects nor to dis­
tinguish between dichromats and anomalous trichromats. However,
the modern so called “HRR plates” can diagnose both blue-yellow
and red-green defects.

Farnsworth D-15 Test


The Farnsworth test (or dichotomous test) consists of arranging 15 colored
chips in a sequential manner which reflects the patient's ability to distinguish
color (4, pp. 160-63).

The Farnsworth D-15 test allows for the diagnosis of protan, deutan
(red-green), and tritan (blue-yellow) anomalies, but cannot distin­
guish between dichromats and anomalous trichromats.

Nagel Anomaloscope
The NA is used to distinguish between dichromacy and anomalous trichromacy
in the diagnosis of red-green defects. A patient is shown a circle: on one half of
the circle is the test field-a 590 nm stimulus whose radiance can be adjusted-
and on the other half of the circle is the mixture field-a mixture of 670 nm
and 546 nm lights whose ratio can be adjusted. The patient is asked to adjust
the proportions of the mixture field and also the test field until both appear
identical. The mixture can be adjusted from a setting of 0 (meaning only 546
nm light) to 73 (meaning only 670 nm light). The test field can be adjusted
from a setting of 0 (meaning very dim yellowish light) to a setting of 87 (very
bright yellowish light) (4, pp. 163-68).

The three wavelengths used in a Nagel Anomaloscope all fall on a


color confusion line seen in both deuteranopes and protanopes.

Diagnosis: The settings used by individuals with different color vision diffi­
culties will vary. Here is a brief summary (4, pp. 163-68):
Normal Individual: A normal trichromat will set the mixture scale to around
45 and the test scale to around 17 (4, pp. 165) to achieve a match (which
appears yellow).
Copyright 2014 by KMK Educational Services, LLC
CHAPTER 9. VISUAL PERCEPTION 403

Protanope: A protanope will be able to achieve a perceived match between


the test and mixture field when the mixture field is set anywhere from
0 to 73. Of course, a normal individual will not see this as a match,
but for the protanope, 670, 546, and 590 nm light all fall on the same
color confusion line and can therefore be made to appear metameric.
In addition, a protanope will see a 670 nm stimulus as being very dim,
as discussed above. If the mixture field is set to 73 (all 670 nm), the
protanope will set the test field to a very low setting to achieve a match.
By contrast, he/she will set the test field to a higher setting if the mixture
field is set to 0 because the dimming only occurs for reddish colors.
Deuteranope: A deuteranope shows similar characteristics to a protanope,
but he/she will not observe a dimming of the longer wavelength stimulus.
Therefore, he/she will use a test scale setting of around 17, regardless of
the setting of the mixture. In this regard, he/she is just like a normal
individual.
Protanomalous trichromat: When asked to match the two samples, this
person will do so by adjusting the mixture field to a higher-than-normal
range. This is because the protanomalous trichromat is red-weak, mean­
ing he/she must add more 670 nm light than a normal individual.
Deuteranomalous trichromat: When asked to match the two samples, this
person will do so by adjusting the mixture field to a lower-than-normal
range. This is because the deuter anomalous trichromat is green-weak,
meaning he/she must add more 546 nm light than a normal individual.

Exam ple 9 .10 : A n d r e w is asked to a d ju s t th e te s t field in an a n o m a lo s co p e u n til he


achieves a m a tc h w ith th e m ix tu re field, w hich is s e t to zero. H e is u n ab le to achieve a
m a tc h fo r a n y value o f th e te s t field. H o w ever, w hen he is allo w e d to a d ju s t th e m ix tu re
field, he can achieve a m a tc h w hen he uses a s e ttin g o f 55. D iagnose A n d re w .

Solution 9 .10 : Because A n d r e w c a n n o t achieve a m a tc h w hen th e m ix tu re fie ld is set


to 0, i t is c le a r t h a t A n d re w is n o t a d ic h ro m a t, b u t in s te a d an a n o m a lo u s tric h ro m a t.
Because he also uses too m u ch red (lo n g w a v e le n g th ) lig h t w hen a c h ie v in g a m a tc h
b etw een th e variable m ix tu re field a n d th e te s t field, i t is c lear th e A n d r e w is “r e d -w e a k ,”
o r a p ro ta n o m a lo u s tric h ro m a t.

Exam ple 9 .1 1 : S h aro n is asked to p e rfo rm th e sa m e m a tc h in g te s t as A n d r e w in th e


previous q uestion, e x cep t t h a t th e m ix tu re field is set to 73. S h e achieves a m a tc h w hen
th e te s t field is set a t a very lo w value. D o es Sharon s h o w signs o f n o rm a l colo r vision?

Copyright 2014 by KMK Educational Services, LLC


404 9.6. COLOR VISION ANOMALIES
Solution 9 .1 1 : S h aro n is d e a r ly a p ro ta n o p e . S h e shows a d im m in g o f reds, as in d ic a te d
b y th e m a tc h in g o f th e p u re 6 7 0 n m m ix tu re w ith very d im te s t field. In a d d itio n , th e
fa c t t h a t she can a ch ieve such a m a tc h a t a ll in d icates t h a t 6 7 0 n m a n d 5 9 0 n m fa ll on
sam e colo r confusion line.

Color vision defects can be either hereditary or acquired via disease (4, pp. 153-
59).
Inherited (Genetic) Defects
Hereditary defects tend to occur in both eyes and do not change drastically over
time. They are more common in males and tend to affect red-green vision. Note
that red-green defects are typically X-linked recessive, which explains why they
occur at a higher rate in males.
Acquired Defects
Acquired defects tend to affect one eye more than the other and may change
over time. They tend to affect blue-yellow vision and occur with nearly equal
frequency in both males and females.
Kollner’s Rule: Kollner’s rule gives a very rough guideline for assessing
acquired diseases. It says that outer retinal disease typically leads to blue-
yellow defects, while inner retinal, optic nerve, and/or visual pathway problems
typically result in red-green defects.
Achromatopsias
Achromatopsias are serious but rare conditions marked by essentially monochro­
matic vision. The most common type is rod monochromacy, which typically
results in greatly reduced visual acuity and color discrimination (4, pp. 158).

Exam ple 9 .1 2 : P a u l shows a colo r vision d efect w hich a ffects o nly his le ft eye. W o u ld
you guess this to be an a c q u ire d o r g e n e tic d efect?

Solution 9 .1 2 : T h e d efe c t in questio n does n o t a ffe c t b o th eyes equally ; a n d i t is


therefore likely a c q u ire d r a th e r th a n g en etic. In a d d itio n , we w ould ex p e c t such a d e fe c t
to a ffe c t b lu e-yello w vision.

Copyright 2014 by I<MK Educational Services, LLC


CHAPTER 9. VISUAL PERCEPTION 405

Chromatopsias
Chromatopsias lead to a distortion of color, but not a deficiency in color dis­
crimination ability (4, pp. 159).

Cataracts act as a yellow filter. After cataract removal, pa­


tients will sometimes speak of chromatopsias-specifieally, of seeing
“blue”-as a result of the relative increase in transmission of blue
light through the lens (4, pp. 159).

- SECTIO N 9.7 ---------------------------------------------- -------------------------

Perception of Form and Space


We introduce ideas related to the perception of space and form (4, ch. 7,8).

We can most easily understand contrast by considering an example. Suppose


we are given a sin wave grating and want to calculate the contrast, A sine wave
grating is an image which looks similar to blurred black and white stripes (see
(4, pp. 178)). The luminance changes sinusoidally (it goes up and down, hence
the stripes). To quantify our intuitive notion of contrast, we use the following
equation:
G = Lmax~ Lavs-,
Lavg
(9.18)
where Lmax is the maximum luminance (this would occur on the white part of
the stripes) and LaVg is the average luminance of the grating (4, pp. 176-79).
Contrast is typically normalized so that it ranges from 0 to 100.
Spatial Contrast Sensitivity Function
This is a type of Spatial Modulation Transfer Function (SMTF). An
SMTF tells us how well a system (e.g. a lens) transforms input into output, in
terms of contrast. That is, if the object we are viewing has high contrast, does
the image (after passing through the lens) have significantly less contrast? The
SMTF helps us answer this question.
SMTF: Assume we are given a square wave grating of known contrast and
frequency. We can then use a lens to produce an image of this grating. We
can measure the contrast of the image, Ci, which will have a lower contrast
Copyright 2014 by KMK Educational Services, LLC
406 9.7. PERCEPTION OF FORM AND SPACE

Contrast
Sensitivity

Figure 9.15: Typical contrast sensitivity function (CSF) for a human.

than the original grating, Co. The ratio ^ gives the value of the SMTP for the
given frequency of the grating, Repeat this procedure for a range of frequencies
to get the entire SMTP function (4, pp. 180-84).

A poorly focused lens leads to poor image contrast. This affects


higher frequencies MORE than moderate frequencies.

Contrast Sensitivity Function (CSF) An SMTF for a human is called


a CSF (see Figure 9.15). To measure a CSF, one would take the following
steps (4, pp. 184):
1 Show a subject a sine grating of very low contrast and known frequency.
To the subject, it will look like a screen of uniform luminance because
the contrast is too low to see the grating.
2 The subject is then told to increase the contrast until he/she sees the
grating. This value of contrast is the threshold value.
3 CSF is the reciprocal of this threshold,
4 Now repeat for many frequencies to get the entire CSF.

Exam ple 9 .1 3 : W o u ld you e x p e c t th e high freq u en cy c u to ff o f th e C S F to b e larg er u n d er


b rig h t o r d im lig h tin g co n d itio n s?

Copyright 2014 by KMK Educational Services, LLC


CHAPTER 9. VISUAL PERCEPTION 407

S olu tion 9 .1 3 : B ecause high frequ ency c u t o ff is re la te d to visual acuity, we k n o w th a t


th e c u t o ff is h ig h e r u n d e r p h o to p ic conditions.

Characteristics of CSF: A CSF shows a high frequency cutoffs which


indicates that there is a limit to our ability to resolve fine spatial detail. This
cutoff corresponds to a patient’s measured visual acuity (see below). As ev­
eryone knows, the typical chart for measuring “Recognition Acuity” consists
of letters (called opotypes) that are at very high contrast. Still, very small
letters cannot be seen. In terms of CSF, this is because the spatial frequency
components of the letters are very high and fall above the high frequency cutoff
of the CSF. Note that the CSF also shows a low frequency cutoff.
Reasons for the CSF high frequency cutoff (4, pp. 184-87):
• Optical limitations (aberrations occur in our visual system even at perfect
focus).
• Density of photoreceptors is finite, and therefore our ability to resolve
spatial detail is physically limited.
Reasons for the CSF low frequency cutoff (4, pp. 190):
• Lateral inhibition in the retina.
• The ganglion cells in the retina show a center-surround visual field. This
means that stimuli which fall in the center of the field cause the cell to
be excited, while those that fall on the periphery cause the cell to be
inhibited. A large (low frequency) stimulus would elicit the inhibitory
effects of the peripheral visual field, and this limits our low-frequency
vision.
CSF and the Clinic: Recall that high frequency cutoff is related to visual
acuity. We can connect this with Snellen fraction and MAR (4, pp, 195-96).
Minimum Angle of Resolution (MAR): The MAR represents the angle
(as measured from the eye) between two just resolvable bars. We can
relate this to the high frequency cutoff of the CSF. First, we recall that
Snellen fraction is given by
1 (9.19)
MAR.
where SF is the Snellen fraction and MAR, is typically measured in ar-
cminutes (1/60 of a degree).

Copyright 2014 by KMK Educational Services, LLC


408 9.7. PERCEPTION OF FORM AND SPACE

A patient with cataracts may show only a moderate loss in visual


acuity as measured by his/her Snellen fraction, even though he/she
speaks of profound visual difficulty. This is because Snellen fraction
only deals with the ability to resolve high spatial frequencies, and
cataracts affects all frequencies (4, pp. 197-98).

.Exam ple 9 .14 : C o nsider a p a tie n t whose high frequ ency c u to ff is 15 cycles p e r degree.
E s tim a te his Sn ellen fractio n .

Solution 9 .14 : We firs t w rite

1 c v c Iq
/ m ax = no. cycles p e r d eg ree — 15 cyc le s /d eg re e = 4----------
arcm ins
:— , (9.20)
We n o w fin d th e p a tie n t's M A R . F o r th is , we w a n t to k n o w how th e m a x im u m resolvable
frequ ency relates to th e m in im u m an g le o f tw o ju s t resolvable bars. A s such, we consider
one cycle to b e o ne lig h t b a r a n d o ne d ark bar. R e la tin g this to the m a x im u m frequency,
we see t h a t th e p a tie n t can resolve 2 bars p e r 4 arcm inu tes, o r 1 b a r p e r 2 arcm in u tes.
B y d efin itio n , then , we h av e t h a t M A R = 2 arcm inu tes. T his corresponds to a Sn ellen
fra c tio n o f 20/40. T h e s h o rt a n s w e r is this: divide the cycles per degree by 30 to g e t
a S nellen fra c tio n in red u ced form .

jMach TSaricls arid tVfiUHer Analysis . ■'i* •■

Fourier Analysis
Fourier decomposition is a simple idea from mathematics that says a function
(given some minimal constraints) can be broken down into a sum of sines and
cosines. This is relevant for vision studies because:
• Every visual scene can be thought of as a function of space: at each
position in space, the “function” supplies a “pixel” value.
• This means that almost any visual scene can be broken into a combination
of simple scenes (like sine wave gratings) in the same way that most
functions can be broken down into a sum of simpler terms.
• As a result, we can study the simple scenes, all the while knowing that ev­
ery complex scene is made up of these simple scenes. This is considerably
easier than trying to study complex scenes directly.
Copyright 2014 by KMK Educational Services, LLC
CHAPTER 9. VISUAL PERCEPTION 409

Visual system and Fourier Analysis: The visual system works somewhat
like Fourier Analyzer (4, pp. 190-93). It breaks scenes down into various fre­
quency components (simple scenes) and then reassembles them to get complete
visual percept.
Mach Bands: As evidence for Fourier analysis in the visual system, consider
the following experiment. Show a subject a stimulus consisting of a strip with
a gradual decrease in luminance as a function of position (it is light on one end,
dark on the other). The person will see two bands (called Mach Bands) at the
junction of the transition; that is, the transition does not appear gradual, but
artificially abrupt.
The explanation is as follows. The gradual change of luminance on the band is
very low frequency. It is therefore hard to perceive. By comparison, the high
frequencies look enhanced, resulting in the appearance of obvious boundaries.

Mach bands suggest that the visual system is breaking down the
visual scene into individual frequency components and handling
these components separately.

|— SECTION 9.8

D epth Perception
Depth perception deals with how we judge relative distances of objects (4,
ch, 10). Humans use a combination of Monocular and Binocular cues to estab­
lish positional relationships between objects.

Figure 9.16: Crossed and uncrossed retinal disparity.

Copyright 2014 by KMK Educational Services, LLC


410 9.9. MOTION PERCEPTION

Monocular cues to depth include (4, pp. 235-39):


Pictorial Depth Cues: PDCs can be represented in 2 dimensions. Examples
include the use of angles, lighting, texture, and shadowing in art.
Motion Parallax: The relative motion between objects gives clues about the
positional relationship between the objects.

Binocular cues to the perception of depth largely rely on the comparison of the
location of the image formed on each retina (4, pp. 239-42). The technical term
for this is binocular retinal disparity (see Figure 9.16).
Uncrossed retinal disparity
If the image falls on the nasal side of the retina, the object is perceived to be
farther away.
Crossed retinal disparity
If the images fall on the temporal side of the retina, the object is perceived to
be closer.
Panum’s fusional area and Diplopia
The retinal disparity must not be too great for stereopsis to occur. The area
of the retina where stereopsis will occur is known as Panum’s fusional area.
If the disparity is too great (that is, the images do not both fall on Panum’s
area), the result is physiological diplopia (double vision). This is covered in
more detail in Physiological Optics and Binocular Vision.
- SECTION 9.9 -----------------------------------------------------------------------------------------------------

M otion Perception
We now discuss the basics of human motion perception (4, ch. 9).

fo iil vs. A pparent M otion ;;vi' ^


Some things appear to be moving, but are not actually in motion. Consider an
example. Around Christmas, we often see decorative lights where, for example,
Copyright 2014 by KMK Educational Services, LLC
CHAPTER 9. VISUAL PERCEPTION 411

Santa appears to wave his arm. Of course, his arm is not really moving; this is
instead merely the result of a sequential flashing of lights.
An Example Experiment
Flash one spot at time t = 0 and then another spot very close (spatially) to
it at time t ~ T. A subject’s perception of these spots will depend on T (4,
pp. 226).
Beta (/?) movement: If T is chosen properly, it looks like the spot is moving.

No Motion: If T is very small or very large, there will be no motion illusion.


Phi {4>) movement: If T is only just a bit too large, there will be a partial
motion illusion.

Motion is mostly processed primarily along the Magno pathway (4, pp. 229).
The area of the brain most associated with motion detection is the part of the
visual cortex called V5 (also known as the middle temporal area (MT)).

V5 / MT are most associated with motion detection.


________________________________ ______________

Experimental Evidence
• Cells in V5 respond to global stimuli (e.g. random dot kinematograms).
Random dot kinematogram: A random dot kinematogram is a stim­
ulus with lots of randomly moving dots. We can create RDKs that
show varying amounts of correlation between the motion of the dots.
For example, we might create an RDK with dots which are more
likely to move left. We can then ask a patient when he/she can per­
ceive some global motion. Alternatively, we can test for maximum
displacement thresholds.
M inimum/M aximum Displacement Threshold: Measures how far
the dots have to move to give a sense of motion.*
* Electrically stimulating cells in monkey V5 alters the animal’s motion
perception.
Copyright 2014 by KMK Educational Services, LLC
412 9AO. TEMPORAL PERCEPTION

Sometimes the perception of motion lingers after the moving stimulus is re­
moved. Often times this perceived motion will be directed opposite to the
stimulus motion. A commonly used example is the waterfall illusion (4).
Biophysical cause of MAE
Direction sensitive motion detectors (linked to cortical cells in V5) become
adapted following prolonged exposure to a stimulus. As a result, one becomes
LESS sensitive to motion in that direction and consequently stationary stimuli
appear to move in the opposite direction.

When a stimulus is in motion, our visual acuity remains fairly constant until
some critical velocity is reached. At that point, acuity drops drastically because
we cannot track objects (by smooth pursuit) when they are moving above a
certain speed (4, pp. 230).
I - SECTION 9.10

Tem poral Perception


Temporal perception deals with how the visual system deals with changes in
a visual scene over time. In this section, we review some fundamentals (4,
ch. 8). To address temporal perceptions, one often uses a flickering stimulus.
The luminance of such a stimulus varies sinusoidally with time. Note that this
is similar to a spatial grating, except now the change in luminance occurs in
time rather than space.

The concepts for describing temporal perception and spatial per­


ception are very similar. Of course, the former deals with space,
the latter with time.

The TMTF is exactly analogous to the SMTF from spatial vision (4, pp. 208-
12). To understand the details, we introduce a new term, percentage modula­
tion.
Copyright 2014 by I<MK Educational Services, LLC
CHAPTER 9. VISUAL PERCEPTION 413

Percentage modulation
Percentage modulation is analogous to contrast in spatial vision. It is given by
_ 100 x A
Pm ~ —f ------

where A is the amplitude of a stimulus (i.e. the height of the sine wave), Lavg
is the time-averaged luminance, and Pm is the percentage modulation. In a
sense, this is nothing but a form of “temporal” contrast.
As in spatial vision, the TMTF is plot of Sensitivity (1/P m ) at different fre­
quencies. The plot will look very similar to the CSF, Figure 9.15. Again, flicker
detection falls off at both low and high frequencies.

Explanation of TM TF Low Frequency Limits


Our visual system cannot detect very slow temporal changes (4, pp.212). For
example, we cannot watch the grass grow because the change happens on a
very slow time scale.

Purkinje Tree: We cannot see the retinal blood vessels that lie on top of
photoreceptors on our retina. However, if we shine a light on our closed eyelid,
we will be able to make out the structure of these vessels (Purkinje tree) because
the light results in higher frequency changes in the images on our retina (4,
pp. 210).

Troxler Phenomenon: Our visual system has poor sensitivity for very low
frequency stimuli because of lateral inhibition in the retina. To combat this,
the eyes are continuously moving, even when fixating. These eye movements
lead to temporal changes in retinal illumination, and therefore we are able to
see our environment because the temporal frequencies exceed threshold thanks
to the continuous eye motion (4, pp.210).

Explanation of TM TF High Frequency Limit


Our visual system shows a high (temporal) frequency cutoff (4, pp. 212). This
is defined as the highest frequency that can be resolved at 100% (maximum)
modulation. Note that this is very similar to the high frequency (spatial) cutoff
that occurs in spatial vision. In the case of temporal vision, the limitation
occurs because neurons take a finite, nonzero time to respond to stimuli, and
therefore the time “pixels” are not infinitesimally small.

Copyright 2014 by KMK Educational Services, LLC


414 9.10. TEMPORAL PERCEPTION

Glaucoma patients may experience profound loss of sensitivity at


moderate and high frequencies (as seen in the TMTF), even when
visual field defects are not present (4, pp. 221-222).

Critical Flicker Fusion Frequency (CFF): Imagine the following experi­


ment: place a flickering spot of light in front of an observer. Depending on the
sensitivity and the frequency of flicker, the light will be seen as either flickering
or constant. The highest (or lowest) frequency that can be resolved at a given
relative sensitivity is the CFF (4, pp. 213-15).
Ferry-Porter Law: The CFF (high frequency) scales linearly with the log
of retinal illumination, likely because biophysical activity underlying retinal
behavior speeds up following increased light adaptation (4, pp. 214).

Exam ple 9 .15 : C o nsider a s tim u lu s t h a t flickers a t a frequ ency 1 .5 tim es g re a te r th a n


y o u r C F F . A t w h a t fre q u e n c y w ill y o u p erceive th e s tim u lu s to flicker?

Solution 9 .15 : A s tim u lu s t h a t flickers a t a fre q u e n c y g re a te r th a n th e C F F w ill n o t


a p p e a r to flicker. T his is s im p ly an a p p lic a tio n o f th e d efin itio n o f C F F.

Other Notable Temporal Effects


Granit-Harper Law: The GH Law states that OFF is increased as stimulus
area is increased. This is because the peripheral retina is better than the central
retina at detecting flicker (4, pp. 214).
Broca-Sulzer Effect: Light flashes (above threshold) appear brightest when
they last for about 50 —100 msec. Longer or shorter flashes are perceived to
be dimmer (4, pp. 216-18).
Brucke-Bartley Effect: Light that is flickering appears brighter than steady
light of the same average luminance (Lavg) (4, pp. 218).
Thlbot-Plateau Law: A stimulus which is flickering at a frequency greater
than the CFF (and therefore fused) is perceived to be as equally bright as a
nonflickering stimulus with a luminance equal to the time-averaged luminance
of the flickering stimulus (4, pp. 219).
Copyright 2014 by KMK Educational Services, LLC
CHAPTER 9. VISUAL PERCEPTION 415

Masking refers to the situation when one stimulus (the mask) decreases the
visibility of another stimulus (the target) (4, pp. 219-20).
Forward (Backward) Masking: Masking is considered forward (backward)
if the mask precedes (follows) the target.
Paracontrast: In paracontrast, the mask appears first and the target appears
second, with both being close to each other in space.
M etacontrast: In metacontrast, the target appears first and the mask ap­
pears second, with both being close to each other in space.
Simultaneous Masking: In simultaneous masking, the mask and target ap­
pear at the same time.

The simultaneous masking effect is more noticeable in amblyopia


patients.

SECTIO N 9,11

M agno vs. Parvo Pathw ays


It is useful to remember that the human primary visual pathway is believed
to consist of two main components: a parvocellular and a magnocellular path­
way (4, pp. 291-98).
Parvocellular: The parvocellular component contributes to detection of de­
tail and color and is therefore associated with perception of object size,
shape, and detail. This pathway includes neurons in the central retina
sensitive to high spatial frequencies.
Magnocellular: The magnocellular component of the visual pathway is pri­
marily associated with the detection of motion. It includes peripheral
retinal neurons sensitive to low spatial frequencies.

Glaucoma may preferentially damage the Magno pathway. As a


result, glaucoma patients may show a decreased ability to process
moving stimuli (4, pp. 232). In addition, malfunctioning of the
Magno pathway has been implicated in developmental dyslexia (4,
pp. 300).

Copyright 2014 by KMK Educational Services, LLC


416 9.11. MAGNO VS. PARVO PATHWAYS
Saccadic suppression: Vision is suppressed just before, during, and
just after saccadic eye movement. This ensures that the image is
not blurred as we make these rapid eye movements. The root of
saccadic suppression is believed to be a suppression of the magno
pathway.

References
[1] http://www.colorvisiontesting.com
[2] Hubei, D (1988). Eye, Brain, and Vision. Scientific American Library.
[3] T. Grosvenor (2002). Primary Care Optometry, Edition. Bu tte rworth-He inemann.
[4] Schwartz, S (2014). Visual Perception: A Clinical Orientation, 4th Edition. McGraw-Hill,

Copyright 2014 by KMK Educational Services, LLC


Chapter 10
General Pharmacology

Melissa A. Cheatham, MPAS, PA -C


E dited by Chad Reade, M.D,

417
c
c
c
c
r
c
c
r
(
c
c
(
(
c
(
(
c
(
(
(
c
(
i
Copyright 2014 by KMK Educational Services, LLC
(
(..
(
c
CHAPTER 10. GENERAL PHARMACOLOGY 419

As of the sixth edition, all systemic drugs have been reviewed again for FDA-
approved indications, mechanism of action, and side effects using Lexi-Comp,
which provides up-to-date clinical content on medications (19).
- SECTION 10.1 -----------------------------------------------------------------------------------------

Basic Principles of Pharmacology *•

This is how the body handles the drug. Includes ADEM... Absorption, Dis­
tribution, Elimination and Metabolism.
ABSORPTION
Please see general physiology chapter for more review. Drugs cross biological
membranes by ...
• Bulk flow
• Passive diffusion
• Carrier mediated transport processes including facilitated diffusion and
active transport.
Absorption is affected by...
First-Pass Metabolism - Metabolism of drug before it reaches the general
circulation. Commonly occurs through the stomach, liver, intestine, rec­
tum and lung (8). Topical ophthalmic drugs obviously avoid first-pass
metabolism.
Blood flow at site of adm inistration - Skin has a relatively poor blood
supply in comparison to the oral cavity hence faster absorption in this
location.
Demographic factors - Patient’s size, weight, gender, age, current disease(s),
and pregnancy are characteristics that can affect absorption.
DISTRIBUTION
Drugs are distributed throughout the body in a manner explained by the fol­
lowing equation regarding volume of distribution (Vd)...
Vd Cp(mXo (mg)
g/L )
( 10. 1)

Copyright 203.4 by KMK Educational Services, LLC


420 10.1. BASIC PRINCIPLES OF PHARMA OOLOGY
This equals the...
Amount of drug in the body
Concentration of drug in plasma ( 10 . 2 )

Drugs with lower Vds are safer due to the fact they do not accu­
mulate in tissue and fat stores very readily. A low Vd may indicate
the drug does not preferentially bind to fatty sites, is hydrophilic, is
large and bound to plasma proteins, and is likely to be eliminated
via the kidneys in a non-metabolized form (8). Volume of distribu­
tion may be increased with liver and kidney disease and decreased
by dehydration (4).

Example 10.1: If you give 10 mg of a drug to a patient and then determine that the
plasma concentration is 3.3 mg/L, what would the Vd he?

Solution 10.1: 10 mg / 3.3 mg/L = 3L

ELIMINATION
Drugs are eliminated through a variety of routes (sweat, saliva, kidneys, etc).
An important term is clearance, defined as the body’s ability to rid itself
of a drug (8) (16). Units can vary, but are always volume/time; clearance is
characterized by the following equation:
CL = Volume
Time (10.3)
Most drugs, including topical ophthalmic drugs, are eliminated by first order
rate kinetics which follow these rules...
• Constant fraction of drug is eliminated per unit time - rate of elimination
depends on initial drug concentration - so plasma concentration decreases
exponentially with time.
• It takes a specific amount of time to eliminate half of the drug from the
body (termed half-life). In general, it takes 4-5 half-lives to remove
a drug from the body (8) (16),•
• Example: Ambien® (a sleep aid) has a 4 hour half-life. In 4 hours half of
the medicine will be eliminated. In another 4 hours half of the medicine
left in the body will be eliminated, etc.
Copyright 2014 by KMK Educational Services, LLC
CHAPTER 10. GENERAL PHARMACOLOGY 421

The following equation is important to know for first order rate kinetics...

„ —------------
Ob 0.693(Vd) (10.4)
^1/2

Drugs may also be eliminated by zero order rate kinetics...


• Constant amount of drug is eliminated per unit time - rate of elimination
is independent of initial drug concentration - so plasma concentration
decreases linearly with time.
• Half-life rules do not apply, as a constant amount of drug is eliminated
per unit of time.
• If the drug concentration doubles, the amount of time required to elimi­
nate the drug from the body will also double.
• Example: Alcohol - The more you consume, the longer it takes to re­
move (8) (16).

The kidney is the primary location for drug removal - renal health
is always the main consideration when prescribing for the elderly.

• • -o : , \ <2: .;•<...........•. . •

Pharmacodynamics refers to how drugs “handle” the body. Drugs generally


bind receptors in order to produce their effect. The relationship between the
dose of a drug (concentration) and the resulting response (effect) is fairly pre­
dictable - more drug, more effect. On the following figure, Curve 1 is the
desired response, and 2 is the dose response curve for an undesirable effect.
ED, or effective dose, is the amount of drug needed to produce a therapeutic
effect. TD, or toxic dose, is the amount of drug required to produce symp­
toms of poisoning. All drugs have the capability of producing both local and
systemic side effects. The therapeutic index (TI) is a numeric way to compare
the toxicity of one drug to another:
Copyright 2014 by KMK Educational Services, LLC
422 10.1. BASIC PRINCIPLES OF PHARMACOLOGY

Figure 10.1:

TI = LD 50 / ED 50
• LD 50 is the dose that kills 50% of a group of experimented animals
• ED 50 is the dose necessary to be effective in 50% of the population (41).

Remember, the higher the TI, the safer the drug.

Terminology
• Potency is the amount of drug required to produce an effect. Example:
If Zoloft® reaches its target goal at 25 mg, and Lexapro reaches its goal
at 10 mg, Lexapro® is more potent.
• Efficacy is the effect of a drug once it is bound.
• Affinity of a drug is how well it will bind to a receptor site - in short it
refers to the binding ability of the drug.

A partial agonist may have a higher binding affinity than a full-


agonist but never a higher efficacy (4).

Copyright 2014 by KMK Educational Services, LLC


CHAPTER 10. GENERAL PHARMACOLOGY 423

r~ S E C T I O N 10.2

A n t im ic r o b ia ls

Bacteria have microbiologic and metabolic features distinct from those of hu­
man cells. These unique features make excellent targets for antibacterial med­
ications and include the following (2);
1 Whereas human intake of folic acid is through the diet, bacterial cells
produce their own folic acid. This allows drugs that inhibit folic acid
synthesis to be much more specific for bacteria. Sulfonamides inhibit a
separate step than Trimethoprim and Pyrimetham ine in folic acid
synthesis, making combinations of these drugs synergistic.
2 Bacteria contain an outermost layer, the cell wall, that is not present
in human cells. Peptidoglycan is a specific layer within the cell wall that
provides essential structural integrity. Bacitracin prevents transfer of
peptidoglycan into the growing bacterial cell wall, while Penicillins and
Cephalosporins inhibit cross-linking of peptidoglycan within bacterial
cell walls.
3 DNA gyrase and topoisomerase IV are enzymes utilized for bacterial
DNA synthesis. Fluoroquinolones, the most clinically important class
of topical ophthalmic antibiotics, inhibit these enzymes.
4 Bacteria have 70S ribosomes, composed of two subunits, a smaller 30S
subunit and a larger SOS subunit. Antibiotics that target these unique
subunits slow bacterial protein synthesis.
• Aminoglycosides and Tetracyclines inhibit bacterial protein
synthesis by binding the 30S subunit.•
• Chloramphenicol, Macrolides, and Clindamycin inhibit bac­
terial protein synthesis by binding the 50S subunit.

For most classes of oral antibiotics, GI effects-from nausea and


vomiting, to bloating and decreased appetite, to diarrhea-are com­
mon. Keep this in mind clinically. For adverse effects listed here,
we sought to focus on those which are unique and testable.

Copyright 2014 by KMK Educational Services, LLC


424 10,2. ANTIMICROBIALS
1. INHIBITORS OF FOLATE SYNTHESIS: Sulfonamides,
Trimethoprim, Pyrimethamine
Sulfonamides: Sulfisoxazole (Gantrisin® ), Sulfacetamide
(Sulamyd®), Sulfamethoxazole (Gantanol®), Sulfadiazine
(Microsulfan®)
MOA: Inhibit dihydropteroate synthase, an enzyme used in the first step
of folic acid synthesis - which is the conversion of para-aminobenzoic acid
(PABA) into dihydrofolic acid. Bacteriostatic agents used to treat gram (+)
and (-) infections.
Ophthalmic Uses/ Adverse Effects: Topical ophthalmic solutions of sulfisoxa-
zole and sulfacetamide were commonly prescribed for blepharitis and conjunc­
tivitis but are rarely used anymore. Most common side effects of ophthalmic
administration are burning, stinging, contact dermatitis and local photosensi­
tization (sunburn on eyelid margins) (2).
Systemic Uses/ Adverse Effects: Sulfadiazine is used with pyrimethamine to
treat toxoplasmosis. Sulfamethoxazole is combined with Trimethoprim in the
pill Bactrim® (19). Oral sulfonamides can cause kernicterus in infants from
excess bilirubin accumulation in the brain (contraindicated in pregnancy) and
can induce myopic changes in refractive error. Stevens-Johnson syndrome
can be caused by the topical ophthalmic and oral sulfonamides (19).

Stevens-Johnson syndrome (SJS) is a disease marked by fever


and lesions on skin and mucous membranes that develops into skin
sloughing on (10%) or less of the body surface area. Ocular man­
ifestations include conjunctival lesions in (85%) of patients and
range from mild conjunctival injection to severe scarring. Most
commonly SJS is associated with medications (Sulfonamides in­
cluding Bactrim®, Amoxicillin, and Allopurinol for example) and
infections (27).

Trimethoprim (Prim sol®), Pyrimethamine (Daraprim ®)


MOA: Inhibit dihydrofolate reductase, an enzyme used in the second
step of folic acid synthesis - which is the conversion of dihydrofolic acid to
tetrahydrofolic acid.
Treatment: Topical ophthalmic Trimethoprim is effective against gram (+)
and (-) infections but is not effective against Pseudomonas; it is available
in combination with Polymyxin B (strong gram (-) agent) as Polytrim®.
Pyrimetham ine can be given orally for toxoplasmosis infections in the eye (19).
Copyright 2014 by KMK Educational Services, LLC
CHAPTER 10. GENERAL PHARMACOLOGY 425

Adverse Effects: Oral Trimethoprim can cause bone marrow suppression


resulting in aplastic anemia, leukopenia and granulocytopenia. Pyrimethamine
can have similar toxicity (19),

Trimethoprim = TM P... “Treats Marrow Poorly” (4).

2. CELL WALL SYNTHESIS BLOCKERS: Bacitracin, Penicillins,


Cephalosporins

Peptidoglycan provides bacterial cell wall structure. Bacitracin


stops transfer of peptidoglycan into the growing cell wall. Peptido­
glycan contains polysaccharide chains that are cross-linked together
via transpeptidases. All penicillins and cephalosporins are
cell wall inhibitors through inhibition of transpeptidase.

Bacitracin
MOA: Stops bacterial cell wall synthesis by inhibiting transfer of mucopeptides
(aka peptidoglycans) into the growing cell wall (2) (19).
Clinical Uses: Bactericidal agent against gram (+) bacteria only. Prescribed
often for blepharitis, available only in ointment form.

Polysporin® is a broad spectrum antibiotic ointment that com­


bines Bacitracin’s gram (+) coverage with PoIyrayxin(-)B’s gram
(-) coverage. Neosporin® is Neomycin added to Polysporin.

Penicillins: Amoxicillin (Principen® ), Dicloxacillin (Dynapen®)


MOA: Transpeptidase inhibitors.
Amoxicillin is NOT resistant to penicillinase like Dicloxacillin but has bet­
ter gram (~) coverage; both agents have good gram (+) coverage. Clavulonic
acid (penicillinase inhibitor, also called B-lactamase inhibitor) can be added to
Amoxicillin to produce the antibiotic Augmentin®.

Copyright 2014 by KMK Educational Services, LLC


426 10,2. ANTIMICROBIALS

Dicloxacillin is penicillinase resistant and is commonly prescribed


for eyelid infections such as hordeolums and blepharitis. Di­
cloxacillin is the drug of choice for “Methicillin-sensitive” Staphy­
lococcal infect ions... especially Methicillin-sensitive Staph aureus
(MSSA) (2). It will not work against resistant Staphylococcal
strains like MRSA (19).

Adverse Effects: Hypersensitivity Reactions are a common adverse effect of


antibiotics in the penicillin class, ranging from urticaria and anaphylaxis (IgE-
mediated type 1 reactions) to contact dermatitis (type 4 reaction). Penicillins
can render oral contraceptives ineffective (although not a consistent response) -
they are generally very safe in all trimesters of pregnancy (19). Stevens-Johnson
is reported with Amoxicillin.

Cephalosporins and penicillins both contain a beta-lactam ring,


but cephalosporins have a 6-member ring instead of the penicillin
class 5-member ring (16). If a patient is allergic to penicillin, there
is a chance that the patient will also be allergic to a cephalosporin!
Caution should be used in any patient with a history of a reac­
tion to a penicillin, a cephalosporin, or any other antibiotic with a
bet a-lactam structure. A history of an IgE-mediated type 1 reac­
tion such as urticaria or especially anaphylaxis to any beta-lactam
makes all cephalosporins and penicillins contraindicated (19).

Cephalosporins: Cephalexin (Keflex®), Ceftriaxone (Rocephin®)


MOA: Transpeptidase inhibitors.
Like penicillins, all cephalosporins have good gram (+) coverage. Later (3rd
and 4th) generation cephalosporins have the added benefit of being more effec­
tive against gram (-) organisms.
Clinical Uses: Cephalexin (1st generation) is commonly used for skin infec­
tions (which of course are usually caused by gram (+) bacteria) including those
around the eye such as dacryoadenitis, dacryocystitis, and preseptal cellulitis
but should not be used in isolation when methicillin-resistant Staph aureus
(MRSA) is suspected or known to be involved (20). Ceftriaxone (third gen­
eration) is indicated for the treatment of a variety of bacterial infections. It
is given as an intramuscular (IM) injection for the treatment of gonorrhea,
including gonococcal conjunctivitis (11).
Adverse Effects: Hypersensitivity reactions. Cephalosporins can destroy the
intestinal microflora, which alters vitamin K absorption and may cause ex­
Copyright 2014 by KMK Educational Services, LLC
CHAPTER 10. GENERAL PHARMACOLOGY 427

cessive “thinning” of the blood in patients taking the vitamin K antagonist


Warfarin (2).

Cephalosporins...
1st Generation: Gram (+)
2nd Generation: Gram (+) and some gram (-)
3rd and 4th Generations: Gram (+) and gram (-)

3. DNA GYRASE AND TOPOISOMERASE BLOCKERS:


Fluoroquinolones
2nd Generation: Ciprofloxacin (Ciloxan®, Cipro®), Ofloxacin
(Ocuflox®). 3rd Generation: Levofloxacin (Quixin®). 4th
Generation: Gatifloxacin (Zymar®), Moxifloxacin (Vigamox®,
Avelox®), Besifloxacin (Besivance®).
MOA: Rapidly inhibit bacterial DNA synthesis by inhibiting DNA gyrase
AND Topoisomerase IV.
Topical Uses: Contact lens related ulcers, corneal abrasions, bacterial conjunc­
tivitis, etc. (2).
Systemic Uses: Ciprofloxacin is used for gram (-) urinary and GI infections.
Moxifloxacin is approved for pneumonia, sinusitis, intra- abdominal and skin
infections (19).

There are 4 generations of fluoroquinolones. It is important


to know that 3rd and 4th generation fluoroquinolones
(Levo/Gati/Moxi/Besifloxacin) have been formulated with im­
proved effectiveness against gram (T) infections; however, these
drugs continue to be potent against gram (-) bacteria as well.

Adverse Effects: ORAL fiuoroquinoLONES can hurt the attachments to your


BONES causing tendinitis (4). These drugs are contraindicated in pregnancy,
children, and adolescents below the age of 18 due to damage in cartilage for­
mation and inhibition of bone growth (19).

All of the topical ophthalmic fluoroquinolones, except Lev­


ofloxacin, are approved for use in patients 1 year of age and
older (2).

Copyright 2014 by KMK Educational Services, LLC


428 10.2. ANTIMICROBIALS
4. RIBOSOMAL BINDING ANTIBIOTICS
BLOCKERS OF PROTEIN SYNTHESIS AT 30S RIBOSOMAL
SUBUNIT: Aminoglycosides and Tetracyclines
Aminoglycosides: Gentamicin (Garamycin®), Tobramycin
(Tobrex®)
MOA; Bind 30S subunit of the bacterial ribosome to inhibit bacterial protein
synthesis. Effective against gram (+) and gram (-) bacteria, but their forte is
in the gram (-) spectrum (23).
Ocular Uses: Prior to fluoroquinolones, aminoglycosides were first-line therapy
for ocular bacterial infections. Tobramycin is available in topical ophthalmic
and ointment form. Gentamicin and tobramycin topical ophthalmic solutions
are available in fortified concentrations, along with fortified cefazolin for the
treatment of sight-threatening corneal ulcers (2). Tobradex® combines to­
bramycin with dexamethasone.
Adverse Effects: Topical aminoglycosides are notorious for causing superficial
punctate keratitis and delayed reepithelialization.

Tetracyclines: Tetracycline, Doxycycline, Minocycline


MOA: Bind to 30 S subunit and prevent access of aminoacyl tRNA (4).
Bacteriostatic.
Clinical Uses: Doxycycline is commonly prescribed for meibomianitis and acne
rosacea- it alters the configuration of the oil glands and decreases the release
of irritating free fatty acids (1). Doxycycline is also prescribed for chlamydial
ocular infections, including trachoma and inclusion conjunctivitis. Minocy­
cline is commonly prescribed (in low doses) for long-term management of acne
vulgaris (28),
Pharmacokinetics: Absorption in the GI tract is impaired by dairy products.
(The cations of milk, antacids, or iron containing compounds inhibit their ab­
sorption.) The primary route of excretion is via the kidney, so this class of
drugs is contraindicated in renal failure patients. Doxycycline is an exception;
it can be taken with food and is fecally eliminated, allowing use in patients
with renal failure.
Adverse Effects: Contraindicated in pregnancy and in children! Side
effects include pseudotumor cerebri, bone growth retardation, discoloring of
teeth. Minocycline can cause blue sclera or pigmented cysts on the conjunctiva.

Copyright 2014 by KMK Educational Services, LLC


CHAPTER 10. GENERAL PHARMACOLOGY 429

Bacteriostatic agents include (not exhaustive list) tetracyclines,


trimethoprim, sodium sulfacetamide and, to some degree, ery­
thromycin. Bactericidal agents include (not exhaustive list) the
penicillins, bacitracin, aminoglycosides, cephalosporins and fluoro­
quinolones (23).

BLOCKERS OF PROTEIN SYNTHESIS AT SOS RIBOSOMAL


SUBUNIT: Chloramphenicol, Erythromycin (Macrolides), and
Lincomycin
Chloramphenicol (Chloroptic®)
MOA: Binds to 50S subunit; effective against gram (+) and gram (-) bac­
teria. Can be formulated in ointment form or eye drop.
Adverse Effects: TOPICAL ophthalmic use has caused fatal aplastic ane­
mia (2). Also, may result in optic neuritis in extended therapy.
Macrolides
Erythromycin, Azithromycin (Zithrom ax® , Azasite®),
Clarithromycin (Biaxin®)
MOA: Inhibit bacterial protein synthesis by binding to the SOS subunit of
the bacterial ribosome.
Clinical uses include:
• Oral Azithromycin (Zithrom ax®) is commonly prescribed for chlamy­
dial infections (trachoma and inclusion conjunctivitis) because of
its convenient single 1-gram dose.
• Topical Azithromycin (Azasite®) is inidcated for bacterial conjunc­
tivitis and is also used to combat blepharitis. Dosing twice daily for
two days, then once daily for five days is recommended (19). Prolonged
contact time is aided by a novel drug delivery system.
• Topical Erythromycin ointment has a poor resistance profile and is not
commonly prescribed to combat active infections; it is more commonly
utilized in a prophylactic role as a nocturnal lubricant (23). It is also
prescribed (instead of silver nitrate) for prophylaxis of gonococcal oph­
thalmia neonatorum.•
• Oral Clarithromycin is utilized for the treatment of respiratory infec­
tions (19).
Copyright 2014 by KMK Educational Services, LLC
430 10.2, ANTIMICROBIALS

Adverse Effects: Azasite© contains BAK, so contact lens wearers are advised
against contact lens use during treatment with Azasite© (19).
Lincomycin, Clindamycin (Cleocin®)
MOA: Inhibit bacterial protein synthesis by reversibly binding to the SOS sub­
unit of the bacterial ribosome.
Clinical Uses: Methicillin resistant Staph aureus (MRSA) and anaerobic infec­
tions.

Clinical Note: Clindamycin, Trimethoprim-Sulfamethoxazole


(Bactrim©) and Doxycycline are all recommended for the treat­
ment of MRSA (20).

TUBERCULOSIS MEDICATIONS
Tuberculosis (TB) is caused by Mycobacterium tuberculosis. Combination
treatment with multiple medications is used for active TB, whereas monother­
apy with Isoniazid or Rifampin is used for treatment of latent TB (19).

Active TB treatment is RIPE: Rifampin, Isoniazid, Pyrazinamide,


and Ethambutol. All except Pyrazinamide are on the NBEO out­
line.

1. Rifampin (Rifadin®)
MOA: Prevents mRNA synthesis (transcription) by binding to the beta subunit
of DNA-dependent RNA Polymerase.
Adverse Effects: Hepatotoxicity (increased AST, ALT) is most common. Ex­
udative conjunctivitis is rarely reported; orange - pink discoloration to urine
and tears can also occur (19).
2. Isoniazid (Nydrazxd®)
MOA: Inhibits mycolic acid synthesis and thereby prevents cell wall syn­
thesis (19).
Adverse Effects: Hepatotoxicity. Isoniazid can cause pyridoxine (vitamin
B6) deficiency that may result in peripheral neuropathy. Rarely, it can cause
optic neuritis and atrophy with loss of vision (19).
Copyright 2014 by KMK Educational Services, LLC
CHAPTER 10. GENERAL PHARMACOLOGY 431

3. Ethambutol (M yambutol®)
MOA: Inhibits arabinosyl transferase, thereby inhibiting synthesis of the
mycobacterial cell wall (19).
Adverse Effects: Optic neuritis that is typically retrobulbar and bilateral -
the initial ocular symptom is reduced visual acuity (2), although color blindness
may also occur. The vision loss is usually reversible but may take months to
normalize, and some case reports describe irreversible damage (22). Baseline
and periodic assessment of visual acuity and red-green color discrimination is
indicated for patients on therapy (19).

1. Influenza Therapy: Oseltamivir (Tamiflu®)


MOA: Inhibitor of influenza A and B viral neuraminidase. Prevents the
spread of the virus along the mucous lining of the respiratory tract. Conjunc­
tivitis is reported in 1% of patients (19).

2. HIV Therapy: Zidovudine (R etrovir® ) (AZT®)


MOA: Nucleoside analog of thymidine that acts as a potent inhibitor of viral
RNA-dependent DNA polymerase, which is also called reverse transcrip­
tase (16).
Clinical Uses: Major component of three-drug therapy for HIV. AZT is also
used during pregnancy to lower risk of transmitting HIV to the fetus (vertical
transmission).
Adverse Effects: 1) Bone marrow suppression (granulocytopenia, anemia, or
even pancytopenia); 2) lactic acidosis; 3) muscle breakdown (myopathy, myosi­
tis). Amblyopia and macular edema have been reported (19).

3. Hepatitis C Therapy: Ribavirin (Copegus® or Rebetol® )


MOA: Viral RNA polymerase inhibitor. Always used in combination with
Interferon for treatment of Hep C.
Adverse effects: Conjunctivitis is the most common ocular effect listed, but
retinopathy (marked by retinal hemorrhage and cotton wool spots), arterial
and venous occlusions are also reported. Baseline ophthalmologic exam is rec­
ommended, and periodic repeat exams are suggested for patients with diabetic
or hypertensive retinopathy who continue on combination treatment with rib­
avirin and interferon (19).

Copyright 2014 by KMK Educational Services, LLC


432 10.2. ANTIMICROBIALS

For patients receiving Ribavirin and Interferon combination ther­


apy, remember that conjunctivitis is most common. Still, do not
forget the other side effects of RIBAViriN: Retinal Ripping (de­
tachment)/ Ischemia (cotton wool spots)/ Bleeding (retinal hem­
orrhage), Arterial and Venous occlusions, (optic) Neuritis.

4. Antiherpes Virus Agents


While the Herpes family of viruses includes HSV-1 and HSV-2, it also includes
Varicella Zoster Virus (VZV), Cytomegalovirus (CMV), and others. All viruses
within the herpes family are DNA viruses. The antiherpetic drugs all have the
same mechanism of action - they inhibit viral DNA synthesis by selectively
inhibiting DNA polymerase.
TVifluridine (Viroptic®)
MOA: DNA polymerase inhibitor indicated for the treatment of herpes simplex
keratitis.
Acyclovir (Zovirax®), Valacyclovir (Valtrex®), Famciclovir
(Famvir®)
MOA: DNA polymerase inhibitors.
Clinical Uses: As systemic agents, all are useful for cold sores (usually HSV-1),
genital sores (usually HSV-2), and shingles (Zoster) infections. All of these can
be used for active Herpes Zoster ophthalmicus (HZO) infections, active HSV
keratitis, and prophytactically for prevention of recurrent Herpes Simplex Virus
(HSV) keratitis.
Adverse Effects: Headache and GI effects are most common and are most likely
to occur in patients with renal dysfunction (19).
Ganciclovir (Zirgan®)
MOA: DNA polymerase inhibitor with improved efficacy (5X per day instead
of 9X per day) and reduced, toxicity (contains BAI< instead of thimerosal) as
compared to viroptic.
Clinical Uses: Topical administration indicated for HSV keratitis. Also in­
dicated for CMV retinitis treatment using intraocular sustained release for­
mulation (19) (2).
Adverse Effects: Topically, blurring and irritation. With intraocular implant
there is a nearly immediate, but temporary, decrease in visual acuity. Retinal
detachment and Intravitreal bleeding arc also reported (19).
Copyright 2014 by KMK Educational Services, LLC
CHAPTER 10. GENERAL PHARMACOLOGY 433

Remember, CMV is an opportunistic pathogen that can reactivate


and cause vision-threatening retinopathy in AIDS patients. It is the
most common ocular infection infection in AIDS patients but has
decreased in incidence since the introduction of medications like
Zidovudine.

Foscarnet (Foscavir®)
MOA: DNA polymerase inhibitor.
Clinical Uses: IV treatment of CMV retinitis when ganciclovir therapy fails, or
IV treatment of acyclovir-resistant HSV infections in AIDS patients (19).
Adverse Effects: Nephrotoxicity, seizures.
ANTIFUNGAL DRUGS
Ergosterol is a component of fungal cell membranes (analogous to cholesterol
in human cell membranes) not found in human cells.
1. Natamycin (N atacyn® ), Amphotericin B (Amphocin®),
Nystatin (M ycostatin®)
MOA: Bind to ergosterol (within cell membrane) of fungi and form
pores allowing cellular contents to leak out which results in cell death.
• Natamycin: approved for treatment of fungal blepharitis, conjunctivitis,
and keratitis caused by susceptible organisms of Candida, Aspergillus,
Fusarium, etc. (19).
• Amphotericin B: broad-spectrum antifungal available in topical formu­
lation (ointment and solution) to treat fungal keratitis and in IV form to
treat systemic and intraocular fungal infections. The topical form (for
treatment of fungal keratitis) is not commercially available, but can be
obtained through a compounding pharmacy. Nephrotoxicity is com­
mon with IV treatment.
• Nystatin: used primarily for Candida oral (thrush) and vaginal (yeast)
infections, not for ophthalmic use (19).
2. Ketoconazole (Nizoral®), Fluconazole (Diflucan®), Miconazole
(Mo nost at-Der m ® )
MOA: Inhibit ergosterol synthesis.
Copyright; 2014 by KMK Educational Services, LLC
434 10.2. ANTIMICROBIALS
Ketoconazole was the first oral azole antifungal. Fluconazole can be pre­
scribed orally, topically and through subconjunctival injections; increasing re­
sistance and ineffectiveness against agents that typically cause endophthalmitis
are reasons for concern (2).
Adverse Effects: Hepatotoxicity (4).
3. Griseofulvin (Grifulvin®)
MOA: Oral agent that inhibits fungal mitosis by interacting with micro­
tubule formation during cell-wall development (4).
Clinical Uses: Infections of the scalp and skin, including fingernails and toe­
nails.
ANTIPARASITIC DRUGS

Sulfadiazine and Pyrimethamine are antiparasitics used in the


treatment of Toxoplasmosis and are discussed at the beginning of
the Antibiotics section with the other inhibitors of folic acid syn­
thesis.

Chloroquine (Aralen®)
MOA: Causes a build-up of heme, the breakdown product of hemoglobin.
This accumulation in the red blood cells is toxic to the “intraerythrocytic”
Plasmodium (malarial) parasite (15). It also inhibits phospholipase-A
but is not generally used as an anti-inflammatory due to side effects including
risk for bull’s eye maculopathy.
Adverse Effects: Reversible whorl keratopathy is actually the most com­
mon ocular effect, but the greatest risk to vision is Bull’s-eye maculopathy
- granular hyperpigmentation surrounded by a zone of depigmentation (19).
Chloroquine binds to melanin with the RPE causing localized RPE damage
and subsequent migration of these RPE cells to the outer nuclear and outer
plexiform layers (2). The initial sign of retinopathy is RPE mottling within
the macula.
Risk of retinopathy increases with:
• Dosage> 3 mg/kg of body weight
• Treatment duration> 5 years
• Abnormal renal function
• High body fat percentage
Copyright 2014 by KMK Educational Services, LLG
CHAPTER 10. GENERAL PHARMACOLOGY 435

• Age> 60
• Liver disease
• Concomitant retinal disease (21).

Central and paracentral scotomas are the most common field de­
fects with chloroquine. Color vision is typically normal in the early
(RPE mottling) stages of the retinopathy.

Kwell (Lindane®)
MOA; Lipophilic structure absorbed through the exoskeleton of insects result­
ing in seizures and death (30).
Clinical Uses: Treatment of lice (with shampoo) and scabies (with lotion).
Adverse Effects: Shampoo treatment can cause conjunctivitis when applied to
eyelashes.
- SECTION 10.3 ----------------------------------------------------------------------------------------------------

A nti-Inflam m atory M edications *•


Before discussing anti-inflammatory medications, it is useful to review the
Arachidonic Acid Pathway. Keep these points in mind as you review the figure:
• Phospholipase A2 is the first step in the pathway; blocking it will prevent
all downstream effects.
• Leukotrienes, synthesized by lipoxygenase, bind receptors in the nose and
bronchi to cause rhinitis (runny nose) and bronchoconstriction.
• Thromboxanes like TXA2, synthesized primarily by Cyclooxygenase-1,
induce platelet aggregation.
• Prostaglandins (PGs) have varying types and function.
- Those which mediate inflammation (pain, fever, swelling, warmth)
are primarily made by Cyclooxygenase-2.
- Cyclooxygenase-1 is responsible for synthesizing others, such as
Prostaglandin E2 (PGE2), that inhibit gastric secretions and influ­
ence gastric motility (15) (16).
Copyright 2014 by KMK Educational Services, LLC
436 10.3. ANTMNFLAMMATORY MEDICATIONS

Figure 10.2: Arachidonic Acid/ Prostaglandin Synthesis Pathway

Hydroxychloroquine (Plaquenil®)

MOA: Inhibits phospholipase A2 and causes a build up of heme (similar


to chloroquine).
Clinical Uses: Lupus, rheumatoid arthritis, malaria (prophylaxis and treat­
ment)
Adverse Effects: Bull’s-eye maculopathy - significantly lower risk than with
chloroquine. Risk of retinopathy is low as long as daily dose does not exceed
400mg/day or 6.5 m g/kg of body weight/day (2). Anyone weighing less
than 135 lbs is at increased risk for macular toxicity if they are taking the
standard 400mg/d dosage of plaquenil (23). Treatment duration of 5 years
and abnormal renal function are important risk factors (2), as are liver dis­
ease, age over 60, high body fat, and retinal disease (21). Like chloroquine,
corneal deposition (causing whorl keratopathy) is reported; however with
hydroxychloroquine it usually only occurs at supratherapeutic doses (19).

Copyright 2014 by KMK Educational Services, LLC


CHAPTER 10. GENERAL PHARMACOLOGY 437

The risk of maculopathy is much higher with Chloroquine than


Hydroxychloroquine. The six risk factors for Bull’s-eye mac­
ulopathy not related to dose are the same in Chloroquine and
Hydroxychloroquine; remember the dose of concern differs between
the two drugs (3m g/kg/day for Chloroquine, 6.5mg/kg/day
for Hydroxychloroquine).

MOA: Inhibit phospholipase A2.


Longterm use of systemic steroids can cause...
* Diabetes and/or insulin resistance, weight gain, fat redistribution, im­
mune suppression, osteoporosis, HTN, and poor wound healing.
• Ocular effects, including PSC cataracts, glaucoma, and increased risk of
secondary infections.

Hydrocortisone (Cortef®)
Used for treatment of adrenal insufficiency but may also be used as a potent
anti-inflammatory.

Triamcinolone (Kenalog®)
• The ocular formulation of Triamcinolone (Triescence©) may be used for
diffuse diabetic macular edema, Graves’ orbitopathy, intermediate and
non-resolving posterior uveitis, chalazion removal and cystoid macular
edema following cataract surgery (assuming topical treatment fails) (19).
• It is also approved for intravitreal implantation for chronic cystoid mac­
ular edema associated with noninfectious posterior uveitis (2).
• Systemic Uses: Injection for the treatment of dermatoses, asthma, exac­
erbations of MS, and arthritis (19).•
• Adverse Effects: Endophthalmitis can occur with intravitreal triamci­
nolone injections, but is not nearly as common as increased IOP. Triam­
cinolone acetonide intralesional chalazion injections can cause depigmen­
tation of the eyelid in dark-skinned individuals.
Copyright 2014 by KMK Educational Services, LLC
438 10.3. ANTI-INFLAMMATORY MEDICATIONS
Fluticasone (Flonase®)
Intranasal (topical) corticosteroid indicated for the treatment of allergic
rhinitis.
Adverse Effects: In addition to cataracts and increase in IOP, conjunctivitis
and dry eye with irritation are also reported (19).

1. Aspirin
MOA: Salicylate that inhibits prostaglandin and thromboxane synthesis by
acting as an irreversible Cox-1 and Cox-2 inhibitor (15) (16).
Clinical Uses: Antipyresis, anti-inflammatory effects and analgesia. Aspirin is
proven to reduce the risk of recurrent heart attacks in patients with known
heart disease (19).
Adverse Effects: GI effects (gastric ulcers and bleeding) are common and clin­
ically important. “Antiplatelet effects” can cause bleeding complications in
the eye. In children, aspirin use is associated with Reye’s Syndrome (a rapidly
progressive brain disease-encephalopathy-marked by confusion, then seizures
and multiorgan failure).

2. Indomethacin (Indocin®), Ibuprofen (Advil®, M otrin® ),


Naproxen (Naprosyn®), Naproxen Sodium (Aleve®), Piroxicam
(Feldene®)
MOA: Reversible Cox~l and Cox-2 inhibitors. Clinical uses are similar to
aspirin, but NSAIDs are not indicated for patients with heart disease (in fact,
they are relatively contraindicated-see below).
Adverse Effects: All increase risk of bleeding complications in the eye,
and all carry a boxed warning in the United States regarding increased risk
of GI bleeding as well as cardiovascular events including fatal M I and
stroke (19). So unlike aspirin, these NSAIDs are contraindicated in patients
with a history of coronary artery disease or stroke. Also unlike aspirin, these
drugs do NOT cause Reye’s Syndrome. Ibuprofen can be used in children 6
months and older.
Unique Effects of Indomethacin: In addition to increased risk of bleeding
complications in the eye seen with other NSAIDs, Indomethacin has unique
ocular effects including whorl keratopathy and pigmentary changes (es­
pecially in macula) (19).

Copyright 2014 by KMK Educational Services, LLC


CHAPTER 10. GENERAL PHARMACOLOGY 439

Misoprostol (Cytotec® ) is synthetic prostaglandin E l-


chemically similar to PGE2, the PG important for stomach
protection-used for prevention and treatment of NSAID-induced
ulcers- The main adverse effect is that it can cause miscarriages or
premature labor (19).

Celecoxib (Celebrex®)
MOA: Selective Cox-2 inhibitor. Unlike other NSAIDS, this spares the
Cox-1 pathway...which helps to protect the gastric mucosa and limits bleeding
effects.
Adverse Effects: Reversible conjunctivitis, blurry vision, and Stevens-Johnson
Syndrome (19). Much better on the gastric mucosa than other NSAIDS.

All topical and systemic NSAIDs are contraindicated in patients


with a history of allergic reaction to other NSAIDs or to aspirin.

- SECTION 10.4 ------ ----------------------------------------------------------------

Cold and Allergy M edications

1. COLD MEDICATION
The "common cold” is caused by a number of viruses for which there is no
specific therapy, so treatment is aimed at amelioration of symptoms. Pseu-
doephedrine (Sudafed®) is intended to limit nasal congestion symptoms.

Pseudoephedrine (Sudafed®)
MOA: Nonspecific alpha and beta adrenergic agonist (19).
Adverse Effects: As expected for general adrenergic agonist (tachycardia, ner­
vousness, agitation, etc.). Ocular effects reported include diplopia and blurred
vision, and package inserts urge caution in.patients with increased IOP (19)
Clinical Note: Pseudoephedrine can be used to synthesize metharnphetamine,
which has led to increased government regulation and monitoring of Sudafed®
sales (including placing the medication behind the pharmacy counter).
Copyright 2014 by KMK Educational Services, LLC
440 1 0 .1 G OLD a n d a l l e r g y m e d ic a t io n s

2. ALLERGY MEDICATIONS
INTRODUCTION TO HISTAMINE
Histamine receptors are found in the following locations:
* H I receptors - found in smooth muscle of bronchi, blood vessels and in­
testine. Activation of HI receptors causes itching, vasodilation, increased
vascular permeability and contraction of smooth muscle in the GI tract.
• H2 receptors - found in gastric parietal cells, heart, pulmonary blood
vessels and cells of the immune system. Activation of H2 receptors causes
itching, vasodilation, mucous discharge and gastric secretion.

The primary histamine receptor in the eye is the HI receptor. His­


tamine release can cause itching, tearing, chemosis of the conjunc­
tiva and eyelids, dilation of conjunctival blood vessels and a pap­
illary reaction (2). Clinically, Hl-receptor blockers are used in the
treatment of allergic disease, including allergic rhinitis and allergic
conjunctivitis. H2-receptor blockers are discussed more in the GI
section.

ANTIHISTAMINES
First Generation Antihistamines (HI Blockers)
Diphenhydramine (Benadryl(R)), Chlorpheniramine maleate
(Chlor-Trimeton®), Brompheniramine (Dimetane®),
Promethazine (Phenergan®)
MOA: Block histamine - HI receptor interaction. First generation antihis­
tamines cause sedation because of their CNS penetration.
Adverse Effects: Sedation as above. Antihistamines, especially first genera­
tion drugs, have anticholinergic effects including mydriasis, dry eyes and
mouth, and tachycardia (19).

Copyright 2014 by KMK Educational Services, LLC


CHAPTER 10. GENERAL PHARMACOLOGY 441

Promethazine (Phenergan® ) is an antihistamine whose chem­


ical structure is that of the antipsychotic medications (phenoth-
iazines). Ocular effects specific to Promethazine not shared by
other antihistamines result from the phenothiazine structure of
Promethazine and include corneal epithelial keratopathy, lentic­
ular changes, and pigmentary retinopathy (19). As you review
antipsychotic medications, Chlorpromazine and Thioridazine, note
the similarities in ocular effects to Promethazine.

Second Generation Antihistamines (H I Blockers)


Loratadine (Claritin® ), Fexofenadine (Allegra®), Cetirizine
(Zyrtec®)
MOA: Second generation HI receptor blockers are mostly used for allergic
rhinitis and chronic urticaria (hives).
Adverse Effects: Less lipid soluble, so less CNS effects (less sedation, dry eye,
dry mouth, etc). Cetirizine can cause abnormal EOM contractions (oculo­
gyric crisis) (2).

- SECTION 10.5 ------------------------------------------------------------------------ --------------------------

G astrointestinal M edications

Recall that the majority of peptic ulcer disease is due to infections


with the gram negative rod Helicobacter pylori and/or NSAID use.

1. H2 Blockers
Cimetidine (Tagamet®), Ranitidine (Zantac® ), Famotidine
(Pepcid®)
MOA: Bind H2 receptor on gastric parietal cells, preventing histamine stimu­
lation of gastric acid (HC1) secretion.
Clinical Uses: Healing and preventing stomach ulcers and acid reflux.
Adverse Effects: Diarrhea. Cimetidine has numerous drug interactions, and it
can cause gynecomastia and loss of libido (19).
Copyright 2014 by KMK Educational Services, LLG
442 10.6. RESPIRATORY MEDICATIONS
2. Proton Pum p Inhibitors
Omeprazole (Prilosec®), Esomeprazole (Nexium®)
MOA: Inhibit H + /K + -ATPase (proton) pump.
Clinical Uses: First-line therapy for peptic ulcer disease and gastro-esophageal
reflux disease (GERD).
3. Gastric Lining Protector
Sucralfate (Carafate® )
MOA: Forms a paste-like substance by binding positively charged proteins in
the stomach and adheres to damaged ulcer tissue, protecting the stomach lining
against acid. Used for acute management of Peptic Ulcer Disease (PUD) to
allow healing of stomach mucosa.
Adverse Effects: May interfere with absorption of other oral medications and
should be taken two hours after other medications (19).
- SECTIO N 10.6 ---------------------------------------------------------------------------------------------------

R espiratory M edications *•
BRONCHODILATORS
Bronchodilators are inhaled medications used for asthma and CO PD treat­
ment; they include long- and short-acting ft2 agonists, as well as cholinergic
muscarinic antagonists.
1. Long-Acting ft2 Agonist
• Salmeterol (Serevent®) is the only long-acting ft2 agonist on the
NBEO outline and is used for maintenance therapy for COPD and asthma
but NOT as a rescue inhaler. It has a 1 —3% incidence of keratits and
conjunctivitis (19).
2. Short-Acting (32 Agonists
• Albuterol (Ventolin®), Levalbuterol (Xopenex®), and Terbu-
taline (Brethine® ) may be used as rescue inhalers for asthma and
COPD patients with acute dyspnea (shortness of breath).
• M etaproterenol (Alupent®) is also listed as a ft2 agonist but is no
longer recommended for asthma therapy due to excessive cardiovascular
effects from ftl stimulation (19).
Copyright 2014 by KMK Educational Services, LLC
CHAPTER 10. GENERAL PHARMACOLOGY 443

• Isoproterenol (Isuprel®) is a nonspecific beta agonist whose clinical


use is primarily in treatment of arrhythmias via p i stimulation; in the
U.S. it is only approved for IV treatment (19). It was previously but is
no longer used in asthma.

Adverse Effects: In general, these medications have greater p2 receptor activity


than P1. Side effects are due to P1 stimulation and include tachycardia, heart
palpitations, nervousness, tremor and nausea.

All short-acting /32 agonists (Albuterol, Levalbuterol, Terbut aline,


Metaproterenol) have warnings about their use in glaucoma pa­
tients due to risk of increased IOP (19).

3. Muscarinic Antagonist
• Ipratropium (Atrovent®) blocks muscarinic receptors in bronchial
smooth muscle, thereby inhibiting bronchoconstriction. Caution is rec­
ommended for patients with narrow-angle glaucoma (19).

1. Zafirlukast (Accolate®), Montelukast (Singulair®)


These are Leukotriene receptor antagonists which are approved for asthma
and bronchoconstriction. Montelukast is also approved for allergic rhinitis (19).

2. Theophylline (Theo-Dur®)
Inhibits phosphodiesterase, leading to increased cAMP and increased re­
lease and effect of epinephrine. Use is limited due to narrow therapeutic in­
dex (4). Beta-blockers, including topical agents like Timolol, have the
potential to mitigate the bronchodilatory effect of the drug (19).

3. Acetylcysteine (M ucomyst®)
Mucolytic agent that works by breaking disulfide bonds in proteins of
mucous to reduce viscosity (19). Topical ophthalmic form can be prescribed
(often qid) for filamentary keratitis, dry eye syndrome and corneal burns.
Copyright 2014 by KMK Educational Services, LLC
444 10.7. CHEMOTHERAPEUTIC AGENTS
- SECTION 10.7 -------------------- ;--------------------------------------------

Chem otherapeutic Agents

IM M UNOSUPPRESSANT MEDICATIONS
1. M ethotrexate (Rheum atrex®)
MOA: Inhibits dihydrofolate reductase which effectively inhibits DNA syn­
thesis (4). Its MOA in rheumatoid arthritis involves immunosuppressant and
anti-inflammatory mechanisms (19).
Adverse Effects: Hepatoxicity, myelosuppression. Myelosuppression increases
risk of opportunistic infections and lymphomas, including in the eye (19).

2. Cyclosporine (Neoral®, Sandimmune®)


MOA: Prevents rejection of organ transplants by inhibiting release and
production of Interleukin-2 (IL-2), which is responsible for T-lymphocyte
activation.
Adverse Effects: Opportunistic infections, malignancies, hypertension and
kidney dysfunction. The topical formulation of Cyclosporine is R estasis® (19)
whose most common side effect is ocular burning.

Clinical Note: Cyclosporine is the most common medication-


associated cause of Reversible Posterior Leukoencephalopa-
thy Syndrome (RPLS). RPLS is marked by headache, altered
consciousness, seizures, and visual disturbances. The visual prob­
lems vary from aura and visual hallucinations to hemianopsia or
even cortical blindness. MRI findings are characteristic; once the
diagnosis is made, Cyclosporine must be immediately and perma­
nently stopped (26).

3. Azathioprine (Im uran® )


MOA: Purine analog that inserts into DNA and RNA and stops replication.
Clinical Uses: Treatment of rheumatoid arthritis and prevention of rejection of
kidney transplants (19). May also be used as a treatment for ocular myasthenia,
but the risk of secondary lymphomas caused by Azathioprine limits its use in
this regard (31).
Copyright 2014 by KMK Educational Services, LLC
CHAPTER 10. GENERAL PHARMACOLOGY 445

ESTROGEN ANTAGONIST
Tamoxifen (Nolvadex®)
Used for treatment of breast cancer during and for at least five years following
the remission of breast cancer,
MOA: A competitive partial agonist inhibitor of estradiol which inhibits
estrogen effects at the breast (16).
Adverse Effects: Crystalline retinopathy, whorl keratopathy, throm­
boembolism (including BRVO and CRVO), fatty liver and hot flashes. In­
creases risk for uterine cancer because of estrogen agonist effect. Prescribing
information reports 7% incidence of cataracts (19), although there is not uni­
form agreement about the true incidence.

Whorl keratopathy may occur on its own or with crystalline


retinopathy. While the keratopathy is generally reversible and
causes no visual symptoms, the crystals may affect vision, espe­
cially if accompanied by macular edema. Unlike the keratopathy,
crystalline changes do not always reverse with cessation of Tamox­
ifen. (2).

- SECTION 10.8 ----------------------------------------------------------------------------------------------

M edications Acting on the Central Nervous System

Oral analgesics include over-the-counter agents like NSAIDs (discussed earlier)


and Acetaminophen. Prescription medications include some NSAIDs, mus­
cle relaxants, and Tramadol. Opiate analgesics are regulated by the Drug
Enforcement Agency (DEA), reserved for severe pain not controlled by other
medications.

1. Acetaminophen (Tylenol®)
MOA: Not completely understood. Does not have anti-inflammatory proper­
ties.
Clinical Uses: Analgesia and antipyresis. Tylenol can be used in patients of all
ages, including infants; it is also used safely during pregnancy.
Adverse Effects: Hepatotoxicity, which can be fatal (19).
Copyright 2014 by KMK Educational Services, LLC
446 10.8. MEDICATIONS ACTING ON CNS
2. Cyclobenzaprine (Flexeril®)
MOA; Decreases somatic (voluntary) motor activity through inhibiting tonic
activity of alpha and gamma motor neurons, which helps treat muscle spasms (19).
Adverse Effects: Drowsiness, loss of coordination, and anticholinergic effects
(caution in glaucoma patients) (19).
3. Tram adol (U lt r a m ® )

MOA: 1. Agonist at Mu opiate receptor (but is not considered a true opiate). 2.


Inhibits serotonin and norepinephrine uptake within ascending pain pathways.
More potent analgesic than Acetaminophen but less potent than opiates.
Adverse Effects: Dry mouth, sedation, dizziness and nausea. Causes miosis
less frequently than regular opiates.

OPIATE im rnssM sm BAsm gM M


M eperidine (Demerol®), Oxycodone (Roxicodone®)
MOA: Agonists at Mu, Kappa and Delta (opiate) receptors. Potent
analgesics with addictive properties.
Adverse Effects: Miosis, respiratory depression, drowsiness and sedation. Ex­
cessive doses of Meperidine can result in the build-up of a metabolite (normeperi-
dine) that can cause increased intracranial pressure and papilledema as well as
seizures (19).

Naloxone (N arcan® ) is an opioid antagonist that is used to


reverse the effects of opiates.

Sum atriptan (Im itrex® )


MOA: 5-HT1B and ID (Serotonin subtype IB and ID) receptor ag­
onist. These receptors are located on the vascular smooth muscle in cranial
arteries; their stimulation by Sumatriptan causes vasoconstriction and reduces
inflammation in the CNS (19).
Adverse Effects: All are related to systemic vasoconstriction and include:
NAION, myocardial infarctions, ischemic strokes. Retinal artery occlusions
and retinal venous thromboses arc also reported (19).
Copyright 2014 by KMK Educational Services, LLC
CHAPTER 10. GENERAL PHARMACOLOGY 447

Phenothiazines: Chlorpromazine (Thorazine®), Thioridazine


(Mellaril®)
MOA: Dopamine receptor antagonists.
Adverse Effects:
• “Pigmentation effects”: pigment on corneal endothelium, anterior stellate
cataracts, and hyperpigmentation of the RPE
• Anticholinergic effects: dry eye, mydriasis, and increase in IOP
• Oculogyric crisis (2)
• Too much of these drugs can result in Parkinson-like effects (15).

Parkinson Disease is due to dopamine deficiency. Classic Parkinson


symptoms: Tremor at rest, Rigidity, Akinesia, and loss of Postural
reflexes. (Parkinson’s is a TRAP) (4).

Amantadine (Symmetrel®)
MOA: Potentiates dopamine effects (either by blocking reuptake or augmenting
release of dopamine) in the brain.
Bromocriptine (Parlodel®)
MOA: Dopamine agonist that can be used to treat Parkinson Disease but is
more commonly used for prolactin-secreting pituitary adenomas.

M ethylphenidate (Ritalin® ), Dextroam phetamine (Dexedrine®)


MOA: Increase dopamine release; used for treatment of ADHD, narcolepsy
and depression (16). High-dose, chronic therapy with these drugs can cause
mydriasis and dry eyes. In patients with narrow anterior chamber angles,
the mydriasis effect can lead to acute or subacute stages of angle closure (2),

Copyright 2014 by KMK Educational Services, LLC


448 10.8. MEDICATIONS ACTING ON CNS

All medications that are dopamine agonists in the CNS (including


ADHD and Parkinson meds) have adrenergic agonist effects and
may cause mydriasis that precipitates an acute angle closure attack
in those with narrow anterior chambers. Glaucoma is a listed as a
contraindication for these medications (19).

Donepezil (Aricept® )
MOA: CNS Acetylcholinesterase inhibitor used to treat Alzheimer's de­
mentia.
Adverse Effects: Ocular side effects include cataracts, blurred vision, and eye ir­
ritation. As a cholinergic (muscarinic) indirect agonist, Donepezil will lower
IOP. Sudden discontinuation of therapy may lead to spikes in IOP (19).
lANTipjdPRESS ANTS
There are 3 major classes of antidepressants. All increase serotonin concentra­
tions in the CNS; TCAs and MAOIs also increase norepinephrine. All share
systemic effects including fatigue/sedation, weight gain, and sexual dysfunc­
tion (19).

Increasing serotonin levels is the primary method for treatment


of depression. Serotonin selective reuptake inhibitors (SSRIs) are
first-line treatments for the condition.

1. Selective Serotonin Reuptake Inhibitor: Fluoxetine (Prozac®)


MOA: Inhibits Serotonin reuptake from the synaptic cleft in the CNS.
Adverse Effects: Fewer side effects than the other classes. Use with caution
in glaucoma patients because Fluoxetine can cause mydriasis (19). Serotonin
syndrome may occur if used with TCAs and especially with MAOIs.

Serotonin syndrome is due to excess serotonin activity in the


CNS and is marked by mental status change (anxiety, confusion),
autonomic hyperactivity (increased blood pressure, pulse, temper­
ature), and neuromuscular problems (tremor, hyperreflexia).

Copyright 2014 by KMK Educational Services, LLC


CHAPTER 10. GENERAL PHARMACOLOGY 449

2. Tricyclic Antidepressants (TCAs): Amitriptyline (Elavil®),


Imipramine (Tofranil®)
MOA: Inhibits NE and Serotonin reuptake
Side Effects: Anticholinergic effects including: dry eye, increased IOP,
blurred vision, and mydriasis (2). TCA overdose is LIFE THREATEN­
ING! DO NOT give to a suicidal patient!
3. Monoamine Oxidase Inhibitor: Phenelzine (Nardil®)
MOA: Inhibits MAO the enzyme responsible for breaking down NE and
Serotonin.
Adverse Effects: Glaucoma and nystagmus are reported ocular effects, and
caution is urged in glaucoma patients (19). MAOIs have many drug interactions
as a class. Foods with tyramine (wine, cheese and dried meats) in combination
with the MAOIs can cause a lethal hypertensive crisis. When MAOIs are
taken with TCAs or SSRIs, serotonin syndrome can occur as noted above.

MAOIs and TCAs can exacerbate systemic effects of topical


phenylephrine and other adrenergic agonists (2).

Diazepam (Valium®)
MOA: Benzodiazepine that binds GABA receptors and causes hyperpolar­
ization of neurons in the CNS by opening chloride channels. Used for acute
anxiety, panic attacks, sedative and alcohol withdrawal.
Adverse Effects: Sedation; when combined with alcohol, these drugs can be
fatal. Occasionally cause mydriasis (anticholinergic activity) and nystag­
mus (2); narrow-angle glaucoma is listed as a contraindication (19).

I. Phenytoin (Dilantin®)
MOA: Acts on multiple neurotransmitters including norepinephrine, acetyl­
choline and GABA (16).
Adverse Effects: Nystagmus, diplopia, extraocular muscle palsies, ataxia
and gingival hyperplasia (19).
Copyright 2014 by KMK Educational Services, LLC
450 10.9. ENDOCR.INE SYSTEM MEDICATIONS
2. Phenobarbital (Luminal©)
MOA: Reduces excitatory transmission (glutamatergic) through the AMPA
receptor blockade, used for seizures and sedation.
Adverse Effects: Sedation, respiratory depression, mydriasis, increased IOP,
and cycloplegia.
3. Topiramat©

MOA: Multiple CNS effects to prevent seizures.


Adverse Effects:
• Effects in 10% or more of patients include blurred vision, diplopia, and
nystagmus.
• Uncommon effects include conjunctivitis, changes in lacrimation, and my­
opia.
• Rarely, choroidal swelling may push the uvea forward and cause acute
secondary angle-closure glaucoma (19).
- SECTION 10.9 ---------------------------------------------------------------------------------------------------

Endocrine System Medications

Insulin
Given to Type I diabetic patients and to patients with Type II when they can
no longer be controlled with oral medications. Insulin promotes the formation
and storage of glycogen, protein and triglycerides; this occurs in the liver, fat
and muscle tissues (15). The main signal for insulin release is the presence of
glucose in the blood.
MOA: Cell surface receptor causes activation of tyrosine kinase receptors
and a phosphorylation cascade.
ORAL AGENTS FOR DIABETES
1, Biguanide: M etformin (Glucophage®)
MOA: Decreases gluconeogenesis (liver glucose production) and increases
glucose uptake. Usually first-line oral med because it does not cause hypo­
glycemia.
Copyright 2014 by KMK Educational Services, LLC
CHAPTER 10. GENERAL PHARMACOLOGY 451

Adverse Effects: Diarrhea is most common. Renal insufficiency or hepatic


impairment increase the chance of lactic acidosis (19).
2. Sulfonylureas: Glipizide (Glucotrol®), Glyburide (Diabeta® ,
M icronase®), Chlorpropamide (Diabinese®)
MOA: Increase secretion of insulin by beta cells, decrease glucagon release
and increase sensitivity to insulin.
Adverse Effects: Hypoglycemia, which may be severe. As suggested by the
class name, these drugs have sulfa moieties.

Beta blocking medications, including ocular beta blockers, increase


the risk of hypoglycemia with sulfonlyureas, and beta blockers
mask the sympathetic response to hypoglycemia in all diabetic pa­
tients (preventing the usual symptoms such as tachycardia and
tremor). The risk is lower but still exists with ocular beta block­
ers (19).

3. Thiazolidinedione: Pioglitazone (Actos®)


MOA: Activates peroxisome proliferator-activated receptor gamma
(PPAR-gamma) to increase glucose uptake in muscle and fat tissues. Thiazo-
lidinediones were introduced to combat insulin resistance (16).
Adverse Effects: Decreased visual acuity caused by new or worsening mac­
ular edema has been reported with Pioglitazone; caution is urged before
prescribing the medication for patients with diabetic retinopathy or macular
edema (19). Other effects include edema, heart failure, hypoglycemia.

Remember that TSH (thyroid stimulating hormone) from the anterior pituitary
binds TSH receptors on the thyroid gland in the neck, stimulating production of
thyroid hormone (primarily T4). Hashimoto’s Thyroiditis is the most common
cause of primary hypothyroidism and requires replacement of T4.
Levothyroxine (Synthroid®)
MOA: Synthetic T4 hormone
Adverse Effects: Hyperthyroidism symptoms and pseudotumor cerebri in chil­
dren (19).
Copyright 2014 by KMK Educational Services, LLC
452 10.10, GENITOURINARY MEDICATIONS

Estrogens are female sex hormones - primarily found as estradiol-17. Progestins


are hormones that promote gestation, primarily progesterone (37).

Review from physiology...


1. Estrogens stimulate growth of endometrium.
2, Progesterone inhibits endometrial growth, maintains vascular
supply and withdrawal leads to menstruation.

Therapeutic uses of estrogens: Used in oral contraceptives, and utilized


for the treatment of hypogonadism, hypopituitarism, post-menopausal symp­
toms, osteoporosis, dysmenorrhea and premenstrual symptoms. In almost all
cases, progestins are given with estrogens to decrease the risk of endometrial
cancer (37).
Adverse ocular effects of estrogens: Dry eyes, optic neuritis, papilledema
due to pseudotumor cerebri. All except dry eye are rare effects. Oral contra­
ceptives also increase risk of venous blood clots. This may present as a
CRVO or BRVO, or it may present systemically as a deep venous thrombosis
(DVT) or pulmonary embolus (PE).
- SECTION 10.10 -------------------------------------------------------------------------------------------------

G enitourinary Medications

Erectile Dysfunction (ED) usually results from endothelial dysfunction and is


a considered a form of peripheral vascular disease. It has the same risk factors
as cardiovascular disease and the same result: inadequate blood flow to end
organs.
Phosphodiesterase inhibitors
Sildenafil (Viagra® ), Vardenafil (Levitra®)
MOA: cGMP relaxes vascular smooth muscle in the penis during an erection
but is broken down by phosphodiesterase type 5 (PDE-5). Sildenafil and
Vardenafil both inhibit PDE-5, thereby increasing blood flow to the penis
by prolonging cGMP effects (19).
Copyright 2014 by KMK Educational Services, LLC
CHAPTER 10. GENERAL PHARMACOLOGY 453

Systemic adverse effects: Flushing and headache are common; priapism (sus­
tained erection) may occur and requires immediate medical attention (19).
Ocular side effects: NAION, color changes (especially cyanopsia, blue tint­
ing), blurred vision and photosensitivity (18) (19).

Alpha-1 Blockers: Prazosin (M inipress®), Terazosin (H ytrin® ),


Tamsulosin (Flomax®)
MOA: a l blockers relax smooth muscle in the bladder neck and prostate
to decrease urinary outflow obstruction in men with BPH. The urine stream
becomes stronger as the obstruction is relieved. Also, the bladder empties
more effectively, so men do not feel the urge to urinate repeatedly. Prazosin
and Terazosin also block a l receptors in blood vessels, preventing peripheral
vasoconstriction.
Clinical Uses: All are indicated for treatment of benign prostatic hypertrophy
(BPH); Terazosin and Prazosin are also indicated for treatment of hypertension
(HTN).
Adverse Effects: 1st dose orthostatic hypotension causing dizziness, headache
(not usually seen with Tamsulosin). Tamsulosin can cause intraoperative
floppy iris syndrome, although all a l blockers carry this warning (19).

- SECTION 10.11 ------------------------------------------------------------------

Cardiovascular M edications

1. ACE Inhibitors (Inhibitors of Angiotensin production) and


Angiotensin II Receptor Antagonists (ARBs)
Recall the Renin-Angiotensin Aldosterone System from physiology. See Figure.

A. ACE inhibitors: Lisinopril (Prinivil® , Zestril® ), Benazepril


(Lotensin®), Enalapril (Vasotec®), Captopril (Capoten® )
MOA: 1. Prevent formation of Angiotensin II by inhibiting Angiotensin con­
verting enzyme (ACE). 2. Block metabolism of bradykinins (vasodilation).
Adverse Effects; Cough. Other effects include hypersensitivity reactions and
angioedema (swelling of the tongue and lips, and sometimes the throat!) (19).
Copyright 2014 by KMK Educational Services, LLC
454 10.11. CARDIOVASCULAR MEDICATIONS

Figure 10.3: Renin-Angiotensin-Aldosterone System

B. Angiotensin II Receptor Antagonist (ARB): Losartan


(Cozaar®)
MOA: Inhibits Angiotensin II contraction of vascular smooth muscle and stim­
ulation of aldosterone secretion, reducing blood pressure.
Adverse Effects: NO COUGH, so these are often prescribed when patients
cannot take an ACE inhibitor due to the cough side effect.
2. Beta Blockers
Propranolol (Inderal®), Labetalol (Trandate®)
MOA: Non-selective Beta 1 and Beta 2 antagonists. They also block release of
renin from the kidney.
Copyright 2014 by KMK Educational Services, LLC
CHAPTER, 10. GENERAL PHARMACOLOGY 455

Metoprolol (Lopressor®), Atenolol (Tenormin®)


MOA: Selective Beta 1 blockers
Side effects of B-blockers include the following:
SYSTEM EFFECT
CNS disorientation, depression, fatigue
Cardiovascular bradycardia, arrhythmias, syncope
Pulmonary dyspnea, wheezing, bronchospasm
Digestive (GI) nausea, vomiting, diarrhea, abdominal pain
Reproductive erectile dysfunction

3. Calcium Channel Blockers: Nifedipine (Procardia® ), Verapamil


(Calan®, Isoptin® ), Diltiazem (Cardizem®)
MOA: Block L-type calcium channels causing a marked decrease in
_ intracellular free calcium, All treat HTN because they decrease periph­
eral vascular resistance, but Verapamil and Diltiazem more commonly
have direct cardiac effects including negative chronotropism (slowing the
heart rate and potentially causing bradycardia) and inotropism (decreasing
contractility and potentially causing or worsening heart failure). For this rea­
son, glaucoma patients on Verapamil or Diltiazem will be at higher risk for
bradycardia and heart failure (than a patient on Nifedipine) with the addition
of Timolol.

Some glaucoma specialists recommend calcium channel blockers


for the treatment of low-tension glaucoma; these drugs may
increase perfusion to the optic nerve (40).

4. Diuretics
A. Loop Diuretic: Furosemide (Lasix®)
MOA: Inhibits Na 2Cl- K-f- Co-transport in the thick ascending LOH (16).
Also increases Ca2+ secretion - causes the Loop of Henle to lose Ca2+.
Adverse Effects: Hypokalemia, nephrotoxicity, and ototoxicity (19).
B. Thiazides: Hydrochlorothiazide (Hydrodiuril® ), Chlorothiazide
(Dxuril®)
MOA: Acts on the early distal convoluted tubule (DCT) and inhibits NaGl
reabsorption and decreases Ca2+ excretion into the loop.
Copyright 2014 by KMK Educational Services, LLC
456 10.11. CARDIOVASCULAR MEDICATIONS

Afferent Arteriole

Figure 10.4: The Nephron

Adverse Effects: Acute transient myopia, acute angle-closure glau­


coma, both of which are unusual

C. Potassium-Sparing Diuretics: Spironolactone (Aldactone®) and


Triamterene (Dyrenium)

Aldosterone is stimulated in response to decreased blood volume


(via Angiotensin II) and increased KT concentration. It functions
to increase Na reabsorption and secretion of I< in the collecting
duct by opening epithelial sodium channels (ENaCs) and by
activating the Na-K ATPase pump. The net effect is increased Na+
and Cl- reabsorption, and increased K+ and H+ secretion.

Spironolactone (Aldactone®)
MOA: Blocks aldosterone’s action at the late DCT and collecting duct
thereby increasing the excretion of sodium and water and decreasing the passive
Copyright 2014 by I<MK Educational Services, LLC
CHAPTER 10. GENERAL PHARMACOLOGY 457

secretion of I<+ - so it “spares K+.” Aldactone® is not entirely specific to


blocking aldosterone receptors-it also blocks androgen receptors.
Adverse Effects: Hyperkalemia, gynecomastia, and antiandrogen effects (19).
Triamterene (Dyrenium®)
MOA: Directly blocks ENaCs (epithelial sodium channels) in the late DCT
and the late collecting ducts (19). The net effects are similar to Spironolactone
(weak diuresis, potassium spared), but Triamterene does not have the anti­
androgen effects.
D. Osmotic Diuretic: Mannitol (Osmitrol®)
MOA: Increases plasma osmolarity, drawing water out of the eye (in acute
angle-closure attack) or the brain (in cerebral edema) (16). In the kidney,
mannitol works on the entire nephron as an osmotic diuretic.
Adverse Effects: IV mannitol to lower IOP is contraindicated in patients with
pulmonary edema, dehydration and CHF.

All diuretics can cause dry eye.

5. “Everything Else”
A. Clonidine (Catapres® )
MOA: CNS a2 agonist. Sympathetic outflow decreases and parasympathetic
tone increases. This will cause decreased vascular resistance and decreased
heart rate.
Adverse Effects: Dry mouth, sedation, impotence, and severe rebound hyper­
tension.

Clonidine is the parent compound of apraclonidine. What effect on


IOP would you anticipate?

B. Hydralazine (Apresoline®)
MOA: Increases cGMP which results in smooth muscle relaxation. It vasodi­
lates arterioles more than veins causing an afterload reduction.
Adverse Effects: Compensatory tachycardia, fluid retention and lupus-like syn­
drome. Conjunctivitis, lacrimation reported (19).
Copyright 2014 by KMK Educational Services, LLC
458 10.11. CARDIOVASCULAR MEDICATIONS
Agents

Digoxin (Lanoxin®)
MOA: Inhibits N a+ /K + ATPase enzyme - which normally pumps Na+
out and K+ in. This leads to increased intracellular Ca2+.
Adverse Effects: Retrobulbar optic neuritis, B/Y color defects and entopic
phenomenon (“snowy” vision, dimming vision, flickering lights) (2).

Other drugs used in congestive heart failure include Beta Blockers,


ACE inhibitors, and diuretics.

ANTIARRHYTHM IC MEDICATION
Amiodarone (Cordarone®)
MOA: Blocks I<+ channels (some Na+ and Ca2+ channels as well).
Clinical Uses: Most effective agent for supraventricular and ventricular tach­
yarrhythmias.
Adverse Effects: Drug-induced nonarteritic ischemic optic neuropathy
(NAION), whorl keratopathy, and anterior subcapsular lens deposits.
Can also cause fatal pulmonary or hepatic toxicity, and can cause thyroid
dysfunction.

Optic neuropathy (uncommon) typically occurs within weeks of


starting amiodarone. Whorl keratopathy is common, especially at
doses >400 mg/day. The anterior subcapsular lens deposits are
relatively common (50%) in higher doses (>600 mg/day) after 6
months of treatment (2).

W arfarin (Coumadin®)
MOA: Vitamin K antagonist, interfering with clotting factors II, VII, IX
and X that depend on vitamin K for synthesis (12).
Clinical Uses: Chronic anticoagulation for patients with previous blood clots,
mechanical heart valves, or atrial fibrillation.
Copyright 2014 by KMK Educational Services, LLC
CHAPTER 10. GENERAL PHARMACOLOGY 459

Adverse Effects: Should be discontinued 96 to 115 hours (4 doses) prior to


cataract surgery (40). Bleeding, NOT for use in pregnancy, skin necrosis (5) (12).

Warfarin decreases the concentration of vitamin K; therefore, its


effects can be reversed by giving the patient vitamin K.

Clopidogrel (Plavix®)
MOA: Inhibits the ADP receptor on platelet cell membranes needed
for platelet aggregation and clot formation.
Clinical Uses: Used after heart attack or stroke to prevent further atheroscle­
rotic events.
Adverse Effects: Increased risk of bleeding, GI upset, and rash (32). Like
aspirin, clopidogrel’s effects on platelets are irreversible.
Dipyridamole (Persantine®)
MOA: Inhibits adenosine deaminase and phosphodiesterase, causing
accumulation of cAMP and adenosine. These inhibit platelet aggregation
and may cause vasodilation.
Adverse Effects: Bleeding risk increased, especially when used in combination
with other medications that increase bleeding risk (19). Aggrenox® used for
patients with a history of ischemic stroke is a just such a combination (aspirin-
dipyridamole) and is probably the most common formulation of Dipyridamole
used in outpatients.

As you prepare for the exam, be sure you can categorize sys­
temic medications that potentially increase bleeding complications
in the eye: Warfarin, Clopidogrel, Dipyridamole, and don’t forget
NSAIDs and aspirin!

HMG CoA reductase inhibitors


Lovastatin (Mevacor®), Simvastatin (Zocor®), Atorvastatin
(Lipitor®)
MOA: Inhibit HMG CoA reductase, an enzyme used for the biosynthesis
of cholesterol. They lower LDL and triglyceride concentrations and increase
Copyright 2014 by KMK Educational Services, LLC
460 10.12. DERMATOLOGIC MEDICATIONS
HDL. First-line therapy for hyperlipidemia.
Adverse Effects: Ilepatotoxicity, myopathy (muscle pain or inflammation),
Contraindicated during pregnancy or lactation (5).

Fibric Acid
Gemfibrozil (Lopid®)
MOA: Binds to peroxisome proliferator activated receptor (PPAR - a) and
increases the activity of lipoprotein lipase for the breakdown of VLDL (16), so
especially useful for lowering VLDL and triglycerides.
Adverse Effects: GI disturbances, skin rash, urticaria, myositis.

Bile Acid Binding Resin


Cholestyramine (Q uestran® )
MOA: Binds to bile acids in the intestine and prevents their reabsorption.
Clinical Uses: Used in combination with a HMG CoA reductase inhibitor to
lower LDL.
Adverse Effects: Constipation and fatty stools.

— SECTION 10.12 ------------------------------------------------------------

Dermatologic Medications

Isotretinoin (Accutane®)
MOA: Reduces oil production from sebaceous glands and reduces the size of
the glands themselves (19).
Adverse Effects: Blepharoconjunctivitis, dry eyes, pseudotumor cere­
bri, lid edema, color vision loss, nyctalopia (night vision decreased). Cataracts
have also been reported (19). Accutane® is teratogenic and not to be used in
pregnancy!

Metronidazole (M etroGel®)
MOA: As a topical agent used for acne rosacea, disrupts DNA and inhibits
nucleic acid synthesis; it also has anti-inflammatory properties (19).
Adverse Effects: Irritation of the skin and eyes is reported.
Copyright 2014 by KMK Educational Services, LLC
CHAPTER 10. GENERAL PHARMACOLOGY 461

- SECTION 10.13 ---------------------------------------------------------------------

Hallucinogens and Drugs o f Abuse

Nicotine
Dependence occurs readily due to the fact that it acts in the pleasure centers
of the brain (16). Withdrawal symptoms include craving, anxiety, irritability
and restlessness, increased appetite.

Alcohol (ethanol)
Dependence occurs readily in those with pre-dispositions to addictive behav­
iors, Withdrawal symptoms include anxiety, tremors, increased blood pressure,
increased heart and respiratory rates, and-when severe-delirium tremens and
seizures. Alcohol withdrawal can be (though is not always) life-threatening!
Chronic alcoholism can lead to thiamine deficiency, resulting in an acute pre­
sentation of Wernicke’s Encephalopathy - a condition characterized by oph­
thalmoplegia, confusion, and ataxia that is reversed with thiamine. Untreated
thiamine deficiency can lead to irreversible Korsakoff syndrome which is
characterized by amnesia and confabulation (42).

Opioids
Abuse occurs with heroin or with prescription opiates like Oxycodone. With­
drawal symptoms include mydriasis, anxiety,, lacrimation, rhinorrhea, sweat­
ing (leaky everywhere), tremor, nausea and vomiting, heart rate and blood
pressure elevation. Unlike alcohol withdrawal, opioid withdrawal is not life-
threatening (9).

Cocaine
Blocks dopamine and norepinephrine uptake in the reward centers of the brain.
As an indirect adrenergic agonist, effects include Mydriasis, elevation of mood,
tremors, chest pain and heart palpitations; heart attacks can occur. With­
drawal is associated with severe craving, depression and fatigue, but again is
not life-threatening.
Copyright 2014 by KMI< Educational Services, LLC
462 10.14. DISINFECTANTS AND ANTISEPTICS
— SECTION 10.14 ------------------------------- --------------------------------

Disinfectants and Antiseptics

Disinfectants
Chemical agents that inhibit or kill microorganisms. Sodium hypochlorite
(household bleach) can be used for disinfecting blood spills. Formaldehyde
is used for disinfection or sterilization of medical instruments.

Antiseptics
Disinfecting agents with low toxicity for human cells. They can be used on
skin, mucous membranes or wounds without harm (16).

Alcohols: Isopropyl alcohol (90%) and ethanol (70%) are most frequently
used; effective immediately, resulting in death of vegetative bacteria, M. tuber­
culosis and many fungi (16).

Iodine: Kills spores in approximately 15 minutes and is the most active an­
tiseptic for intact skin. Not commonly used due to hypersensitivity.

Oxidizing Agents (hydrogen peroxide): Used for cleaning wounds and


disinfecting contact lenses.

Heavy Metals: Silver sulfadiazine is bactericidal and„used for burn wounds (16).
“ SECTION 10.15 ---------------------------------------------------------------- ---------------------------
Autonomic and/or Neuromuscular Junction Drugs
(N M J )

:cU(n.INKRGrC AOONISTS
Important ocular drugs to know include direct agents (Pilocarpine, Acetyl­
choline, Bethanechol, Carbachol) and indirect agents (Edrophonium, Echoth-
iophate, Pyridostigmine, and Neostigmine.)

Donepezil is a CNS acetylcholinesterase inhibitor (indirect cholinergic ago­


nist) used in treatment of Alzheimer dementia.
Copyright; 2014 by KMK Educational Services, LLC
CHAPTER 10. GENERAL PHARMACOLOGY 463

Important ocular drugs that “STop ACH” include Scopolamine, Tropic amide,
Atropine, Cyclopentalate, and Homatropine. Recall that several classes of
systemic medications have anticholinergic properties:
• First generation HI blockers: Diphenhydramine (Benadryl®), Brompheni­
ramine (Dimetane®), Chlorpheniramine (Chlor-Trimeton®), Promet­
hazine (Phenergan®)
• Antipsychotics (Phenothiazines): Chlorpromazine (Thorazine®), Thiori­
dazine (Mellaril®). Remember that Promethazine is also a phenothiazine
but is not used as an antipsychotic.
• Antidepressants: TCAs Amitriptyline (Elavil®) and Imipramine (Tofranil®);
MAOI Phenelzine (Nardil®)
• Muscle relaxant: Cyclobenzaprine (Flexeril®)
• Anxiolytic: Diazepam (Valium®)
• Ipratropium (Atrovent®): while the other drugs listed above have an­
ticholinergic side effects, remember that ipratopium’s primary MOA is
muscarinic blockade (resulting in bronchodilation!).

Important ocular adrenergic agonists include alpha-1 agonist Phenylephrine


and alpha-2 agonists Apraclonidine and Brimonidine, as well as nonspecific
alpha agonists Naphazoline and Tetrahydrozoline (used as topical ocular de­
congestants). Systemic drugs include:
• Alpha-2 agonist: Clonidine (Catapres©)
• Beta-l/Beta-2 agonist: Isoproterenol (Isuprel®)
• Beta-2 agonists (with variable beta-1 activity): Salmeterol (Serevent®),
Albuterol (Ventolin®), Levalbuterol (Xopenex®), Terbutaline (Brethine®),
Metaproterenol (Alupent®). Of course these drugs would be expected
to augment aqueous production. Salmeterol is the only long-acting
drug of these five and the only one which does not carry a specific warning
about use in patients with glaucoma.
• Nonspecific alpha and beta agonist: Pseudoephedrine (Sudafed®)
• Recall that drugs with dopamine agonist activity, including Bromocrip­
tine (Parlodel®), used to treat hyperprolactinemia; ADHD drugs Methylphenidate
(Ritalin®) and Dextroamphetamine (Dexedrine®); and drugs used in
Copyright 2014 by KMK Educational Services, LLC
464 10.16. TOXICOLOGY
the treatment of Parkinson Disease such as Amantadine (Symmetrel®),
all have adrenergic agonist properties and are of particular concern for
precipitating acute angle closure attacks in patients with narrow anterior
chambers.

Topical /3 blockers include (non-selective) Timolol, Carteolol, Metipranolol,


Levobunolol, and (beta-1 specific) Betaxolol. Systemic adrenergic antagonists
include a l blockers and /3 blockers:
• Alpha-1 antagonists used for BPH: Tamsulosin (Floma-x®), Terazosin
(Hytrin®), Prazosin (Minipress®)
• Beta-1 and Beta-2 antagonists (non-selective): Labetolol (Trandate®),
Propranolol (Inderal®)
• Beta-1 specific antagonists: Atenolol (Tenormin®), Metoprolol (Lopressor®)
r - SECTION 10.16 ---------------------------------------------------------------------------------------------

Toxicology

Each drug has specific toxicities that can arise if given too much of the medi­
cation. This often results in nephrotoxicity or hepatotoxicity. Elderly patients
are at a much higher risk of reaching a toxic level because of their decreased
body mass and drug elimination. The safest way to monitor for toxicity is to
start by prescribing low doses and proceed slowly through the treatment. The
lowest dosage that shows effectiveness is the desired choice (16).
- SECTION 10.17 --------------------------------------------------------------------------------------------

Drug Use in Pregnancy and Breast Feeding

All drugs are required to undergo trials to test their safety for use in pregnancy.
They are given a rating in terms of how safe they are to the developing fetus.
The following ratings are listed in the physicians drug reference (PDR) (33).
A - Studies in pregnant women have not shown a risk to the fetus in any
trimester of pregnancy.
B - Animal studies have failed to show a risk to the fetus but there are no in­
sufficient studies in pregnant women. Or animal studies have shown an adverse
effect, but human studies have not shown adverse effects to the fetus in any
trimester.
Copyright 2014 by KMK Educational Services, LLC
CHAPTER 10. GENERAL PHARMACOLOGY 465

C - Adverse effects on the fetus were found in animal studies, but there are no
adequate human studies. Or there are no animal or human studies to determine
the safety.
D - Evidence of human fetal risk but the benefits may outweigh the risks.
X - Human or animal studies have shown fetal abnormalities and the risks are
much more significant than the benefits.

Ocular structures are most damaged by teratogens if exposed dur­


ing the first trimester.

References
[1] Bartlett, Jimmy D., Jaanus, Siret D, Clinical Ocular Pharmacology. Boston: Butterworth,
1984.
[2] Bartlett, Jimmy D., Jaanus, Siret D. Clinical Ocular Pharmacology. Boston: Butterworth,
2008.
[3] Bernstein HN. Chloroquine ocular toxicity. Surv Ophthalmol. Oct 1967; 12(5);415-47.
[4] Bhushan, Vikas, Le, Tao, Amin, Chirag. First Aid for the USMLE Step 1. New York:
McGraw-Hill, 2003,
[5] Briggs, Gerald B., Freeman, Roger K., and Sumner J. Yaffe. Drugs in Pregnancy and Lacta­
tion. 6th ed. Philadelphia: Lippincott, Williams and Wilkins: 2002.
[6] Bylund, David. Pharmacology 507. “The Receptor Concept in Pharmacology.” Class Notes.
University of Nebraska Medical Center. 2002.
[7] Cardiovascular Prescribing Guide, 6th ed. Montvale: Thompson Medical Economics, 2002.
[8] Dupree, Jean D. Pharmacology 507. “Basic Principles of Pharmacokinetics,” Class Notes.
University of Nebraska Medical Center. 2002.
[9] Dupree, Jean D. Pharmacology 507. “Pharmacology of Benzodiazepines, Barbiturates, Hyp­
notics, Ethanol, Antiepileptics and Local anesthetics.” Class Notes. University of Nebraska
Medical Center, 2002.
[10] Ellis, Philip, Ocular Therapeutics and Pharmacology, 5th ed. St, Louis: Mosby, 1977.
[11] Gilbert, David et ai. Stanford Guide to Antimicrobial Therapy. Vienna: Antimicrobial Ther­
apy Inc.
[12] Green, Gopa B. Ed et. al. The Washington Manual of Medical Therapeutics, 31st ed. Philadel­
phia: Lippincott Williams and Wilkins, 2004.
[13] Guidelines for the programmatic management of drug-resistant tuberculosis: emergency up­
date 2008 (WHO/HTM/TB/2008.402). Geneva, Switzerland: World Health Organization.
2008. p. ix. ISBN 978 92 4 164758 1.
[14] Havener WH. Ocular Pharmacology. St Louis: Mosby, 1978.
[15] Katzung, Bertram G. and Anthony J. Trevor. Examination and Board Review Pharmacology,
4lh ed. Norwalk: Appleton and Lange, 1993.

Copyright 203.4 by KMK Educational Services, LLC


466 10.17. DR UG USE IN PREGNANCY AND BREAST FEEDING
[16] Katzung, Bertram G. Basic and Clinical Pharmacology, 8th ed. Now York: Lange Medical
Books/McGraw Hill, 2001.
[17] Kawahara, Rodney. Pharmacology 507. “Cancer Chemotherapy.” Class Notes. University of
Nebraska Medical Center. 2002.
[18] Laties, AM, et al. “Viagra (sildenafil citrate) and ophthalmology." Progress in Retinal and
Eye Research. 2002;21: 485-506.
[19] Lexi-Comp Online, Lexi-Druga, Hudson, Ohio: Lexi-Comp, Inc.; December 2011.
[20] Lowy, PD. Treatment of skin and soft tissue infections due to methicillin-resistant Staph
aureus infections in adults. In: UpToDate, Basow, DS (Ed), Up'lbDate, Waltham, MA,
2011 .

[21] Marmot', MF, et al. “ Recommendations on screening for chloroquine and hydroxychloro­
quine retinopathy: a report by the American Academy of Ophthalmology,” Ophthalmology.
2002;109:1377-82.
[22] Melamud, A, et al. “Ocular Ethambutol Toxicity.” Mayo Clinic Proceedings. 2003;78:1409-
1411.
[23] Melton, Ron. Thomas, Randall, Guide to Ophthalmic Drugs. Review of Optometry, June,
2009.
[24] Moore, Keith and Arthur Dailey. Clinically Oriented Anatomy, 4th ed, Baltimore: Lippincott
Williams and Wilkins, 1999.
[25] Murrin, Charles L. Pharmacology 507. “Pharmacotherapy of Migraine and Pharmacotherapy
in Parkinson’s Disease.” Class Notes. University of Nebraska Medical Center. 2002,
[26] Neill, TA, et al. Reversible posterior leukoencephalopathy syndrome. In: UpToDate, Basow,
DS (Ed), UpToDate, Waltham, MA, 2011.
[27] Nirken, MH, et al, Stevens-Johnson syndrome and toxic epidermal necrolysis: Clinical
manifestations; pathogenesis; and diagnosis. In: UpToDate, Basow, DS (Ed), UpToDate,
Waltham, MA, 2012.
[28] Noble, John, Ed. Textbook of Primary Care Medicine, 3rd ed. St. Louis: Mosby, 2001.
[29] Olson, James. Clinical Pharmacology Made Ridiculously Simple, 2nd ed. Miami: Medmaster.
Inc., 2001.
[30] Parish, Witltowski. Guide to the management of scabies. Drug Therapies. 1978; 134-136.
[31] Pelak, VS, et al. Ocular Myasthenia Gravis. In: UpToDate, Basow, DS (Ed), UpToDate,
Waltham, MA, 2011.
[32] Physician Assistant Prescribing Reference, Spring 2008, vol. 15, num. 1, p. 146.
[33] Physicians’ Desk Reference, 56th Ed. Montvale: Thompson Medical Economics, 2002.
[34] Physicians’ Desk Reference For Nonprescription Drugs and Dietary Supplements, 22nd Ed.
Montvale: Thompson Medical Economics, 2001.
[35] Rang, H. P. (2003). Pharmacology. Edinburgh: Churchill Livingstone. ISBN 0-443-07145-4.
Page 103
[36] Roth N. “Refractive State after Instillation of Paredrine and Neosynephrine.” Br J Ophthal­
mol 1968; 52: 763-767.
[37] Roy, Shymal. I<. 306/606/806 “Endocrine and Reproductive Physiology" Class Notes. Uni­
versity of Nebraska Medical Center. 2001,
[38] Stenchever, Morton et al. Comprehensive Gynecology, 4th ed. St. Louis: Mosby, 2001.
[39] Sugar SH, “Pitfalls in the medical treatment of simple glaucoma." Ann Ophthalmol 1979;
11:1041-1050.

Copyright 2014 by KMK Educational Services, LLC


CHAPTER 10. GENERAL PHARMACOLOGY 467

[40] Tamesis, Richard R. Ophthalmology Board Review., 2nd Edition. McGraw-Hill, 2006.
[41] Terry, Jack. Ocular Disease - Detection, Diagnosis, and Treatment. Springfield: Thomas,
1984.
[42] Tierney. Lawrence M., McPhee, McPhee, Stephen, and Maxine A. Papadakis Eds. Current
Medical Diagnosis and Treatment, 41st. ed. New York: McGraw-Hill, 2002.
[43] Thoreson, Wallace B. Pharmacology 507. “Antiarrythmic Agents and Cardiac Glycosides,’*
Glass Notes. University of Nebraska Medical Center. 2002.

Copyright 2014 by KMK Educational Services, LLC


(
c~

((

c( '
i
;

(
j

( i

((
'
'
~

(( '

((
((
((
((
((
((
((
((
((
((
((
((
((
((
((
((
((
((
((
((
(c
/
Chapter

Ocular Pharmacology

by Kyle Cheatham O.D., F.A.A.O.

469
470 11.1. GENERAL PRINCIPLES
r ~ SECTION 11.1

General Principles

Factors AIIc.JiiiK Oru 8 IJioavailai.ility


We start by summarizing drug bioavailability in the eye (3). Around 25%
of topical drug concentration is lost via evaporation upon instillation. The
remaining amount has one of three possible fates (3, pp. 17):
1 Drainage into the nasolacrimal apparatus.
2 Absorption into the systemic circulation by the conjunctival and lid vas­
culature.
3 Penetration into the cornea.
Molecular differences in the various layers of the cornea require drugs to contain
both lipophilic and hydrophilic properties for suitable bioavailability.
Bioavailability
The percent of unchanged drug delivered to the desired site. Drugs must have
a very specific composition to make it through tears and the cornea to be
available within the eye. Let’s look at each layer to understand how the drugs
need to be formulated for increased bioavailability.
Tear layer: Each layer has unique characteristics.
• Lipid layer: Lipid soluble.
• Aqueous layer: Water soluble.
• Mucous: Combination of water and lipid soluble.
Corneal layers: Consists of three barriers to drug penetration.
• Epithelium and Endothelium: Lipid soluble.
• Stroma: Water soluble.
Drug Formulation
• Drugs must have a combination of lipid and non-lipid soluble components
to maximize bioavailability; small, uncharged (non-ionized), lipid-soluble
molecules will penetrate the best.
Copyright 2014 by KMK Educational Services, LLC
CHAPTER 11. OCULAR PHARMACOLOGY 471

• For example, homatropine has a pH of 6.0 and is formulated with an


ionized/nonionized ratio of 1,000: 1. When placed on the cornea, tears
immediately buffer the drug to a pH of 7.4 and the ratio changes to
40:1 (3).
- The non-ionized (lipid soluble) portion will penetrate the epithelium,
while the ionized portion (water soluble) is left behind.
- Upon reaching the stroma, a new equilibrium is established and now
the ionized portion readily passes through.
- Equilibrium is again established upon reaching the endothelium, and
finally the non-ionized lipid portion reaches the aqueous humor.
• Most ocular drugs are formulated as weak bases because this formula­
tion allows betters penetration and bioavailability due to the presence of
more non-ionized portions of the drug reaching the aqueous humor.

Route Pros Potential Cons


Topical: Applied to At site of desired effects Site irritation. Systemic
mucosa, conjunctiva, side effects
cornea or epidermis
Oral: Taken by mouth Simple dosage, easily GI distress, drug degradation,
administered, time released. absorption problems
Subconjunctival: Inject Rapid, effectively absorbed Fear, pain,
between conjunctiva inflammation
and sclera
Intravenous: Injected Very rapid, dose accuracy, Danger of cardiotoxicity
into vein bypass digestive tract (bolus), sterility
Intramuscular: Injected Rapid, controlled Pain, necrosis
into muscle and absorption
subcutaneous areas
Chart modified from Terry, Jack. Ocular Disease - Detection, Diagnosis, and
Treatment. Springfield: Thomas, 1984. page 13.
- SECTIO N 11.2 --------------------------------------------------------

Autonom ic Drugs

jFMnctional: Types
We now summarize the CNS before overviewing ocular receptors (29, ch. 2).
Recall that efferent motor nerves can be either somatic (voluntary) or auto­
nomic (involuntary). Somatic neurons innervate skeletal muscles, while auto­
nomic neurons innervate smooth and cardiac muscle and glands.
There are two major divisions of the autonomic pathway:
Copyright 2014 by KMK Educational Services, LLC
472 11.2. AUTONOMIC DRUGS
1 Parasympathetic: Cholinergic (muscarinic)
2 Sympathetic: Adrenergic
Some general characteristics about the autonomic divisions are as follows:
• Two cell bodies are involved - one always in the CNS - one outside the
CNS. Sympathetic cell bodies are located in the thoraco-lumbar regions
of the CNS. Parasympathetic cell bodies are located in the cranio-sacral
regions of the CNS.
• Preganglionic neurons extend from the CNS cell body to synapse at
the cell body outside the CNS. In both the sympathetic and parasympa­
thetic division, acetylcholine is released at this junction. Preganglionic
neurons are longer in the parasympathetic pathway.
• Postganglionic neurons extend from the second cell body to an af­
fected organ. The neurotransmitter here varies depending on the au­
tonomic division. Parasympathetic releases Ach again. Sympathetic re­
leases norepinephrine and epinephrine. Postganglionic neurons are longer
in the sympathetic pathway. /
• The affected organ has receptors on its surface that are bound by the neu­
rotransmitters. Organs that are innervated by the sympathetic system
contain alpha and beta receptors, while the parasympathetic system
acts on muscarinic receptors.
• Major actions of the parasym pathetic nervous system include bron-
choconstriction and miosis, “rest and digest” functions, and an increase in
secretions (SLUD)... salivation, lacrimation, urination, and defecation.
• Major actions of the sympathetic nervous system include bronchodila-
tion and mydriasis, “fight or flight” functions, and an overall decrease in
secretions.

Cholinergic Receptors in the Eye


Location Receptor General Effects
Iris sphincter M3 Miosis
Ciliary Muscle M2, M3 Accommodation
Lacrimal gland M2, M3 f Tear production
Copyright 2014 by KMK Educational Services, LLC
CHAPTER 11. OCULAR PHARMACOLOGY 473

Adrenergic Receptors in the Eye


Location Receptor General Effects
Iris dilator al Dilation
Trabecular Meshwork /?2 Relaxation, f Outflow
Ciliary Muscle /32 Relaxation, opposes accommodation
NPCE 42,41 Increases aqueous formation
CB vasculature a2 Constricts, reducing aqueous formation
Note: The above charts summarize the main points of autonomic innervation.
Some locations and receptors are not mentioned. Information was gathered
from the following sources: (3, ch. 1) (18, ch. 6).

Cholinergic Agonists (CA)


We now introduce drugs that promote parasym pathetic activity (6, ch. 4) (29,
ch. 2) (3, ch. 2). They include direct and indirect agonists.

The three main structures in the eye that receive parasympa­


thetic innervation are the sphincter muscle (miosis), ciliary muscle
(accommodation), and the lacrimal gland (lacrimation).

Cholinergic agonists are used therapeutically for the treatment of glaucoma


and accommodative esotropia. They are less commonly used to induce miosis
prior to surgery and for pupil abnormality testing (3).

1st ever glaucoma drug introduced in the 1870s; provides up to 30% IOP re­
duction, but has a relatively short half-life so frequent administration (qid) is
required for effectiveness (23) (3). Pilocarpine comes in concentrations ranging
from 0.5 —12%; 1%, 2% and 4% are the most commonly prescribed concentra­
tions (31).

Mechanism of Action (MOA): Pilocarpine and other CAs stimulate the


longitudinal muscle of the ciliary body, which pulls posteriorly on the scleral
spur and secondarily opens up the trabecular spaces for t outflow and \ IOP.
Clinical uses include:
Copyright 2014 by KMK Educational Services, LLC
474 11.2, AUTONOMIC DRUGS
• Utilized after an angle closure attack in preparation for a laser periph­
eral iridotomy (LPI). The miotic action of the drug pulls the iris taut and
allows the LPI to be more effective.
• 1% pilocarpine is used to differentiate a 3rd nerve palsy from a sphincter
tear in a patient with a fixed, dilated pupil - 3rd nerve palsies will constrict
with pilocarpine.
• 0.125% pilocarpine is used in the diagnosis of Adie’s tonic pupil. The
iris sphincter is supersensitized in Adie’s and will respond with miosis to
a diluted form of the drug.
Pilocarpine’s main side effects include:
• Browaches, headaches and myopic shifts - Headaches are extremely
common in pre-presbyopes, as are several diopter shifts in myopia.
• Miosis - smaller pupils allow better depth of focus and can improve read­
ing in presbyopes. However, in older patients with cataracts, inducing
miosis and further reducing the amount of light reaching the retina can
be problematic,
• Cataracts - After long term use. When/if the cataracts form centrally,
the miotic action of the drug (which obviously narrows the field of view)
causes symptomatic visual decline by making the cataracts more signifi­
cant. This is why a common side effect of pilocarpine is “dimmed vision.”
• Retinal Detachments - Uncommon but well-recognized side effect. CAs
can cause retinal breaks, which can lead to a detachment.
• Secondary angle-closure glaucoma - by causing contraction of the ciliary
muscle, CAs secondarily relax the lens zonules and the lens moves ante­
riorly. Also, the miotic action of the drugs can cause adhesion between
the iris and the lens.
- The combination of these factors can cause pupillary block, imped­
ing the flow of aqueous from the posterior chamber to the anterior
chamber.
— This build-up of pressure bows the iris forward peripherally, closing
off the angle and preventing aqueous outflow.

AchE inhibitors act as indirect agonists by inhibiting acetylcholinesterase,


the enzyme that breaks down acetylcholine.
Copyright 2014 by KMK Educational Services, LLC
CHAPTER 11. OCULAR PHARMACOLOGY 475

NEOSTIGM INE (Prostigmin®)


Clinical uses: Treatment of myasthenia gravis and for limb strength evaluation
(Neostigmine test) in patients with suspected myasthenia (2).
EDROPHONIUM (Enlon®)
Used in the diagnosis of Myasthenia gravis (Tensilon Test). Has a rapid onset
(via injection) of action (30-60 seconds) and short duration of action (approx­
imately 10 minutes). If ptosis improves (1-2 minutes after injection), the test
is positive for myasthenia (11) (17).
ECHOTHIOPHATE (Phospholine®)
Clinical Uses: Can be used for the diagnosis and/or treatment of accommoda­
tive esotropia and rarely for glaucoma (3).
PYRIDOSTIGM INE (M estinon®)
Clinical Uses: Pyridostigmine is used for the treatment of myasthenia gravis.

Echothiophate and Isofluorophate are irreversible AchE agents


not used for glaucoma treatment because of their high incidence of
side effects. Agents in this family are found in pesticides.

Used therapeutically for cycloplegic refractions, pupillary dilation, and the


management of uveitis. These drugs block Ach at muscarinic sites in the
ciliary body and iris.

Know all of the drugs in this class and their MOA. Remember the
following acronym:
• STop ACH = Scopolamine, Tropicamide, Atropine,
Cyclopentolate, Homatropine.
• Note that none of the drugs in this class are used for glaucoma
treatment.

Copyright 2014 by KMK Educational Services, LLC


476 11.2. AUTONOMIC DRUGS
SCOPOLAMINE
■ Has similar clinical indications as atropine, but rarely used in topical oph­
thalmic formulation due to the potential for severe side effects.
• CNS toxicity is more likely with scopolamine (penetrates blood brain
barrier better) than atropine.
• Adverse effects include hallucinations, amnesia, unconsciousness, confu­
sion, restlessness, incoherence, vomiting, and urinary incontinence (2).
• The contraindications for scopolamine are the same as those for atropine
(see atropine side effects) (2).

TROPICAM IDE
Has the fastest onset and shortest duration of mydriatic effects. Has
a much stronger mydriatic than cycloplegic effect. For these reasons, it is the
standard drug used for dilation. Clinical facts include:
• Maximum mydriatic effect occurs quickly (20-35 minutes) and lasts for
approximately 6 hours; onset and duration of effects vary depending on
race and iris color. Maximum cycloplegic effect occurs between 20-45
minutes and lasts for up to 6 hours (2).
• 1% and 0.5% have very similar effects on mydriasis; however, cyclo­
plegic effects are dose-related (higher concentrations affect accommoda­
tion more) (2).
• Adverse systemic reactions are rare; very safe agent overall, even in pa­
tients with cardiovascular disease.

ATROPINE
• Atropine’s onset (60-180 minutes) and duration (7-12 days) of cycloplegic
effects are too prolonged for routine cycloplegic refractions (2).
• Atropine can be used for the treatment of uveitis; however, homatropine
is much more commonly utilized for this purpose.
• Amblyopia treatm ent - the good eye is treated with atropine (called
penalization); often reserved for mild and moderate amblyopia (acuity
better than 20/100 in the amblyopic eye) (2).
• Systemic side effects: Atropine is safe when the correct dosage is
utilized; however, there are six reported cases of death in young children
(under 3 years of age), most of whom were sick and/or handicapped and
had been given an incorrect dosage (2).
Copyright 2014 by KMK Educational Services, LLC
CHAPTER 11. OCULAR PHARMACOLOGY 477

— Special caution is recommended in patients with Down’s syndrome,


small children, and the elderly (29) (2),

Atropine toxicity, although rare, can elicit the following symp­


toms (29): Dry mouth (usually the 1st sign), dry flushed skin,
rapid pulse, and disorientation and fever due to CNS effects on the
hypothalamus.

CYCLOPENTOLATE
Among drugs with significant cycloplegic effects, cyclopentalate has the fastest
onset and shortest duration of cycloplegic effects. Cyelopentolate is the
standard cycloplegic agent utilized in clinic. Clinical uses include:
• Cycloplegic effects - better than homatropine, similar overall effects to
atropine (but faster onset with less duration of action).
• Maximum effects - mydriatic (20-45 minutes) and cycloplegic (20-45 min­
utes), depending on race and iris color.
• Routine cycloplegic refractions for all ages, especially kids.
• Treatment of anterior uveitis.

HOMATROPINE
Has prolonged mydriatic and cycloplegic effects (although only 10% as potent
as atropine). It has a weak cycloplegic action, so it is not recommended clin­
ically for routine dilation (tropicamide is better) or cyclopegic examination
(cyclopentalate is better).
Homatropine is the standard for treating anterior uveitis.
1 Dilates pupil and keeps the iris mobile, which decreases the likelihood of
posterior synechiae formation.
2 Reduces pain by paralyzing the ciliary and sphincter muscles.
3 Stabilizes the blood aqueous barrier by constricting the iris and ciliary
body vasculature to limit passage of blood contents into the aqueous
humor.
Copyright 2014 by KMK Educational Services, LLC
478 11.2. AUTONOMIC DRUGS

Anticholinergic toxicity = hot as a hare, red as a beet, dry as


a bone, mad as a hatter, blind as a bat.

Drug Mydriasis (Effects) Cycloplegic (Effects)


Atropine 7-10 days 7-12 days
Homatropine 1-3 days 1-3 days
Scopolamine 3-7 days 5-7 days
Cyclopentolate 24 hours 24 hours
Tropicamide 4-6 hours 4 hours
Chart modified from Bartlett, Jimmy D. Jaanus, Siret D. Clinical Ocular Phar­
macology. Boston: Butterworth, 1984, pp. 107.

BOTULINA TOXIN (Botox) is a somatic drug that blocks the


release of Ach at the neuromuscular junction, inhibiting muscle
contraction.
• When utilized for blepharospasm treatment, orbicularis func­
tion returns quickly and effects only last a few months (18).
• Single injections of Botox have been effective in providing
permanent correction for strabismus (18); botox is also used
cosmetically to reduce wrinkling.

'Ocular Adrenergic Agents


Adrenergic Agonists
We now introduce drugs that promote sympathetic activity (6, ch. 4) (7, ch. 2).
Used therapeutically in a variety of ways, including dilation, conjunctival con­
striction, and management of minor allergic conditions. Also used for tem­
porary control of IOP spikes (post-surgery, angle closure glaucoma), and for
treatment of primary open angle glaucoma.

Norepinephrine differs from epinephrine in that it does not act on


B2 receptors. Recall that epinephrine acts on all four adrenergic
receptors (Al, A2, Bl, B2) (4).

Copyright 2014 by KMK Educational Services, LLC


CHAPTER 11. OCULAR PHARMACOLOGY 479

PHENYLEPHRINE (Neo-Synephrine®)
2.5% phenylephrine is routinely used in combination with tropicamide for rou­
tine dilation. In isolation, phenylephrine is unable to provide a fixed dilated
pupil.
MOA: a l agonist, no effects on B receptors. This allows it to cause dilation
(radial muscle) without cyclopiegia.
Clinical Uses:
• Dilation without cyclopiegia.
• Palpebral widening - acts on Muller’s muscle to retract the upper
eyelid (widens the palpebral fissure).
• Differentiates scleritis from episcleritis- blanched conjunctival ves­
sels (redness clears) indicates episcleritis.
• H orner’s syndrome - Phenylephrine may have value in diagnosing
Homer’s syndrome.
• Phenylephrine 10% is primarily limited to breaking posterior synechiae
because of its adverse cardiovascular effects, including hypertension
(risk of hypertensive crisis) and cardiac arrhythmias.
— Contraindicated in patients taking monoamine oxidase inhibitors
(MAOIs), tricyclic antidepressants (TCAs), and atropine.
— Contraindicated in patients with Graves’ disease.
NAPHAZOLINE (Naphcon®) and TETRAHYDROZOLINE
(Visine®)
Both are used clinically as topical ocular decongestants to constrict the con­
junctival blood vessels. They have greater alpha than beta effects so, unlike
other adrenergic agents, they have the potential to depress the central nervous
system.

If a patient presents with a fixed dilated pupil, ask about


Visine® use. Excessive Visine® can cause dilation because of
the alpha effects of tetrahydrozoline on the radial muscle. Naph­
con A® is a popular OTC drug that combines napliazoline (reduces
hyperemia) and an antihistamine for relief of itching.

Copyright 2014 by KMK Educational Services, LLC


480 11.2. AUTONOMIC DRUGS

a 2 Agonists (Brimonidine, Apraclonidine) - decrease aqueous humor produc­


tion and increase uveoscleral outflow (30).
BRIM ONIDINE (Alphagan 0.20%®)
Highly selective ck2 agonist (30X more selective than apraclonidine), allowing
effective IOP lowering and long-term treatment of glaucoma (2).
• Alphagan® has shown neuroprotective properties in a crushed-rat
nerve model (14).
• In the past, around 30% of patients had to discontinue the drug because
of follicular conjunctivitis (3); however, Alphagan-P® (0.10% and
0.15%) has purite as the preservative, which is believed to decrease the
incidence of allergic reactions.
• When Alphagan® is the only glaucoma medication being prescribed, TID
dosing is recommended because of its relatively short duration of action.
• Brimonidine causes miosis and can be used to reduce glare, halos, and
other night vision symptoms for patients suffering from postrefractive
(LASIK, PRK) complaints.

Brimonidine and Apraclonidine can cause systemic side effects,


including dry mouth (most commonly). Brimonidine is contraindi­
cated in patients taking monoamine oxidase inhibitors (2).

APRACLONIDINE (Iopidine®)
Primarily an a 2 agonist with limited ai activity. Commonly used to control
IOP spikes before and after ocular surgery, including laser iridotomy, trabecu­
loplasty, and posterior capsulotomy. Also used during an acute angle closure
attack to provide a rapid, potent decrease in intraocular pressure. Clinical facts
include:
• 30%-40% IOP reduction (3).
• Onset within 1 hour, peak effect within 3 to 5 hours (3).
• Not effective in chronic therapy - within 8 days of continuous treatment,
efficacy is reduced (tachyphylaxis) (2). Long-term therapy (over 1 year)
has a high risk for an allergic response (approx 50%).
• Can be used for diagnosis of Horner’s syndrome.
Copyright 2014 by KMK Educational Services, LLC
CHAPTER 11. OCULAR PHARMACOLOGY 481

H orner’s syndrome
Recall that the anisocoria present in Horner’s syndrome will be greater in the
dark. When evaluating these patients, turn off the lights and observe the miotic
pupil; a delayed dilation (most apparent within the first 5 seconds) will exist
due to abnormal sympathetic innervation to the dilator muscle; if this “dilation
lag” exists, along with a ptosis, Horner’s syndrome can be diagnosed without
pharmacological testing (33).
The following is an overview of the traditional approach that focuses on phar­
maceutical agents:
1 Cocaine or apraclonidine is instilled. Since apraclonidine is easier to
obtain, it is often utilized instead of cocaine.
• Apraclonidine, when instilled in a healthy eye, has no effect on
the pupil (32). In Horner’s syndrome, it causes the miotic pupil
to dilate. Research postulates that Homer’s syndrome leads to a
denervation hypersensitivity of a l receptors; the weak ctl activity
of apraclonidine is enough to cause dilation in these patients.
• Cocaine, when instilled in a healthy eye, always causes dilation. In
Horner’s syndrome, instillation of cocaine has no effect on the miotic
pupil (regardless of the sympathetic pathway lesion location).
2 Hydroxyamphetamine is instilled.
• In healthy eyes, or those with preganglionic damage, hydroxyam­
phetamine acts on the postganglionic neuron to release norepinephrine
and mydriasis will occur (3).
• If the patient fails to dilate upon instillation, the postganglionic
neuron is presumably damaged.

Phenylephrine 1% may have value for diagnosis of lesion location


in Horner’s syndrome. The mechanism does not appear to be com­
pletely understood, but research suggests that phenylephrine 1%
gives full dilation in patients with postganglionic damage because
adrenergic receptors are super sensitized (8).

Beta-Adrenergic Blocking Agents (/5-Blockers)


Introduced topically in the late 1970s and have remained very effective in treat­
ing glaucoma. Timolol is the most common /3-Blocker still used in clinical
practice; others include levobunolol, betaxolol, metipranolol, and carteoiol.
Copyright 2014 by I<MK Educational Services, LLC
482 11.2. AUTONOMIC DRUGS
When prescribed topically, the drugs can be absorbed into the systemic circu­
lation and can cause the following effects:
SYSTEM EFFECT
CNS disorientation, depression, fatigue
Cardiovascular bradycardia, arrhythmias, syncope
Pulmonary dyspnea, wheezing, bronchospasm
Digestive (GI) nausea, vomiting, diarrhea, abdominal pain
Reproductive erectile dysfunction
Topical /3-Blockers are contraindicated with certain respiratory (asthma, COPD)
and cardiovascular disorders (bradycardia, congestive heart failure) (2). In
order to minimize some of the adverse effects, topical /?-Blockers have been
formulated with specificity toward /31 (minimizing lung side effects) and /32
(minimizing heart side effects) receptors.

• MOA: Block /3-adrenergic receptors throughout the body. Topically, they


act primarily on /3-receptors (mainly (12) in the non-pigmented ciliary
epithelium to decrease aqueous production.

Betaxolol is the only topical (31 selective drug.

TIMOLOL (Timoptic®)
Non-selective /?-Blocker, 1st introduced /3-Blocker in 1978 and continues to be
the most effective at lowering IOP (around 25% reduction) (2). Also available
in gel-form (Timoptic XE@).
• Maximum efficacy is similar between 0.25% and 0.50% solution concen­
trations.
• Often dosed BID, but a once-daily regimen is effective (2nd drop does
not provide much additional effect) (2).
• If dosed daily, morning dosage is recommended (has better daytime
efficacy).
• Similar to other /3-Blockers, unilateral use of Timolol commonly reduces
IOP in the contralateral eye (28); this is often referred to as a crossover
effect.
• Similar to other /3-Blockers, extended use of Timolol often results in long­
term drift (IOP starts to gradually rise) or short-term escape (IOP
initially lowers but returns to normal within weeks after starting therapy).
Copyright 2014 by KMK Educational Services, LLC
CHAPTER 11. OCULAR PHARMACOLOGY 483

• Timolol and other /5-Blockers should be used cautiously in diabetics;


/3-Bloekers mask symptoms of hypoglycemia.
• /3-Blockers also mask signs and symptoms of hyperthyroidism.
• Symptoms of weakness in patients with myasthenia gravis can be ex­
acerbated with /3-Blockers.

Combination therapies for glaucoma include COSOPT® (0.5%


Timolol and 2% Dorzolamide) and COM BIGAN® (0.5% Tim­
olol and 0.2% Brimonidine). Both of these medications are dosed
q!2hrs.

CARTEOLOL (Ocupress®)
Non-selective /3-Blocker. Not used much clinically because it does not lower
IOP as well as the other /3-Blockers.
• Has intrinsic sympathomimetic activity (ISA); carteolol has been shown
to significantly reduce nocturnal bradycardia (compared to timolol), and
is generally more comfortable (less stinging) than timolol (2).
• Has been proven to provide a modest reduction in cholesterol in patients
with hypercholesterolemia (10).
• In summary, note that carteolol potentially has less side effects than other
/3-Blockers.
BETAXOLOL (Betoptic-S®)
“Cardioselective” 81 specific /3-Blocker supplied as a 0.25% suspension.
• Limited B2 activity, so it minimizes the risk of respiratory effects; how­
ever, there are reported cases of respiratory adverse effects with betaxolol
in patients with COPD and asthma.
• Questionable neuroprotective qualities, BUT overall not as effective as
Timolol at lowering IOP.
• j3l activity of Betaxolol can worsen congestive heart failure (2).
LEVOBUNOLOL (Betagan®)
Non-selective B blocker / Similar to Timolol in effectiveness.
Copyright 2014 by KMK Educational Services, LLC
484 11.3. GLA UCOMA DR UGS
METIPRANOLOL (Optipranolo 1®)
Non-selective /5-Blocker. Not used anymore because was not nearly as effective
as Timolol.
- SECTION 11.3 -------------------------------------------------------------------------------------------- -

G la u c o m a D r u g s

Cholinergic Agonists: t Corneoscleral Outflow


Alpha-adrenergic agonists: 4 Production and t Uveoscleral Outflow
Beta-Blockers: 4- Production
Carbonic anhydrase inhibitors: 4 Production
Prostaglandins: t Outflow via the Uveoscleral route

||; j

Carbonic anhydrase is an enzyme that acts in the ciliary body epithelium (non-
pigmented and pigmented) to catalyze the joining of CO2 4- H 2O to yield bi­
carbonate.
• Bicarbonate ions are believed to increase aqueous production by in­
creasing Cl- and Na+ flux into the posterior chamber (24) (18).
• These drugs are sulfa-based; they should not be used in patients with
sulfa allergies.
• Topicals: BRINZOLAMIDE 1% (Azopt®) and DORZOLAMIDE 2%
(Trusopt©): these drugs are rarely a primary medication, but are effec­
tive in combination (e.g, Cosopt®).
• Orals: ACETAZOLAMIDE (Diamox®), METHAZOLAMIDE (Neptazane®).
- Acetazolamide is commonly given with liquid during acute angle
closure attacks.
- It is quickly absorbed into the GI tract shortly after oral admin­
istration and provides a potent decrease in IOP. Can be used in
treating primary open angle glaucoma, but is employed as a last
resort because of its side effects.
Copyright 2014 by KMK Educational Services, LLC
CHAPTER 11. OCULAR PHARMACOLOGY 485

Oral CAIs are notorious for systemic side effects. Short-term use, if
possible, is always advised. The following is a summary of noteworthy
adverse effects and contraindications:
- Common adverse effects - metallic taste, tingling in hands and
feet, and metabolic acidosis.
- Most serious adverse effects - thrombocytopenia, agranulocyto­
sis, and.aplastic anemia.
- Other adverse effects - malaise, fatigue, weight loss, anorexia,
impotence, depression, diarrhea, and myopic shifts in refractive er­
ror.
- Contraindications - severe chronic obstructive pulmonary disease
(COPD), pregnancy, sulfa allergy; strongly consider other options
or use with caution in patients with liver disease and renal disease.

Fatal side effects of oral CAIs include bone marrow suppression


and aplastic anemia.

Currently the first line of treatment for POAG. First introduced in 1996 with
Latanoprost (Xalatan 0.005%®) (31); Bimatoprost (Lumigan 0.03%®) and
Travoprost (Travatan 0.004%®) were released in 2001.
• On average, the three drugs provide a 27-35% lowering of IOP from base­
line - the highest among glaucoma drugs. Travatan Z® is a new formu­
lation of Travoprost with Sofzia as the preservative (instead of BAK).
• MOA: Prostaglandin analogs act on FP receptors (PGF2a receptors) on
the ciliary muscle, which causes reduction of neighboring collagen (via
metalloproteinases), decreasing resistance within the uveoscleral mesh-
work for increased outflow. Also acts on skin receptors (activating phos­
pholipase C) to alter hair follicles, contributing to the side effects listed
below (3).
• Bedtime dosing is recommended, as it allows for better diurnal control
than morning instillation; has a daytime peak effect (12-24 hours after
administration) (2).
• Contraindications: Patients who are at risk for CME (especially cataract
surgery post-op), cases of active inflammation (e.g. uveitis), and pa­
tients with previous episodes of herpes simplex keratitis (2).
Copyright 2014 by KMK Educational Services, LLC
486 11.4- PROPERTIES OF TOPICAL OCULAR ANESTHETICS
• Side Effects: Iris heterochromia (often permanent, even after discontin­
uation of the drug), increased pigmentation and growth (length, thick­
ness, and number) of the eyelashes, and skin darkening around the eyes
(most common in mixed-color irises).
— Conjunctival hyperemia can occur with all three drugs but is worse
with Lumigan® and least common with Xalatan® (31).
- Pruritis is also noted more frequently with Lumigan® (2).

Typical | in IOP (7):


• Prostaglandins: 33% / /5-Blockers: 25%
• Brimonidine: 18% AND Dor zolamide: 18%
• These are all commonly used in glaucoma treatment.

- S E C T I O N 11.4 --------------------------------------------------------------------------------------------

Properties of Topical Ocular A nesthetics


MO A: Local anesthetics block nerve conduction and change membrane perme­
ability by stopping the influx of Na-L ions into the nerve cytoplasm.
Without Na+ entry, the nerve can no longer be depolarized.

Injected anesthetics are given with epinephrine so that local blood


vessels are constricted and systemic absorption is minimal. This
keeps the drug localized, allowing more potent effects.

Their structure consists of three parts (3):


1 Aromatic residue (provides lipophilic properties)
2 Intermediate chain (t the length, t the potency)
3 Amino group (provides hydrophilic properties)
The bond between the chain and the amino group (usually tertiary amine) is
called an ester or amide linkage and is how anesthetics are categorized.
Copyright 2014 by KMK Educational Services, LLC
CHAPTER 11. OCULAR PHARMACOLOGY 487

AMIDES: Longer duration of action / Metabolized by the liver so are less


toxic.
ESTERS: Shorter duration of action / Metabolized locally / All topical
anesthetics are esters!

PROPARACAINE (Ophthaine®) / BENOXINATE: Esters / Onset


10-20 sec / Duration 10-20 min

Pluoress® solution is a combination of fluorescein and benoxi-


nate. Proparacaine is a commonly used topical anesthetic in clini­
cal practice. Cocaine was the first topical anesthetic developed.

I — SECTION 11.5
Antihistam ines
MO A: Block Type 1 hypersensitivity reactions. Antihistamines do not pre­
vent the release of histamine from mast cells and basophils - they block
the cell receptors that histamine acts upon. This minimizes the red,
watery, itchy effects of allergies.

Summary of Type 1 Hypersensitivity Reactions:


• Upon first exposure to particular antigens, IgE antibodies are produced
but no symptoms occur.
• Between exposures, IgE antibodies bind to mast cells and basophils.
• When the antigen is reintroduced, it binds to the IgE / mast cell complex
resulting in the opening of calcium channels.
• Calcium influx depolarizes the cell, resulting in degranulation of mast
cells, causing the release of histamine and other inflammatory mediators
into the bloodstream.
• The binding of histamine to histamine receptors results in allergic symp­
toms.
Copyright 20.1.4 by KMK Educational Services, LLC
488 11.6, ANTI-INFLAMMA TOE, Y A GENTS

Emedastine (Emadine®)
Employed in mild to moderate cases of allergic conjunctivitis. In more seri­
ous cases of inflammation and irritation, corticosteroids are the recommended
therapy (3). These solutions can be used in isolation, but are more commonly
prescribed clinically in combination with a vasoconstrictor agent (example:
Naphcon A®). H2 antihistamines arc of no benefit topically.

rAi-r.V"'-A- A : A-VT a 1A. A .£ ’1A A d A-h: A,V A ojjiV u'A.fA A , A

Cromolyn sodium (Crolom ®), Lodoxamide (Alomide®),


Pemirolast (A lam ast® ), Nedocromil (Alocril®)
Not effective in acute allergic symptoms. Acts on exposed mast cells and in­
hibits their degranulation upon re-exposure to the antigen. Stabilizes mast cell
membranes, preventing Ca2+ influx and degranulation. Effects begin days
to weeks after beginning therapy. Can be used for chronic allergic conjunctivi­
tis, vernal conjunctivitis, and atopic keratoconjunctivitis (3) (12).
ftiast cell - A ntihistam ine Gohibinations
Bepotastine (Bepreve®), Epinastine (Elestat® ), Ketotifen
(Zaditor®), Olopatadine 0.10% (Patanol® ), Azelastine
(Optivar®), Olopatadine 0.20% (Pataday® )
The dual mechanism of action for these drugs allows effectiveness in long-term
management of ocular itching and allergic conjunctivitis, as well as relief of
acute symptoms.
- SECTION 11.6 ------------------------------------------------------------------

A nti-Inflam m atory Agents

P S W iB Ilsi 7~-' ‘A t Jtc A-:

During “fight or flight” conditions, ACTH is released and acts on the adrenal
cortex to secrete cortisol, the body’s endogenous glucocorticoid.
• Overall Actions: Anti-inflammatory and immunosuppressive. Inhibits
phospholipase A2 and thus the arachidonic acid pathway.
* 4 Inflammatory mediators and 4- capillary permeability - this causes a
significant j, in the immune system response.
Copyright 2014 by KMK Educational Services, LLC
CHAPTER 1L OCULAR PHARMACOLOGY 489

• \ Fibroblast and collagen formation - this prevents healing.


Side effects: The literature lists many ocular side effects. The most clinically
relevant are increased risk of secondary infections, PSC cataracts, and
glaucoma.

• Potent steroids: Prednisolone 1% Acetate, Rimexolone


(Vexol®), Difluprednate (Durezol®), and Dexamethasone
(Maxidex®) 0.1%.
• “Soft steroids:” Fluorometholone (FML®) 0.1% and
Loteprednol (Lotemax®) 0.5%.
— Soft steroids and are less likely to cause a spike in IOP
(steroid response) during treatment.
— 5% of the general population are high steroid responders;
90% of patients with established PO AG are high steroid
responders (28).

Diclofenac sodium 0.1% (Voltaren®), Ketorolac Tromethamine 0.4%


(Acular LS® ), Nepafenac 0.1% (Nevanec®), Bromfenac 0.09%
(Xibrom®, Bromday®), Flurbiprofen 0.03% (Ocufen®)
MOA: NSAIDs block cyclooxygenase I and II, which decreases inflamma­
tion by inhibiting the conversion of arachidonic acid into prostaglandins and
thromboxanes.
• Voltaren® and Acular LS® are dosed qid, Nevanec® is dosed tid,
and Xibrom ® is dosed bid. Brom day® is the only topical NSAID
approved for qd dosage. Ocufen® is utilized prior to ocular surgery (1
drop every 30 minutes, beginning 2 hours prior to surgery).
• Clinical Uses: Post-op cataract patients - decreases the risk of post­
operative inflammation, particularly that of the macula (cystoid mac­
ular edema), recurrent corneal erosions, corneal abrasions, and allergic
conjunctivitis (Non-exhaustive list).

Copyright 2014 by KMK Educational Services, LLC


490 11.6. ANTI-INFLAMMATORY A GENTS

Figure 11.1: Arachidonic Acid/ Prostaglandin Synthesis Pathway

Ketorolac is the only NSAID approved for topical treatment of


seasonal allergic conjunctivitis; an alternative option (besides an
NSAID) should be considered if the patient has any corneal in­
volvement.

Noteworthy ocular side effects include:


• Corneal toxicity - not a consistent occurrence; corneal melting can
occur but was most problematic in a generic form of Voltaren®, which
was subsequently withdrawn from the market (3).

Corneal melting can result from repeated use of topical anes­


thetics and generic Voltaren®.

• Stinging upon instillation - transient, most common with ketorolac.

Copyright 2014 by KMK Educational Services, LLC


CHAPTER 11. OCULAR PHARMACOLOGY 491

Xibrom® contains the preservatives BAK and sodium sulfite;


therefore, beware of prescribing in a patient with a sulfa allergy.
Ocufen® contains thimerosal as one of its preservatives.

I - SECTIO N 11.7

Dyes

1
Fluorescein: Water-soluble compound that is quickly dissolved in the aque­
ous portion of the tears, allowing effective evaluation of tear film quality and
epithelial defects.
Rose Bengal: Stains dead and devitalized cells as well as cells that have
lost their mucous surface (28). It does not enter epithelial defects like fluores­
cein (29). Can be helpful in evaluating herpetic corneal ulcers because it stains
the edges of the dendritic lesion.

Lissamine green and Rose Bengal both have mild anti-viral


properties, whereas Methylene blue is bacteriostatic (2). If cell
culturing is indicated, do not instill these topical diagnostic agents
prior to culture if the organism you are suspecting (e.g. virus,
bacteria) is altered by the agent.

Lissamine Green: Similar staining patterns as Bose Bengal but causes less
discomfort and ocular irritation; therefore, lissamine green is more commonly
utilized for dry eye evaluations (5) (9) (19).
M ethylene blue: Staining properties similar to Rose Bengal, but also stains
corneal nerves. Used to outline glaucoma filtering blebs and for staining the
lacrimal sac before dacryocystorhinostomy (2).

Fluorescein Dye
Fluoresces, or emits longer wavelength light, when exposed to visible spectrum
light (29). During fluorescein angiography, approximately 5 cc are injected into
Copyright 2014 by KMK Educational Services, LLC
492 11.8. A GENTS FOR EXUDA TIVE ARMD
the brachial vein and photographs are taken of the fundus as the dye enters
the choroidal and retinal vessels. It takes approximately 10-20 seconds for this
to occur.
The procedure is commonly used in macular degeneration to determine if a
choroidal neovascular membrane (CNVM) is present and treatable. Also com­
monly employed in diabetes to determine if macular thickening is from hypoxia
or edema (non-exhaustive list).
- SECTION 11.8 -----------------------------------------------------------------------------------------------------
Agents for Exudative ARM D
This review focuses only on agents for exudative ARMD that are listed on the
NBEO outline.
Pegaptanib (M acugen®)
Antineoplastic agent that decreases angiogenesis by binding to and inhibit­
ing the actions of vascular endothelial growth factor (VEGF). Macugen® is
administered as an intravitreal injection.
Ranibizumab (Lucentis®)
Monoclonal antibody (Fab portion) engineered from mouse antibodies that
targets VEGF (2). Lucentis® is administered as an intravitreal injection.

VEGF is a group of proteins that promote vascular permeability


and new blood vessel formation.

r - SECTION 11.9
H yperosm otic Agents
Glycerine (Osmoglyn®): High molecular weight, water-soluble compound
that is unable to cross the blood aqueous barrier; this creates an osmotic gra­
dient in which the plasma in the ciliary stroma region is hypertonic to the
aqueous humor, lowering IOP (6).
Glycerine is used to lower fluid volume during an acute angle closure attack.
About 4-6 ounces are mixed with a soft drink - on crushed ice - and should be
sipped by the patient to avoid vomiting.
Copyright 2014 by KMK Educational Services, LLC
CHAPTER 11. OCULAR PHARMACOLOGY 493

Glycerine is rapidly absorbed after oral administration and is metabolized into


carbohydrates, increasing blood sugar levels. For this reason, glycerine is not
recommended in diabetics; isosorbide solution should be utilized instead.
Sodium Chloride (Muro 128®): Hypertonic solution that is prescribed
for reduction of corneal edema. Available as eye drops (2% and 5% solutions)
or in ointment form (5%).
- SECTION 11.10 -------- ------------------------------------------------------------------------------------------

Tear Substitutes
Synthetic water-based solutions that are used to lubricate the eye and replace
the aqueous portion of the tears. There are several different types of sub­
stances used in tears to enhance lubrication, including: cellulose esters - (car-
boxymethylcellulose and hydroxymethylcellulose) and polyvinyl alcohol (PVA).
• Examples include Optive®, Systane®, and Refresh Tears®. Artificial
tears can be administered as the patient requires them, commonly 3-4
times/day.

One of the main disadvantages of artificial tears is that 90% of the


drop volume is eliminated from the eye within the first minute or
two after application. Methylcellulose increases the viscosity of
solutions, allowing more contact time on the cornea. Polyvinyl al­
cohol is also commonly incorporated into tear solutions, although
it is less viscous than methylcellulose (6).

Ointments: High viscosity solutions that provide longer duration of action on


the eye with minimal discomfort upon instillation.
• Ointments obstruct vision so they are commonly prescribed for bedtime
use.
• Increased contact time results in an increased risk for secondary infections
and reactions.
• Ophthalmic ointments are especially helpful in children, because they are
less likely to be diluted and washed out by the tears.
— Examples: Celluvisc®, Lacrilube®.

Copyright 2014 by KMK Educational Services, LLC


494 11.11. PREPARATIONS USED WITH CONTACT LENSES

Restasis® (cyclosporine 0,05%) inhibits T-cell activation by stop­


ping the production of interleukin-2.

i - SECTION 11.11

Preparations Used with Contact Lenses

Prevent and kill bacteria, viruses, and other contaminates in cases where they
enter the solution (29). Common preservatives include the following:
• Benzalkonium chloride (BAK) - very common, well-known to cause corneal
toxicity (increases drug penetration).
• Thimerosal - rarely used anymore, sensitivity to the mercury component
caused noncompliance. Thimerosal is the preservative used in TVifluri-
dine. Prolonged use of the drug (3 weeks or more) is not recommended
because of the toxicity caused by thimerosal.
• Ethylenediaminetetraacetic acid (EDTA) - chelating agent (binds and
inactivates) that most commonly sequesters calcium.
• Purite / Sodium perborate - oxidative preservatives found in Refresh
Tears® and GenTeal®, respectively. Favored over traditional chemical
preservatives because they are effective and with less toxicity (2).

Diffuse SPK and follicles are common if a patient has a toxic


reaction to a preservative.

r - SECTION 11.12

Toxicology
We now introduce side effects of systemic and topical drugs, as they pertain to
the eye (3, ch. 29,30).
Copyright 2014 by KMK Educational Services, LLC
CHAPTER 11. OCULAR PHARMACOLOGY 495

Many systemic drugs can affect the eye - it is important to consider the amount,
route; and frequency of the drug, as well as the age, sex, and past history of
the patient. This review focuses only on the most prominent examples. The
best resource for this section was the textbook Clinical Ocular Pharmacology
by Bartlett and Jaanus.
Drugs affecting the Cornea
Several different effects can occur from drug toxicity on the cornea - the most
common include (2, pp. 704):
• Whorl keratopathy: Chloroquine, Hydroxychloroquine, Amiodarone, Ta­
moxifen, Indomethacin.

Fabry’s disease is a lysosomal storage disease that can result in


cornea verticillata (“whorl keratopathy”) and spoke-like lens opac­
ities (28) (16).

• Superficial punctate keratitis (SPK): Isotretinoin (Accutane®), topical


aminoglycosides.
• Endothelial/Descemet’s pigmentation: Chlorpromazine, Thioridazine.
• Stromal gold deposits: Gold salts - visually asymptomatic yellow-brown
deposits located on the posterior stroma and Descemet’s membrane; re­
ferred to as ocular chrysiasis (15).
• Delayed healing: Corticosteroids.

Amiodarone is an antiarrhythmic drug with the following note­


worthy ocular side effects (2):
1 “W horl keratopathy”: visually insignificant toxicity of the
epithelium; can cause complaints of glare and photophobia.
2 Anterior sub capsular lens deposits
3 Non-arteritic ischemic optic neuropathy - 1-2% of
cases (16).

Copyright 2014 by KMK Educational Services, LLC


496 11.12. TOXICOLOGY
Drugs affecting the Lens
The most common effects from drug toxicity on the lens include (2):
• Anterior subcapsular effects: Chlorpromazine, Thioridazine, Amiodarone
(deposits), and Miotics (vacuoles).
• Posterior subcapsular cataracts: Corticosteroids.

PSC cataract formation from steroids is dose dependent and irre­


versible. Hispanics appear to be at highest risk (2).

Drugs affecting the Conjunctiva and Lids


The most common drugs that cause toxicity of the conjunctiva and eyelids
include (2, pp. 712):
• Isotretinoin (Accutane®): Blepharoconjunctivitis, dryness, lid edema.
• NSAIDS: Subconjunctival hemorrhage.
• Sulfonamides: Ocular findings rare, but include Stevens-Johnson syn­
drome and lid edema .
• Tetracyclines: Pigmented cysts on the conjunctiva.
• Sildenafil (Viagra®): Subconjunctival hemorrhage, conjunctival hyper­
emia.
• Prostaglandin analogs: Conjunctival hyperemia, increased growth and
pigmentation of the eyelashes, increased pigmentation of the periorbital
skin.
Drugs affecting the Lacrimal System
Recall that drugs that affect the autonomic nervous system can alter tear pro­
duction. Some of the drug classes that decrease tear secretion include (2,
pp.7'14) (16) (29):
• Drugs with anticholinergic effects:
—Anticholinergics: Atropine, scopolamine.
- Tricyclic antidepressants (TCAs): amitriptyline (Elavil®), imipramine
(Tofranil®).
Copyright 2014 by KMK Educational Services, LLC
CHAPTER 11. OCULAR PHARMACOLOGY 497

- Antihistamines: Chlorpheniramine (Chlor-Trimeton®), bropheni-


ramine (Dimetane®), diphenhydramine (Benadryl®), promethazine
(Phenergan®).
- Phenothiazines: Chlorpromazine (Thorazine®), thioridazine (Mellaril®).
• Isotretinoin (Accutane®).
• /7-Blockers: Most commonly caused by timolol, atenolol, propranolol.
• Hormone therapies: Oral contraceptives, hormone replacement therapy.
• ADHD medications: Methylphenidate (Ritalin®), dextroamphetamine
(Dexedrine®).
• Diuretics: Hydrochlorothiazide (Hydrodiuril®).
Drugs affecting Pupil Size
Drugs that can cause mydriasis (2, pp. 718):
• Drugs with anticholinergic effects:
- Anticholinergics: Atropine, scopolamine.
- Tricyclic antidepressants (TCAs): amitriptyline (Elavil®), imipramine
(Tofranil®),
- Antihistamines: Chlorpheniramine (Chlor-Trimeton®), bropheni-
ramine (Dimetane®), diphenhydramine (Benadryl®), promethazine
(Phenergan®).
- Phenothiazines: Chlorpromazine (Thorazine®), thioridazine (Mellaril®),
- Benzodiazepine: Diazepam (Valium®).
• Dopamine agonists: methylphenidate (Ritalin®), dextroamphetamine
(Dexedrine®), amantadine (Symmetrol®), bromocriptine (Parlodel®).

Drugs with mydriasis side effects can contribute to an angle-


closure event in patients with narrow anterior chamber angles.

Drugs that can cause miosis (2, pp.718);


• Opiates (Morphine, Heroine, Codeine).
* Acetylcholinesterase inhibitors.
Copyright 2014 by KMK Educational Services, LLC
498 11.12, TOXICOLOGY
Drugs affecting Extraocular Muscles and Eye Movements
Below is a summary of drug-induced extraocular muscle abnormalities (2,
pp. 719):
• Nystagmus: Phenytoin (Dilantin®), phenobarbital (Luminal®), salicy­
lates (NSAIDS).
• Diplopia: Antidepressants, antianxiety agents, phenytoin (Dilantin®).
• Smooth pursuit impairment: Alcohol.
• Oculogyric crisis: Phenothiazines, cetirizine (Zyrtec®),

Oculogyric crisis occurs when the extraocular muscles undergo


spastic, abnormal muscle contractions that leave the eye abnor­
mally positioned (typically elevated). This condition occurs most
commonly with phenothiazine toxicity, but can also occur with ce­
tirizine (2).

Drugs affecting the Sclera and Uvea


Below is a summary of the most common effects from drug toxicity on the
sclera and uvea (2, pp. 716):
• Tamsulosin (Flomax®): Intraoperative floppy iris syndrome.
• Blue sclera: Corticosteroids, minocycline.
Drugs affecting the Optic Nerve
The following drugs can affect the optic nerve (2, pp.736):
• Digoxin: Retrobulbar optic neuritis, B/Y color defects, entopic phe­
nomenon (“snowy” vision, dimming vision, flickering lights).
• Ethambutob Optic neuritis - typically retrobulbar and bilateral (1) (2).
Chloramphenicol, streptomycin and sulfonamides are other antibiotics
that can cause optic neuritis (2).
• Isoniazid / Methotrexate: Unlikely culprits of optic neuritis (2).
• Sildenafil (Viagra®), sumatriptan (Imitrex®), amiodarone (Cordarone®):
all causes of NAION.
• Oral contraceptives: Effects are rare but may include optic neuritis, pa­
pilledema, and pscudotumor cerebri.
Copyright 2014 by KMK Educational Services, LLC
CHAPTER 11. OCULAR PHARMACOLOGY 499

Drugs affecting the Retina


The following drugs can cause retinal toxicity (2, pp. 725):
• Chloroquine: Bull’s-eye raaculopathy (much more common than with
hydroxychloroquine).
• Epinephrine: Cystoid macular edema (topical epinephrine in aphakic
eyes).
• Tamoxifen: White or yellow crystalline deposits (commonly in the mac­
ula) with or without macular edema.
• Thioridazine/Chlorpromazine: Pigmentary retinopathy that can have an
appearance similar to Bull’s eye maculopathy.
• Indomethacin: Retinal hemorrhage, pigmentary changes (especially in
thte macula).
• Talc: Retinopathy (white, shiny emboli within arterioles).
• Isotretinoin (Accutane®): Color vision loss, nyctalopia (night blindness).
• NSAIDS: Retinal hemorrhage.
• Oral contraceptives: Vasculopathy including arterial and vein occlusions,
retinal hemorrhage.

Drugs causing Intracranial Hypertension


Tetracyclines (minocycline and doxycycline) and isotretinoin (Accutane®) can
cause pseudotumor cerebri.
Drugs affecting Intraocular Pressure
Drugs that decrease IOP include:
* Systemic /3-Blockers, cardiac glycosides (digoxin), alcohol, cannabinoids.

M arijuana is a cannabinoid derived from the plant Cannabis


sativa. The maximum effect of marijuana on IOP occurs 60-90
minutes after inhalation and lasts approximately 4 hours (2).

Copyright 2014 by KMK Educational Services, LLC


500 11.12. TOXICOLOGY
Drugs that increase IOP include corticosteroids and the following drugs with
anticholinergic activity:
• Atropine and scopolamine.
• Antihistamines: bropheniramine (Dimetane©), diphenhydramine (Benadryl©).
• Tricyclic antidepressants: amitriptyline (Elavil®), imipramine (Tofranil®).
• Antipsychotic agents: phenothiazines - Chlorpromazine (Thorazine®),
thioridazine (Mellaril®).

Corticosteroids can increase IOP by decreasing aqueous humor


outflow. Several mechanisms are believed to occur, including: de­
creased ability for TM cells to replace matrix and phagocytose
debris, and a thickening of trabecular fibrils and juxtacanalicular
tissue (2).

References
[1] Aftab Zafar, MD. Toxic/Nutritional Optic Neuropathy, eMedicine. Aug 5, 2005.
[2] Bartlett, Jimmy D., Jaanus, Siret D, Clinical Ocular Pharmacology. Boston: Butterworth,
2008.
[3] Bartlett, Jimmy D,, Jaanus, Siret D. Clinical Ocular Pharmacology. Boston: Butterworth,
1984.
[4] Bhushan, Vikas, Le, Tao, Amin, Chirag. First Aid for the USMLE Step 1. New York:
McGraw-Hill, 2003.
[5] Chodosh J, Dix R, Howell RC, Stoop WG, Tseng SCG. Staining characteristics and antiviral
activities of sulfonhodamine B and Lissamine green. Invest Ophthalmol Vis Sci. 1994;35:104G-
58.
[6] Ellis, Philip, Ocular Therapeutics and Pharmacology, 5th ed. St, Louis: Mosby, 1977.
[7] Epstein, David L. Chandler and Grant’s Glaucoma, 4th ed. Baltimore: Williams and Wilkins,
1997.
[8] Falzon, K. Denervation supersensitivity to 1 percent phenylephrine in Horner syndrome can
be demonstrated 10 days after the onset of symptoms. Br J Ophthalmol 2009;93:130, Volume
93, Issue 1.
[9] Feenstra RBG, Tseng SCG. Comparison of fluorescein and rose bengal staining. Ophthal­
mology. 1992;110:984-93.
[10] Freedman, Sharon F. and all. “Effects of Ocular Carteolol and Timolol on plasma high density
lipoprotein cholesterol levels.” American Journal of Ophthalmology, Nov, 1993; 116; 600-611.

Copyright 2014 by KMK Educational Services, LLC


CHAPTER 11. OCULAR PHARMACOLOGY 501

[11] Green, Gopa B. Ed et. al, The Washington Manual of Medical Therapeutics, 31st ed, Philadel­
phia: Lippincott Williams and Wilkins, 2004,
[12] Harkins, Timothy, “Drug Therapy for Ocular Allergy,” Optometry Clinics Vol. 2, Number 4,
Ocular Pharmacology Update, Eds. John G, Glasse. Norwalk: Appleton and Lange, 1994,
[13] Havener WH. Ocular Pharmacology. St Louis: Mosby, 1978; Ch. 12
[14] Hernandez M, Urcola JH, Vecino E, “Retinal ganglion cell neuroprotection in a rat model of
glaucoma following brimonidine, latanoprost or combined treatments.” Exp Eye Res, 2008
May;86(5):798-806. Epub 2008 Mar 4.
[15] Holland, E., Cornea, 2nd edition, volume 1, Fundamentals, Diagnosis and Management
(2005), pp 1335-1340.
[16] Kanski, Jack. Clinical Ophthalmology 4th ed. Woburn: Butter worth and Heinmann, 1999.
[17] Katzung, Bertram G, and Anthony J, Trevor. Examination and Board Review Pharmacology,
4th ed. Norwalk: Appleton and Lange, 1993.
[18] Kaufman, P. Aim, A. Adler’s Physiology of the Eye, 10th ed. St, Louis: Mosby, 2003.
[19] Kim J, Foulks G. Evaluation of the effect of Lissamine green and Rose Bengal on human
corneal epithelial cells. Cornea. 1999;183:328-32.
[20] Lawrenson JG, C Kelly, A L Lawrenson, J Birch. Acquired colour vision deficiency in patients
receiving digoxin maintenance therapy, British Journal of Ophthalmology 2002;86:1259-1261
[21] Moorthy, Ramana S., Valluri, Shailaja. Ocular toxicity associated with systemic drug ther­
apy. Current Opinion in Ophthalmology 1999, 10: 438-446.
[22] Quinn A, et.al. Pediatric tetracycline-induced pseudotumor cerebri. Journal of AAPOS. Vol­
ume 3, Issue 1, Pages 53-57 (February 1999).
[23] Quinn C. A Field Guide to Glaucoma Drugs, Review of Optometry, 2003 July.
[24] Remington, Lee Ann. Clinical Anatomy and Physiology of the Visual System, 3rd Ed. Boston:
Butter worth-Heinemann, 1988.
[25] Roth N. Refractive state after instillation of paredrine and neosynephrine. Br J Ophthalmol
1968; 52: 763-767.
[26] Sigelman J, The clinical diagnosis of retinal drug toxicity. In: Srinivasan BD, ed. Ocular
therapeutics. New York: Masson, 1980; Ch, 17
[27] Sugar SH. Pitfalls in the medical treatment of simple glaucoma. Ann Ophthalmol 1979;
11:1041-1050.
[28] Tamesis, Richard R. Ophthalmology Board Review., 2nd Edition, McGraw-Hill, 2006.
[29] Terry, Jack. Ocular Disease - detection, diagnosis, and treatment. Springfield: Thomas, 1984.
[30] The Eye Digest, University of Illinois Eye and Ear Infirmary, Chicago, IL. 2003. Aging Eye
and Ear Times.
[31] The Glaucoma Handbook, Optometric Glaucoma Society. Review of Optometry, August
2008.
[32] Morales J, Brown SM, Abdul-Rahim AS, Crosson CE. Ocular effects of apraclonidine in
Horner syndrome. Arch Ophthalmol. 2000;118951- 954
[33] Woods, Albert D. Neuro-Eye Rounds and Clinical Updates. Lecture given at Nebraska Op­
tometric Association March 28th, 2010.

Copyright 2014 by KMK Educational Services, LLG


((
c
(
(f
c
( i

((
r( . '

(f :
((
r
( I

c(
(
c(
((
((
((
((
((
(( I

((
((
((
(
(
(.
(( .,

(( , '

(( ,
((
Index

4 base out test, 351 metabolic, 17


respiratory, 17
Abney’s law of additivity, 378 acquired brain injury
AC/A ratio, 321, 341 symptoms, 360
calculated, 321 acquired brain injury, ABI, 359
gradient, 321 acromegaly, 47
accommodation, 95 actos, 451
comfortable use, 277 acuity
optics of, 277 Allen vision test, 268
accommodative amplitude, 327, birthday cake slide, 267
342 HOTV, 268
accommodative excess, 339, 341, Landolt C chart, 268
342 Lea symbols, 268
accommodative facility, 327, 329, Sheridan Gardiner, 268
342 STYCAR, 268
accommodative infacility, 340- tumbling E chart, 268
342 acular, 489
accommodative insufficiency, 341, acute angle closure, 105
342 acyclovir, 432
accommodative paralysis, 339 adjustment method, 370
accommodative response, 327 adrenal gland, 55
accomodative insufficiency, 339 adrenergic agonists, 478
amplitude of, 277 adrenergic antagonists
disorders of, 327 systemic drugs, 464
ill-sustained accommodation, 339, topical ophthalmic summary of,
341, 342 481
accommodative infacility, 338 adrenergic receptors
accutane, 460 mechanism of action of, 5
adverse effects of, 495, 496, 499 advil, 438
acetaminophen, 445 after image test, 353
acetylcysteine, 443 alamast, 488
acidosis aldosterone, 28, 30, 31, 39, 56
503
504 INDEX
alkalosis anticholinergics
metabolic, 17 adverse effects of, 496
respiratory, 17 anticoagulants, 458
allegra, 441 antidepressants
alocril, 488 adverse effects of, 496
alomide, 488 antidiuretic hormone (ADH), 28, 30
alphagan, 480 antihistamines, 440
alphagan P, see alphagan adverse effects of, 496
amantadine, 447 antihyperlipidernic agents, 459
amblyopia, 340, 345 antihypertensive medications, 453
anisometropic amblyopia, 345 antimetropia, 225
critical period of, 343 aperture stop, 158
isometropic amblyopia, 345 aphakia, 280
meridional amblyopia, 345 aphasia, 363
occlusion amblyopia, 344 apparent depth, 148
refractive amblyopia, 344 apraclonidine, 480
strabismic amblyopia, 345 apraxia, 363
types of, 343 Apresoline, 457
ametropia, 260 aquaporins, 90
clinical tests, 265 aqueous
optics rule for correction, 263 general characteristics of, 134
aminoglycosides, 428 aqueous humor
amiodarone, 458 active secretion, 135
adverse effects of, 495, 496 composition of, 137
adverse effects summary of, 458 diffusion, 135
amitriptyline, 449 formation of, 134
amoxicillin, 425 functions of, 134
amphotericin B, 433 ultrafiltration, 135
anesthetics, 486, 487 aqueous outflow
angiotensin converting enzyme (ACE) controlling factors of, 132
inhibitors, 453 summary of, 133
angiotensin II, 28, 30 aqueous production
receptor antagonists (ARBs), 453 controlling factors of, 132
angle alpha, 351 aqueous rate of flow
angle gamma, 351 influencing factors of, 136
angle kappa, 351 arc of contact, 289
angle lambda, 351 argyll-robertson pupil, 129
angle of anomaly, 347 arteriovenous fistula, 133
angle recession, 136 aspirin, 438
angular magnification, 242 asthma, 11
aniseikonia, 223 astigmatic dial, 274
estimating amount of, 224 astigmatism
anisocoria, 129 optics definition, 161
anisometropia, 224 ATR, 282
anomalous retinal correspondence, compound, 282
347, 349 mixed, 282
Copyright 2014 by KMK Educational Services, LLC
INDEX 505

oblique, 282 blood retinal barrier, 108


simple, 282 botulina toxin (botox), 478
WTR, 282 boxing system, 199
Atenolol, 454 breathing
at orvast atm, 459 mechanics of, 8
atropine, 476 Brewster’s Law, 180
auditory pathway, 115 sunglasses, 180
autonomic nervous system brimonidine, see alphagan
pathways of, 119 Broca Sulzer effect, 414
autonomic pathway bromocriptine, 447
summary of, 472 Brown’s syndrome, 307
autorefractor, 272 Brucke Bartley effect, 414
autoregulation, 104 Bruckner test, 349, 351
axis of rotation, 289 bull’s-eye maculopathy, 434, 499
Azathioprine, 444
azithromycin, 429 calcitonin, 52
calcium channel blockers, 455
bacitracin, 425 calmodulin, 62
back vertex power, 153, 227 Caloric testing, COWS, 295
Bagolini lenses, 349, 354 captopril, 453
BCC, see fused cross cylinder carbon dioxide, 12
bell’s phenomenon,. 77 transport of, 12
benadryl, 440 carbonic anhydrase inhibitors, 484
benign essential blepharospasm, 76 cardiac muscles, 63
benoxinate, 487 cardiac output, 29
benzalkonium chloride (BAK), 494 cardinal points, 154
benzapril, 453 solving problems with, 156
besifloxacin, 427 carteolol, 483
besivance, 427 catapres, 457
beta blockers, 464 cataracts
betaxolol, 483 posterior subcapsular, 496
Rezold Brucke phenomenon, 396 ceftriaxone, 426
bichrome test, see duochrome test celebrex, 439
Bielchoswky’s head tilt test, 307 celecoxib, 439
bile, 68 cell
binocular cues, 410 transport mechanisms, 4
binocular prism, see vertical imbal­ central retinal artery occlusion, 105
ance cephalexin, 426
bioavailability, 470 cephalosporins, 425, 426
birefringence cerebrovascular accident, CVA, 359,
Haidinger’s brush, 284 363
bitot’s spots, 82 cetirizine, 441
blinking, 75 adverse effects of, 498
types of, 75 chase the red, 273
Bloch’s law, 391 chloramphenicol, 429
blood aqueous barrier, 137 chloroquine, 434
Copyright 2014 by KMK Educational Services, LLC
506 INDEX
adverse effects of, 495, 499 achromatopsias, 404
chlorothiazide, 455 anomalies
chlorpheniramine maleate, 440 color matching behavior, 402
chlorpromazine, 447 brightness, 395
adverse effects of, 447, 495, 496 characteristics of color, 395
chlorpropamide, 451 chromatic discrimination, 401
cholestyramine, 460 chromatopsias, 405
cholinergic agonists, 473 CIE system, 396, 398
indirect agents (AchE Inhibitors), color confusion lines, 401
474 color constancy, 396
cholinergic antagonists, 475 color opponency, 394
cholinergic receptors cellular studies, 394
mechanism of action of, 5 effects on sensitivity, 394
choriocapillaris colorimetric purity, 395
fenestrations within, 107 Farnsworth D-15, 402
choroid Grassman’s laws, 394
aging changes of, 101 heredity and acquisition, 404
functions of, 100 hue, 395
chromatic aberration, 209 Kollner’s rule, 404
use in refraction, 210 metamers, 393
achromatic doublet, 210 Munsell color system, 395
chronic obstructive pulmonary dis­ Nagel anomaloscope, 402
ease, 11 pseudoisochromatic plates, 401
cidofovir saturation, 395
adverse effects of, 498 trichromatic theory, 392
ciliary body anomalies, 398
aging changes of, 100 anomalous trichromacy, 399
functions of, 100 trends of dichromacy, 399 .
cimetidine, 441 dichromacy, 398
ciprofloxacin, 427 coma, 205
circle of least confusion, 163 comitancy, 303
clarithromycin, 429 contact lenses
claritin, 441 accommodation effects, 242
cleocin, 430 exploded system, 228
clindamycin, 430 flexure, 238
clock dial fluorescein staining, 238
seeastigmatic dial, 274 LARS, 239
clonidine, 457 magnification effects, 242
clopidogrel, 459 rigid contact lenses
GN IV palsy, 304 choosing prescription, 229
cocaine, 461 bitoric lenses, 237
collecting duct, 39 gas permeability, 240
colon, 67 SAM-FAP, 229, 232
color vision, 392 tear lens power, 229
absorption peaks of photorecep­ toric lenses, 236
tors, 392 tear lens power
Copyright 2014 by KMK Educational Services, LLC
INDEX 507

estimate, 232 critical flicker fusion frequency (CFF),


eontraocular view, 162 414
contrast cromolyn sodium, 488
CSF, 405, 406 crossed and uncrossed diplopia, 318
high frequency cutoff, 407 crossed retinal disparity, 410
low frequency cutoff, 407 crowding phenomenon, 343
relation to MAR, 407 CT scan, 121
relation to Snellen acuity, 407 curvature of field, 207
definition, 405 cyciobenzaprine, 446
SMTF for lens, 405 cyclopentolate, 477
convergence excess, 336 cyclosporine, 444
convergence excess, CE, 323, 341, cylinder lens: optics definition, 162
342
convergence insufficiency, 334 dark adaptation, 382
convergence insufficiency, Cl, 341, not merely photopigment regen­
342 eration, 383
cornea rod-cone break, 382
aging changes of, 94 decentration per lens, 199
corneal regeneration, 92 depth of field, 160
effective refractive index, 231 depth of focus, 160
endothelial pump mechanisms depth perception, 409
of, 89 detectability, 372
epithelial pump mechanisms of, deuteranomalous trichromat, 399
89 deuteranope, 399
nerve functions, 94 developmental eye movement test,
nutrient characteristics for lay­ DEM, 300
ers of, 91 DeVries Rose law, see light adapta­
oxygen requirements for layers tion
of, 90 dexamethasone, 489
permeability of, 86 dextroamphetamine, 447, 463
corneal deturgescence, 89 diabetes mellitus, 54
oral medications for, 450
corneal transparency diabetic retinopathy, 108
theories of, 87 diamox
corneoscleral outflow, 132 adverse effects of, 485
corresponding retinal points, 313 diastole, 19
corticosteroids diazepam, 449
adverse effects of, 496, 498 diclofenac sodium, 489
Cosine diffuser, 379 dicloxacillin, 425
cotton swab testing, 76 diffraction
coumadin, 458 definition, 176
covariance, 348 Rayleigh’s criteria, 177
cover test, 314, 341 digestion
alternating, 316 organs of, 68
subjective, 316 digoxin, 458
unilateral, 315 adverse effects of, 458, 498
Copyright 2014 by KMK Educational Services, LLC
508 INDEX
dilantin, 449 Arden ratio of, 126
diltiazem, 455 electroretinogram (ERG), 127
diphenhydramine, 440 elestat, 488
diplopia, 303, 345 emedastine, 488
Dipyridamole, 459 emmetropia
dissimilar segs, see vertical imbal­ optics of, 261
ance emphysema, 11
dissociated vertical deviation, DVD, enalapril, 453
357 endocrine hormones, 42
distal convoluted tubule, 38 entrance port, 160
distortion, 208 entrance pupil, 159
barrel, 208 EOMS, see extraocular muscles
pincushion, 208 epinephrine
diurnal variation, 132 adverse effects of, 499
divergence excess, DE, 336, 341, 342 episcleral venous pressure, 133
divergence insufficiency, DI, 335, 341, equivalent power, 153
342 erectile dysfunction medications, 452
Donder’s law, 292 erythromycin, 429
Donepezil, 448 erythropoiesis, 42
Dowling-Rushton equation, 384 esomeprazole, 442
doxycycline, 428 esophagus, 65
Duane’s syndrome, 307, 357, 358 esophoria, EP, 337, 341, 342
ductions, 289 esotropia
abduction, 289, 291 accommodative, 357
adduction, 289, 291 acquired, 357
depression, 289, 291 acute, 357
elevation, 289, 291 consecutive, 358
excycloduction, 289, 291 mechanical, 357
forced, 304, 306 micro, 358
incycloduction, 289, 291 sensory, 357
monocular, 304 types of, 356
duochrome test, 275 estrogens, 452
duodenum, 67 ethambutol, 430, 431
dynamic contour tonometry, 131 adverse effects of, 498
ethylenediaminetetraacetic acid (EDTA),
eccentric 494
viewing, 364 exit port, 160
eccentric fixation, 346, 352 - exit pupil, 159
eccentric viewing, 347 exophoria, XP, 337, 341, 342
edigner westphal nucleus, 129 exotropia
edrophonium, 475 acquired, 358
effective vergence, 151 congenital, 358
ejection fraction, 28 consecutive, 358
electrocardiogram (EKG), 19-21 infantile, 358
electromagnetic spectrum, 171 ■ mechanical, 358
electrooculogram (EOG), 126 micro, 358
Copyright 2014 by KMK Educational Services, LLC
INDEX 509

sensory, 358 fixation, 293


expiration, 9 fixation disparity, FD, 314, 319
exploded system, 228 FD curve, 321
extraocular muscles, 290 paradoxical, 320
dysfunction, 303 flat fusion, 355
inferior oblique, 291 flexeril, 446
inferior rectus, 291 floaters, 103
lateral rectus, 290 flomax, 453
medial rectus, 290 adverse effects of, 498
superior oblique, 291 fluconazole, 433
superior rectus, 291 fluorescein, 491
vestibular control of, 84 fluorescein staining, see contact lenses
extrastriate cortex, 124 fluoress, 487
eye fluorometholone (FML), 489
unique environment of, 106 fluoroquinolones, 427
eye movements fluoxetine, 448
dynamics of, 292 fluticasone, 438
eyelids, 75 focal points, 146
protective functions of, 78 equivalent, 155
role in tear processes, 77 front and back, 153, 155
focal ratio, 160
false negative, 373 forced choice method, 370
false negative rate, 374 foscar.net, 433
false positive, 373 Fourier analysis in visual system,
false positive rate, 374 409
famciclovir, 432 frame adjustments, 203
famotidine, 441 frame specifications, 197
far point, 261 frequency invariance of light, 171
far point sphere, 261 Fresnel’s law, 226
FCC, see fused cross cylinder frontal eye fields, 125
Fechner’s log law, 386 fuchs’ heterochromic irits, 136
Ferry Porter Law, 414 fundus lens, 192
fexofenadine, 441 furosemide, 455
field of view, 161 fused cross cylinder, FCC, 327, 331
trends, 161 fusional vergence dysfunction, FVD,
field stop, 158 338
filters, 378
broad band, 378 GABA, 111
half height bandwidth, 378 gall bladder, 70
interference, 378 ganciclovir, 432
long pass, 379 gastrointestinal function, 63
narrow band, 378 gastrointestinal hormones, 64
neutral density, 379 gatifloxacin, 427
viewing pendulum through, 379 Gauss system, see cardinal points
first order rate kinetics, 420 gemfibrozil, 460
first -pass metabolism, 419 gentamicin, 428
Copyright 2014 by KMK Educational Services, LLC
510 INDEX
geometrical center, 199 summary of histamine receptors,
GI 440
function of, 63 homatropine, 477
GI motility, 70 hormone therapies
GI secretions, 65 adverse effects of, 496
GI system hormones
control, 63 GI tract, 64
gigantism, 47 horopter, 314
glaucoma drugs horror fusionis, 356
adrenergic agonists, 478 hydralazine, 457
adrenergic antagonists, 481 hydrochlorothiazide, 455
carbonic anhydrase inhibitors, hydroxychloroquine, 436
484 adverse effects of, 495
cholinergic agonists, 473, 475 hyperacuity, 266
hyperosmotic agents, 492 hyperopia
prostaglandin analogs, 485 absolute, 264
summary of, 484 epidemiology, 264
glaucomatocyclitic crisis, 136 facultative, 264
glipizide, 451 latent, 263
glucagon, 52 manifest, 264
glucocorticoids, 55 optics of, 263
glutamate, 111 hyperosmotic agents, 492
glyburide, 451 hypertropia, 359
glycerine, 492 hyphema, 136
glycine, 111 hypotropia, 359
goldmann applanation tonometry, 130 hytrin, 453
Granit Harper law, 414 ibuprofen, 438
griseofulvin, 434 ileum, 67
growth hormone, 46 illuminance, 377
Gullstrand’s eye, 259 illumination
dependence on distance, 381
Haidinger’s brush, 284, 349 dependence on tilt, 381
head tilt, 305 image
head turn, 305 definition, 143
heart image jump, 217
anatomy of, 18 imipramine, 449
fast action potentials of, 23 imitrex, 446
slow action potentials of, 25 adverse effects of, 498
sounds of, 20 Imuran, 444
hemiparesis, 362 ' Inderal, 454
hemoglobin, 11 index of refraction
Hering’s law, 292, 305 relation to velocity and wave­
Hess-Lancaster test, 306 length, 171
Hirschberg/Krimsky, 349 indomethacin, 438
histamine adverse effects of, 438, 495, 499
Copyright 2014 by KMK Educational Services, LLC
INDEX 511

inspiration, 8 lasers
insulin, 53 argon, 174
actions of, 53 clinical application, 173
interval of Sturm, 163 helium neon, 175
intraocular pressure Holmium laser, 176
factors influencing, 134 krypton, 175
methods of measurement for, NdrYAG, 175
130 excimer, 174
ipratropium, 443 function, 172
iris LASIK, 174
aging changes of, 100 lasix, 455
functions of, 100 latent nystagmus, 357
isoniazid, 430 lateral magnification, 148
adverse effects of, 498 lens
isoproterenol, 463 aging changes of, 98
isothickness curves, 197 functions of, 95
isotretinoin, 460 glutathione function, 96
adverse effects of, 495, 496, 499 metabolism of, 95
sorbitol production within, 96
Jackson cross cylinder, 273 transparency theories of, 97
Javal’s Rule, 234 vitamin C function, 97
jejunum, 67 lens clock, 191
juxtaglomerular cells, 40 calibration, 191
lens epithelium
keflex, 426 mitotic activity of, 97
kenalog, 437 lens materials, 204
keratometer lens mirror combinations, 167
description, 189 lens proteins
keratometry, 268 regulation of, 96
DK notation, 230 lens size
power and radius of curvature B distance, 199
relation, 231 DBL, 199
Kestenbaum’s rule, 250 GCD, 199
ketoconazole, 433 lens thickness, 196
ketorolac tromethamine, 489 conceptual approach, 197
kidney, 32 relation to power, 196
blood flow through, 33 lens transposition, 164
nephron, 34 lensometer, 185
Knapp’s law, 221 finding bifocal power, 202
Korsakoff syndrome, 461 levitra, 452
kwell, 435 levofloxacin, 427
levothyroxine, 451
Labetalol, 454 LGN
lacrimal pump theory, 77 function of, 122
lag of accommodation, 271 receptive fields of, 123
Lambert surface, 379 light adaptation, 384
Copyright 2014 by KMK Educational Services, LLC
512 INDEX
DeVries Rose law, 385 macugen, 492
neural noise, 385 Maddox rod, 314, 317, 318, 349
rod saturation, 385 double Maddox rod, 318
Weber’s law, 385 magnifiers
light scattering, 178 hand held, 246
Rayleigh, 178 magnification at standard dis­
Tyndall, 179 tance, 245
lindane, 435 maximum possible magnifica­
lipitor, 459 tion, 245
lisinopril, 453 stand, 245
lissamine green, 491 thin lens, 243
Listing’s plane, 289, 292 Magnocellular pathway, 415
liver, 68 major amblyoscope, 349
functions of, 68 major arterial circle of the iris, 107
LogMAR, 267 major reference point, MRP, 199
loop of Henle mannitol, 457
ascending loop of, 37 MAR, 250, 267
descending loop of, 37 marginal astigmatism, see radial astig­
lopid, 460 matism
Lopressor, 454 masking, 415
loratadine, 441 mast cell stabilizers, 488
losartan, 453 Maxwell spot, 349
lotemax, 489 Maxwell’s spot, 285
lovastatin, 459 measuring interpupillary distance,
low vision 265
central vision loss, 252 medial lemniscus pathway, 118
classification, 253 MEM, see monocular estimation method
contrast sensitivity, 254 meperidine, 446
general blur, 252 mesopic conditions, 381
legal blindness (USA), 253 metabolic acidosis, 17
peripheral vision loss, 252 metabolic alkalosis, 17
visual field testing, 254 metacontrast, 415
epidemiology, 252 metaproterenol, 463
lucentis, 492 metformin, 450
lumen, see luminous power method of ascending/descending lim­
lumigan, 485 its, 370
luminance, 377 method of constant stimuli, 370
luminous intensity, 377 methotrexate, 444
luminous power, 376 adverse effects of, 498
lung Methylene blue, 491
anatomy of, 7 methylphenidate, 447, 463
volumes of, 9, 10 Metoprolol, 454
metrogel, 460
M print, 251 metronidazole, 460
Mach bands, 408, 409 miconazole, 433
macrolides, 429 microdrifts, 293
Copyright 2014 by KMK Educational Services, LLC
INDEX 513

microstrabismus, 351 sliding filament theory of con­


microtremors, 293 traction of, 57
midline shift syndrome, 362 smooth, 62
minimum blank size, 199 muscle plane, 289
minimum detectable acuity, 266 myasthenia gravis
minipress, 453 edrophonium tensilon test, 475
minocycline, 428 myopia
adverse effects of, 498, 499 epidemiology, 262
minus lens test, 327, 329 night, 262
miotics optics of, 261
adverse effects of, 496 myosin light chain kinase, 62
mirrors, 166
cornea properties, 190 naproxen, 438
power, 166 naproxen sodium, 438
Misoprostol, 439 natamycin, 433
modified Thorington, 314, 318 near point, 278
monoamine oxidase inhibitors, 449 near point of convergence, NPC, 314,
adverse ocular effects, 479 326, 341
monochromatic aberrations, 205 near reflex triad, 129, 303
monocular cues, 410 negative predictive value, 374
monocular estimation method, MEM, negative relative accommodation, NRA,
327, 330, 342 314, 327, 331, 342
monofixation syndrome, 358 neglect, 362, 363
Montelukast, 443 neovascular glaucoma, 136
Moore’s lightning streaks, 285
motility testing, 304 neurosyphilis, 129
motion after-effects, 412 neurotrophic keratitis, 94
motion perception, 410 nexium, 442
beta movement, 411 nifedipine, 455
MT, 411 nodal points
phi movement, 411 for thick lenses, 157
V5, 411 non-strabismic binocular vision dis­
motor fusion, 313 orders, 314
moxifloxacin, 427 noncontact tonometry, 131
MRI scan, 121 normal retinal correspondence, 347,
mucomyst, 443 355
mucous NPC, see near point of convergence
GI tract, 65 NRA, see negative relative accom­
mucous fishing syndrome, 82 modation
multifocal lenses, 199 NSAIDS
Muro 128, 493 adverse effects of, 496, 499
muscle nuclear sclerosis, 99
review of excitation-contraction nystagmus, 295
coupling of skeletal mus­ pathologic, 296
cle, 57 types of, 295
skeletal, 56 nystatin, 433
Copyright 2014 by KMK Educational Services, LLC
514 INDEX
object parathyroid hormone, 52
definition, 143 Park’s 3 step, 304, 307
oblique astigmatism, see radial astig­ Parvocellular pathway, 415
matism pascal tonometry, 131
obstructive airway disease, 11 pegaptanib, 492
occlusion, 364 Pelli-Robson chart, 254
binasal, 364 penicillin, 425
ocuflox, 427 pepcid, 441
ocular rotations, 289 Percival’s criterion, 324
oculocentric localization, 314 perfusion pressure, 104
oculogyric crisis, 498 Persantine, 459
ocupress, 483 PET scan, 121
ofloxacin, 427 Petzval Surface, 208
olopatadine, 488 phenelzine, 449
omeprazole, 442 phenobarbital, 450
oncotic pressure, 27 phenylephrine, 479
ophthalmoscope, 185 phenytoin, 449
optical properties of eye, 259 adverse effects of, 498
optical thickness, 177 phi phenomenon, see cover test
optivar, 488 phorias, 303, 314
optokinetic system, 293, 298 photo chromatic interval, 382
OKN drum, 298 zero at 650 nm, 382
oral contraceptives photometry, 376
adverse effects of, 498, 499 photopic system, 381, 388, 391
orthoscopic doublet, 208 photoreceptors
oscillopsia, 295 phototransduction, 110
oseltamivir, 431 stages of visual cycle, 109
osmoglyn, 492 phototransduction, 110
osmosis, 7 pigment dispersion glaucoma, 137
osteoblasts, 51 pilocarpine, 473, 474
osteoclasts, 51 side effects of, 474
over-refraction, 235 pioglitazone, 451
overactive inferior oblique, OIO, 357 piroxicam, 438
oxycodone, 446 pituitary gland
oxygen dissociation curve, 13 hormones of anterior, 43
oxygen permeability (Dk), 91 plaquenil, 436
oxytocin, 50 plavix, 459
polarization
pachymeter, 269 description, 179
pancreas, 68 polarizers, 180
Panum’s fusional area, 314, 319, 410 effect on unpolarized light, 181
papilledema, 106 Malus Law, 181
paracontrast, 415 positive predictive value, 374
parasympathetic nervous system positive relative accommodation, PRA,
actions of, 471 314, 327, 331, 342
parasympathetic pathway, 119 post trauma vision syndrome, 363
Oopyright 2014 by KMK Educational Services, LLG
INDEX 515

posterior vitreous detachment, 103 pupillary pathways, 129


power in an oblique meridian, 165 pupilometer, 266
PRA, see positive relative accom­ Purkinje images, 283
modation Purkinje shift, 382
prazosin, 453 Purkinje tree, see temporal percep­
Prentice’s rule , see prisms tion
presbyopia, 279 pursuits, 86, 293, 301
prescribing adds, 280 abnormalities of, 301
prilosec, 442 push-up test, 327
principal planes pyramidal motor pathway, 113
locations, 157 pyrimethamine, 424
principle of univariance, 381
prism, 364 quixin, 427
INWAVE, 364
yoked, 362 radial astigmatism, 206
prisms, 212 radiometry, 375
apex angle, 213 radiuscope, 189
combining, 216 RAM-GAP, 276
orientation, 214 range of clear vision, 278
power, definition, 212 ranibizumab, 492
thickness, 213 ranitidine, 441
decentering a lens, 214 reading caps, see telescopes
Prentice’s rule, 214 receptive fields
progestins, 452 properties of, 125
progressives, 200 recognition acuity, 266
promethazine, 440 rectum, 67
proparacaine, 487 red lens testing, 306
Propranolol, 454 red-green glasses, 355
prostaglandins, 485 reduced eye, 260
adverse effects of, 496 refraction
general actions of, 435 objective, 269
protanomalous trichromat, 399 subjective, 272
protanope, 399 astigmatism, 273
proximal convoluted tubule (PCT), cycloplegic techniques, 277
36 equalization techniques, 276
prozac, 448 refining sphere, 275
pseudo Cl, 334 vertex distance problem, 152
pseudoephedrine, 439 relative distance magnification, 243
pseudo exfoliative glaucoma, 137 relative size magnification, 243
pseudomyopia, 339 relative spectacle magnification, 220
pseudophakia, 280 residual astigmatism
pseudotumor cerebri, 499 rigid contact lens, 233
psychometric function, 369 resolution acuity, 266
Pulfrich phenomenon, 379 respiration
pull-away test, 327, 328 neural control of, 16
pupillary block, 95 regulation of, 16
Copyright 201.4 by KMK Educational Services, LLC
516 INDEX
respiratory acidosis, 17 selection of, 202
respiratory alkalosis, 17 seg inset, 200, 201
restrictive airway disease, 11 sensitivity, 374
reticulospinal pathway, 114 sensory fusion, 313
retina, 108 serotonin selective reuptake inhibitors
aging changes of, 112 (SSRTs)
blood flow of, 107 adverse effects of, 496
functions of, 111 Sheard’s criterion, 324
retinal cells Sherrington’s law, 292
receptive fields of, 111 side effects
retinal illumination, 380 summary of topical drugs, 494
retinal image sizes signal detection theory, 371
ametropes, 221 sildenafil, 452
retinitis pigmentosa, 128 adverse effects of, 496
retinoscopy simultaneous contrast, 386
accounting for working distance, simvastatin, 459
270 single spherical refracting interfaces,
dynamic, 271 145
static, 269 concave/convex convention, 145
retrovir, 431 sinoatrial node (SA) node, 22
rhodopsin skeletal muscle, 56
bleaching, 385 slab off, see vertical imbalance
Ribavirin, 431 slit lamp, 191
Ricco’s law, 388 small intestine, 66
rifabutin smooth muscle, 62
adverse effects of, 498 Snell’s Law, 149
rifampin, 430 Snellen fraction, 267
ritalin, 463 soft contact lenses, 241
adverse effects of, 496 spatial summation, 387
adverse effects on pupil size, 497 biophysics, 387
ROC curve, 374 toy example, 387
rocephin, 426 specificity, 374
rose bengal, 491 speckle effect, 176
spectacle magnification, 219
saccades, 86, 293, 298 aniseikonia considerations, 223
abnormalities of, 299 power factor, 219
microsaccades, 293 shape factor, 219
suppression of, 416 spherical aberration, 205
scopolamine, 476 spherical equivalent, 163
scotopic lumens, 377 spherical lens
scotopic system, 377, 381, 388, 390, optics definition, 162
391 spinothalamic pathway, 117
secretions Spiral of Tillaux, 291
mouth, 65 spirograms, 11
seg depth, 200 spironolactone, 456
seg height, 200 stairstep method, 370
Copyright 2014 by KMK Educational Services, LLC
INDEX 517

stand magnifier TBI


add needed, 151 testing, 360
Starling equation, 26 treatment, 364
stenopaic slit, 273 tbut, 82
stereopsis, 349, 355, 356 tear lens, see contact lenses
contour, 356 tears, 78
global, 356 distribution of, 82
steroids, 488 elimination of, 83
mechanism for increasing IOP, function of, 78
500 layers of, 79
ocular adverse effects of, 489 mucous layer of, 81
systemic adverse effects of, 437 osmolarity of, 83
triamcinolone, 437 tectospinal pathway, 115
Stevens’ power law, 386 telemicroscope, 249
Stiles-Crawford effect, 392 telescopes, 246
stomach, 66 using for near object, 247
cells of, 66 choosing power, 250
strabismus, 345, 356 finding exit pupil size, 248
other, 357, 358 finding lens separation, 247
paralytic, 359 Galilean, 247
sturge-weber syndrome, 133 Kepler, 246
successive imaging, 154 magnification, 248
sucralfate, 442 reading caps, 249
sudafed, 439 temporal perception
sulfonamides, 424 OFF, 414
adverse effects of, 496, 498 percentage modulation, 413
sulfonylureas, 451 TMTF, 412
sumatriptan, 446 high frequency cutoff, 413
superior colliculus, 125 low frequency cutoff, 413
suppression, 355 Purkinje tree, 413
sympathetic nervous system Troxler phenomenon, 413
actions of, 471 temporal summation, 390
sympathetic pathway, 119 toy example, 390
synthroid, 451 Tenormin, 454
systole, 19 terazosin, 453
tetracycline, 428
tagamet, 441 adverse effects of, 496
Talbot Plateau law, 414 theophylline, 443
talc therapeutic index, 422
adverse effects of, 499 thiazide diuretics, 39
tamiflu, 431 thick lenses, 152
tamoxifen, 445 thimerosal, 494
adverse effects of, 495, 499 thin films
tamsulosin, 453 best index of refraction, 178,
adverse effects of, 498 226
tangential point, 289 minimum thickness, 177
Copyright 2014 by KMK Educational Services, LLC
518 INDEX
description, 177 uveoscleral outflow, 133
thioridazine, 447
adverse effects of, 499 valacyclovir, 432
threshold, 369 valium, 449
criteria, 372 vardenafil, 452
thyroid vasa recta, 41
stimulating hormone of, 43 vascular endothelial growth factor
timolol, 482 (VEGP), 492
tobramycin, 428 verapamil, 455
topiramate, 450 vergence, 86
total inset, 200 definition, 143
total internal reflection, 149 vergence eye movements, 290, 293,
tramadol, 446 302
Trandate, 454 accommodative convergence, 303
transmissibility (Dk/t), 91 fusional vergence, 303
transmittance, 225 proximal convergence, 303
traumatic brain injury, TBI, 359 smooth vergence testing, 314,
travatan, 485 323
triamcinolone, 437 step vergence testing, 314, 326
triamterene, 456 tonic convergence, 303
tricyclic antidepressants, 449 vergence facility, 314, 326
adverse ocular effects, 479 vergence ranges, 314, 323
trifocals, 200 versions, 290
trigeminothalamic pathway, 117 vertex distance
trimethoprim, 424 power considerations, 151
tritanope, 399 correcting refraction, 227
trochlear nerve vertical imbalance, 216
palsy, 361 correcting, 217
trolands dissimilar segs, 217
seeretinal illumination, 380 slab off, 217
tropia, 345, 356 vertical phoria, 338
tropicamide, 476 vestibular pathway, 115
Troxler effect, 293 vestibulo-ocular reflex, VOR, 292,
Troxler phenomenon, see temporal 294
perception vestibulo-ocular system
true negative, 373 abnormalities of, 294
true positive, 373 vestibulospinal pathway, 115
tylenol, 445 viagra, 452
type I error, see false positive adverse effects of, 496, 498
type II error, see false negative vigamox, 427
visine, 479
ultraviolet keratopathy, 87 vision therapy, 365
uncrossed retinal disparity, 410 Vistech contrast system, 254
unilateral inattention,' 363 visual acuity, 266
uvea, 99 types, 266
uveitis, 136 visual confusion, 346
Copyright 2014 by KMK Educational Services, LLC
INDEX 519

visual cortex, 124


visual cycle, 109
visual evoked potentials (VEP), 128
visual pigments, 109
formation of, 109
visuoscopy, 349, 352
vitreous, 101
aging changes of, 103
composition of, 101
functions of, 101
metabolism of, 103
vitreous syneresis, 103
voltaren, 489
volume of distribution, 419
von Graefe phoria, 314, 317
VOR, see vestibulo-ocular reflex
warfarin, 458
Weber’s law, see light adaptation
Wernicke’s encephalopathy, 461
whorl keratopathy, 495
Worth 4 dot, 349, 355
X-chrom lenses, 399
xalatan, 485
yoked muscles, 292
zaditor, 488
Zafirlukast, 443
zantac, 441
zero order rate kinetics, 421
zidovudine, 431
zocor, 459
zymar, 427
zyrtec, 441

Copyright 2014 by KMI< Educational Services, LLC


The KMK Part One Applied Science
Review Guide is a concise, organized
resource specifically tailored to board
exam preparation. Formatted with a
focus on exam review, the text offers
a structured approach to study,
including summary diagrams, con-
densed scientific explanations, and
an increased emphasis on clinical
applications. As a companion text
for board exam preparation or an
additional resource for classroom
success, the KMK Part One
Applied Science Review Guide is a
single, compact resource ideal for
self-study and student achievement.

You might also like